Download as pdf or txt
Download as pdf or txt
You are on page 1of 370

TABLE OF CONTENTS

GOLDEN SUN FINANCE CORP. VS. RICARDO ALBANO........................................................................ 2


GENEROSA ALMEDA LATORRE VS. LUIS ESTEBAN LATORRE ........................................................... 4
GAVINO TUMALAD AND GENEROSA VS. ABERTO VICENCIO AND EMILIANO SIMEON .................... 7
PRUDENTIAL BANK VS. HONORABLE DOMINGO PANIS ...................................................................... 8
MERALCO SECURITIES INDUSTRIAL CORPORATION VS. CBAA ...................................................... 10
STAR TWO, INCOPORATION VS. PAPER CITY CORPORATION ......................................................... 12
SERG’S PRODUCTS, INC., ET AL. VS. PCI LEASING AND FINANCE, INC. ......................................... 14
RUBY L. TSAI VS. COURT OF APPEALS, ET AL. .................................................................................. 16
MAKATI LEASING, ET AL., VS. WEAREVER TEXTILE MILLS, INCORPORATION., ET AL. ................. 18
LIGHT RAIL TRANSIT AUTHORITY VS. CENTRAL BOARD OF ASSESSMENT APPEALS, ET AL. ..... 19
CALTEX (PHILIPPINES) INC. VS. CENTRAL BOARD OF ASSESSMENT APPEALS, ET AL. ............... 21
FELS ENERGY, INC. VS. THE PROVINCE OF BATANGAS , ET AL. ..................................................... 22
JIMMY T. GO VS. UNITED COCONUT PLANTERS BANK ..................................................................... 25
LUIS MARCOS P. LAUREL VS. HON. ZEUS C. ABROGAR ................................................................... 27
BPI FAMILY BANK VS. AMADO FRANCO and COURT OF APPEALS ................................................... 28
REPUBLIC OF THE PHILIPPINES VS. APOSTOLITA SAN MATEO, ET. AL>........................................ 31
REMMAN ENTERPRISES, INC. VS. REPUBLIC OF THE PHILIPPINES ................................................ 32
REPUBLIC OF THE PHILIPPINES VS. MICHAEL C. SANTOS, ET. AL. ................................................. 33
REPUBLIC OF THE PHILIPPINES VS. CORAZON C. SESE and FE C. SESE ....................................... 34
REPUBLIC OF THE PHILIPPINES VS. EMMANUEL C. CORTEZ ........................................................... 36
REPUBLIC OF THE PHILIPPINES VS. FRANCISCA SANTOS, ET AL. .................................................. 37
REPUBLIC VS. AFP RETIREMENT AND SEPARATION BENEFITS SYSTEM ...................................... 39
JEAN TAN, ET AL. VS. REPUBLIC .......................................................................................................... 41
VICENTE YU CHANG AND SOLEDAD YU CHANG VS. REPUBLIC ...................................................... 43
RAMON ARANDA VS. REPUBLIC OF THE PHILIPPINES ...................................................................... 45
REPUBLIC OF THE PHILIPPINES VS. BANTIGUE POINT DEVELOPMENT CORPORATION ............. 46
JOYCE Y. LIM VS. REPUBLIC OF THE PHILIPPINES ............................................................................ 47
HEIRS OF MARIO MALABANAN VS. REPUBLIC OF THE PHILIPPINES............................................... 49
THE SECRETARY OF DENR ET. AL.VS. MAYOR JOSE S. YAP ........................................................... 50
HEIRS OF SPOUSES PEDRO S. PALANCA VS. REPUBLIC OF THE PHILIPPINES, ET. AL. ............. 53
LUNINGNING P. DEL ROSARIO-IGBITEN, ET. AL.. VS. REPUBLIC OF THE PHILIPPINES................. 54
REPUBLIC OF THE PHILIPPINES VS. CELESTINA NAGUIAT .............................................................. 56
THE DIRECTOR OF FORESTRY VS. RUPERTO A. VILLAREAL ........................................................... 58
EDUBIGIS GORDULA, CELSO VS. FERNANDEZ, JR., ET. AL. ............................................................. 59
SPOUSES GEMINIANO AND AMPARO DE OCAMPO, ET. AL. VS. FEDERICO ARLOS, ET. AL. ........ 61
ZARATE VS. THE DIRECTOR OF LANDS .............................................................................................. 63
REPUBLIC OF THE PHILIPPINES VS. COURT OF APPEALS, HON. JOSE D. AZARRAGA AND ANGEL
T. YU ......................................................................................................................................................... 65
REPUBLIC OF THE PHILIPPINES VS. CANDY MAKER, INC. ................................................................ 67
JOSE MENCHAVEZ, ET. AL. VS. FLORENTINOTEVES JR. .................................................................. 71
REPUBLIC OF THE PHILIPPINES VS. SOCORRO P. JACOB ............................................................... 73
FRANCISCO I. CHAVEZ, VS. PUBLIC ESTATES AUTHORITY AND AMARI COASTAL BAY
DEVELOPMENT CORPORATION ........................................................................................................... 75
ARBIAS VS. REPUBLIC OF THE PHILIPPINES ...................................................................................... 78
REPUBLIC VS. CAYETANO SERRANO .................................................................................................. 79
REPUBLIC VS. HEIRS OF JUAN FABIO ................................................................................................. 80
FLORENCIA DIAZ VS. REPUBLIC ........................................................................................................... 81
MIAA VS. CITY OF PASAY ...................................................................................................................... 83
MIAA VS. COURT OF APPEALS ............................................................................................................. 84
PHILIPPINE FISHERIES DEVELOPMENT AUTHORITY VS. COURT OF APPEALS ............................. 86
MACASIANO VS. DIOKNO....................................................................................................................... 89
REPUBLIC VS. RTC, CITY OF ROXAS ................................................................................................... 91
ANECO REALTY AND DEVELOPMENT CORPORATION VS. LANDEX DEVELOPMENT .................... 94
OCAMPO VS. DIOKNO ............................................................................................................................ 97
BONIFACIO PEIDAD VS. SPOUSES GURIEZA .................................................................................... 101
LIM VS. LIGON ....................................................................................................................................... 103
GABRIEL VS. CRISOLOGO ................................................................................................................... 105
SUAREZ VS. EMBOY ............................................................................................................................. 107
DELA CRUZ VS. CAPCO ....................................................................................................................... 109
G.R. No. 176055 ..................................................................................................................................... 109
March 17, 2014 ....................................................................................................................................... 109
RIVERA-CALINGASAN VS. RIVERA ..................................................................................................... 111
TOLENTINO VS. LAUREL ...................................................................................................................... 113
ROGELIO J. JAKOSALEM and GODOFREDO B. DULFO VS. ROBERTO S. BARANGAN .................. 115
SPOUSES ELEGIO CAÑEZO VS. SPOUSES APOLINARIO AND CONSORCIA L. BAUTISTA ........... 117
MODESTO PALALI VS. JULIET AWISAN .............................................................................................. 119
NELSON LAGAZO VS. GERALD B. SORIANO and GALILEO B. SORIANO ........................................ 122
HEIRS OF PEDRO LAURORA VS. STERLING TECHNOPARK and SP PROPERTIES ....................... 124
PHILIPPINE NATIONAL BANK VS. COURT OF APPEALS ................................................................... 126
SPS. RONALD HUTCHISON AND VALENTINE NAVALLE-HUTCHISO VS. ENRIQUE M. BUSCAS ... 128
RUDY LAO VS. JAIME LAO ................................................................................................................... 130
RENE GANILA VS. HON. COURT OF APPEALS .................................................................................. 132
RUBEN SANTOS VS. SPOUSES TONY AYON and MERCY AYON ..................................................... 134
HEIRS OF DEMETRIO MELCHOR VS. JULIO MELCHOR.................................................................... 136
PABLO D. ACAYLAR, JR., VS. DANILO G. HARAYO ........................................................................... 138
HEIRS OF ANACLETO B. NIETO, ET AL. VS. MUNICIPALITY OF MEYCAUAYAN ............................. 140
GLORIA S. DY VS. MANDY COMMODITIES CO., INC. ........................................................................ 142
VILMA G. ARRIOLA AND ANTHONY RONALD G. ARRIOLA VS. JOHN NABOR C. ARRIOLA ........... 145
MAGDALENA T. VILLASI VS. SPOUSES FILOMENO GARCIA AND ERMELINDA HALILI-GARCIA ... 147
REPUBLIC OF THE PHILIPPINES VS. HOLY TRINITY REALTY DEVELOPMENT CORP. ................. 150
EQUATORIAL REALTY DEVELOPMENT, INC. VS. MAYFAIR THEATER, INC. .................................. 153
ELVIRA ARANGOTE VS. SPOUSES MARTIN and LOURDES MAGLUNOB AND ROMEO SALIDO ... 156
TORBELA VS SPOUSES ROSARIO ...................................................................................................... 158
FILOMENA R. BENEDICTO VS. ANTONIO VILLAFLORES .................................................................. 160
LUCIANO BRIONES AND NELLY BRIONES VS. JOSE MACABAGDAL, ET. AL. ................................ 161
BARSTOWE PHILIPPINES CORPORATION VS. REPUBLIC OF THE PHILIPPINES .......................... 162
SPOUSES NUGUID VS. COURT OF APPEALS .................................................................................... 164
PHILIPPINE NATIONAL BANK VS. DE JESUS ..................................................................................... 166
PLEASANTVILLE DEVELOPMENT CORPORATION VS. COURT OF APPEALS ................................ 167
EVADEL REALTY AND DEVELOPMENT CORPORATION VS. SPOUSES SORIANO ........................ 169
TECNOGAS PHILIPPINES MANUFACTURING CORPORATION VS. COURT OF APPEALS ............. 171
SPOUSES REYNATO AND EDITHA LOPEZ VS. MARGARITA SARABIA............................................ 173
PROGRAMME INCORPORATED VS. PROVINCE OF BATAAN ........................................................... 175
FEDERICO GEMINIANO, ET. AL., VS. COURT OF APPEALS ............................................................. 177
SULO SA NAYON, INCORPORATED VS. NAYONG PILIPINO FOUNDATION .................................... 179
SERAFIN CHENG VS. SPOUSES DONINI ............................................................................................ 181
SPOUSES JIMENEZ VS. PATRICIA, INCORPORATED ....................................................................... 182
NATIONAL HOUSING AUTHORITY VS. GRACE BAPTIST CHURCH .................................................. 184
FERNANDO ET. AL., VS. ACUNA ET. AL. ............................................................................................. 187
OFFICE OF THE CITY MAYOR OF PARAÑAQUE CITY VS. MARIO D. EBIO ..................................... 189
DIONISIA P. BAGAIPO VS. COURT OF APPEALS ............................................................................... 191
HEIRS OF EMILIANO NAVARRO VS. INTERMEDIATE APPELLATE COURT ..................................... 192
DESAMPARADO VDA. DENAZARENO ET. AL., VS. COURT OF APPEALS ....................................... 194
SPOUSES CACHOPERO VS. RACHEL CELESTIAL ............................................................................ 195
MAXIMO JAGUALING, ET. AL., VS. COURT OF APPEALS .................................................................. 196
CYNTHIA CRUZ KHEMANI VS. THE HEIRS OF ANASTACIO TRINIDAD ........................................... 198
EULOGIO AGUSTIN, ET. AL., VS. INTERMEDIATE APPELLATE COURT .......................................... 200
MARCELINO C. AGNE VS. THE DIRECTOR OF LANDS...................................................................... 202
LEOGARIO RONQUILLO, ET AL. VS. JOSE ROCO, ET AL.................................................................. 204
HILARIO VS. CITY OF MANILA ............................................................................................................. 205
RESIDENTS OF LOWER ATAB, ET. AL. VS. STA. MONICA INDUSTRIAL & DEVELOPMENT
CORPORATION ..................................................................................................................................... 209
HEIRS OF PACIFICO POCDO VS. ARSENIA AVILA AND EMELINDA CHUA ...................................... 211
TEOFISTO ONO, ETC. VS. VICENTE LIM............................................................................................. 212
INOCENCIO Y. LUCASAN VS. PHILIPPINE DEPOSIT INSURANCE CORPORATION ........................ 214
SPOUSES MARCOS VS. HEIRS OF ISIDRO BANGI ............................................................................ 218
RAUL V. ARAMBULO VS. GENARO NOLASCO ................................................................................... 221
TEODORO S. TEODORO VS. DANILO ESPINO, ET. AL. ..................................................................... 222
ANTIPOLO INING VS. LEONARDO R. VEGA ........................................................................................ 224
CAROLINA VDA. DE FIGURACION ET. AL. VS. EMILIA FIGURACION-GERILLA ............................... 227
SPOUSES PASCUAL, ET. AL. vS. SPOUSES BALLESTEROS ............................................................ 228
G.R. No. 186269 ..................................................................................................................................... 228
February 15, 2012................................................................................................................................... 228
NENITA GONZALES, ET. AL. VS. MARIANO BUGAAY, ET. AL. .......................................................... 230
JOSE FERNANDO, JR., ET. AL. VS. LEON ACUNA, ET. AL. ............................................................... 231
JULITA ROMBAUA PANGANIBAN, ET. AL. VS. JULITA S. OAMIL ...................................................... 232
SPOUSES DEL CAMPO VS. BERNARDA FERNANDEZ ABESIA ........................................................ 234
TITAN CONSTRUCTION VS. DAVID ..................................................................................................... 235
AGUIRRE, ET AL. VS. COURT OF APPEALS, ET AL. .......................................................................... 237
IGLESIA NI KRISTO VS. PONFERRADA............................................................................................... 239
SPOUSES MENDOZA VS. MARIA CORONEL ...................................................................................... 242
DE GUIA VS. COURT OF APPEALS ..................................................................................................... 244
VILLANUEVA VS. COURT OF APPEALS .............................................................................................. 245
AVILA VS. BARABAT ............................................................................................................................. 248
AGUILAR VS. AGUILAR ......................................................................................................................... 249
FERNANDEZ VS. TARUN ...................................................................................................................... 250
MONTEROSO VS. COURT OF APPEALS ............................................................................................. 251
LEONORA B. CRUZ VS. TEOFILA M. CATAPANG ............................................................................... 253
REPUBLIC OF THE PHILIPPINES VS. HEIRS OF DIGNOS-SORONO ................................................ 255
CELESTINO BALUS VS. SATURNINO & LEONARDA BALUS VDA. DE CALUNOD ............................ 256
ELY QUILATAN ET. AL VS. HEIRS OF LORENZO QUILATAN............................................................. 258
REILLO ET.AL. VS. SAN JOSE .............................................................................................................. 260
FELICIDAD DADIZON, ET AL. VS. SOCORRO BERNADAS ................................................................ 262
SPOUSES LITA DE LEON & FELIX RIO TARROSA VS. ANITA B. DE LEON, ET. AL. ........................ 264
ROLANDO S. ABADILLA, JR. VS. SPS. BONIFACIO P. OBRERO ....................................................... 267
HEIRS OF BIENVENIDO & ARACELI TANYAG VS. SALOME E. GABRIEL, ET. AL. ........................... 269
CELERINO E. MERCADO VS. BELEN ESPINOCILLA & FERDINAND ESPINOCILLA......................... 271
JULITA V. IMUAN, ET. AL. VS. JUANITO CERENO, ET. AL. ................................................................ 273
SEMIRARA COAL CORPORATION VS. HGL DEVELOPMENT CORPORATION ................................ 275
SOLEDAD CALICDAN VS. SILVERIO CENDANA ................................................................................. 277
LINA LUBOS VS. MARITES GALUPO ................................................................................................... 278
ERNANDA MENDOZA CEQUEÑA VS. HONORATA MENDOZA BOLANTE ........................................ 280
PHILIPPINE NATIONAL BANK VS. COURT OF APPEALS ................................................................... 282
CARMEN CANIZA VS. COURT OF APPEALS....................................................................................... 284
HABAGAT GRILL VS. DMC – URBAN PROPERTY DEVELOPER, INC. .............................................. 285
PEDRO SEPULVEDA, SR. VS. ATTY. PACIFICO S. PELAEZ .............................................................. 287
NATIONAL HOUSING AUTHORITY VS. GRACE BAPTIST CHURCH .................................................. 289
EMILIO GANCAYCO VS. CITY GOVERNMENT OF QUEZON CITY & MMDA ..................................... 291
GUILLERMO M. TELMO VS. LUCIANO M. BUSTAMANTE ................................................................... 293
CITY OF MANILA VS. LAGUIO .............................................................................................................. 295
JAIME S. PEREZ VS. SPOUSES FORTUNITO L. MADRONA & YOLANDA B. PANTE ....................... 297
LUCENA GRAND CENTRAL TERMINAL, INC. VS. JAC LINER, INC. .................................................. 298
PEDRO J. VELASCO VS. MANILA ELECTRIC CO. .............................................................................. 299
HIDALGO ENTERPRISES, INC. VS. GUILLERMO BALANDAN, ET. AL. ............................................. 301
JOSE "PEPITO" TIMONER VS. PEOPLE OF THE PHILIPPINES ......................................................... 302
RESTITUTO CALMA VS. COURT OF APPEALS ................................................................................... 303
CONCEPCION PARAYNO VS. JOSE JOVELLANOS ............................................................................ 304
PILAR DEVELOPMENT CORPORATION VS. RAMON DUMADAG ...................................................... 306
BICOL AGRO-INDUSTRIAL PRODUCERS COOPERATIVE, INC. VS. OBIAS, ET AL. ........................ 307
SPOUSES SABIO VS. THE INTERNATIONAL CORPORATE BANK, INC. ........................................... 309
EDILBERTO ALCANTARA VS. CORNELIO B. RETA, JR...................................................................... 311
NATIONAL IRRIGATION ADMINISTRATION VS. COURT OF APPEALS ............................................. 313
CAMARINES NORTE ELECTRIC COOPERATIVE, INC. VS. COURT OF APPEALS ........................... 316
BRYAN U. VILLANUEVA VS. HON. TIRSO D.C. VELASCO ................................................................. 318
SPOUSES STA. MARIA VS. COURT OF APPEALS .............................................................................. 320
REMMAN ENTERPRISES, INC. VS. COURT OF APPEALS ................................................................. 322
COSTABELLA CORPORATION VS. COURT OF APPEALS, ET. AL. ................................................... 324
TOMAS ENCARNACION VS. COURT OF APPEALS ............................................................................ 326
ELISEO FAJARDO, JR., and MARISSA FAJARDO VS. FREEDOM TO BUILD, INC. ........................... 329
SPOUSES DE LA CRUZ VS. OLGA RAMISCAL ................................................................................... 331
ENEDINA PRESLEY VS. BEL-AIR VILLAGE ASSOCIATION, INC. ...................................................... 335
SOLID MANILA CORPORATION VS. BIO HONG TRADING CO. ......................................................... 337
REPUBLIC OF THE PHILIPPINES VS. LEON SILIM and ILDEFONSA MANGUBAT ............................ 340
DECS VS. JULIA DEL ROSARIO, ET. AL. ............................................................................................. 342
GONZALES VS. COURT OF APPEALS ................................................................................................. 344
MARIA ALVAREZ VDA. DE DELGADO VS. COURT OF APPEALS ...................................................... 345
CIRILA ARCABA VS. ERLINDA TABANCURA VDA. DE BATOCAEL, ET. AL. ..................................... 347
RICKY QUILALA VS. GLICERIA ALCANTARA, ET. AL. ........................................................................ 349
APOLINARIA AUSTRIA-MAGAT VS. COURT OF APPEALS ................................................................ 351
HEIRS OF SEVILLA FLORENCIO VS. HEIRS OF SEVILLA DE LEON ................................................. 353
C-J YULO & SONS, INC. VS. ROMAN CATHOLIC BISHOP OF SAN PABLO, INC. ............................. 355
MA. ESTELA MAGLASANG, ET. AL. VS. HEIRS OF CORAZON CABATINGAN .................................. 357
MANUEL G. ABELLO, ET. AL. VS. COMMISSIONER OF INTERNAL REVENUE ................................ 359
SPOUSES GESTOPA VS. COURT OF APPEALS ................................................................................. 360
REPUBLIC OF THE PHILIPPINES VS. DAVID REY GUZMAN ............................................................. 362
IMPORTANCE
OF KNOWING
CLASSIFICATION
OF PROPERTY

1
GOLDEN SUN FINANCE CORP. VS. RICARDO ALBANO

A.M. No. P-11-2888


July 27, 2011

Facts:

A complaint was filed for the recovery of a Honda Civic Sedan, against one Lucila S.
Reyes, The subject motor vehicle, registered in the name of Reyes, was encumbered in
its favor, The RTC decided in favor of the complainant and issued a writ of replevin.
However, the complainant found out that the motor vehicle had already been levied
upon by the respondent by virtue of a writ of execution for violation of Batas Pambansa
Bilang 22 against Reyes.

It was sold at a public auction conducted by the respondent on April 29, 2009, with the
Royal Makati Credit Resource as the highest bidder. On the same day, a Certificate of
Sale was issued in favor of the Royal Makati Credit Resource.The complainant averred
that the levy and sale of the motor vehicle by the respondent was illegal; and that the
encumbrance on the motor vehicle having been made prior to the suit filed by the Royal
Makati Credit Resource, the complainant posited that its claim should have priority over
the formers claims.

The sheriff (respondent) contended that he had no knowledge that the car was
encumbered because the Certificate of Registration was never shown to him. He also
had no knowledge that the car was the subject of a writ of replevin. Thus, the
respondent asked for the dismissal of the complaint, stressing that he had acted within
the scope of his duty as sheriff when he enforced the writ of execution.

Issue:

Whether or not the sheriff is liable for simple neglect of duty

Ruling:

No. The Court held that: the officer shall levy upon the properties of the judgment
obligor of every kind and nature whatsoever which may be disposed of for value and not
otherwise exempt from execution giving the latter the option to immediately choose
which property or part thereof may be levied upon, sufficient to satisfy the judgment. If
the judgment obligor does not exercise the option, the officer shall first levy on the
personal properties, if any, and then on the real properties if the personal properties are

2
insufficient to answer for the judgment. More so, the duty of the sheriff to execute a writ
is simply ministerial and he is bound to perform only those tasks stated under the Rules
of Court and no more.

3
GENEROSA ALMEDA LATORRE VS. LUIS ESTEBAN LATORRE

G.R. No. 183926


March 29, 2010

Facts:

In October 2000, petitioner filed before the RTC of Muntinlupa City a Complaint for
Collection and Declaration of nullity of Deed of Absolute Sale with application for
Injunction against her own son, herein respondent, and one Ifzal Ali Petitioner alleged
that respondent leased a parcel of land that they co-owned to Ifzal in Dasmarinas
Village and that respondent declared that he is the sole owner of the said parcel of land.

Respondent immediately filed a Motion to Dismiss on the sole ground that the venue of
the case was improperly laid. He stressed that while the complaint was denominated as
one for Collection and Declaration of nullity of Deed of Absolute Sale with application for
Injunction, in truth the case was a real action affecting title to and interest over the
subject property. Since the subject property is located in Makati City, respondent argued
that petitioner should have filed the case before the RTC of Makati City and not of
Muntinlupa City'.

The RTC denied the Motion to Dismiss of the respondent on January 2, 2001. However,
on April 28, 2008, the RTC dismissed the case for lack of Jurisdiction because the case
should have been filed in RTC Makati. Petitioner filed an Motion for Review which was
denied. Hence, this petition.

Issue:

Whether or not the Petitioner‗s contention is correct

Ruling:

No. The court held that actions affecting title to or possession of real property or an
interest therein (real actions) shall be commenced and tried in the proper court that has
territorial jurisdiction over the area where the real property is situated. On the other
hand, all other actions (personal actions) shall be commenced and tried in the proper
courts where the plaintiff or any of the principal plaintiffs resides or where the defendant
or any of the principal defendants resides

4
Hence, The action in the RTC, other than for Collection, is located in Makati City. The
venue for such action is unquestionably the proper court of Makati City, where the real
property or part thereof lies, not the RTC of Muntinlupa City.

5
IMMOVABLE
PROPERTY
BY
INCORPORATION

6
GAVINO TUMALAD AND GENEROSA VS. ABERTO VICENCIO AND EMILIANO
SIMEON

G.R. No. L-30173


September 30, 1971

Facts:

Vicencio and Simeon executed a chattel mortgage in favor of plaintiffs Tumalad over
their house, which was being rented by Madrigal and company. This was executed to
guarantee a loan, payable in one year with a 12% per annum interest. The mortgage
was extrajudicially foreclosed upon failure to pay the loan. The house was sold at a
public auction and the plaintiffs were the highest bidder. A corresponding certificate of
sale was issued. Thereafter, the plaintiffs filed an action for ejectment against the
defendants, praying that the latter vacate the house as they were the proper owners.

Issues:

Whether or not the chattel mortgage was null and void ab initio because only personal
properties can be subject of a chattel mortgage.

Ruling:

Certain deviations have been allowed from the general doctrine that buildings are
immovable property such as when through stipulation, parties may agree to treat as
personal property those by their nature would be real property. This is partly based on
the principle of estoppel wherein the principle is predicated on statements by the owner
declaring his house as chattel, a conduct that may conceivably stop him from
subsequently claiming otherwise. In the case at bar, though there be no specific
statement referring to the subject house as personal property, yet by ceding, selling or
transferring a property through chattel mortgage could only have meant that defendant
conveys the house as chattel, or at least, intended to treat the same as such, so that
they should not now be allowed to make an inconsistent stand by claiming otherwise.

7
PRUDENTIAL BANK VS. HONORABLE DOMINGO PANIS

G.R. No. L-50008


August 31, 1987

Facts:

Spouses Magcale secured a loan from Prudential Bank. To secure payment, they
executed a real estate mortgage over a residential building. The mortgage included also
the right to occupy the lot and the information about the sales patent applied for by the
spouses for the lot to which the building stood. After securing the first loan, the spouses
secured another from the same bank. To secure payment, another real estate mortgage
was executed over the same properties. The Secretary of Agriculture then issued a
Miscellaneous Sales Patent over the land which was later on mortgaged to the bank.
The spouses then failed to pay for the loan and the REM was extrajudicially foreclosed
and sold in public auction despite opposition from the spouses. The respondent court
held that the REM was null and void.

Issue:

What are the statuses of the mortgages involved, with respect to immovable properties?

Ruling:

A real estate mortgage can be constituted on the building erected on the land belonging
to another. The inclusion of building distinct and separate from the land in the Civil Code
can only mean that the building itself is an immovable property. While it is true that a
mortgage of land necessarily includes in the absence of stipulation of the improvements
thereon, buildings, still a building in itself may be mortgaged by itself apart from the land
on which it is built. Such a mortgage would still be considered as a REM for the building
would still be considered as immovable property even if dealt with separately and apart
from the land. The original mortgage on the building and right to occupancy of the land
was executed before the issuance of the sales patent and before the government was
divested of title to the land. Under the foregoing, it is evident that the mortgage
executed by private respondent on his own building was a valid mortgage. As to the
second mortgage, it was done after the sales patent was issued and thus prohibits
pertinent provisions of the Public Land Act.

8
IMMOVABLE
PROPERTY
BY
DESTINATION

9
MERALCO SECURITIES INDUSTRIAL CORPORATION VS. CBAA

GR No. L-46245
May 31, 1982

Facts:

Pursuant to a pipeline concession issued under the Petroleum Act of 1949, Republic Act
No. 387, Meralco Securities installed from Batangas to Manila a pipeline system
consisting of cylindrical steel pipes joined together and buried not less than one meter
below the surface along the shoulder of the public highway. The pipes are embedded in
the soil and are firmly and solidly welded together so as to preclude breakage or
damage thereto and prevent leakage or seepage of the oil. The valves are welded to
the pipes so as to make the pipeline system one single piece of property from end to
end.

In order to repair, replace, remove or transfer segments of the pipeline, the pipes have
to be cold-cut by means of a rotary hard-metal pipe-cutter after digging or excavating
them out of the ground where they are buried. In points where the pipeline traversed
rivers or creeks, the pipes were laid beneath the bed thereof. Hence, the pipes are
permanently attached to the land. Pursuant to the Assessment Law, Commonwealth Act
No. 470, the provincial assessor of Laguna treated the pipeline as real property and
issued tax declarations, containing the assessed values of portions of the pipeline.

Meralco appealed the assessments to the defendants, but the latter ruled that pipeline
is subject to realty tax. The defendants argued that the pipeline is subject to realty tax
because they are contemplated in Assessment Law and Real Property Tax Code; that
they do not fall within the category of property exempt from realty tax under those laws;
that Articles 415 & 416 of the Civil Code, defining real and personal property have no
applications to this case because these pipes are constructions adhered to soil and
things attached to the land in a fixed manner, and that Meralco Securities is not exempt
from realty tax under petroleum law.

Meralco insists that its pipeline is not subject to realty tax because it is not real property
within the meaning of Art. 415.

Issue:

Whether the aforementioned pipelines are subject to realty tax.

10
Ruling:

Yes, the pipelines are subject to realty tax. Section 2 of the Assessment Law provides
that the realty tax is due ―on real property, including land, buildings, machinery, and
other improvements. This provision is reproduced with some modification in Section 38,
Real Property Tax Code, which provides that ―there shall be levied, assessed, and
collected xxx annual ad valorem tax on real property such as land, buildings, machinery,
and other improvements affixed or attached to real property xxx. It is incontestable that
the pipeline of Meralco Securities does not fall within any of the classes of exempt real
property enumerated in section 3 of the Assessment Law and section 40 of the Real
Property Tax Code.

Pipeline means a line of pipe connected to pumps, valves and control devices for
conveying liquids, gases or finely divided solids. It is a line of pipe running upon or in the
earth, carrying with it the right to the use of the soil in which it is placed. Article 415[l]
and [3] provides that real property may consist of constructions of all kinds adhered to
the soil and everything attached to an immovable in a fixed manner, in such a way that
it cannot be separated therefrom without breaking the material or deterioration of the
object.The pipeline system in question is indubitably a construction adhering to the soil.
It is attached to the land in such a way that it cannot be separated therefrom without
dismantling the steel pipes which were welded to form the pipeline.

11
STAR TWO, INCOPORATION VS. PAPER CITY CORPORATION

G.R. No. 169211


March 6, 2013

Facts:

Respondent Paper City is a domestic corporation engaged in the manufacture of paper


products. Paper City applied for and was granted loans and credit accommodations in
peso and dollar denominations by RCBC secured by 4 Deeds of Continuing Chattel
Mortgages on its machineries and equipments found inside its paper plants.However, a
unilateral Cancellation of Deed of Continuing Chattel Mortgage on Inventory of
Merchandise/Stocks-in-Trade was executed by RCBC over the merchandise and
stocks-in-trade covered by the continuing chattel mortgages.

RCBC, Metrobank and Union Bank (creditor banks with RCBC instituted as the trustee
bank) entered into a Mortgage Trust Indenture (MTI) with Paper City. In the said MTI,
Paper City acquired an additional P170, 000,000.00 from the creditor banks in addition
to the previous loan from RCBC amounting to P110, 000,000.00.The old loan of
P110,000,000.00 was partly secured by various parcels of land situated in Valenzuela
City. The new loan obligation of P170,000,000.00 would be secured by the same five
(5) Deeds of Real Estate Mortgage and additional real and personal properties
described in an annex to MTI, Annex "B" which covered the machineries and
equipments of Paper City.

The MTI was later amended to increase the contributions of the RCBC and Union Bank.
As a consequence, they executed a Deed of Amendment to MTI but still included as
part of the mortgaged properties by way of a first mortgage the various machineries and
equipments located in and bolted to and/or forming part of buildings. A Second
Supplemental Indenture to the MTI was executed to increase the amount of the loan
secured against the existing properties composed of land, building, machineries and
equipments and inventories described in Annexes "A" and "B." Finally, a Third
Supplemental Indenture to the MTI was executed to increase the existing loan
obligation with an additional security composed of a newly constructed two-storey
building and other improvements, machineries and equipments located in the existing
plant site.

Paper City was able to comply with its loan obligations but economic crisis ensued
which made it difficult for Paper City to meet the terms of its obligations leading to
payment defaults. Consequently, RCBC filed a Petition for Extrajudicial Foreclosure.

12
The petition was for the extra-judicial foreclosure of eight parcels of land including all
improvements thereon which were sold in favor of the creditor banks RCBC, Union
Bank and Metrobank as the highest bidders.

This foreclosure sale prompted Paper City to file a Complaint against the creditor banks
alleging that the extra-judicial sale of the properties and plants was null and void due to
lack of prior notice and attendance of gross and evident bad faith on the part of the
creditor banks. Acting on the said motion, the trial court issued an Order denying the
prayer and ruled that the machineries and equipments were included in the annexes
and form part of the MTI. Paper City filed its Motion for Reconsideration which was
favorably granted by the trial court with justification that the disputed machineries and
equipments are chattels by agreement of the parties through their inclusion in the four
Deeds of Chattel Mortgage and the deed of cancellation executed by RCBC was not
valid because it was done unilaterally and without the consent of Paper City.

Issue:
Whether the subject machineries and equipments were included in the mortgage,
extrajudicial foreclosure and in the consequent sale.

Ruling:

Yes. By contracts, all uncontested in this case, machineries and equipments are
included in the mortgage in favor of RCBC, in the foreclosure of the mortgage and in the
consequent sale on foreclosure also in favor of petitioner. The real estate mortgage
over the machineries and equipments is even in full accord with the classification of
such properties by the Civil Code of the Philippines as immovable property. Thus:
Article 415. The following are immovable property:
(1) Land, buildings, roads and constructions of all kinds adhered to the soil;
xxxx
(5) Machinery, receptacles, instruments or implements intended by the owner of
the tenement for an industry or works which may be carried on in a building or on a
piece of land, and which tend directly to meet the needs of the said industry or works.

13
SERG’S PRODUCTS, INC., ET AL. VS. PCI LEASING AND FINANCE, INC.

G.R. No. 137705


August 22, 2000

Facts:

Respondent filed with the RTC for a complaint for sum of money with an application for
a writ of replevin against petitioner. Upon an ex-parte motion, the judge issued the writ,
pursuant to which the sheriff proceeded to the petitioner‗s factory, and seized a
machinery while promising to return for the others. Due to this, petitioners filed for a
special protective order to defer the enforcement of the writ.

Respondents argue that the properties were personal and hence, subject to seizure.
Petitioners, on the other hand, argue that the properties sought to be seized were
immovable, despite the agreement of the parties to the contrary. They held that the
respondents were estopped form treating the machineries as personal property
because the contracts in which the alleged agreement were embodied were farcical.
The CA ruled in favor of the respondent, hence this petition.

Issue:

Whether or not the machineries purchased and imported by petitioner became real
property by virtue of immobilization?

Ruling:

No, they are not. In the present case, the machines that were the subjects of the Writ of
Seizure were placed by petitioners in the factory built on their own land. Indisputably,
they were essential and principal elements oftheir chocolate-making industry. Hence,
although each of them was movable or personal property on its own, all of them have
become immobilized by destination because they are essential and principal elements
in the industry, and hence, the said machines are real, not personal, property pursuant
to Article 415 (5) of the Civil Code.

However, The Court has held that contracting parties may validly stipulate that a real
property be considered as personal. After agreeing to such stipulation, they are
consequently estopped from claiming otherwise. In the present case, the Lease
Agreement clearly provides that the machines in question are to be considered as
personal property, under Section 12.1 of the Agreement.
Hence, the petitioners are estopped from denying the characterization of the subject
machines as personal property. Under the circumstances, they are proper subjects of

14
the Writ of Seizure. It must be noted however that the machines should be deemed
personal property pursuant to the Lease Agreement is good only insofar as the
contracting parties are concerned.

15
RUBY L. TSAI VS. COURT OF APPEALS, ET AL.

G.R. No. 120098


October 2, 2001

Facts:

EVERTEX entered into two loans with petitioner PBCom, the two having been secured
with real and chattel mortgages. After executing said loans, EVERTEX purchased
various machines and equipments. Subsequently, however, due to business reverses,
EVERTEX had to file for insolvency. As such, the court declared EVERTEX insolvent
and took all of its assets, including the collaterals securing the mortgage, into custody.
In the meantime, due to EVERTEX‗s failure to meet its obligations, PBCom commenced
extrajudicial foreclosure proceedings on the mortgages made. By virtue of the auctions
held, PBCom became the owner of the lot and other properties auctioned thereat. After
consolidating its ownership, PBCom leased and consequently sold to petitioner Tsai,
the entire factory premises, lock, stock and barrel, including the contested machineries
therein.

Subsequently, EVERTEX filed for annulment of sale, and reconveyance with damages
before the RTC on certain grounds including that PBCom appropriated the contested
properties which were not included in the mortgages made. The RTC ruled in favor of
EVERTEX, and the same was affirmed by the CA, hence this petition.

Issue:

Whether or not the inclusion of the questioned properties in the foreclosed properties is
proper?

Ruling:

No, it is not. While it is true that the controverted properties appear to be immobile, a
perusal of the contract of Real and Chattel Mortgage executed by the parties gives a
contrary indication. In the case at bar, both the trial and the appellate courts reached the
same finding that the true intention of PBCOM and the owner, EVERTEX, is to treat
machinery and equipment as chattels. In the absence of any showing that the
conclusion is baseless, erroneous or uncorroborated by the evidence on record, there is
no compelling reason to depart therefrom.

Too, assuming arguendo that the properties in question are immovable by nature,
nothing detracts the parties from treating it as chattels to secure an obligation under the

16
principle of estoppel. An immovable may be considered a personal property if there is a
stipulation as when it is used as security in the payment of an obligation where a chattel
mortgage is executed over it, as in the case at bar.

17
MAKATI LEASING, ET AL., VS. WEAREVER TEXTILE MILLS, INCORPORATION.,
ET AL.

G.R. No. L-58469


May 16, 1983

Facts:

In order to obtain financial accommodations from the petitioner, the respondent


discounted and assigned several receivables with the former under a Receivable
Purchase Agreement. The same is secured with a chattel mortgage over certain raw
materials inventory as well as certain machinery. Upon the respondent‗s default,
petitioner filed for the extrajudicial foreclosure of the mortgage. Upon implementation of
the foreclosure, however, the sheriff failed to gain entry into the building, and hence,
was unable to seize the particular machinery. As a result, the petitioner filed for a
judicial foreclosure whereby a writ of seizure was issued. Subsequently, the sheriff was
able to break into the premises of the respondent by virtue of an order to break open the
premises, whereby the said sheriff as able to remove the main motor drive of the
subject machinery.

Issue:

Whether or not the machinery in suit can be the subject of replevin, much less of a
chattel mortgage?

Ruling:

Yes, it may be. Examining the records of the instant case, there is no logical justification
to exclude the rule out, as the appellate court did, the present case from the application
of the pronouncement in the Tumalad case. In said case, if a house of strong materials
may be considered as personal property for purposes of executing a chattel mortgage
thereon as long as the parties to the contract so agree and no innocent third party will
be prejudiced thereby, there is absolutely no reason why a machinery, which is movable
in its nature and becomes immobilized only by destination or purpose, may not be
likewise treated as such. This is really because one who has so agreed is estopped
from denying the existence of the chattel mortgage.

From what has been said above, the nature of the machinery and equipment involved
therein as real properties never having been disputed nor in issue, and not having been
the subject of a chattel mortgage, the Tumalad case bears more nearly perfect parity
with the instant case to be the more controlling jurisprudential authority.

18
LIGHT RAIL TRANSIT AUTHORITY VS. CENTRAL BOARD OF ASSESSMENT
APPEALS, ET AL.

G.R. No. 127316


October 12, 2000

Facts:

The LRTA is a government-owned and controlled corporation created and organized


under Executive Order No. 603 and is primarily responsible for the construction,
operation, maintenance and/or lease of light rail transit system in the Philippines. By
reason of EO No. 603, LRTA acquired real properties constructed structural
improvements, such as buildings, carriageways, passenger terminal stations, and
installed various kinds of machinery and equipment and facilities for the purpose of its
operations.

On 1984, respondent assessed the real properties of petitioner under the Real Property
Tax Code, to commence with the year 1985. The respondent found that the petitioner
paid its real property taxes on all its real property holdings, except the carriageways and
passenger terminal stations including the land where it is constructed on. The petitioner
contends that the same are not real properties under the Real Property Tax Code, and if
the same are real property, these are for public use/purpose, therefore, exempt from
realty taxation. Such claim was denied, reasoning that such pieces of property did not
fall under any of the exemptions listed in Section 40 of the aforementioned law, that
they were not owned by the government or any government-owned corporation which,
as such, was exempt from the payment of real property taxes.

Issue:
Whether petitioner's carriageways and passenger terminal stations are subject to real
property taxes?

Ruling:

Yes, they are. Under the Real Property Tax Code, real property is classified for
assessment purposes on the basis of actual use, which is defined as "the purpose for
which the property is principally or predominantly utilized by the person in possession of
the property."

In this case, unlike public roads which are open for use by everyone, the LRT is
accessible only to those who pay the required fare. It is thus apparent that petitioner
does not exist solely for public service, and that the LRT carriageways and terminal

19
stations are not exclusively for public use. Although petitioner is a public utility, it is
nonetheless profit-earning. It actually uses those carriageways and terminal stations in
its public utility business and earns money therefrom.

Also, there is another legal justification for upholding the assailed CA Decision. Under
the Real Property Tax Code, real property "owned by the Republic of the Philippines or
any of its political subdivisions and any government-owned or controlled corporation so
exempt by its charter, provided, however, that this exemption shall not apply to real
property of the named entities the beneficial use of which has been granted, for
consideration or otherwise, to a taxable person."

20
CALTEX (PHILIPPINES) INCORPORATION. VS. CENTRAL BOARD OF
ASSESSMENT APPEALS, ET AL.

G.R. No. L-50466


May 31, 1982

Facts:

Some machinery equipments installed by CALTEX in its gas stations located on a


leased land were loaned by Caltex to gas station operators under an appropriate lease
agreement or receipt. The city assessor of Pasay City characterized the said items of
gas station equipment and machinery as taxable realty. The city board of tax appeals
ruled that they are personalty. The Board held that the said machines and equipment
are real property within the meaning of sections 3(k) & (m) and 38 of the Real Property
Tax Code, Presidential Decree No. 464.

Issue:
Whether or not the pieces of gas station equipment and machinery already enumerated
are subject to realty tax?

Ruling:

Yes, they are subject to realty tax. The said equipment and machinery, as
appurtenances to the gas station building or shed owned by Caltex and which fixtures
are necessary to the operation of the gas station, for without them the gas station would
be useless, and which have been attached or affixed permanently to the gas station site
or embedded therein, are taxable improvements and machinery within the meaning of
the Assessment Law and the Real Property Tax Code. That ruling is an interpretation of
paragraph 5 of article 415 of the Civil Code regarding machinery that becomes real
property by destination. In the Davao Saw Mills case the question was whether the
machinery mounted on foundations of cement and installed by the lessee on leased
land should be regarded as real property for purposes of execution of a judgment
against the lessee. The sheriff treated the machinery as personal property. This Court
sustained the sheriff's action. The Central Board of Assessment Appeals did not commit
a grave abuse of discretion in upholding the city assessor's is imposition of the realty tax
on Caltex's gas station and equipment.

21
FELS ENERGY, INC. VS. THE PROVINCE OF BATANGAS , ET AL.

G.R. No. 168557


August 22, 2000

Facts:

NPC entered into a lease contract with Polar Energy, Inc. over 3x30 MW diesel engine
power barges moored at Batangas. The contract, denominated as an Energy
Conversion Agreement (Agreement), was for a period of five years. Subsequently, Polar
Energy, Inc. assigned its rights under the Agreement to FELS. The NPC initially
opposed the assignment of rights, citing paragraph 17.2 of Article 17 of the Agreement.
On August 7, 1995, FELS received an assessment of real property taxes on the power
barges from Provincial Assessor Lauro C. Andaya of Batangas City. The assessed tax,
which likewise covered those due for 1994, amounted to P56, 184,088.40 per annum.
FELS referred the matter to NPC, reminding it of its obligation under the Agreement to
pay all real estate taxes. It then gave NPC the full power and authority to represent it in
any conference regarding the real property assessment of the Provincial Assessor. A
reconsideration of the decision to assess real property taxes on the power barges was
aked for, however the same was denied.

Issue:

Whether or not the power barges, which are floating and movable, are personal
properties and therefore, not subject to real property tax.

Ruling:

The power barges are real property and are thus subject to real property tax. In
Consolidated Edison Company of New York, Inc., et al. v. The City of New York, et al.,
,a power company brought an action to review property tax assessment. On the city‗s
motion to dismiss, the Supreme Court of New York held that the barges on which were
mounted gas turbine power plants designated to generate electrical power, the fuel oil
barges which supplied fuel oil to the power plant barges, and the accessory equipment
mounted on the barges were subject to real property taxation.

Moreover, Article 415 (9) of the New Civil Code provides that "docks and structures
which, though floating, are intended by their nature and object to remain at a fixed place
on a river, lake, or coast" are considered immovable property. Thus, power barges are
categorized as immovable property by destination, being in the nature of machinery and

22
other implements intended by the owner for an industry or work which may be carried
on in a building or on a piece of land and which tend directly to meet the needs of said
industry or work.

23
IMMOVABLE
PROPERTY
BY
ANALOGY

24
JIMMY T. GO VS. UNITED COCONUT PLANTERS BANK

G.R. No. 156187


November 11, 2004

Facts:

Petitioner Jimmy T. Go and Alberto T. Looyuko are co-owners of Noahs Ark


International. Sometime in August 1996, petitioners Go and Looyuko applied for an
Omnibus Line accommodation with respondent United Coconut Planters Bank in the
amount of P900,000,000, and was favorably acted upon by the latter. The transaction
was secured by Real Estate Mortgages over parcels of land located in Mandaluyong
City covered by Transfer Certificate of Title No. 64070 and No. 3325, registered in the
name of Mr. Looyuko and Noahs Ark Sugar Refinery, respectively. In 1997, the
approved Omnibus Line accommodation granted to petitioner was subsequently
cancelled by respondent UCPB. However, UCPB refused to return the TCTs covered by
Real Estate Mortgages earlier executed. The UCP thereafter filed for the extrajudicial
foreclosure. On the other hand, Go filed for the cancellation of the real estate mortgage
with prayer for real estate mortgage before the RTC of Pasig City. The trial court issued
an order enjoining the auction sale. The CA reversed the trial court‗s decision and
ordered the dismissal of Go‗s complaint.

Issue:

Whether or not the petition for cancellation of the REM is a real of personal action for
the purpose of determining the venue

Ruling:

In a real action, the plaintiff seeks the recovery of real property, or as provided for in
Section 1, Rule 4, a real action is an action affecting title to or possession of real
property, or interest therein. These include partition or condemnation of, or foreclosure
of mortgage on, real property. The venue for real actions is the same for regional trial
courts and municipal trial courts -- the court which has territorial jurisdiction over the
area where the real property or any part thereof lies. The cancellation of the real estate
mortgage, subject of the instant petition, is a real action, considering that a real estate
mortgage is a real right and a real property by itself. An action for cancellation of real
estate mortgage is necessarily an action affecting the title to the property. It is,
therefore, a real action which should be commenced and tried in Mandaluyong City, the
place where the subject property lies.

25
PERSONAL/
MOVABLE
PROPERTY

26
LUIS MARCOS P. LAUREL VS. HON. ZEUS C. ABROGAR

G.R. No. 155076


January 13, 2009

Facts:

Petitioner Laurel was charged with theft under Article 308 of the Revised Penal Code for
taking the international long distance calls belonging to PLDT by conducting
International Simple Resale which is a method of routing and completing international
long distance calls effectively stealing this business from PLDT while using its facilities
in the estimated amount of P20,370,651.92 to the damage and prejudice of PLDT.
Laurel filed a Motion to Quash on the ground that the facts alleged do not constitute a
felony. The trial court denied the motion, and the CA likewise dismissed petitioner‗s
special civil action for certiorari.

Issue:

Whether or not long distance calls are personal property susceptible of theft

Ruling:

The elements of theft under Article 308 of the Revised Penal Code are as follows: (1)
that there be taking of personal property; (2) that said property belongs to another; (3)
that the taking be done with intent to gain; (4) that the taking be done without the
consent of the owner; and (5) that the taking be accomplished without the use of
violence against or intimidation of persons or force upon things. Any personal property,
tangible or intangible, corporeal or incorporeal, capable of appropriation can be the
object of theft. In this case, while the telephone calls are actually a form of electrical
energy and are therefore personal property, it cannot be said that such international
long distance calls were personal properties belonging to PLDT since the latter could
not have acquired ownership over them. However, the business of providing
telecommunication and the telephone service are personal property under Article 308 of
the Revised Penal Code, and the act of engaging in ISR is an act of subtraction
penalized under said article. Hence, the crime is properly designated as one of theft.

27
BPI FAMILY BANK VS. AMADO FRANCO and COURT OF APPEALS

G.R. No. 123498


November 23, 2007

Facts:

Salazar had in her possession three crossed checks with an aggregate amount of
P267,692.50. These checks were payable to the order of JRT Construction and Trading
which was the name of Templonuevo‗s business. Despite lack of knowledge and
endorsement of Templonuevo, Salazar was able to deposit the checks in her personal
savings account with BPI and encash the same. The three checks were deposited in
three different occasions over the span of eight months. A year after the last
encashment, Templonuevo protested the purportedly unauthorized encashments and
demanded from BPI the aggregate amount of the checks. BPI complied with
Templonuevo‗s demand. Since the money could no longer be debited from the account
of Salazar where she deposited the checks, they froze her other account with them.
Later on, BPI issued a cashier‗s check in favor of Templonuevo for the aggregate
amount and debited P267, 707.70 from Salazar‗s account representing the aggregate
amount and the bank charges for the cashier‗s check. Salazar filed a complaint against
BPI. Trial court ruled in favor of her which was affirmed by CA. Hence, this petition.

Issue:

Whether or not BPI has the authority to unilaterally withdraw from Salazar‗s account the
amount it has previously paid upon certain unendorsed order instrument?

Ruling:

Records show that no prior arrangement existed between Salazar and Templonuevo
regarding the transfer of ownership of the checks. This fact is crucial as Salazar‗s
entitlement to the value of the instruments is based on the assumption that she is a
transferee within the contemplation of Section 49 of the NIL. Section 49 of the NIL
contemplates a situation where the payee or endorsee delivers a negotiable instrument
for value without endorsing it. The underlying premise of this provision, however, is that
a valid transfer of ownership of the negotiable instrument in question has taken place.
Transferees in this situation do not enjoy the presumption of ownership in favor of
holders since they are neither payees nor endorsees of such instruments. Mere
possession of a negotiable instrument does not in itself conclusively establish either the

28
right of the possessor to receive payment, or of the right of one who has made payment
to be discharged from liability. Something more than mere possession is necessary to
authorize payment to such possessor. The one-year delay of Templonuevo in asserting
ownership over the checks is not enough to prove that there has a valid transfer of
ownership has taken place. Salazar failed to discharge the burden of presumption of
ownership in Templonuevo‗s favor as the designated payee. Thus, the return of the
check proceeds to Templonuevo was therefore warranted.

29
CHARACTERISTICS
OF PROPERTY
OF
PUBLIC DOMINION

30
REPUBLIC OF THE PHILIPPINES VS. APOSTOLITA SAN MATEO, BRIGIDA
TAPANG, ROSITA ACCION, AND CELSO MERCADO

G.R. No. 203560


November 10, 2014

Facts:

On January 27, 1999 respondents Apostolita San Mateo, Brigida Tapang, Rosita
Accion, and Celso Mercado, filed before the RTC a Petition for Registration of Title.
GSC contended that the application might have encroached on its properties, because it
owned the adjoining parcels of land. NDCNAI argued that it had a better right of
possession to apply for registration of ownership, because the lot would have been unfit
for human habitation, were it not for the fillings introduced by the association to the
lot.LLDA, on the other hand,claimed that the petition should be denied because the lot
is located below the reglementary lake elevation of 12.50 meters, and, thus, the lot
forms part of the Laguna Lake bed, and is considered inalienable and indisposable
public land, and within the jurisdiction of the LLDA. The RTC granted the petition for
registration. The CA affirmed.

Issue:

Whether or not the petition for registration should be granted

Ruling:

The petition for registration is an action in rem. Moreover, there was proper publication
of the Notice of Initial Hearing, along with the technical description of the property.
Given that this is an action in rem, the publication of the notice is sufficient notice to all
claimants to the property.However, on the issue of whether the respondents were able
to prove that the subject property is alienable and disposable, the SC held that the
respondents failed to prove that the property sought to be registered is indeed alienable
and thus subject to registration. Respondents merely relied on the certification of
DENR-South CENRO to the effect that the subject property is alienable. But this is
insufficient, as respondents failed to present any proof that the DENR Secretary
approved such certification. The certification issued by the Regional Technical Director,
FMS-DENR, in the form ofa memorandum to the trial court, has no probative value.
Clearly, therefore, a CENRO certification that a certain property is alienable, without the
corresponding proof that the DENR Secretary had approved such certification, is
insufficient to support a petition for registration of land. Both certification and approval
are required to be presented as proofs that the land is alienable.

31
REMMAN ENTERPRISES, INC. VS. REPUBLIC OF THE PHILIPPINES

G.R. No. 188494


November 26, 2014

Facts:

The petitioner, through its authorized representative Ronnie P. Inocencio filed with the
RTC on June 4, 1998 an application for registration of the subject properties situated in
Barangay Napindan, Taguig, Metro Manila. The State, through the Office of the Solicitor
General, interposed its opposition to the application.1 Inocencio, the petitioner‗s sales
manager, testified that the subject properties were purchased on by the petitioner from
sellers Magdalena Samonte, Jaime Aldana and Virgilio Navarro. The properties were
declared for taxation purposes on August 9, 1989. After the sale, the petitioner occupied
the properties and planted thereon crops like rice, corn and vegetables. The RTC
granted the application for registration. The CA reversed.

Issue:

Whether or not the petitioner complied with the requirements to warrant the granting of
the application

Ruling:

The petition is dismissible. On the matter of proof of the subject property‗s identity,
jurisprudence provides that the presentation of the original tracing cloth plan may be
dispensed with, subject however to certain conditions. Contrary to the petitioner‗s claim,
the original clothing plans that cover the subject properties do not form part of the case
records. The identity of the land, its boundaries and location can be established by other
competent evidence apart from the original tracing cloth such as a duly executed
blueprint of the survey plan and technical description. Notwithstanding the foregoing,
the CA‗s dismissal of the petitioner‗s application for original registration was proper
considering the latter‗s failure to sufficiently establish that the subject properties were
already declared alienable and disposable by the government. Its reliance on a Report,
issued by the CENRO, DENR National Capital Region, West Sector, was misplaced. It
is not enough for the PENRO or CENRO to certify that a land is alienable and
disposable. The applicant for land registration must prove that the DENR Secretary had
approved the land classification and released the land of the public domain as alienable
and disposable, and that the land subject of the application for registration falls within
the approved area per verification through survey by the PENRO or CENRO. Petition
was denied.

32
REPUBLIC OF THE PHILIPPINES VS. MICHAEL C. SANTOS, ET. AL.

G.R. No. 180027


July 18, 2012

Facts:

In October 1997, the respondents purchased three parcels of unregistered land in


Cavite. Sometime after the said purchase, the respondents caused the survey and
consolidation of the parcels of land, thereafter named Lot 3 with a determined total area
of 9,577 square meters. The respondents filed with the RTC an Application for Original
Registration of Lot 3. The RTC issued an order setting the application for initial hearing,
but also required the DENR to submit a report on the status of Lot 3. The DENR
reported that the land is alienable and disposable land. The RTC granted the petition.
The CA affirmed.

Issue:

Whether or not Lot 3 should be granted in favour of respondents

Ruling:

Under the Regalian Doctrine, all claims of private title to land, save those acquired from
native title, must be traced from some grant, whether express or implied, from the State.
Absent a clear showing that land had been let into private ownership through the State‗s
imprimatur, such land is presumed to belong to the State. Being an unregistered land,
Lot 3 is therefore presumed as land belonging to the State. Original registration of title
to land is allowed by Section 14 of Presidential Decree No. 1529, or otherwise known as
the Property Registration Decree. In this case, the respondents were not able to satisfy
the third requisite under Section 14(1), i.e., that the respondents failed to establish that
they or their predecessors-in-interest, have been in possession and occupation of Lot 3
―since June 12, 1945 or earlier. The evidence presented by the respondents does not
qualify as the ―well-nigh incontrovertible kind that is required to prove title thru
possession and occupation of public land since 12 June 1945 or earlier. Clearly,
respondents are not entitled to registration under Section 14(1) of Presidential Decree
No. 1529. Section 14(2) also does not apply because Lot 3 only became ―Alienable or
Disposable Land on 15 March 1982. The requirement under Section 14(2) allows
acquisition of ownership over private lands by prescription. The Supreme Court held
that the respondents‗ 12 March 2002 application is still premature. Hence, petition was
granted and the RTC decision was reversed.

33
REPUBLIC OF THE PHILIPPINES VS. CORAZON C. SESE and FE C. SESE

G.R. No. 185092


June 4, 2014

Facts:

On September 17, 2002, Corazon C. Sese and Fe C. Sese (respondents) filed with the
MTC an application for original registration of land over a parcel of land with an area of
10, 792 square meters, situated in Barangay Sto. Cristo, Municipality of Pulilan,
Province of Bulacan, and more particularly described as Lot 11247, Cad. 345,
PulilanCadastre, under Plan No. AP-03-004226.

Respondents alleged that on July 22, 1972, they acquired such land through a donation
inter vivos from their mother, Resurreccion L. Castro. Thereafter, respondents
submitted the documents to prove their claim. Then, the MTC granted the application.
And such decision was affirmed by the CA.

Issue:
Whether or not the respondents are entitled to the registration of land.

Ruling:

The Supreme Court ruled in the negative since he burden of proof in overcoming the
presumption of State ownership of the lands of the public domain is on the person
applying for registration, who must prove that the land subject of the application is
alienable or disposable. To overcome this presumption, incontrovertible evidence must
be established that the land subject of the application is alienable or disposable. The
applicant must establish the existence of a positive act of the government such as a
presidential proclamation or an executive order; an administrative action; investigation
reports of Bureau of Lands investigators; or a legislative act or a statute. The applicant
may also secure a certification from the government that the land claimed to have been
possessed for the required number of years is alienable and disposable. Furthermore,
reliance on an annotation to prove that the lot is alienable is insufficient and does not
constitute incontrovertible evidence to overcome the presumption that it remains part of
the inalienable public domain.

Moreover, the Court ruled that under Section 14(2) of P.D. No. 1529, for acquisitive
prescription to commence and operate against the State, the classification of land as
alienable and disposable alone is not sufficient. The applicant must be able to show that
the State, in addition to the said classification, expressly declared through either a law

34
enacted by Congress or a proclamation issued by the President that the subject land is
no longer retained for public service or the development of the national wealth or that
the property has been converted into patrimonial.

35
REPUBLIC OF THE PHILIPPINES VS. EMMANUEL C. CORTEZ

G.R. No. 186639


February 5, 2014

Facts:

On February 28, 2003, respondent Emmanuel C. Cortez (Cortez) filed with the RTC an
application4 for judicial confirmation of title over a parcel of land located at Barangay
(Poblacion) Aguho, P. Herrera Street, Pateros, Metro Manila. The said parcel of land
has an area of 110 square meters and more particularly described as Lot No. 2697-B of
the Pateros Cadastre. In support of his application, Cortez submitted, inter alia, the
following documents: (1) tax declarations; (2) survey plan of the property; (3) technical
description of the property; (4) tax clearance certificate; (5) extrajudicial settlement of
estate; and (6) escritura de particion extrajudicial dated July 19, 1946, allocating the
subject property to FelicisimaCotas – Cortez‗ mother. The RTC granted the application
of the respondent. And such decision was affirmed by the CA.

Issue:

Whether or not the CA erred in affirming the RTC Decision dated February 7, 2006,
which granted the application for registration filed by Cortez.

Ruling:

The Supreme Court ruled in the affirmative since under Section 14(1) of P.D. No. 1529,
applicants for registration of title must sufficiently establish first, that the subject land
forms part of the disposable and alienable lands of the public domain; second, that the
applicant and his predecessors-in-interest have been in open, continuous, exclusive,
and notorious possession and occupation of the same; and third, that it is under a bona
fide claim of ownership since June 12, 1945, or earlier. And the reliance on the
foregoing annotation in the survey plan is amiss; it does not constitute incontrovertible
evidence to overcome the presumption that the subject property remains part of the
inalienable public domain.

36
REPUBLIC OF THE PHILIPPINES VS. FRANCISCA SANTOS, ET AL.

G.R. No. 191516


June 4, 2014

Facts:

Francisca, Geronimo and [Crispin], all surnamed Santos, filed an Application for
Registration of title for four parcels of land described as Lot Nos. 536, 1101, 1214,
1215, all Mcadm 590-D of the Taguig Cadastre, covering areas of 12,221, 4,218, 9,237
and 1,000 square meters, respectively. Lot No. 536, described in SWO-13-000480, is
situated in Barrio Wawa, Taguig,while Lot Nos. 1101, 1214 and 1215, described in
SWO-13-000464, are located in Barrio Sta. Ana, Taguig.

The Application of the respondents was accompanied by the following required


documents: (1) Original or tracing cloth of Survey plan; (2) Technical Description; (3)
Surveyor‗s Certificate; (4) Tax Declaration; and (5) Deed of Extrajudicial Settlement.
Subsequently, the MTC granted the application. And such decision was affirmed by the
CA.

Issues:

Whether or not the subject lots had been declared alienable and disposable lands of the
public domain at the time the application was filed.

Ruling:

The Supreme Court ruled in the negative since anyone who applies for registration of
ownership over a parcel of land has the burden of overcoming the presumption that the
land sought to be registered forms part of the public domain.

In this case, an applicant must prove that the land subject of an application for
registration is alienable and disposable by establishing the existence of a positive act of
the government such as a presidential proclamation or an executive order; an
administrative action; investigation reports of Bureau of Lands investigators; and a
legislative act or a statute. The applicant may also secure a certification from the
government that the land claimed to have been possessed for the required number of
years is alienable and disposable.

37
Hence, the failure of the respondent to establish by sufficient proof that the subject
parcels of land had been classified as part of the alienable and disposable land of the
public domain, his application for registration of title should be denied.

38
REPUBLIC VS. AFP RETIREMENT AND SEPARATION BENEFITS SYSTEM

G.R. No. 180463


January 16, 2013

Facts:

Lots X, Y-1 and Y-2 were lands of the public domain pursuant to Proclamation No. 168
(Proc. 168). In 1983, Proclamation No. 2273 (Proc. 2273) was issued which removed
and segregated Lots Y-1 and Y-2 from the reservation and declaring them open for
disposition to qualified applicants. As a result, only Lot X which consists of 15,020
square meters remained part of the reservation now known as Magsaysay Park

The record discloses that the heirs of CabaloKusop and Atty. Flaviano petitioned the
President to have Lots Y-1 and Y-2 taken out of the reservation for the reason that
through their predecessor CabaloKusop (Kusop), they have acquired vested private
rights over these lots. This campaign resulted in Proc. 2273, which re-classified and
returned Lots Y-1 and Y-2 to their original alienable and disposable state.

In 1997, the heirs and Flaviano filed applications for the issuance of individual
miscellaneous sales patents over the whole of Lot X. Consequently, 16 original
certificates of title (OCTs) covering Lot X were issued in the names of the heirs,
Flaviano and several others. These 16 titles were simultaneously conveyed to
respondent AFP-Retirement and Separation Benefits System (AFP-RSBS).
Petitioner Republic of the Philippines a complaint for reversion, cancellation and
annulment of the AFP-RSBS titles, on the thesis that they were issued over a public
park which is classified as inalienable and non-disposable public land.

The heirs and Flaviano intervened, and, together with the AFP-RSBS, argued that their
predecessor-in-interest Kusop had acquired vested interests over Lot X for having
occupied the same for more than 30 years. The RTC ruled in favor of the Republic. The
CA reversed the RTC and ruled that the lands in dispute are alienable and disposable
lands. Hence this present appeal.

Issue:

Whether or not the CA erred in ruling that the lands in question are alienable and
disposable lands.

39
Ruling:

The Supreme Court ruled in the affirmative. The Court held that from the wording of
Proc. 168, the land it comprises is subject to sale or settlement, and thus alienable and
disposable. However, this alienable and disposable character of the land covered by the
proclamation was subsequently withdrawn, and the land was re-classified by then
President Macapagal to pave the way for the establishment of a park reservation,
subject only to previously acquired private rights. The heirs then lobbied for the
exclusion of certain portions of the reservation which they claimed to be theirs, allegedly
acquired by their predecessor Kusop through prescription. They were successful, for in
1983, then President Marcos issued Proc. 2273, which excluded and segregated Lots
Y-1 and Y-2 from the coverage of Proc. 168. In addition, Proc. 2273 declared Lots Y-1
and Y-2 open for distribution to qualified beneficiaries which included the heirs.
However, Lot X was retained as part of the reservation.

The heirs and Flavianos actions betray their claim of ownership to Lot X. When Proc.
168 was issued, they did not institute action to question its validity, using as cause of
action their claimed ownership and title over the land. The same is true when Proc.
2273 came out. They did not file suit to invalidate it because it contravenes their claimed
ownership over Lot X. They simply sat and waited for the good graces of the
government to fall on their laps. They simply waited for the State to declare them
beneficiaries of the land.

The principle of estoppel bars one from denying the truth of a fact which has, in the
contemplation of law, become settled by the acts and proceedings of judicial or
legislative officers or by the act of the party himself, either by conventional writing or by
representations, express or implied or in pais. Finally, as regards AFP-RSBS rights, the
Court sustains the petitioners view that any title issued covering non-disposable lots
even in the hands of an alleged innocent purchaser for value shall be cancelled.

40
JEAN TAN, ET AL. VS. REPUBLIC

G.R. No. 193443


April 16, 2012

Facts:

On June 14, 2001, the petitioners filed with the Regional Trial Court (RTC) of Naic,
Cavite, an application for land registration covering a parcel of land identified as Lot
9972, Cad-459-D of Indang Cadastre, situated in Barangay Bancod, Indang, Cavite and
with an area of 6,920 square meters. The petitioners alleged that they acquired the
subject property from GregonioGatdula pursuant to a Deed of Absolute Sale dated April
25, 1996; and they and their predecessors-in-interest have been in open, continuous
and exclusive possession of the subject property in the concept of an owner for more
than 30 years. After trial and hearing, the RTC issued a Decision on July 29, 2006,
granting the petitioners application. However, the CA reversed the decision of the RTC.

Issue:

Whether or not the petitioners have proven themselves qualified to the benefits under
the relevant laws on the confirmation of imperfect or incomplete titles.

Ruling:

The Supreme Court ruled that Section 14(1) covers alienable and disposable lands
while Section 14(2) covers private property. Thus, for one‗s possession and occupation
of an alienable and disposable public land to give rise to an imperfect title, the same
should have commenced on June 12, 1945 or earlier. On the other, for one to claim that
his possession and occupation of private property has ripened to imperfect title, the
same should have been for the prescriptive period provided under the Civil Code.
Without need for an extensive extrapolation, the private property contemplated in
Section 14(2) is patrimonial property as defined in Article 421 in relation to Articles 420
and 422 of the Civil Code.

Moreover, the possession and occupation of an alienable and disposable public land for
the periods provided under the Civil Code will not convert it to patrimonial or private
property. There must be an express declaration that the property is no longer intended
for public service or the development of national wealth. In the absence, thereof, the
property remains to be alienable and disposable and may not be acquired by
prescription.

41
However, it is jurisprudentially clear that the thirty (30)-year period of prescription for
purposes of acquiring ownership and registration of public land under Section 14 (2) of
P.D. No. 1529 only begins from the moment the State expressly declares that the public
dominion property is no longer intended for public service or the development of the
national wealth or that the property has been converted into patrimonial.

42
VICENTE YU CHANG AND SOLEDAD YU CHANG VS. REPUBLIC

G.R. No. 171726


February 23, 2011

Facts:

On March 22, 1949, petitioners father, L. Yu Chang and the Municipality of Pili,
Camarines Sur, through its then Mayor, Justo Casuncad, executed an Agreement to
Exchange Real Property wherein the former assigned and transferred to the
Municipality of Pili his 400-square-meter residential lot in Barrio San Roque, Pili,
Camarines Sur, in exchange for a 400-square-meter piece of land located in San Juan,
Pili. Thereafter, L. Yu Chang and his family took possession of the property thus
obtained and erected a residential house and a gasoline station thereon.

On March 1, 1978, a Deed of Transfer and Renunciation of their rights over the property
was executed by L. Yu Chang's five children, Rafaela, Catalina, Flaviana, Esperanza,
and Antonio, in favor of herein petitioners.

On February 21, 1997, petitioner Soledad Yu Chang, for herself and in representation of
her brother and co-petitioner, Vicente Yu Chang, filed a petition for registration of title
over the aforementioned lots under the Property Registration Decree. The RTC granted
the application of the petitioners but the CA reversed such decision.

Issue:

Whether or not the petitioners have proven their open, continuous, exclusive and
notorious possession of the subject lots for the period of time required by law.

Ruling:

The Supreme Court ruled in the Section 48 (b) of the Public Land Act as amended by
PD 1073 provides that the following described citizens of the Philippines, occupying
lands of public domain or claiming to own any such lands or an interest therein, but
whose title have not been perfected or completed, may apply to Regional Trial Court of
the province or city where the land is located for confirmation of their claims and the
issuance of a Certificate of title thereof, under the Property Registration Decree.

Moreover, in order that petitioner‗s application for registration of title may be granted,
they must first establish the following: (1) That the subject land forms part of the
disposable and alienable lands of the Public Domain: and (2) That they have been in

43
open, continuous, exclusive and notorious possession and occupation of the same
under a bona fide claim of ownership, since June 12, 1945.

Furthermore, Jose Amunategui vs Director of Forestry provides that: A forested area


classified as forest land of the public domain does not lose such classification simply
because loggers or settlers may have stripped it of its forest cover.

44
RAMON ARANDA VS. REPUBLIC OF THE PHILIPPINES

G.R. No. 172331


August 24, 2011

Facts:

A petition for land registration was filed by Ramon Aranda over a parcel of land situated
in San Andres, Malvar, Batangas with an area of 9,103 square meters and designated
as Lot 3730, Psc 47, Malvar Cadastre. He prayed that should the Land Registration Act
be not applicable to this case, he invokes the liberal provisions of Section 48 of
Commonwealth Act No. 141, as amended, having been in continuous possession of the
subject land in the concept of owner, publicly, openly and adversely for more than thirty
(30) years prior to the filing of the application.

During the trial, he presented as witnesses his sister and one Lucio Olan. Based upon
their testimonies, the lower court granted the application of Aranda. However, on
appeal, the CA reversed the ruling of the trial court. The CA held that petitioner‗s
evidence does not satisfactorily establish the character and duration of possession
required by law, as petitioner failed to prove specific acts showing the nature of the
possession by his predecessors-in-interest. Hence, this petition.

Issue:

Whether or not the land applied for was alienable and disposable.

Ruling:

No. Petitioner failed to prove the alleged possession of his predecessors-in-interest. His
witness Luis Olan testified that he had been visiting the land along with his father Lucio
since he was 6 years old (he was 70 years old at the time he testified), or as early as
1936. Yet, there was no evidence that Lucio Olan declared the property for tax
purposes at anytime before he sold it to Anatalio Aranda. There is also no showing that
Anatalio Aranda declared the property in his name from the time he bought it from Lucio
Olan. And even assuming that Lucio actually planted rice and corn on the land, such
statement is not sufficient to establish possession in the concept of owner as
contemplated by law. Mere casual cultivation of the land does not amount to exclusive
and notorious possession that would give rise to ownership. Specific acts of dominion
must be clearly shown by the applicant.

45
REPUBLIC OF THE PHILIPPINES VS. BANTIGUE POINT DEVELOPMENT
CORPORATION

G. R. No. 162322
March 14, 2012

Facts:

Respondent Bantigue Point Development Corporation filed with the RTC an application
for original registration of title over a parcel of land. Petitioner Republic filed its
opposition. Thereafter, the RTC Clerk of Court transmitted motu propio the records of
the case to the MTC because the assessed value of the property was allegedly less
than P100,000.00. After trial, the MTC awarded the land to respondent.

On appeal, the CA ruled that since the former had actively participated in the
proceedings before the lower court, but failed to raise the jurisdictional challenge
therein, petitioner is thereby stopped from questioning the jurisdiction of the lower court
on appeal.

Issue:

Whether or not the Republic is stopped from questioning the court‗s jurisdiction.

Ruling:

No. The Court ruled that the Republic is not stopped. The rule is settled that lack of
jurisdiction over the subject matter may be raised at any stage of the proceedings.
Jurisdiction over the subject matter is conferred only by the Constitution or the Law. It
cannot be acquired through a waiver or enlarged by the omission of the parties or
conferred by the acquiescence of the court. Consequently, questions of jurisdiction may
be cognizable even if raised for the first time on appeal.

46
JOYCE Y. LIM VS. REPUBLIC OF THE PHILIPPINES

G.R. No. 158630 & G.R. No. 162047


September 4, 2009

Facts:

Joyce Lim (petitioner) filed on September 7, 1998 before the Regional Trial Court (RTC)
of Tagaytay City an Application for Registration of Title over two parcels of land. He
invokes the provisions of the Property Registration Decree for both applications. No
opposition was filed by the prosecutor. After due hearing, the RTC awarded the two
parcles of land to Lim

On appeal to the CA, the Republic argued that Lim was short of the thirty years
possession required by both the Property Registration Decree and the Civil Code. By
Decisions of November 20, 2002 and April 28, 2003, the appellate court reversed and
set aside the decisions of the RTC and dismissed petitioner‗s applications. Hence, this
petition.

Issue:

Whether or not Lim has established registerable title over the lot applied for.

Ruling:

No. While the subject lots were verified to be alienable or disposable lands since March
15, 1982, there is no sufficient proof that open, continuous and adverse possession
over them by petitioner and her predecessors-in-interest commenced on June 12, 1945
or earlier. Petitioner‗s applications cannot thus be granted.

While a property classified as alienable and disposable public land may be converted
into private property by reason of open, continuous, exclusive and notorious possession
of at least 30 years,41 public dominion lands become patrimonial property not only with
a declaration that these are alienable or disposable but also with an express
government manifestation that the property is already patrimonial or no longer retained
for public use, public service or the development of national wealth.42 And only when
the property has become patrimonial can the prescriptive period for the acquisition of
property of the public dominion begin to run.

47
While the subject lots were declared alienable or disposable on March 15, 1982, there is
no competent evidence that they are no longer intended for public use or for public
service. The classification of the lots as alienable and disposable lands of the public
domain does not change its status as properties of the public dominion. Petitioner
cannot thus acquire title to them by prescription as yet.

48
HEIRS OF MARIO MALABANAN VS. REPUBLIC OF THE PHILIPPINES

G.R. No. 179987


April 29, 2009

Facts:

Mario Malabanan applied for the registration of 71,324 sq. meters of land. He claims
that he bought the land from Eduardo Velazco who also claims that his great
grandfather owned the land. Malabanan submitted a certification from DENR CENRO
stating that the land is alienable and disposable in 1982. The prosecutor did not oppose
the registration. RTC granted Malabanan‗s request for registration.

Republic interposed an appeal claiming that Malabanan did not adhere to the
requirement of time required by the law and the he failed to prove that the land is an
alienable and disposable land. The CA ruled in favor of the republic reasoning that the
possession of the land before it is declared alienable and disposable cannot be included
in the computation of possession of the land, thus Malabanan did not adhere to the
period requirement of the law.

Issue:

Can the heirs of Malabanan register the land?

Ruling:

No. Artcle 422 is controlling in the conversion of the land of public dominion to
patrimonial property. It is only when a land becomes patrimonial that it becomes
susceptible to prescription. There must be an express declaration by the State that an
alienable and disposable land is no longer intended for public service. It is only after
such express declaration that the period may begin to run. Section 14(2) of the
Property Registration Decree applies for the case at bar. Possesion of the land is traced
back to 1948. Since the land in question has no express declaration of being
patrimonial, Malabanan failed to adhere to the period as required by law.

49
THE SECRETARY OF THE DEPARTMENT OF ENVIRONMENT AND NATURAL
RESOURCES et.al VS. MAYOR JOSE S. YAP

G.R. No. 167707


October 8, 2008

Facts:

On November 10, 1978, then President Marcos issued Proc. No. 1801declaring
Boracay Island, among other islands, caves and peninsulas in the Philippines, as tourist
zones and marine reserves under the administration of the Philippine Tourism Authority
(PTA). President Marcos later approved the issuance of PTA Circular 3-82 dated
September 3, 1982, to implement Proclamation No. 1801.

Claiming that Proclamation No. 1801 and PTA Circular No 3-82 precluded them from
filing an application for judicial confirmation of imperfect title or survey of land for titling
purposes, respondents-claimants Mayor . Yap, Jr., and others filed a petition for
declaratory relief with the RTC in Kalibo, Aklan

In their petition, respondents-claimants alleged that Proc. No. 1801 and PTA Circular
No. 3-82 raised doubts on their right to secure titles over their occupied lands. They
declared that they themselves, or through their predecessors-in-interest, had been in
open, continuous, exclusive, and notorious possession and occupation in Boracay since
June 12, 1945, or earlier since time immemorial. They declared their lands for tax
purposes and paid realty taxes on them. Respondents-claimants posited that
Proclamation No. 1801 and its implementing Circular did not place Boracay beyond the
commerce of man. Since the Island was classified as a tourist zone, it was susceptible
of private ownership. Under Section 48(b) of the Public Land Act, they had the right to
have the lots registered in their names through judicial confirmation of imperfect titles.

The Republic, through the OSG, opposed the petition for declaratory relief. The OSG
countered that Boracay Island was an unclassified land of the public domain. It formed
part of the mass of lands classified as ―public forest,‖ which was not available for
disposition pursuant to Section 3(a) of the Revised Forestry Code, as amended. The
OSG maintained that respondents-claimants‗ reliance on PD No. 1801 and PTA Circular
No. 3-82 was misplaced. Their right to judicial confirmation of title was governed by
Public Land Act and Revised Forestry Code, as amended. Since Boracay Island had
not been classified as alienable and disposable, whatever possession they had cannot
ripen into ownership.

50
On July 14, 1999, the RTC rendered a decision in favor of respondents-claimants,
declaring that, ―PD 1810 and PTA Circular No. 3-82 Revised Forestry Code, as
amended. The OSG moved for reconsideration but its motion was denied. The Republic
then appealed to the CA. On In 2004, the appellate court affirmed in toto the RTC
decision. Again, the OSG sought reconsideration but it was similarly denied. Hence, the
present petition under Rule 45.

Issue:
Whether or not private claimants have a right to secure titles over their occupied
portions in Boracay.

Ruling:

Petition DENIED. The CA decision is reversed. PD No. 705 issued by President Marcos
categorized all unclassified lands of the public domain as public forest. Section 3(a) of
PD No. 705 defines a public forest as ―a mass of lands of the public domain which has
not been the subject of the present system of classification for the determination of
which lands are needed for forest purpose and which are not.‖ Applying PD No. 705, all
unclassified lands, including those in Boracay Island, are ipso factoconsidered public
forests. PD No. 705, however, respects titles already existing prior to its effectivity.

The 1935 Constitution classified lands of the public domain into agricultural, forest or
timber, such classification modified by the 1973 Constitution. The 1987 Constitution
reverted to the 1935 Constitution classification with one addition: national parks. Of
these, only agricultural lands may be alienated.Prior to Proclamation No. 1064 of May
22, 2006, Boracay Island had never been expressly and administratively classified
under any of these grand divisions. Boracay was an unclassified land of the public
domain.

A positive act declaring land as alienable and disposable is required. In keeping with the
presumption of State ownership, the Court has time and again emphasized that there
must be a positive act of the government, such as a presidential proclamation or an
executive order; an administrative action; investigation reports of Bureau of Lands
investigators; and a legislative act or a statute. The applicant may also secure a
certification from the government that the land claimed to have been possessed for the
required number of years is alienable and disposable. The burden of proof in
overcoming such presumption is on the person applying for registration (or claiming
ownership), who must prove that the land subject of the application is alienable or
disposable.

51
In the case at bar, no such proclamation, executive order, administrative action, report,
statute, or certification was presented to the Court. The records are bereft of evidence
showing that, prior to 2006, the portions of Boracay occupied by private claimants were
subject of a government proclamation that the land is alienable and disposable. Matters
of land classification or reclassification cannot be assumed. They call for proof. Proc.
No. 1801 cannot be deemed the positive act needed to classify Boracay Island as
alienable and disposable land. If President Marcos intended to classify the island as
alienable and disposable or forest, or both, he would have identified the specific limits of
each, as President Arroyo did in Proclamation No. 1064. This was not done in
Proclamation No. 1801.

52
HEIRS OF THE LATE SPOUSES PEDRO S. PALANCA AND SOTERRANEA
RAFOLS VDA. DE PALANCA VS. REPUBLIC OF THE PHILIPPINES, et. al.

G.R. No. 151312


August 30, 2006

Facts:

On July 19, 1973, the heirs of Pedro S. Palanca, (petitioners herein), filed an application
to bring the pieces of land they allegedly owned under the operation of the Land
Registration Act. They acquired said realties by inheritance from the late Pedro S.
Palanca, who had occupied and possessed said land openly and continuously in the
concept of an owner since 1934, or 39 years before the filing of said application, and
planted on said lands about 1,200 coconut trees on each land, declared the same for
taxation purposes and paid the taxes thereof. The first parcel of land is presently
occupied by Lopez, Libarra, an encargado of herein while the second is occupied by
Candelaria Punzalan.

During the initial hearing of the case, verbal oppositions to the application were made by
the Provincial Fiscal of Palawan purportedly for and in behalf of the Bureau of Forest
Development, the Bureau of Lands, and the Department of Agrarian Reform, some
inhabitants of the subject properties and a businessman by the name of Alfonso
Guillamac. After the lapse of three years from the date of the initial hearing, however, no
valid and formal opposition was filed by any of the oppositors in the form and manner
required by law.

Issue:

Whether or not the subject properties of Palanca are private properties.

Ruling:

No. The court ruled that the Executive Proclamation No. 219 classified these lands as a
national reserve and it appears that the said property was never released for public
disposition. At the time of the petitioner‗s predecessor‗s occupancy, the land remained
inalienable. For a public dominion to be subject for a registration proceeding, the
government should exercise a positive act to re-classify inalienable land to alienable for
proper disposition. In an application for a land registration of a property of public
domain, the applicant for land registration must secure a certification from the
government that the land claimed (1) has been possessed in the concept of an owner
for more than 30 years and (2) alienable and disposable. Moreover, action to recover
property of public domain never prescribes.

53
LUNINGNING P. DEL ROSARIO-IGBITEN, JOSE REYES IGBITEN, JOSE DEL
ROSARIO-IGBITEN, JR. AND THERESA TOPACIO MEDINA VS. REPUBLIC OF THE
PHILIPPINES

G.R. No. 158449


October 22, 2004

Facts:

Petitioners filed with the trial court an application for registration of land under
Presidential Decree (PD) No. 1529, covering a parcel of land. Petitioners alleged that
they acquired the Subject Property purchase, and that they, by themselves through their
predecessors-in-interest, had been in actual, continuous, uninterrupted, open, public,
and adverse possession of the Subject Property in the concept of owner for more than
30 years. The Tonido family sold the Subject Property to petitioners, as evidence by a
Deed of Absolute Sale.

The history possession of the Subject Property, was supported by tax declarations in
the name of petitioners and their predecessors-in-interest from 1958-1998. The trial
court rendered a decision approving petitioners‗ application for registration of the
Subject Property. The Republic of the Philippines, represented by the Office of the
Solicitor General, appealed the decision of the trial court to the Court of Appeals.

In its appeal, the Republic alleged that the trial court erred in approving the application
for registration despite petitioners‗ failure to prove open, continuous, exclusive and
notorious possession and occupation of the Subject Property since 12 June 1945, or
earlier, as required by Section 48(b) of Commonwealth Act. No. 141, otherwise known
as the Public Land Act, as amended by PD No. 1073. Moreover, petitioners also failed
to produce monuments of title to tack their possession to those of their predecessors-in-
interest in compliance with the prescriptive period required by law.

On December 2002, the Court of Appeals rendered a decision finding the appeal
meritorious, setting aside the decision of the trial court, and dismissing the application
for registration of petitioners. The Court of Appeals denied petitioners Motion for
Reconsideration in its resolution dated 22 May 2003. Petitioners filed this petition for
review on certiorari.

Issue:

Whether petitioners have complied with the period of possession and occupation
required by the Public Land Act.

54
Ruling:

(1) Section 44 of the Public Land Act, as amended by RA No. 6940, which provides for
a prescriptive period of thirty (30) years possession, applies only to applications for free
patents; (2) The case a bar is a judicial application for confirmation of an imperfect or
incomplete title over the Subject Property covered by Section 48(b) of the Public Land
Act; and (3) Section 48(b) of the Pubic Land Act requires for judicial confirmation of an
imperfect or incomplete title the continuous possession of the land since 12 June 1945,
or earlier, which petitioners herein failed to comply with.

55
REPUBLIC OF THE PHILIPPINES VS. CELESTINA NAGUIAT

479 SCRA 585


January 24, 2006

Facts:

This is an application for registration of title to four (4) parcels of land located in Panan,
Botalan, Zambales, filed by Celestina Naguiat on December 1989 with the Regional
Trial Court of Zambales, Branch 69. Applicant alleges, interalia, that she is the owner of
the said parcels of land having them by purchase from the LID Corporation which
likewise acquired the same from Demetria Calderon, Josefina Moraga and Fausto
Monje and their predecessors –in-interest who have been possession thereof for more
than thirty (30) years; and that to the best of her knowledge, said lots suffer no
mortgage or encumbrance of whatever kind nor is there any person having any interest,
legal or equitable, or in possession thereof.

On 29 June 1990, the Republic of the Philippines [herein petitioner] filed an opposition
to the application on the ground that neither the applicant nor her predecessors-in-
interest have been in open, continuous, exclusive and notorious possession and
occupation of the lands I question since 12 June 1945 or prior thereto; that the
monuments of title and tax payment receipts of application do not constitute competent
and sufficient evidence of bona-fide acquisition of the lands applied for or his open,
continuous, exclusive and notorious possession and occupation thereof in the concept
of (an) owner; that the applicant‗s claim of ownership in fee simple on the basis Spanish
title or grant can no longer be availed of; and that the parcels of land applied for are part
of the public domain belonging to the Republic of the Philippines not subject to private
appropriation.The trial court rendered judgment herein respondent Celestina Naguiat,
adjudicating unto her the parcels of land in question and decreeing the registration
thereof in her name,

With its motion for reconsideration having been denied by the trial court, petitioner
Republic went on appeal to the CA in CA-G.R CV No. 37001. As stated at the outset
hereof, the CA, in the herein assailed decision of May 29, 1998, affirmed that of the trial
court. Hence, the Republic‗s present recourse on its basic submission that the CA‗s
decision ―is not an accordance with law, jurisprudence and the evidence, since
respondent has not established with the required evidence her title in fee simple or
imperfect title in respect of the subject lots which would warrant their registration under
(P.D. 1529 or Public Land Act (C.A) 141. In particular, petitioner Republic faults the
appellate court on its finding respecting the length of respondent‗s occupation of the
property subject of her application for registration and for not considering the fact that

56
she has not established that the lands in question have been declassified from forest or
timber zone to alienable and disposable property.

Issue:

Whether or not the areas in question have ceased to have the status of forest or other
inalienable lands of the public domain.

Ruling:

Public lands not shown to have been reclassified or released as alienable agricultural
land or alienated to a private person by the State remain part of the inalienable public
domain. Under Section 6 of the Public Land Act, the prerogative of classifying or
reclassifying lands of the public domain, i.e., from forest or mineral to agricultural and
vice versa, belongs to the Executive Branch of the government and not the court.
Needless to stress, the onus to overturn, by incontrovertible evidence, the presumption
that the land subject of an application for registration as alienable or disposable rests
with the applicant. Respondent never presented the required certification from the
proper government agency or official proclamation reclassifying the land applied for as
alienable and disposable. Declassification of forest and mineral lands, as the case may
be, and their conversion into alienable and disposable lands need an express and
positive act from government.

The foregoing considered, the issue of whether or not respondent and her predecessor-
in-interest have been in open, exclusive and continuous possession of the parcels of
land in question is now of little moment. For, unclassified land, as here, cannot be
acquired by adverse occupation or possession; occupation thereof in the concept of
owner, however long, cannot ripen into private ownership and be registered as title.

57
THE DIRECTOR OF FORESTRY VS. RUPERTO A. VILLAREAL

170 SCRA 598


February 27, 1989

Facts:

Ruperto Villareal applied for its registration on 25 January 1949, a land consisting of
178,113 sq. m. of mangrove swamps located in the municipality of Sapian, Capiz,
alleging that he and his predecessors-in-interest had been in possession of the land for
more than 40 years. He was opposed by several persons, including the Directors of
Forestry on behalf of the Republic of the Philippines. After trial, the application was
approved by the Court of First Instance Capiz. The decision was affirmed by the Court
of Appeals. The Director of Forestry then came to the Supreme Court in a petition for
review on certiorari.

Issue:

Whether or not the land in dispute was forestall in nature and not subject to private
appropriation.

Ruling:

The Supreme Court set aside the decision of the Court of Appeals and dismissed the
application for registration of title of Villareal. Administrative Code of 1917; Mangrove
swamps form part of the public forests of the country. Subsequently, the Philippine
Legislature categorically declared that mangrove swamps form part of the public forest
of this country. This it did in the Administrative Code of 1917, which became effective on
1 October 1917, providing in Section 1820 of said code that for the purpose of this
chapter ‗public forest‗ includes, except otherwise specially indicated, all unreserved
public land, including nipa and mangrove swamps, and all forest reserves of whatever
character. The legislative definition embodied in Section 1820 of the Revised
Administrative Code of 1917, remains unamend up to now, provides the mangrove
swamps or manglares form part of the public forests of the Philippines. As such, they
are not alienable under the Constitution and may not be the subject of private ownership
until and unless they are first released as forest land and classified as alienable
agricultural land.

58
EDUBIGIS GORDULA, CELSO VS. FERNANDEZ, JR., ET. AL.

284 SCRA 617


January 22, 1998

Facts:

Former President Ferdinand E. Marcos issued Proclamation No. 573[3] withdrawing


from sale and settlement and setting aside permanent forest reserves, subject to private
rights, certain parcels of the public domain which include Parcel No. 9 – Caliraya-Lumot
River Forest Reserve.

They were primarily for use as watershed area.The parcel of land subject of the case at
bar is, by petitioner‗ explicit admission, within Parcel No. 9, the Caliraya-Lumot River
Forest Reserve.Petitioner Edubigis Gordula filed with the Bureau of lands, an
Application for free Patent over the land. Manuel Fernandez and several others also
filed free patent application covering other parcels of land in the area.

Mr. Antonio Aquino, Jr., the Civil Security Officer of the Cavinti reservoir complex, sent a
Memorandum to the President of the Napocor informing him of the fences and roads
being constructed in the saddle area, more particularly, in the lots sold by petitioner
Fernandez to petitioner Estrellado.

Respondent Republic, through the Napocor, filed against petitioners a Complaint for
Annulment of free Patent and Cancellation of Titles and Reversion with Writ of
Preliminary Injunction in the RTC of Sta. Cruz, Laguna. The trial court rendered
judgment in favour of petitioners. Respondent Republic, through the Napocor, elevated
the case to the respondent Court of Appeals. On June 20, 1996, the respondent Court
of Appeals ruled against petitioners. Hence, this petition.

Issue:

Whether or not the subject parcel of lands are non-disposable and inalienable public
land.

Ruling:

The two (2) parcels of land were public disposable and alienable lands before the
issuance, by the former President, of Proclamation No. 573, on June 26, 1969. The
property was, however, later reserved, under Proclamation No. 573, as a permanent
forest, on June 26 1969. Since then, the property became non-disposable and alienable

59
public land. By their very nature or by executive or statutory fiat, they are outside the
commerce of man, unsusceptible of private appropriation in any form. And inconvertible
into any character less than of inalienable public domain, regardless of their actual
state, for as long as reservation subsists and is not revoked by a subsequent valid
declassified. Petitioners do not contest the nature of the land in the case at bar. It is
admitted that it lies in the heart of the Caliraya-Lumot River Forest Reserve, which
Proclamation No. 573 classified as alienable and in disposable.

No public land can be acquired by private persons without any grant, express or implied
from the government; it is indispensable that there be a showing of a title from the state.
The facts show that petitioner Gordula, did not acquire title to the subject land prior to its
reservation under proclamation No. 573. He filed this application for free patent only in
January, 1973, more than three (3) years after the issuance of Proclamation No. 573 in
June, 1969. At that time, the land, as part of the Caliraya-Lumot River Forest Reserve,
was no longer open to private ownership as it has been classified as public forest
reserve for the public good.

60
SPOUSES GEMINIANO AND AMPARO DE OCAMPO AND SPOUSES PEDRO AND
CRISANTA SANTOS VS. FEDERICO ARLOS, MARY ARLOS, TEOFILO OJERIO
AND ABELLA OJERIO

343 SCRA 716


October 19, 2000

Facts:

Federico S. Arlos and Teofilo D. Ojerio filed an application for registration, docketed as
Land Registration Case No. N-340, wherein they seek judicial confirmation of their titles
to three parcels of land, all located at Cabcaben, Mariveles, Bataan.

Spouses Geminiano De Ocampo and Amparo De Ocampo and spouses Pedro Santos
and Crisanta Santos opposed the application for registration, alleging that they are the
co-owners of Lots 1 and 2 of Plan SGS 3062, situated at Cabcaben, Mariveles, Bataan
and their ownership is evidenced by Transfer Certificate of Title Nos. T-43298 and T-
44205, and those they became owners of said lots by purchase from the government
through sales patents.

The Republic of the Philippines also opposed the application, contending that neither
the applicants not their predecessors-in-interests have been in open, continuous,
exclusive and notorious possession and have occupation of the lands in question for at
least 30 years immediately preceding the filing of the application; and that the parcels of
land applied for are portions of the public domain belonging to the Republic of the
Philippines not subject to private appropriation. The CA ruled that the petitioners had
failed to comply with the Public Land At, which required sales patent applicants to be
the actual occupants and cultivators of the land. It held that the testimonies of the
petitioners, which were ―incongruous with reality. Bolstered the ―finding that [they
had] never occupied cultivated or made improvements of the property.

Issue:

Whether or not the registration of respondents‗ title under the Public Land Act is proper.

Ruling:

Respondents application for registration of the title to the three parcels of land that were
once part of the public domain is governed by the Public Land Act, a title may be
judicially confirmed under Section 48 of the Public Land Act only if it pertains to
alienable lands of the public domain. Unless such assets are reclassified and

61
considered disposable and alienable, occupation thereof the concept of owner, no
matter how long cannot ripen into ownership and be registered as a title. Verily,
Presidential Decree No. 1073 clarified Section 48 (b) of the Public Land Act by
specifically declaring that the latter applied only to alienable and disposable lands of the
public domain.

In the present case, the disputed land which was formerly a part of a US military
reservation that had been turned to the Philippine government in 1965 was declared
disposable and alienable only in 1971. Second, respondents and their predecessors-in-
interest could not have occupied the subject properly form 1947 until 1971 when the
land was declared alienable and disposable, because it was a military reservation at the
time. Hence, it was not subject to occupation, entry or settlement. We reiterate that the
land was declared alienable only in 1971; hence, respondents have not satisfied the
thirty-year requirement under the Public Land Act. Moreover, they could not have
occupied the property for thirty years, because it formed part of a military reservation.
Clearly then, their application for the registration of their title was erroneously granted by
the appellate and the trial courts.

62
ZARATE VS. THE DIRECTOR OF LANDS

434 SCRA 322


July 14, 2004

Facts:

This is a proceeding to register the title to lands described in the petition. The
government of the Philippine Islands interposed an objection to the registration of title
on the ground ―that said parcel of land was part of the public domain and is occupied
by Apolonio Gamido and Bibiana Olivite by virtue of applications made by them for
homesteads Nos. 2061 and 5626, respectively. Registration of title to that portion of the
land found to be occupied by the persons and highway named was denied; and from
that judgment the applicant appealed.

The lands in question belong to the applicant who has shown by a strong
preponderance of the evidence that he is the owner thereof. The land I question not
being public and, the Government of the Philippine Islands had no authority to declare it
open homesteads; and as a necessary consequence, whatever concessions the
Government has made with respect to such land are without force and effect, except as
to the homesteads of Apolonio Gamido who, prior to the commencement of this
proceeding, appears to have received his homestead patent from the Government.
Under Act No. 926 a patent issued under the Homestead Law has all the force and
effect to a Torrens title acquired under Act No. 496; and that being the case, and no
question having been raised here or in the court below as to the validity of that Act in
connection with the proceedings for homesteads mentioned in this case, we must
respect the title so secured, provided it be a fact that a patent has been secured in any
of said homestead proceedings.

It clearly appears that the applicant and hid predecessors in interest were the owners of
and had a good title thereto. In our judgment the evidence falls far short of showing
abandonment, the record discloses no acts of the owners on which abandonment can
be case. Nor is there any claim of title by adverse possession.

Issue:

Whether or not the applicant has the right to register the title described in the
application.

63
Ruling:

It is declared that the applicants has the right to register title to all of the land described
in the application, with the exception of that portion claimed as a homestead by
Apolonio Gamido, which homestead shall be excluded from registration by the applicant
provided the Court of Land Registration shall find that said Apolonio Gamido has
obtained a patent for said land; but if the Court of Land registration finds that said
Gamido has not yet obtained a patent thereof, then the court shall register title in favour
of the applicant to all lands described in the application.

64
REPUBLIC OF THE PHILIPPINES VS. COURT OF APPEALS, HON. JOSE D.
AZARRAGA AND ANGEL T. YU

G.R. No. 126316


June 25, 2004

Facts:

On June 22, 1994, respondent Angel T. Yu filed a petition for registration of a parcel of
land. The petition was later amended to include the adjoining lots and the corresponding
owners name. Initial hearing was scheduled on February 9, 1995. On February 22,
1995, the RTC received a letter from the Land Registration Authority (LRA) requesting
the court to require the Land Management Bureau, Manila and the Community
Environment and Natural Resources Office (CENRO) at Barotac Viejo, Iloilo to report on
the status of the subject land considering that a discrepancy was noted after plotting the
land. On March 6, 1995, the RTC issued an Order to the effect. Based on this
certification and after reception of evidence, the RTC rendered judgment confirming the
title of the applicant/petitioner ANGEL T. YU, Filipino, of legal age, married and a
resident of Estancia, Iloilo, over a parcel of land. No motion for reconsideration was filed
by the City Prosecutor on behalf of the Solicitor General. Hence, the said decision
became final and executor. An order was consequently issued by the RTC directing the
issuance of the corresponding decree of registration and certificate of title to respondent
Angel T. Yu.

On June 22, 1995, the OSG received a letter from Regional Executive Director Jose P.
Catus of the DENR, stating that an investigation was conducted on the instant case,
and it was found that there were grounds for opposition to the respondents land
application. Land Investigator Fabio O. Catalan, Jr., who conducted an ocular inspection
of the subject land, found the same to be a reclaimed foreshore area.After discovering
the actual status of Lot 524, the Republic filed a petition for the annulment of judgment
with a prayer for a writ of preliminary injunction with the Court of Appeals on July 20,
1995.On February 5, 1996, respondent Angel T. Yu filed a motion with the CA, praying
that he be allowed to submit to the Land Registration Authority the corrected technical
description and the republication in the Official Gazette of the corrected technical
description of the plan. The OSG filed its objection thereto.

65
Issue:

Whether or not he Court of Appeals erred in denying the Republic‗s Petition for
Annulment of Judgment on the mere supposition that the lot is not foreshore land, but
agricultural land.

Ruling:

The Catalan Report, which states that the subject land is foreshore land, was received
by the OSG only on June 22, 1995, long after the RTC rendered its judgment on May 3,
1995. Angel T. Yu had, in fact, filed a foreshore lease application in 1977 and paid the
corresponding fees thereon. There is, therefore, doubt to the respondents claim that he
had been in actual, open, notorious, continuous possession, in the concept of an owner.
Moreover, the Rosal Report datedApril 12, 1995 was received by the OSG only on May
29, 1995. Although the report states that Lot No. 524, Cad-633-D is declared public land
and is alienable and disposable per L.C. Map 1020, Project 44 dated July 26, 1933, the
same report buttresses the contention that the subject land is foreshore land and
covered by a foreshore lease application filed by Angel T. Yu. Finding the reports to be
revealing and significant as to the real status of the land being foreshore, the petitioner
lost no time in filing the petition for annulment of judgment with the Court of Appeals.
The court cannot uphold the respondent courts finding regarding the character of the
land. The Rosal Report clearly states that the subject land is not an agricultural land.
Despite such declaration, the respondent court continued to rule that the subject land is
agricultural on the basis that out of the total area of 1,194 square meters, 850 square
meters is dry land and that 334 square meters is now a reclaimed area.

66
REPUBLIC OF THE PHILIPPINES VS. CANDY MAKER, INC., AS REPRESENTED
BY ITS PRESIDENT, ONG YEE SEE

G.R. No. 163766


June 22, 2006

Facts:

Sometime in 1998, Candy Maker, Inc. decided to purchase Lot No. 3138 Cad. 688 of
the Cainta-TaytayCadastre, a parcel of land located below the reglementary lake
elevation of 12.50 meters, about 900 meters away from the Laguna de Bay, and
bounded on the southwest by the Manggahan Floodway, and on the southeast by a
legal easement. On April 1, 1998, Geodetic Engineer Potenciano H. Fernandez,
prepared and signed a Subdivision Plan of the property for Apolonio Cruz. On April 29,
1999, Antonio, Eladia, and Felisa, all surnamed Cruz, executed a Deed of Absolute
Sale in favor of Candy Maker, Inc.On June 16, 1999, Candy Maker, Inc., as applicant,
filed an application with the MTC of Taytay, Rizal, for the registration of its alleged title
over Lot No. 3138-A and Lot No. 3138-B under Presidential Decree (P.D.) No. 1529.The
Community Environment and Natural Resources Officer (CENRO) of Antipolo City filed
on August 18, 1999 his Report declaring that the land falls within the Alienable and
Disposable Zone, under Land Classification Project No. 5-A, per L.C. Map No. 639
certified released on March 11, 1927 and that the property is the subject of CENRO
Case No. 520(97) entitled Perpetua San Jose v. Almario Cruz. On the other hand, the
LRA, in its September 21, 1999 Report,recommended the exclusion of Lot No. 3138-B
on the ground that it is a legal easement and intended for public use, hence, inalienable
and indisposable.

On September 30, 1999, the Laguna Lake Development Authority (LLDA) approved
Resolution No. 113, Series of 1993, providing that untitled shoreland areas may be
leased subject to conditions enumerated therein. The applicant filed its Amended
Application

On July 20, 2001, the Republic of the Philippines, the LLDA filed its Opposition17 to the
Amended Application in which it alleged that the lot subject of the application for
registration may not be alienated and disposed since it is considered part of the Laguna
Lake bed, a public land within its jurisdiction pursuant to Republic Act (R.A.) No. 4850,
as amended. According to the LLDA, the projection of Lot No. 3138-A, Cad-688-D Csd-
04-018302 in its topographic map based on the Memorandum18 of Engineer
Christopher Pedrezuela of the Engineering and Construction Division of the LLDA
indicated that it is "located below the reglementary lake elevation of 12.50 meters
referred to datum 10.00 meters below mean lower water" and under Section 41(11) of

67
R.A. No. 4850, the property is a public land which forms part of the bed of the Laguna
Lake. This Memorandum was appended to the application.On October 12, 2001, the
MTC rendered a Decision granting the application for registration over the lots. On
appeal to the CA, it dismissed the appeal and affirmed in toto the Decision of the MTC
and upheld the applicant‗s claim that the parcels of land were alienable and not part of
the public domain, and that it had adduced preponderant evidence to prove that its
predecessors had been tilling the land since 1937, during which palay and vegetables
were planted.

Issue:

Whether or not the property subject of the amended application is alienable and
disposable property of the State

Ruling:

The property subject of this application was alienable and disposable public agricultural
land until July 18, 1966. However, respondent failed to prove that it. To prove that the
land subject of an application for registration is alienable, an applicant must conclusively
establish the existence of a positive act of the government such as a presidential
proclamation or an executive order, or administrative action, investigation reports of the
Bureau of Lands investigator or a legislative act or statute. As gleaned from the Survey
Report of Magalonga, Polanco and Medenilla of the LLDA based on the ocular
inspection dated September 14, 2001 as well as the Memorandum of Engineer
Christopher Pedrezuela, the property is located below the reglementary level of 12.50
m.; hence, part of the bed of the Laguna de Bay, and, as such, is public land. Although
the Report and Memorandum were not offered as evidence in the MTC, the respondent
admitted in its Manifestation in this Court that the property is situated below the 12.50
elevation based on the survey of Magalonga, Polanco and Medenilla, the same survey
team who conducted an ocular inspection of the property on April 12, 2005, which thus
confirmed the September 14, 2001 survey report. This is a judicial admission in the
course of judicial proceedings which is binding on it. Under R.A. No. 4850 and the
issuances of LLDA, registerable rights acquired by occupants before the effectivity of
the law are recognized. However, the respondent failed to adduce proof that its
predecessors-in-interest had acquired registerable title over the property before July 18,
1966.

68
NIMFA USERO VS. COURT OF APPEALS AND SPS. HERMINIGILDO & CECILIA
POLINAR

G.R. No. 152115


January 26, 2005

Facts:

Petitioners Lutgarda R. Samela and NimfaUsero are the owners respectively of lots 1
and 2, Block 5, Golden Acres Subdivision, Barrio Almanza, LasPias City.Private
respondent spouses Polinar are the registered owners of a parcel of land behind the
lots of petitioners Samela and Usero.Situated between the lots of the parties is a low-
level strip of land, with a stagnant body of water filled with floating water lilies.
Apparently, every time a storm or heavy rains occur, the water in said strip of land rises
and the strong current passing through it causes considerable damage to the house of
respondent Polinars. Private respondent spouses, on July 30, 1998, erected a concrete
wall on the bank of the low-level strip of land about three meters from their house and
rip-rapped the soil on that portion of the strip of land.Claiming ownership of the subject
strip of land, petitioners Samela and Usero demanded that the spouses Apolinar stop
their construction but the spouses paid no heed, believing the strip to be part of a creek.
Nevertheless, for the sake of peace, the Polinars offered to pay for the land being
claimed by petitioners Samela and Usero. However, the parties failed to settle their
differences. On November 9, 1998, petitioners filed separate complaints for forcible
entry against the Polinars at the Metropolitan Trial Court of Las Pias City.

The trial court rendered a decision in favor of Samela and Usero. The Polinar spouses
appealed the decisions of the two Municipal Trial Courts to the Regional Trial Court.
The Regional Trial Court, reversed the decision of the trial court and ordered the
dismissal of the complaint. It confirmed the existence of the creek between the
northwestern portion of the lot of petitioner Samela and the southwestern portion of the
lot of the spouses Polinar.

Issue:

Whether or not the disputed strip of land, allegedly encroached upon by the spouses
Polinar, is the private property of petitioners or part of the creek and therefore part of the
public domain

69
Ruling:

A careful scrutiny of the records reveals that the assailed decisions are founded on
sufficient evidence. That the subject strip of land is a creek is evidenced by: a barangay
certification that a creek exists in the disputed strip of land; a certification from the
Second Manila Engineering District, NCR-DPWH, that the western portion of Pilar
Village where the subject strip of land is located is bounded by a tributary of Talon
Creek and photographs showing the abundance of water lilies in the subject strip of
land. The Court of Appeals was correct: the fact that water lilies thrive in that strip of
land can only mean that there is a permanent stream of water or creek there. The
phrase others of similar character includes a creek which is a recess or an arm of a
river. It is property belonging to the public domain which is not susceptible to private
ownership. Being public water, a creek cannot be registered under the Torrens System
in the name of any individual. Accordingly, the Polinar spouses may utilize the rip-
rapped portion of the creek to prevent the erosion of their property.

70
JOSE MENCHAVEZ, JUAN MENCHAVEZ JR., SIMEON MENCHAVEZ, ET. AL. VS.
FLORENTINOTEVES JR.

G.R. No. 153201


January 26, 2005

Facts:

On February 28, 1986, a Contract of Lease was executed by Jose S. Menchavez, Juan
S. Menchavez Sr., Juan S. Menchavez Jr., Rodolfo Menchavez, Simeon Menchavez,
Reynaldo Menchavez, Cesar Menchavez, Charito M. Maga, Fe M. Potot, Thelma R.
Reroma, Myrna Ybaez, Sonia S. Menchavez, Sarah Villaver, Alma S. Menchavez, and
Elma S. Menchavez, as lessors; and FlorentinoTeves Jr. as lessee.On June 2, 1988,
Cebu RTC Sheriffs GumersindoGimenez and Arturo Cabigon demolished the fishpond
dikes constructed by respondent and delivered possession of the subject property to
other parties.As a result, he filed a Complaint for damages with application for
preliminary attachment against petitioners. In his Complaint, he alleged that the lessors
had violated their Contract of Lease, specifically the peaceful and adequate enjoyment
of the property for the entire duration of the Contract. Respondent further asserted that
the lessors had withheld from him the findings of the trial court in a case involving the
same property, subject of the lease, the Menchavez spouses were ordered to remove
the dikes illegally constructed and to pay damages and attorneys fees.Petitioners filed a
Third Party Complaint against Benny and Elizabeth Allego, Albino Laput, AdrinicoChe
and Charlemagne Arendain Jr., as agents of EufraciaColongan and Paulino Pamplona
and maintained that the Complaint filed against them was unfounded. As agents of their
elderly parents, they could not be sued in their personal capacity.

After trial on the merits, the RTC dismissed the complaint stating that the parties are in
pari delicto. On appeal, the CA disagreed with the RTCs finding that petitioners and
respondent were in pari delicto.

Issue:

Whether or not the Parties are in Pari Delicto?

Ruling:

Being merely applicants for the lease of the fishponds, petitioners had no transferable
right over them. And even if the State were to grant their application, the law expressly
disallowed sublease of the fishponds to respondent. Void are all contracts in which the
cause, object or purpose is contrary to law, public order or public policy. A void contract

71
is equivalent to nothing; it produces no civil effect. It does not create, modify or
extinguish a juridical relation. Parties to a void agreement cannot expect the aid of the
law; the courts leave them as they are, because they are deemed in pari delicto or in
equal fault. Unquestionably, petitioners leased out a property that did not belong to
them, one that they had no authority to sublease. The trial court correctly observed that
petitioners still had a pending lease application with the State at the time they entered
into the Contract with respondent. The evidence of respondent himself shows that he
negotiated the lease of the fishpond with both Juan Menchavez Sr. and Juan
Menchavez Jr. in the office of his lawyer, Atty. Jorge Esparagoza. His counsels
presence during the negotiations, prior to the parties meeting of minds, further debunks
his claim of lack of knowledge. Lawyers are expected to know that fishponds belong to
the State and are inalienable. It was reasonably expected of the counsel herein to
advise his client regarding the matter of ownership. The evidence presented by
respondent demonstrates the contradictory claims of petitioners regarding their alleged
ownership of the fishpond. On the one hand, they claimed ownership and, on the other,
they assured him that their fishpond lease application would be approved. This
circumstance should have been sufficient to place him on notice. It should have
compelled him to determine their right over the fishpond, including their right to lease
it.The Contract itself stated that the area was still covered by a fishpond application.
Nonetheless, although petitioners declared in the Contract that they co-owned the
property, their erroneous declaration should not be used against them. A cursory
examination of the Contract suggests that it was drafted to favor the lessee. It can
readily be presumed that it was he or his counsel who prepared it -- a matter supported
by petitioners evidence. The ambiguity should therefore be resolved against him, being
the one who primarily caused it.

The CA erred in finding that petitioners had failed to prove actual knowledge of
respondent of the ownership status of the property that had been leased to him. On the
contrary, as the party alleging the fact, it was he who had the burden of proving through
a preponderance of evidence that they misled him regarding the ownership of the
fishpond. His evidence fails to support this contention. Instead, it reveals his fault in
entering into a void Contract. As both parties are equally at fault, neither may recover
against the other.

72
REPUBLIC OF THE PHILIPPINES VS. SOCORRO P. JACOB

G.R. No. 146874


July 20, 2006

Facts:

On August 14, 1970, then President Ferdinand E. Marcos issued Proclamation No. 739,
"Establishing as Reservation for the Purpose of the Exploration, Development,
Exploitation and Utilization of Geothermal Energy, Natural Gas and Methane Gas a
Parcel of Land in the Province of Albay, Island of Luzon, Philippines." Lot No. 4094 of
the MalinaoCadastre, consisting of 15,520 square meters, is covered by the said
proclamation.Nevertheless, on May 6, 1994, private respondent, a retired public school
teacher, filed an application with the RTC of Albay for the confirmation and registration
of her alleged title over Lot No. 4094. The Republic of the Philippines, through the Office
of the Solicitor General (OSG), opposed the application for the reasons that the claim of
ownership in fee simple on the basis of Spanish title or grant can no longer be availed of
by the applicant/s who have failed to file an appropriate application for registration within
the period of six (6) months from February 16, 1976 as required by P.D. No. 892 and
the parcel/s applied for is/are portions of the public domain belonging to the Republic of
the Philippines not subject to private appropriation.Lot No. No. 4094 was declared for
taxation purposes under the name of Socorro under T. D. No. 00530effective 1985. On
July 7, 1983, she paid the realty taxes over the property from 1960 to 1983, and from
1983 to 1995.

On January 30, 1996, the trial court rendered judgment in favor of the applicant. On
appeal, the CA rendered judgment affirming the appealed decision. It declared that
although private respondent failed to adduce in evidence the deed of sale executed by
SoteroBondal in favor of MacarioMonjardin, her testimony that the sale took place was
enough. Her claims were likewise buttressed by her documentary evidence, and thus
she was able to muster the requisite quantum of evidence to prove exclusive, open, and
continuous possession under a bona fide claim of ownership for the requisite period of
time before August 14, 1970. According to the appellate court, the bare fact that private
respondent failed to present any evidence to corroborate such testimony did not render
it self-serving.

73
Issue:

Whether or not the Court of Appeals gravely erred in ruling that respondent acquired a
vested right over the subject parcel of land even before the effectivity of Proclamation
No. 739 of august 14, 1970.

Ruling:
Applicants for confirmation of imperfect title mustprove the following: (a) that the
land forms part of the disposable and alienable agricultural lands of the public domain;
and (b) that they have been in open, continuous, exclusive, and notorious possession
and occupation of the same under a bona fide claim of ownership either since time
immemorial or since June 12, 1945.No public land can be acquired by private persons
without any grant from the government, whether express or implied. It is indispensable
that there be a showing of a title from the State. The rationale for the period "since time
immemorial or since June 12, 1945" lies in the presumption that the land applied for
pertains to the State, and that the occupants or possessorclaim an interest thereon only
by virtue of their imperfect title as continuous, open and notorious possession.

In the case at bar, when private respondent filed her application with the RTC on May 6,
1994, Lot No. 4094 was no longer alienable and disposable property of the public
domain, since as of August 14, 1970, by virtue of Proclamation No. 739, it was
segregated from the public domain and declared part of the reservation for the
development of geothermal energy. Private respondent filed her application for
confirmation 24 years after the said proclamation was issued; thus, the period of her
possession and occupancy after such proclamation can no longer be tacked in favor of
the claimant.While she claimed that her mother was designated as encargado, private
respondent failed to even mention the portion of the property that was cultivated, or at
least where and who planted the palay. Such declaration (that Macario designated her
mother as encargado) without more does not constitute preponderant evidence to prove
adverse, continuous, open, public, and peaceful possession in the concept of owner.
Private respondent's testimony that after her parents purchased the lot, they began
receiving the share of the produce of the property does not in itself constitute proof of
such adverse possession. There is thus no evidence that the parents of private
respondent ever had open, continuous, adverse and actual possession of Lot No. 4094.

74
FRANCISCO I. CHAVEZ, VS. PUBLIC ESTATES AUTHORITY ANDAMARI
COASTAL BAY DEVELOPMENT CORPORATION

G.R. No. 133250


July 9, 2002

Facts:

On November 20, 1973, the government, through the Commissioner of Public


Highways, signed a contract with the Construction and Development Corporation of the
Philippines to reclaim certain foreshore and offshore areas of Manila Bay. The contract
also included the construction of Phases I and II of the Manila-Cavite Coastal Road.
CDCP obligated itself to carry out all the works in consideration of fifty percent of the
total reclaimed land.On February 4, 1977, then President Ferdinand E. Marcos issued
Presidential Decree No. 1084 creating PEA. PD No. 1084 tasked PEA "to reclaim land,
including foreshore and submerged areas," and "to develop, improve, acquire, lease
and sell any and all kinds of lands." On the same date, then President Marcos issued
Presidential Decree No. 1085 transferring to PEA the "lands reclaimed in the foreshore
and offshore of the Manila Bay" under the Manila-Cavite Coastal Road and Reclamation
Project.On January 19, 1988, then President Corazon C. Aquino issued Special Patent
No. 3517, granting and transferring to PEA "the parcels of land so reclaimed under the
Manila-Cavite Coastal Road and Reclamation Project (MCCRRP) containing a total
area of one million nine hundred fifteen thousand eight hundred ninety four (1,915,894)
square meters." Subsequently, on April 9, 1988, the Register of Deeds of the
Municipality of Parañaque issued Transfer Certificates of Title Nos. 7309, 7311, and
7312, in the name of PEA, covering the three reclaimed islands known as the "Freedom
Islands".

On April 25, 1995, PEA entered into a Joint Venture Agreement ("JVA" for brevity) with
AMARI, a private corporation, to develop the Freedom Islands. The JVA also required
the reclamation of an additional 250 hectares of submerged areas surrounding these
islands to complete the configuration in the Master Development Plan of the Southern
Reclamation Project-MCCRRP. PEA and AMARI entered into the JVA through
negotiation without public bidding.On June 8, 1995, then President Fidel V. Ramos,
through then Executive Secretary Ruben Torres, approved the JVA.On November 29,
1996, then Senate President Ernesto Maceda delivered a privilege speech in the
Senate and denounced the JVA as the "grandmother of all scams." As a result, the
Senate Committee on Government Corporations and Public Enterprises, and the
Committee on Accountability of Public Officers and Investigations, conducted a joint
investigation. On April 27, 1998, petitioner Frank I. Chavez as a taxpayer, filed the

75
instant Petition for Mandamus with Prayer for the Issuance of a Writ of Preliminary
Injunction and Temporary Restraining Order. On May 28, 1999, the Office of the
President under the administration of then President Joseph E. Estrada approved the
Amended JVA.Due to the approval of the Amended JVA by the Office of the President,
petitioner now prays that on "constitutional and statutory grounds the renegotiated
contract be declared null and void."

Issue:

Whether or not Amari, a private corporation, can acquire and own under the Amended
JVA 367.5 hectares of reclaimed foreshore and submerged areas in Manila Bay in view
of Sections 2 and 3, Article XII of the 1987 Constitution

Ruling:

Under Section 2, Article XII of the 1987 Constitution, the foreshore and submerged
areas of Manila Bay are part of the "lands of the public domain, waters and other natural
resources" and consequently "owned by the State." As such, foreshore and submerged
areas "shall not be alienated," unless they are classified as "agricultural lands" of the
public domain. The mere reclamation of these areas by PEA does not convert these
inalienable natural resources of the State into alienable or disposable lands of the public
domain. There must be a law or presidential proclamation officially classifying these
reclaimed lands as alienable or disposable and open to disposition or concession.
Moreover, these reclaimed lands cannot be classified as alienable or disposable if the
law has reserved them for some public or quasi-public use.Article 5 of the Spanish Law
of Waters must be read together with laws subsequently enacted on the disposition of
public lands. In particular, CA No. 141 requires that lands of the public domain must first
be classified as alienable or disposable before the government can alienate them.
These lands must not be reserved for public or quasi-public purposes. Moreover, the
contract between CDCP and the government was executed after the effectivity of the
1973 Constitution which barred private corporations from acquiring any kind of alienable
land of the public domain. This contract could not have converted the Freedom Islands
into private lands of a private corporation. The Amended JVA covers not only the
Freedom Islands, but also an additional 592.15 hectares which are still submerged and
forming part of Manila Bay. There is no legislative or Presidential act classifying these
submerged areas as alienable or disposable lands of the public domain open to
disposition. These submerged areas are not covered by any patent or certificate of title.
There can be no dispute that these submerged areas form part of the public domain,
and in their present state are inalienable and outside the commerce of man. Until

76
reclaimed from the sea, these submerged areas are, under the Constitution, "waters
owned by the State," forming part of the public domain and consequently inalienable.

77
ARBIAS VS. REPUBLIC OF THE PHILIPPINES

G.R. No. 173808


September 17, 2008

Facts:

Arbias filed an Application for Registration of Title over a lot which he purchased from
Lourder Jardeleza. She attached to her application the Tracing Cloth with Blue Print
copies, the Deed of Absolute Sale involving the subject property, the Surveyors
Certification, the Technical Description of the land, and Declaration of Real Property in
the name of petitioner and her spouse Jimmy. During trial, Arbias merely relied on an
annotation of the blueprint to prove that the land is alienable and disposable.The court
ruled in favor of Arbias. The CA, however, reversed said decision due to Arbias' failure
to prove that the land is alienable and disposable.

Issue:

Whether or not petitioner has a registerable title on the subject lot.

Ruling:

No. First, the documentary evidence that petitioner presented before the RTC did not in
any way prove the length and character of her possession and those of her
predecessor-in-interest relative to the subject property.

The Deed of Sale merely stated that the vendor of the subject property, Jardeleza, was
the true and lawful owner of the subject property, and that she sold the same to
petitioner on 12 March 1993. The Deed did not state the duration of time during which
the vendor (or her predecessors-in-interest) possessed the subject property in the
concept of an owner.

Petitioners presentation of tax declarations of the subject property for the years 1983,
1989, 1991 and 1994, as well as tax receipts of payment of the realty tax due thereon,
are of little evidentiary weight. Well-settled is the rule that tax declarations and receipts
are not conclusive evidence of ownership or of the right to possess land when not
supported by any other evidence. The fact that the disputed property may have been
declared for taxation purposes in the names of the applicants for registration or of their
predecessors-in-interest does not necessarily prove ownership. They are merely indicia
of a claim of ownership.

78
REPUBLIC VS. CAYETANO SERRANO

G.R. No. 183063


February 24, 2010

Facts:

Cayetano filed before the RTC an application for registration. He claimed that he
inherited the property from his parents. Thus, Cayetano claimed to have been in open,
continuous, exclusive and notorious possession of the lot under a claim of ownership
before 1917 by himself and through his deceased parents predecessors-in-interest or
for more than 70 years. The heirs of Catlino Alaan also intervened and filed an
application for registration. They claimed that Catalino bought a part of the land in
question from Cayetano. Cayetano did not oppose said intervention.

At the trial, the following pieces of documentary evidence were by the Cayetano and the
Heirs of Alaan: original survey plan certified by the Department of Environment and
Natural Resources (DENR) in which it was stated that the survey was conducted inside
the alienable and disposable area, the technical description of the lot, Tax Declarations
for the years 1924 (in the name of Simeon) and 1948-1997 (in the name of either
Simeon [deceased] or Cayetano), official receipts showing real estate tax payments
(from 1948-1997), and Surveyors Certificate No. 157485 dated January 1957.
The Republic did not present any evidence to oppose the application. After trial, the
RTC awarded the lot to respondents. The OSG then appealed the case to the CA . The
OSG claimed that Cayetano was not able to prove that the property is alienable and
disposable. The CA affirmed the decision of the RTC.

Issue:

Whether or not Cayetano was able to prove that the land is alienable and disposable.

Ruling:

Yes. While Cayetano failed to submit any certification which would formally attest to the
alienable and disposable character if the land applied for, the Certification by DENR
Regional Technical Director , as annotated on the subdivision plan submitted in
evidence by Paulita, constitutes substantial compliance with the legal requirement. It
clearly indicates that Lot 249 had been verified as belonging to the alienable and
disposable area as early as July 18, 1925.

79
REPUBLIC VS. HEIRS OF JUAN FABIO

G.R. No. 159589


December 23, 2008

Facts:

The Heirs of Juan Fabio filed an application for registration of title to the Lot with an
approximate area of 109.6 hectares. Among the document they presented, particularly
for the purpose of proving that the land in question is alienable and disposable, was a
certification made by Arnaldo Conlu who is a Land Management Inspector of DENR
which established that the land is alienable and disposable. During trial, respondents
presented their witnesses who then testified that since time immemorial, the family of
Juan Fabio occupied the lot in the concept of an owner and adverse to the public. One
of the witness, Pangyarihan, however, testified that the land falls within the Calumpang
Point Naval Reservation. The land was declared as such through U.S. War Department
General Order No. 56 since 1904.

Sometime during the time of President Marcos, the President issued 2 proclamations
which affirms the fact that said land is a military reservation but recognized vested rights
by persons occupying said land. The RTC awarded the lands respondents. The
Republic through the OSG appealed the case to the CA. The CA affirmed the decision
of the court a quo.

Issue:

Whether or not the land in question may be registered in the names of Juan Fabio's
heirs.

Ruling:

No. To prove that the Lot is alienable and disposable land of the public domain,
respondents presented a certification made by Conlu. This letter-certification is
insufficient. Conlu is merely a land investigator of the DENR. It is not enough that he
alone should certify that the Lot is within the alienable and disposable zone. Under
Section 6 of the Public Land Act, the prerogative of classifying or reclassifying lands of
the public domain belongs to the President. The President, through a presidential
proclamation or executive order, can classify or reclassify a land to be included or
excluded from the public domain. The DENR Secretary is the only other public official
empowered by law to approve a land classification and declare such land as alienable
and disposable.

80
FLORENCIA DIAZ VS. REPUBLIC

G.R. No. 181502


February 2, 2010

Facts:

This is a letter-motion praying for reconsideration for the third time of the resolution of
the Supreme Court denying the petition for review filed by petitioner Florencia Diaz. The
petitioner filed an application for registration of a vast tract of land in Nueva Ecija. She
alleged that she possessed the land as owner and worker, developed, and harvested
the agricultural products and benefits of the same continuously, publicly, and adversely
for more than 26 years.

OSG opposed the application because the land in question was within the Fort
Magsaysay Military Reservation. Thus, it was inalienable as it formed part of the public
domain. Prior to this case, the Supreme Court already ruled in the case of Director of
Lands vs Reyes that the property was inalienable as it formed part of a military
reservation and the existence of a Possessory Information Title No. 216 (registered in
the name of a certain Melecio Padilla) which the respondent in the sited cased
anchored its claim on the land, was not proven. CFI ruled in favor of the petitioner. Upon
appeal, the CA ruled in favor of the Republic.

Issue:

Whether or not the land in dispute can be registered.

Ruling:

Before the military reservation was established, the evidence is inconclusive as to


possession, for it is shown by the evidence that the land involved is largely mountainous
and forested. As a matter of fact, at the time of the hearing, it was conceded that
approximately 13, 957 hectares of said land consist of public forest. It is well-settled that
forest land is incapable of registration; and its inclusion in a title, whether such title be
one issued during the Spanish era or under the present Torrens system of registration,
nullifies the title.

However, it is true that land may be registered when they have been reclassified as
alienable by the President in a clear and categorical manner (upon the recommendation
of the proper department head who has the authority to classify the land of the public

81
domain into alienable or disposable, timber and miner lands) coupled with possession
by the claimant as well as that of her predecessors-in-interest. Unfortunately for the
petitioner, she was not able to produce such evidence. Her occupation thereof could not
have ripened into ownership of the subject land.

82
MIAA VS. CITY OF PASAY

G.R. No. 163072


April 02 2009

Facts:

Petitioner Manila International Airport Authority (MIAA) operates and administers the
Ninoy Aquino International Airport (NAIA) Complex under Executive Order No. 903 (EO
903),3 otherwise known as the Revised Charter of the Manila International Airport
Authority, issued by then President Ferdinand E. Marcos. The NAIA Complex is located
along the border between Pasay City and Paranaque City. MIAA received Final Notices
of Real Property Tax Delinquency from the City of Pasay for the taxable years 1992 to
2001. The Court of Appeals upheld the power of the City of Pasay to impose and collect
realty taxes on the NAIA Pasay properties. MIAA filed a motion for reconsideration,
which the Court of Appeals denied.

Issue:

The issue raised in this petition is whether the NAIA Pasay properties of MIAA are
exempt from real property tax.

Ruling:

The Supreme Court held that the Airport Lands and Buildings of MIAA are properties
devoted to public use and thus are properties of public dominion. Properties of public
dominion are owned by the State or the Republic. Article 420 of the Civil Code provides:
Art. 420. The following things are property of public dominion:
(1) Those intended for public use, such as roads, canals, rivers, torrents, ports and
bridges constructed by the State, banks, shores, roadsteads, and others of similar
character;
(2) Those which belong to the State, without being for public use, and are intended for
some public service or for the development of the national wealth.

The term "ports x x x constructed by the State" includes airports and seaports. The
Airport Lands and Buildings of MIAA are intended for public use, and at the very least
intended for public service. Whether intended for public use or public service, the Airport
Lands and Buildings are properties of public dominion. As properties of public dominion,
the Airport Lands and Buildings are owned by the Republic and thus exempt from real
estate tax under Section 234(a) of the Local Government Code.

83
MIAA VS. COURT OF APPEALS

G.R. No. 155650


July 20, 2006

Facts:

The Manila International Airport Authority (MIAA) operates the Ninoy Aquino
International Airport (NAIA) Complex in Paranaque City under Executive Order No. 903
(MIAA Charter), as amended. As such operator, it administers the land, improvements
and equipment within the NAIA Complex. In March 1997, the Office of the Government
Corporate Counsel (OGCC) issued Opinion No. 061 to the effect that the Local
Government Code of 1991 (LGC) withdrew the exemption from real estate tax granted
to MIAA under Section 21of its Charter. Thus, MIAA paid some of the real estate tax
already due. In June 2001, it received Final Notices of Real Estate Tax Delinquency
from the City of Paranaque for the taxable years 1992 to 2001. The City Treasurer
subsequently issued notices of levy and warrants of levy on the airport lands and
buildings. At the instance of MIAA, the OGCC issued Opinion No. 147 clarifying Opinion
No. 061, pointing out that Sec. 206 of the LGC requires persons exempt from real
estate tax to show proof of exemption. According to the OGCC, Sec. 21 of the MIAA
Charter is the proof that MIAA is exempt from real estate tax. MIAA, thus, filed a petition
with the Court of Appeals seeking to restrain the City of Paranaque from imposing real
estate tax on, levying against, and auctioning for public sale the airport lands and
buildings, but this was dismissed for having been filed out of time.

Hence, MIAA filed this petition for review, pointing out that it is exempt from real-estate
tax under Sec. 21 of its charter and Sec. 234 of the LGC. It invokes the principle that the
government cannot tax itself as a justification for exemption, since the airport lands and
buildings, being devoted to public use and public service, are owned by the Republic of
the Philippines. On the other hand, the City of Paranaque invokes Sec. 193 of the LGC,
which expressly withdrew the tax exemption privileges of government-owned and
controlled corporations (GOCC) upon the effectivity of the LGC. It asserts that an
international airport is not among the exceptions mentioned in the said law. Meanwhile,
the City of Paranaque posted and published notices announcing the public auction sale
of the airport lands and buildings. In the afternoon before the scheduled public auction,
MIAA applied with the Court for the issuance of a TRO to restrain the auction sale. The
Court issued a TRO on the day of the auction sale, however, the same was received
only by the City of Paranaque three hours after the sale.

84
Issue:

Whether or not the airport lands and buildings of MIAA are exempt from real estate tax?

Ruling:

The airport lands and buildings of MIAA are exempt from real estate tax imposed by
local governments. Sec. 243(a) of the LGC exempts from real estate tax any real
property owned by the Republic of the Philippines. This exemption should be read in
relation with Sec.133(o) of the LGC, which provides that the exercise of the taxing
powers of local governments shall not extend to the levy of taxes, fees or charges of
any kind on the National Government, its agencies and instrumentalities.

85
PHILIPPINE FISHERIES DEVELOPMENT AUTHORITY VS. COURT OF APPEALS

G.R. No. 169836


July 31, 2007

Facts:

The controversy arose when respondent Municipality of Navotas assessed the real
estate taxes allegedly due from petitioner Philippine Fisheries Development Authority
(PFDA) for the period 1981-1990 on properties under its jurisdiction, management and
operation located inside the Navotas Fishing Port Complex (NFPC). The assessed
taxes had remained unpaid despite the demands made by the municipality which
prompted it, through Municipal Treasurer Florante M. Barredo, to give notice to
petitioner on October 29, 1990 that the NFPC will be sold at public auction on
November 30, 1990 in order that the municipality will be able to collect on petitioner‗s
delinquent realty taxes which, as of June 30, 1990, amounted to P23,128,304.51,
inclusive of penalties. Petitioner sought the deferment of the auction sale claiming that
the NFPC is owned by the Republic of the Philippines, and pursuant to Presidential
Decree (P.D.) No. 977, it (PFDA) is not a taxable entity. In view of the refusal of PFDA
to pay the assessed realty taxes, the matter was referred to the Department of Finance
(DOF). On July 14, 1990 the DOF stated that: This Department takes cognizance of the
allegations of [the Office of the Mayor of Navotas] that PFDA has leased its properties
to beneficial users, such as ―businessmen, private persons and entities who are
taxable persons. For this reason, it is imperative that the Municipality should conduct an
ocular inspection on the real properties (land and building owned by PFDA) in order to
identify the properties actually leased and the taxable persons enjoying the beneficial
use thereof. The ocular inspection is necessary for reason that the real properties, the
use of which has been granted to taxable persons, for consideration or otherwise, are
subject to the payment of real property taxes which must be paid by the grantees
pursuant to the provisions of the Real Property Tax Code.

Therefore, it is imperative to determine who the actual users of the properties


concerned are if used by a non-taxable person other than PFDA itself, it remains to be
non-taxable. Otherwise, if said properties are being used by taxable persons, same
becomes taxable properties. For this purpose, it is also incumbent upon PFDA to furnish
the Municipality copies of the deed of lease or other relevant documents showing the
leased properties and their beneficial users for proper assessment.

Notwithstanding the DOF‗s instruction, respondent Municipality proceeded to publish


the notice of sale of NFPC in the November 2, 1990 issue of Balita, a local newspaper.
Petitioner instituted a Civil Case in the Regional Trial Court (RTC) of Malabon, Metro

86
Manila against respondent Municipality, its Municipal Treasurer and the Chairman of the
Public Auction Sale Committee. Petitioner asked the RTC to enjoin the auction of the
NFPC on the ground that the properties comprising the NFPC are owned by the
Republic of the Philippines and are, thus, exempt from taxation. According to petitioner,
only a small portion of NFPC which had been leased to private parties may be
subjected to real property tax which should be paid by the latter. Respondent
Municipality insisted that:
1) the real properties within NFPC are owned entirely by petitioner which, despite the
opportunity given, had failed to submit proof to the Municipal Assessor that the
properties are indeed owned by the Republic of the Philippines;
2) if the properties in question really belong to the government, then the complaint
should have been instituted in the name of the Republic of the Philippines, represented
by the Office of the Solicitor General; and
3) the complaint is fatally defective because of non-compliance with a condition
precedent, which is, payment of the disputed tax assessment under protest.

Issues:

Whether petitioner is liable to pay real property tax.

Ruling:

Local government units, pursuant to the fiscal autonomy granted by the provisions of
Republic Act No. 7160 or the 1991 Local Government Code, can impose realty taxes on
juridical persons subject to the limitations enumerated in Section 133 of the Code:

SEC. 133. Common Limitations on the Taxing Power of Local Government Units. –
Unless otherwise provided herein, the exercise of the taxing powers of provinces, cities,
municipalities, and barangays shall not extend to the levy of the following: (o) taxes,
fees, charges of any kind on the national government, its agencies and
instrumentalities, and local government units.

Nonetheless, the above exemption does not apply when the beneficial use of the
government property has been granted to a taxable person. Section 234 (a) of the Code
states that real property owned by the Republic of the Philippines or any of its political
subdivisions is exempted from payment of the real property tax ―except when the
beneficial use thereof has been granted, for consideration or otherwise, to a taxable
person. Thus, as a rule, petitioner PFDA, being an instrumentality of the national
government, is exempt from real property tax but the exemption does not extend to the

87
portions of the NFPC that were leased to taxable or private persons and entities for their
beneficial use.

The real property tax assessments issued by the City of Iloilo should be upheld only
with respect to the portions leased to private persons. In case the Authority fails to pay
the real property taxes due thereon, said portions cannot be sold at public auction to
satisfy the tax delinquency. The port built by the State in the Iloilo fishing complex is a
property of public dominion and cannot therefore be sold at public auction. Article 420 of
the Civil Code provides:

ARTICLE 420. The following things are property of public dominion: (1) Those intended
for public use, such as roads, canals, rivers, torrents, ports and bridges constructed by
the State, banks, shores, roadsteads, and others of similar character; (2) Those which
belong to the State, without being for public use, and are intended for some public
service or for the development of national wealth.

The Iloilo [F]ishing [P]ort [Complex/IFPC] which was constructed by the State for public
use and/or public service falls within the term ―port in the aforecited provision. Being a
property of public dominion the same cannot be subject to execution or foreclosure sale.
… Whether there are improvements in the fishing port complex that should not be
construed to be embraced within the term – port involves evidentiary matters that
cannot be addressed in the present case. As for now, considering that the Authority is a
national government instrumentality, any doubt on whether the entire IFPC may be
levied upon to satisfy the tax delinquency should be resolved against the City of Iloilo.

Similarly, for the same reason, the NFPC cannot be sold at public auction in satisfaction
of the tax delinquency assessments made by the Municipality of Navotas on the entire
complex. Additionally, the land on which the NFPC property sits is a reclaimed land,
which belongs to the State. In Chavez v. Public Estates Authority, the Court declared
that reclaimed lands are lands of the public domain and cannot, without Congressional
fiat, be subject of a sale, public or private.

88
MACASIANO VS. DIOKNO

G.R. No. 97764


August 10, 1992

Facts:

Respondent Municipality passed Ordinance No. 86 which authorized the closure of


J.Gabriel, G.G. Cruz, Bayanihan, Lt. Garcia Extension and Opena Streets and the
establishment of a flea market thereon. This was passed pursuant to MMC Ordinance
No.2 and was approved by the Metropolitan Manila Authority on July 20, 1990.

On August 8, 1990, respondent municipality and Palanyag entered into a contract


agreement whereby the latter shall operate, maintain & manage the flea markets and/or
vending areas in the aforementioned streets with the obligation to remit dues to the
treasury of the municipal government of Parañaque.

On September 13, 1990 Brig. Gen. Macasiano ordered the destruction and confiscation
of stalls along G.G. Cruz & Gabriel Street in Baclaran. He also wrote a letter to
Palanyag ordering the destruction of the flea market. Hence, respondent filed a joint
petition praying for preliminary injunction. The trial court upheld the assailed Ordinance
and enjoined petitioner from enforcing his letter-order against Palanyag.

Issue:

WON an ordinance/resolution issued by the municipal council of Parañaque authorizing


the lease & use of public streets/thoroughfares as sites for the flea market is valid.

Ruling:

No. J. Gabriel, G.G. Cruz, Bayanihan, Lt. Garcia Extension and Opena Streets are local
roads used for public service and are therefore considered public properties of
respondent municipality. Properties of the local government devoted to public service
are deemed public and are under the absolute control of Congress. Hence, local
governments have no authority to control/regulate the use of public properties unless
specific authority is vested upon them by Congress.

Sec. 10, Chapter II of the LGC should be read and interpreted in accordance with basic
principles already established by law. The closure should be for the sole purpose of
withdrawing the road or other public property from public use when circumstances show
that such property is no longer intended/necessary for public use/service. Once

89
withdrawn, the property then becomes patrimonial property of the LGU concerned and
only then can said LGU use the property as an object of an ordinary contract. Roads
and streets available to the public and ordinarily used for vehicular traffic are still
considered public property devoted to public use. The LGU has no power to use it for
another purpose or to dispose of or lease it to private persons.

Also, the disputed ordinance cannot be validly implemented because it can‗t be


considered approved by the Metropolitan Manila Authority due to non-compliance with
the conditions it imposed for the approval of said ordinance.

The powers of an LGU are not absolute, but subject to the limitations laid down by the
Constitution and laws such as the Civil Code. Every LGU has the sworn obligation to
enact measures that will enhance the public health, safety & convenience, maintain
peace & order and promiote the general prosperity of the inhanbitants pf the local units.

As in the Dacanay case, the general public have the right to demand the demolition of
the illegally constructed stalls in public roads & streets. The officials of the respondent
municipality have the corresponding duty arising from public office to clear the city
streets and restore them to their specific public purpose.

The ordinance is void and illegal for lack of basis in authority in laws applicable during
its time.

90
REPUBLIC VS. RTC, CITY OF ROXAS

G.R. No. 172931


June 18, 2009

Facts:

Then President Quezon issued proclamation No. 678 converting forest land measuring
around 928 hectares, situated in San Teodoro, Oriental Mindoro, Matchwood Forest
Reserve. He withdrew the 928 ha of forest land from entry, sale, or settlement, subject
to private rights.

Petitioner Republic, through the DANR, entered into Matchwood Plantation Lease
Agreement No. 1 with petitioner PTFI, wherein Republic lease the entire Matchwood
Forest Reserve to petitioner PTFI for a period of 25 years, which would expire June 30,
1990.

Roxas filed with the Bureau of Lands on December 29, 1959 Homestead Application.
The Director of Lands issued the Homestead Patent to Roxas.
Petitioner Republic represented by the BED filed with the RTC a Complaint for
Cancellation of Title and/or Reversion against respondents Roxas and the ROD over
the subject property.

Petitioner Republic alleged that the subject property was within the Matchwood Forest
Reserve and could not be the subject of private appropriation and ownership; and
possession of said property, no matter how long would not convert the same into private
property.

Issue:

Whether or not petitioner Republic may cause the reversion of the Property despite
there being no fraud or misrepresentation on the part of Roxas.

Ruling:

A mistake or oversight was committed on the part of respondent Roxas, as well as the
Government, resulting in the grant of a homestead patent over inalienable forest land.
Hence, it can be said that the subject property was unlawfully covered by the
Homestead Patent and OCT in Roxas‗ name, which entitles petitioner Republic to the
cancellation of said patent to the public domain. As we have ruled herein, the subject
property is part of the Matchwood Forest Reserve and is inalienable and not subject to

91
disposition. Being contrary to the Public Land Law, Homestead Patent No. and OCT
issued in respondent Roxas‗ name are void; and the right of petitioner Republic to seek
cancellation of such void ptent/title and reversion of the subject property to the State is
imprescriptible.

92
ATTRIBUTES OF
OWNERSHIP

Right to Possess

93
ANECO REALTY AND DEVELOPMENT CORPORATION VS. LANDEX
DEVELOPMENT

G.R. No. 165952


July 28, 2008

Facts:

Fernandez Hermanos Development, Inc. (FHDI) is the original owner of a tract of land in
San Francisco Del Monte, Quezon City. FHDI subdivided the land into thirty-nine (39)
lots.[3] It later sold twenty-two (22) lots to petitioner Aneco and the remaining seventeen
(17) lots to respondent Landex.

The dispute arose when Landex started the construction of a concrete wall on one of its
lots. To restrain construction of the wall, Aneco filed a complaint for injunction with the
RTC in Quezon City. Aneco later filed two (2) supplemental complaints seeking to
demolish the newly-built wall and to hold Landex liable for two million pesos in
damages.

Issue:

Whether or not Aneco may enjoin Landex from constructing a concrete wall on its
property (W/N Aneco should be given the right of way.)

Ruling:

No. What is involved here is an undue interference on the property rights of a landowner
to build a concrete wall on his own property. It is a simple case of a neighbor, petitioner
Aneco, seeking to restrain a landowner, respondent Landex, from fencing his own land.

Article 430 of the Civil Code gives every owner the right to enclose or fence his land or
tenement by means of walls, ditches, hedges or any other means. The right to fence
flows from the right of ownership. As owner of the land,Landex may fence his property
subject only to the limitations and restrictions provided by law. Absent a clear legal and
enforceable right, as here, We will not interfere with the exercise of an essential
attribute of ownership.

Well-settled is the rule that factual findings and conclusions of law of the trial court when
affirmed by the CA are accorded great weight and respect. Here, We find no cogent
reason to deviate from the factual findings and conclusion of law of the trial court and
the appellate court. We have meticulously reviewed the records and agree thatAneco

94
failed to prove any clear legal right to prevent, much less restrain, Landex from fencing
its own property.

Aneco cannot rely on the road lot under the old subdivision project of FHDI because it
knew at the time of the sale that it was buying ordinary lots, not subdivision lots, from
FHDI. This is clear from the deed of sale between FHDI and Aneco where FHDI
manifested that it was no longer interested in pursuing its own subdivision project. If
Anecowants to transform its own lots into a subdivision project, it must make its own
provision for road lots. It certainly cannot piggy back on the road lot of the defunct
subdivision project of FHDI to the detriment of the new ownerLandex. The RTC and the
CA correctly dismissed the complaint for injunction of Aneco for lack of merit.

95
ATTRIBUTES OF
OWNERSHIP

Right to Recover

96
OCAMPO VS. DIOKNO

G.R. No. 176830


February 11, 2014

Facts:
On 26 August 2006, a mass grave was discovered by elements of the 43rd
Infantry Brigade of the Philippine Army at Sitio Sapang Daco, Barangay Kaulisihan,
Inopacan, Leyte.1The mass grave contained skeletal remains of 67 individuals believed
to be victims of "Operation Venereal Disease" (Operation VD) launched by members of
the Communist Party of the Philippines/New Peoples Army/National Democratic Front
of the Philippines (CPP/NPA/NDFP) to purge their ranks of suspected military informers.
P C/Insp. Almaden of the (PNP) Regional Office 8 and Staff Judge Advocate Captain
Allan Tiu (Army Captain Tiu) of the 8th Infantry Division of the Philippine Army sent 12
undated letters to the Provincial Prosecutor of Leyte through Assistant Provincial
Prosecutor Rosulo U. Vivero (Prosecutor Vivero).The letters requested appropriate legal
action on 12 complaint-affidavits attached therewith accusing 71 named members of the
Communist Party of the Philippines/New Peoples Army/National Democratic Front of
the Philippines (CPP/NPA/NDFP) of murder, including petitioners herein along with
several other unnamed members.

Also attached to the letters were the affidavits of Zacarias Piedad,Leonardo C. Tanaid,
Floro M. Tanaid, Numeriano Beringuel, Glecerio Roluna and Veronica P. Tabara. They
narrated that they were former members of the CPP/NPA/NDFP.According to them,
Operation VD was ordered in 1985 by the CPP/NPA/NDFP Central
Committee.Allegedly, petitioners Saturnino C. Ocampo (Ocampo),Randall B. Echanis
(Echanis),Rafael G. Baylosis (Baylosis),and Vicente P. Ladlad (Ladlad)were then
members of the Central Committee.

From 1985 to 1992, at least 100 people had been abducted, hog-tied, tortured and
executed by members of the CPP/NPA/NDF pursuant to Operation VD. On the basis of
the 12 letters and their attachments, Prosecutor Vivero issued a subpoena requiring,
among others, petitioners to submit their counter-affidavits and those of their
witnesses.Petitioner Ocampo submitted his counter-affidavit.Petitioners Echanisand
Baylosis did not file counter-affidavits because they were allegedly not served the copy
of the complaint and the attached documents or evidence. Counsel of petitioner Ladlad
made a formal entry of appearance on 8 December 2006 during the preliminary
investigation. However, petitioner Ladlad did not file a counter-affidavit because he was
allegedly not served a subpoena. In a Resolution, Prosecutor Vivero recommended the

97
filing of an Information for 15 counts of multiple murder against 54 named members of
the CPP/NPA/NDFP, including petitioners herein

Prosecutor Vivero also recommended that Zacarias Piedad, Leonardo Tanaid,


Numeriano Beringuel and Glecerio Roluna be dropped as respondents and utilized as
state witnesses, as their testimonies were vital to the success of the prosecution.
The Information was filed before the (RTC) of Hilongos, Leyte, Branch 18 (RTC
Hilongos, Leyte) presided by Judge Ephrem S. Abando (Judge Abando). On 6 March
2007, Judge Abando issued an Order finding probable cause "in the commission by all
mentioned accused of the crime charged." He ordered the issuance of warrants of
arrest against them with no recommended bail for their temporary liberty.

On 16 March 2007, petitioner Ocampo filed a special civil action for certiorari and
prohibition under Rule 65 of the Rules of Court seeking the annulment of the 6 March
2007 Order of Judge Abando and the Resolution of Prosecutor Vivero.The petition
prayed for the unconditional release of petitioner Ocampo from PNP custody, as well as
the issuance of a temporary restraining order/ writ of preliminary injunction to restrain
the conduct of further proceedings during the pendency of the petition.

Petitioner Ocampo argued that a case for rebellion against him and 44 others (including
petitioners Echanis and Baylosisand Ladlad) was then pending before the RTC Makati,
Branch 150 (RTC Makati).Putting forward the political offense doctrine, petitioner
Ocampo argues that common crimes, such as murder in this case, are already
absorbed by the crime of rebellion when committed as a necessary means, in
connection with and in furtherance of rebellion.

While the proceedings were suspended, petitioner Echanis was arrested by virtue of the
warrant of arrest issued by Judge Abando. On 1 February 2008, petitioners Echanis and
Baylosis filed a Motion for Judicial Reinvestigation/ Determination of Probable Cause
with Prayer to Dismiss the Case Outright and Alternative Prayer to Recall/ Suspend
Service of Warrant. Judge Abando issued an Order denying the motion.Petitioners
Echanis and Baylosis filed a Motion for Reconsideration but before being able to rule
thereon, Judge Abando issued an Order transmitting the records of Criminal Case to the
Office of the Clerk of Court, RTC Manila. Petitioner Ladlad and Baylosis filed an Urgent
Motion to Fix Bail and a Motion to Allow Petitioner to Post Bail respectively.The OSG
interposed no objection to the grant of aP100,000 cash bail to them. The Court granted
the motions of petitioners Ladlad and Baylosis and fixed their bail in the amount
ofP100,000, subject to the condition that their temporary release shall be limited to the
period of their actual participation in the peace negotiations

98
Issue:
1. Whether or not petitioners were denied due process during preliminary
investigation and in the issuance of the warrant of arrest;
2. Whether the murder charges against petitioners should be dismissed under
the political offense doctrine.

Ruling:

The petition was denied. "The essence of due process is reasonable opportunity to be
heard and submit evidence in support of one's defense." What is proscribed is lack of
opportunity to be heard. Thus, one who has been afforded a chance to present ones
own side of the story cannot claim denial of due process. Majority of the respondents
did not submit their counter-affidavits because they could no longer be found in their last
known address, per return of the subpoenas. On the other hand, Saturnino Ocampo @
Satur, Fides Lim, Maureen Palejaro and Ruben Manatad submitted their Counter-
Affidavits. However, Vicente Ladlad and Jasmin Jerusalem failed to submit the required
Counter Affidavits in spite entry of appearance by their respective counsels.

Section 3(d), Rule 112 of the Rules of Court, allows Prosecutor Vivero to resolve the
complaint based on the evidence before him if a respondent could not be subpoenaed.
As long as efforts to reach a respondent were made, and he was given an opportunity
to present countervailing evidence, the preliminary investigation remains valid. In this
case, the Resolution stated that efforts were undertaken to serve subpoenas on the
named respondents at their last known addresses. This is sufficient for due process. It
was only because a majority of them could no longer be found at their last known
addresses that they were not served copies of the complaint and the attached
documents or evidence.

Petitioner Ladlad, through his counsel, had every opportunity to secure copies of the
complaint after his counsels formal entry of appearance and, thereafter, to participate
fully in the preliminary investigation. Instead, he refused to participate. Neither can we
uphold petitioner Ocampos contention that he was denied the right to be heard. For him
to claim that he was denied due process by not being furnished a copy of the
Supplemental Affidavit of Zacarias Piedad would imply that the entire case of the
prosecution rested on the Supplemental Affidavit. The OSG has asserted that the
indictment of petitioner Ocampo was based on the collective affidavits of several other
witnesses attesting to the allegation that he was a member of the CPP/NPA/NDFP
Central Committee, which had ordered the launch of Operation VD.

99
B. Issuance of the Warrants of Arrest Article III, Section 2 of the Constitution
provides that "no search warrant or warrant of arrest shall issue except upon probable
cause to be determined personally by the judge after examination under oath or
affirmation of the complainant and the witnesses he may produce."Petitioner Ocampo
alleges that Judge Abando did not comply with the requirements of the Constitution in
finding the existence of probable cause for the issuance of warrants of arrest against
petitioners. Probable cause for the issuance of a warrant of arrest has been defined as
"such facts and circumstances which would lead a reasonably discreet and prudent man
to believe that an offense has been committed by the person sought to be arrested."
Allado v. Diokno, G.R. No. 113630, May 5, 1994.Although the Constitution provides that
probable cause shall be determined by the judge after an examination under oath or an
affirmation of the complainant and the witnesses, we have ruled that a hearing is not
necessary for the determination thereof. In fact, the judges personal examination of the
complainant and the witnesses is not mandatory and indispensable for determining the
aptness of issuing a warrant of arrest.

It is enough that the judge personally evaluates the prosecutors report and
supporting documents showing the existence of probable cause for the indictment and,
on the basis thereof, issue a warrant of arrest; or if, on the basis of his evaluation, he
finds no probable cause, to disregard the prosecutor's resolution and require the
submission of additional affidavits of witnesses to aid him in determining its existence.
Delos Santos-Reyes v. Montesa, Jr. 317 Phil. 101 The determination of probable cause
for the issuance of warrants of arrest against petitioners is addressed to the sound
discretion of Judge Abando as the trial judge.

100
BONIFACIO PEIDAD VS. SPOUSES GURIEZA

G.R. No. 207525


June 18, 2014

Facts:

The instant case stemmed from a Complaint for Unlawful Detainer and Damages5 filed
by Bonifacio against Sps.Gurieza before the Municipal Trial Court of Bayombong,
Nueva Vizcaya (MTC), docketed as Civil Case No. 3877. In his complaint, Bonifacio
alleged that heis the absolute owner of the 1/3 middle portion of a parcel of residential
land designated as Lot 1227, located at La Torre, Bayombong, Nueva Vizcaya, withan
area of 4,640.98 square meters (subject lot) which he acquired through intestate
succession from his late father who inherited the same from the latter‗s parents,
Alejandro Piedad (Alejandro)and Tomasa Villaray (Tomasa). He also claimed that his
ownership of the subject lot took place even before his father‗s death and was validated
through a Deed of Confirmation of an Adjudication and Partition (Deed of Confirmation)
executed by Alejandro and Tomasa‗s legal heirs. Further, Bonifacio alleged that before
migrating to Hawaii, he built a bungalow on the subject lot and assigned numerous
caretakers to look after it, the last of which were Sps. Gurieza. Sometime in 2005,
however, Sps. Gurieza allegedly took interest of the bungalow and the subject lot after
learning from an employee of the Department of Environment and Natural Resources
(DENR) that Lot 1227 is public land. Using such information, Sps. Gurieza had the
subject lot declared under their name for tax purposes, caused a subdivision survey of
Lot 1227, and filed an application for survey authority and titling with the Bureau of Land
Management, Community Environment and Natural Resources Office of the DENR,
Bayombong, Nueva Vizcaya (CENRO – DENR Nueva Vizcaya).

Issue:

The primordial issue for the Court‗s resolution is whether or not the CA correctly
reversed the RTC ruling and, consequently, dismissed Bonifacio‗s Complaint for
Unlawful Detainer and Damages against Sps. Gurieza.

Ruling:

The Court shall solely resolve the issue as to who between the parties has the better
right of possession de facto over the subject lot. Corollary thereto, issues pertaining to
ownership are better threshed out in another action instituted for such purpose.

101
After a judicious perusal of the records, the Court holds that Bonifacio had clearly
established his cause of action for unlawful detainer. The following established facts
impel this conclusion:

First, the evidence shows that as early as the 1950s, Bonifacio already had possession
of the subject lot and even built a bungalow-type house thereon. Moreover, when he
migrated to Hawaii, Bonifacio appointed numerous caretakers to the said house and lot,
the last being Sps. Gurieza. Thus, despite his migration to Hawaii, Bonifacio never
relinquished said possession over the house and lot. Consistent with Article 52420 of
the Civil Code, it is well-settled that "[i]t is not necessary that the owner of a parcel of
land should himself occupy the property as someone in his name may perform the act.
In other words, the owner of real estate has possession, either when he himself is
physically in occupation of the property, or when another person who recognizes his
rights as owner is in such occupancy."21 Thus, the Sps. Gurieza‗s stay on the subject
lot was only made possible through the mere tolerance of Bonifacio.

Second, when Bonifacio learned that Sps. Gurieza declared the subject lot under their
name for tax purposes, caused a subdivision survey of Lot 1227, and filed an
application for survey authority and titling with the CENRO–DENR Nueva Vizcaya, he
immediately took steps to terminate their tolerated stay on the subject lot and house and
demanded that they leave immediately, rendering the Sps.Gurieza‗s stay on the subject
lot illegal.

Third, instead of vacating the subject lot, Sps. Gurieza defied Bonifacio‗s demand and
asserted their ownership over the same.1âwphi1 Moreover, they even challenged
Danao to go to the courts to have them removed from such lot. In effect, Sps. Gurieza
was able to unlawfully withhold possession of the subject lot from Bonifacio.

Lastly, Bonifacio, through Danao, made his final demand to Sps. Gurieza on January
14, 2008, as evidenced by a Certificate to File Action issued by the Barangay Captain of
the area where the subject lot was located, stating that the Sangguniang Barangay had
tried to settle the dispute between the parties but failed to do so,22 and filed his
complaint on June 24, 2008, or within the one (1) year period from his last demand.23
In view of the foregoing, the Court thus holds that the CA erred in dismissing Bonifacio'
s Complaint for Unlawful Detainer and Damages against Sps. Gurieza. Perforce, a
reversal of its ruling is proper.

102
LIM VS. LIGON

G.R. No. 183589


June 25, 2014

Facts:

Sometime in 1970, one Tomas Fernandez filed a Free Patent Application over a parcel
of land situated in Sitio Kuala, Barangay Wawa, Nasugbu, Batangas with an area
9,[478] sq. meters. After the death of Tomas Fernandez, his son Felicisimo pursued the
application and on 25 April 1984. In 1985, the spouses Isaac and Concepcion Ronulo
asked the assistance of the Office of the President and requested investigation of their
claim that a parcel of land containing 1,000 square meters which they have been
occupying since the 1950s was included in the approved survey plan PSU-04-008565 in
the name of Tomas Fernandez. The plan PSU-04-008565 approved in the name of
Tomas Fernandez is hereby, as it is, ordered CANCELLED and whatever amount paid
on account thereof forfeited in favor of the Government.The above order was appealed
by Felicisimo Fernandez to the Office of the DENR Secretary. Spouses Danilo Ligon
and Generosa Vitug-Ligon purchased the subject property from Felicisimo Fernandez
and introduced improvements thereon, including a beach house.defendant Lim filed a
complaint for forcible entry against the petitioners with the Municipal Trial Court of
Nasugbu, Batangas involving the subject property. The case was docketed as Civil
Case No. 1275. On May 26, 1997, the trial court rendered judgment (Exh. A-5) in favor
of private respondent and ordered petitioners to vacate the subject land.

On 10 April 2000, this Court denied plaintiffs‗ application for injunctive relief as a result
of which plaintiffs‗ beach house was demolished by the Branch Sheriff on the motion of
defendants. On 16 April 2000, plaintiffs filed a supplemental complaint for additional
damages as a result of the demolition of their beach house worth about P7 million.
Defendants did not answer the supplemental complaint despite being ordered to do so.

Issue:

Whether or not the finality of the judgment in the ejectment case served as res judicata
with respect to the issue of prior possession of the spouses ronulos (the predecessors-
in-interest of the petitioners)? Whether or not the court of appeals erred in affirming the
ownership ofthe respondents.

Ruling:

103
For a judgment to constitute res judicata, the following requisites must concur: x x x (a)
the former judgment was final; (b) the court that rendered it had jurisdiction over the
subject matter and the parties; (c) the judgment was based on the merits; and (d)
between the first and the second actions, there was an identity of parties, subject
matters, and causes of action.

Res judicata embraces two concepts: (1) bar by prior judgment and (2) conclusiveness
of judgment.

104
GABRIEL VS. CRISOLOGO

G.R. No. 204626


June 09, 2014

Facts:

Crisologo alleged, among others, that she was the registered owner of two parcels of
land. The properties were covered by an Assessment of Real Property; that the
payments of realty taxes on the said properties were updated; that sometime in 2006,
she discovered that petitioners unlawfully entered, occupied her properties by stealth,
by force and without her prior consent and knowledge, and constructed their houses
thereon; that upon discovery of their illegal occupation, her daughter, Atty. Carmelita
Crisologo, and Isican personally went to the properties and verbally demanded that
petitionersvacate the premises and remove their structures thereon; that the petitioners
begged and promised to buy the said properties for ?3,500.00 per square
meter;Petitioners countered that the titles of Crisologo were products of Civil
Registration Case No. 1, Record 211, which were declared void by the Supreme Court
in Republic v. Marcos. Petitioners had been in open, actual, exclusive, notorious,
uninterrupted, and continuous possession of the subject land, in good faith; and that
Crisologo was never in prior possession and had no valid title over the subject land.

Issue:

Whether the honorable court of appeals gravely erred in ruling that respondent‗s
supposed possession over the subject lots should be preferred despite the nature or
condition of the property as part of the public domain.

Ruling:

At any rate, petitioners, as private individuals, are not the proper parties to question the
status of the respondent‗s registered titles. Section 6 of P.D. No. 127114 expressly
states that the ―Solicitor General shall institute such actions or suits as may be
necessary to recover possession of lands covered by all void titles not validated under
this Decree.” The respondent’s certificates of title give her the better right to possess the
subject parcels of land It is settled that a Torrens title is evidence of indefeasible title to
property in favor of the person in whose name the title appears. It is conclusive
evidence with respect to the ownership of the land described therein. It is also settled
that the titleholder is entitled to all the attributes of ownership of the property, including
possession.

105
As a holder of a Torrens certificate of title, the law protects Crisologo from a collateral
attack on the same. Section 48 of P.D. No. 1529, otherwise known as the Property
Registration Decree, provides that a certificate of title cannot be the subject of a
collateral attack. As the lawful possessor, the respondent has the right to eject the
petitioners

106
SUAREZ VS. EMBOY

G.R. No. 187944


12 March 2014

Facts:

A parcel of land was partitioned into 5 among the heirs of the Carlos and Asuncion. Lot
No. 1907-A-2 was occupied by Felix and Marilou Emboy, who were claiming that they
inherited it from their mother Claudia Emboy, who inherited it from her parents Carlos
and Asuncion. Felix and Marilou were asked by their cousins to vacate Lot No. 1907-A-
2 and transfer to Lot No. 1907-A-5. They refused to comply and insisted that Claudia's
inheritance pertained to Lot No. 1907-A-2.

In 2004, Felix and Marilou received a demand letter from Carmencita requiring them to
vacate the lot and informed them that she had already purchased the lot from the
former's relatives. Felix and Marilou did not heed the demand so Carmencita filed
before the MTCC a complaint against unlawful detainer against them. Felix and Marilou
argued that the complaint for unlawful detainer was fundamentally inadequate. There
was practically no specific allegation as to when and how possession by tolerance of
them began.

Issue:

Whether or not the complaint for unlawful detainer was inadequate.

Ruling:

In a complaint for unlawful detainer, the following requisites must be alleged:


(1) initially, possession of property by the defendant was by contract with or by
tolerance of the plaintiff;
(2) eventually, such possession became illegal upon notice by plaintiff to defendant of
the termination of the latter‗s right of possession;
(3) thereafter, the defendant remained in possession of the property and deprived the
plaintiff of the enjoyment thereof; and
(4) within one year from the last demand on defendant to vacate the property, the
plaintiff instituted the complaint for ejectment.

In ejectment cases, it is necessary that the complaint must sufficiently show a statement
of facts to determine the class of case and remedies available to the parties. When the
complaint fails to state the facts constituting a forcible entry or unlawful detainer, as

107
where it does not state how entry was effected or how the dispossession started, the
remedy should either be an accion publiciana or accion reinvidicatoria. In this case, the
first requisite was absent. Carmencita failed to clearly allege and prove how Emboy
entered the lot and constructed a house upon it. She was also silent about the details
on who specifically permitted Emboy to occupy the lot, and how and when such
tolerance came about. Hence, the complaint should not have been for unlawful detainer
and the CA did not commit an error in dismissing Carmencita's complaint.

108
DELA CRUZ VS. CAPCO

G.R. No. 176055


March 17, 2014

Facts:

On October 6, 2003, the spouses Dela Cruz filed a Complaint6 for Unlawful Detainer
against the spouses Capco before the Metropolitan Trial Court (MeTC) of Pateros. They
alleged that Teodora T. Concio (Teodora), mother of petitioner Amelia Concio-Dela
Cruz (Amelia), acquired ownership over a piece of land by virtue of a Decision dated
October 3, 1983 rendered by the RTC of Pasig, Branch 151 in Land Registration Case
No. 9511. The said property was eventually registered in her name under Transfer
Certificate of Title (TCT) No. 31873. Teodora, out of neighborliness and blood
relationship, tolerated the spouses Capco‗s occupation thereof.Subsequently, the
subject property was conveyed to the spouses Dela Cruz. Intending to construct a
house thereon and utilize the space for their balut and salted eggs business, the
spouses Dela Cruz thus demanded that the spouses Capco vacate the property. As the
spouses Capco refused, the matter was brought before the Barangay Lupon for
conciliation wherein several meetings were held but to no avail.7 Hence, the said
Complaint.

Spouses Capco asserted that they have all the rights to occupy the subject property
since respondent Rufino Capco (Rufino) is an heir of its true owner. In fact, they
established their balutan business and built their house thereon as early as 1947. By
way of counterclaim, the spouses Capco prayed that the spouses Dela Cruz be ordered
to pay them exemplary damages, attorney‗s fees and litigation expenses.

Issue:

Whether or not the spouses have the right over questioned lot.

Ruling:

The spouses Capco appear to have acknowledged the fact that the spouses Dela Cruz
owned the lot that they are occupying. As shown by the records during the first meeting
before the Barangay Lupon, respondent Marty C. Capco asked Amelia if the latter could
just sell the lot to them [the spouses Capco] so that their business built thereon would
not suffer.It is thus clear that the lot being occupied by the spouses Capco is the same
lot over which the spouses Dela Cruz claim to have a better right to possess. Contrary
therefore to the CA‗s pronouncement, there is no need to physically determine the

109
extent of the land covered by T.C.T. No. 31873. The spouses Dela Cruz are able to
establish by preponderance of evidence that they are the rightful possessors of the
property.

It has repeatedly been emphasized that when the property is registered under the
Torrens system, the registered owner's title to the property is presumed legal and
cannot be collaterally attacked, especially in a mere action for unlawful detainer. It has
even been held that it does not even matter if the party's title to the property is
questionable.

110
RIVERA-CALINGASAN VS. RIVERA

G.R. No.171555
17 April 2013

Facts:

During their lifetime, respondent Wilfredo Rivera and his wife, Loreto Inciong, acquired
several parcels of land in Lipa City. About eleven (11) years later, or on March 29, 1993,
Loreto‗s heirs executed an extrajudicial settlement of her one-half share of the conjugal
estate, adjudicating all the properties in favor of Evangeline and Brigida Liza; Wilfredo
waived his rights to the properties, with a reservation of his usufructuary rights during
his lifetime.8 On September 23, 1993, the Register of Deeds of Lipa City, Batangas
cancelled TCT Nos. T-22290 and T-30557 and issued TCT Nos. T-87494 and T-87495
in the names of Evangeline and Brigida Liza, with an annotation of Wilfredo‗s
usufructuary rights.9 Almost a decade later, or on March 13, 2003,10 Wilfredo filed with
the Municipal Trial Court in Cities (MTCC) of Lipa City a complaint for forcible entry
against the petitioners and Star Honda, Inc., docketed as Civil Case No. 0019-03.
Wilfredo claimed that he lawfully possessed and occupied the two (2) parcels of land
located along C.M. Recto Avenue, Lipa City, Batangas, covered by TCT Nos. T-87494
and T-87495, with a building used for his furniture business. Taking advantage of his
absence due to his hospital confinement in September 2002, the petitioners and Star
Honda, Inc. took possession and caused the renovation of the building on the property.
In December 2002, the petitioners and Star Honda, Inc., with the aid of armed men,
barred him from entering the property.11 Both the petitioners and Star Honda, Inc.
countered that Wilfredo voluntarily renounced his usufructuary rights in a petition for
cancellation of usufructuary rights dated March 4, 1996,12 and that another action
between the same parties is pending with the RTC of Lipa City, Branch 13 (an action for
the annulment of the petition for cancellation of usufructuary rights filed by Wilfredo),
docketed as Civil Case No. 99-0773.

Issue:

The case presents to us the issue of who, between the petitioners and Wilfredo, had
been in prior physical possession of the property.

Ruling:

The petitioners‗ claim of physical possession cannot find support in the March 11, 2003
order32 of the RTC of Lipa City, Branch 13, in Civil Case No. 99-0773 stating that the

111
petitioners ―have been occupying the premises since 1997. We note that the order
was a mere interlocutory order on Wilfredo‗s motion for the issuance of a cease and
desist order. An interlocutory order does not end the task of the court in adjudicating the
parties' contentions and determining their rights and liabilities against each other. ―[I]t
is basically provisional in its application.‖33 It is the nature of an interlocutory order that
it is subject to modification or 96 reversal that the result of further proceedings may
warrant. Thus, the RTC‗s pronouncement on the petitioners‗ occupation ―since 1997‖
is not res judicata on the issue of actual physical possession.

The death of Wilfredo introduces a seeming complication into the case and on the
disposition we shall make. To go back to basics, the petition before us involves the
recovery of possession of real property and is a real action that is not extinguished by
the death of a party. The judgment in an ejectment case is conclusive between the
parties and their successors-ininterest by title subsequent to the commencement of the
action; hence, it is enforceable by or against the heirs of the deceased.

112
TOLENTINO VS. LAUREL

G.R. No. 181368


22 February 2012

Facts:

Respondents alleged that they are the registered owners of a parcel of land situated in
Barangay Balugo, Tagkawayan, Quezon, with an area of 1,056,275 square meters. For
several years, petitioners have been in actual possession of the western portion of the
said property with a total area of 620,000 square meters which they tried to develop into
fishponds. In the years 1993 and 1994, respondents informed petitioners, through
Gustavo C. Tolentino, Sr. (Gustavo) who was then representing them, that the area they
are occupying was inside the respondents' property and, therefore, they should vacate
and leave the same. Gustavo, however, asked for time to verify respondents' claim.
Petitioners continued to develop the area they were occupying into fishponds, thereby
manifesting their unwillingness to vacate the premises and restore the possession
thereof in favor of respondents.

On August 27, 1996, petitioners were declared in default, for failure to appear at the
pre-trial conference. However, the trial court set aside the default order and reset the
pre-trial conference. Despite several resetting of the pre-trial conference of which
petitioners were notified, petitioners failed to appear. Hence, on March 21, 2000, the
trial court issued an Order allowing respondents to present their evidence ex parte,
instead of declaring petitioners in default.

Issue:

Whether or not accion publiciana was the proper action to be instituted in this case.

Ruling:

Petitioners' attack on the legality of TCT No. T-43927, issued in the name of
respondents, is incidental to their quest to defend their possession of the property in an
accion publiciana, not in a direct action whose main objective is to impugn the validity of
the judgment granting the title. To permit a collateral attack on the title, such as what
petitioners attempt, would reduce the vaunted legal indefeasibility of a Torrens title to
meaningless verbiage.

Respondents' title over the subject property is evidence of their ownership thereof. It is a
fundamental principle in land registration that the certificate of title serves as evidence

113
of an indefeasible and incontrovertible title to the property in favor of the person whose
name appears therein. It is conclusive evidence with respect to the ownership of the
land described therein. It is also settled that the titleholder is entitled to all the attributes
of ownership of the property, including possessin. Thus, the Court held that the age-old
rule is that the person who has a Torrens title over a land is entitled to possession
thereof.

114
ROGELIO J. JAKOSALEM and GODOFREDO B. DULFO VS. ROBERTO S.
BARANGAN

G.R. No. 175025


February 15, 2012

Facts:

On August 13, 1966, respondent Col. Roberto S. Barangan (respondent Barangan)


entered into a Land Purchase Agreement with Ireneo S. Labsilica of Citadel Realty
Corporation whereby respondent Barangan agreed to purchase on installment a 300
square meter parcel of land, located in Antipolo, Rizal. Upon full payment of the
purchase price, a Deed of Absolute Sale was executed on August 31, 1976 in his favor.
Consequently, the old title was cancelled and a new one was issued in his name. Since
then, he has been dutifully paying real property taxes for the said property. He was not,
however, able to physically occupy the subject property because as a member of the
Philippine Air Force, he was often assigned to various stations in the Philippines.
On December 23, 1993, when he was about to retire from the government service,
respondent Barangan went to visit his property, where he was planning to build a
retirement home. It was only then that he discovered that it was being occupied by
petitioner Godofredo Dulfo (petitioner Dulfo) and his family.

On February 4, 1994, respondent Barangan sent a letter to petitioner Dulfo demanding


that he and his family vacate the subject property within 30 days. In reply, petitioner
Atty. Rogelio J. Jakosalem (petitioner Jakosalem), the son-in-law of petitioner Dulfo,
sent a letter claiming ownership over the subject property.

On February 19, 1994, respondent Barangan filed with Barangay San Luis, Antipolo,
Rizal, a complaint for Violation of Presidential Decree No. 772 or the Anti-Squatting Law
against petitioners. No settlement was reached; hence, the complaint was filed before
the Prosecutor‗s Office of Rizal. The case, however, was dismissed because the issue
of ownership must first be resolved in a civil action.

On May 28, 1994, respondent Barangan commissioned Geodetic Engineer Lope C.


Jonco (Engr. Jonco) of J. Surveying Services to conduct a relocation survey of the
subject property based on the technical description appearing on respondent
Barangan‗s TCT. The relocation survey revealed that the property occupied by
petitioner Dulfo and his family is the same property covered by respondent Barangan‗s
title.

115
On November 17, 1994, respondent Barangan filed a Complaint for Recovery of
Possession against petitioners Dulfo and Jakosalem with the Regional Trial Court
(RTC) Antipolo City. Respondent Barangan prayed that petitioners Dulfo and Jakosalem
be ordered to vacate the subject property and pay a monthly rental of P3,000.00 for the
use and occupancy of the subject property from May 1979 until the time the subject
property is vacated, plus moral and exemplary damages and cost of suit.

In their Answer with Counterclaim, petitioners Dulfo and Jakosalem claimed that the
subject property was assigned to petitioner Jakosalem by Mr. Nicanor Samson
(Samson); that they have been in possession of the subject property since May 8, 1979;
and that the property covered by respondent Barangan‗s title is not the property
occupied by petitioner Dulfo and his family.

Issues:
1. Whether or not respondent barangan is entitled to possession of the subject
property?
2. Whether or not prescription/laches may apply in order for petitioner to retain
possession of the subject property?

Ruling:

Article 434 of the Civil Code provides that ―[i]n an action to recover, the property must
be identified, and the plaintiff must rely on the strength of his title and not on the
weakness of the defendant‗s claim.‖ In other words, in order to recover possession, a
person must prove (1) the identity of the land claimed, and (2) his title.
In this case, respondent Barangan was able to prove the identity of the property and his
title. To prove his title to the property, he presented in evidence the following
documents: (1) Land Purchase Agreement; (2) Deed of Absolute Sale; (3) and a
Torrens title registered under his name.

Neither is there any discrepancy between the title number stated in the Land Purchase
Agreement and the Deed of Absolute Sale. As correctly found by the CA, the title stated
in the Deed of Absolute Sale, is the title stated in the Land Purchase Agreement.
Hence, both TCTs pertain to the same property.

Finally, as to the issue of laches and prescription, we agree with the CA that these do
not apply in the instant case. Jurisprudence consistently holds that prescription and
laches can not apply to registered land covered by the Torrens system because under
the Property Registration Decree, no title to registered land in derogation to that of the
registered owner shall be acquired by prescription or adverse possession.

116
SPOUSES ELEGIO CAÑEZO AND DOLIA CAÑEZO VS. SPOUSES APOLINARIO
AND CONSORCIA L. BAUTISTA

G.R. NO. 170189


September 1, 2010

Facts:

Spouses Elegio and Dolia Cañezo (appellees) are the registered owners of a parcel of
land with an area of 186 square meters, covered by TCT No. 32911. Whereas, Spouses
Apolinario and Consorcia Bautista (appellants) are the registered owners of a parcel of
land, containing an area of 181 square meters, covered by TCT No. 31727. Both
parcels of land are located at Coronado Heights, Barangka Ibaba, Mandaluyong City
and registered with the Registry of Deeds of Mandaluyong City. Appellants‗ lot is
adjacent to that of appellees. Sometime in 1995, appellees started the construction of a
building on their lot. During the construction, appellees discovered that their lot was
encroached upon by the structures built by appellants without appellees‗ knowledge and
consent. Three surveys were conducted which confirmed the fact of encroachment.
However, despite oral and written demands, appellants failed and refused to remove the
structures encroaching appellees‗ lot. Spouses Cañezo filed their complaint for the
issuance of a writ of demolition with damages on 13 April 2000. In an Order dated 15
August 2000, the trial court declared the spouses Bautista in default for failure to
answer within the reglementary period. The trial court promulgated its Decision in favor
of the spouses Cañezo. The trial court found that the spouses Bautista built structures
encroaching on the land owned by the spouses Cañezo. The spouses Bautista also
refused to remove the structures and respect the boundaries as established by the
various surveyors. A referral to the Barangay Lupon failed to settle the controversy
amicably. The trial court thus ruled that the spouses Bautista are builders in bad faith,
such that spouses Cañezo are entitled to an issuance of a writ of demolition with
damages. On appeal, the appellate court rendered its Decision which reversed the Trial
Court‗s Decision. The appellate court ruled that since the last demand was made on 27
March 2000, or more than a year before the filing of the complaint, the spouses Cañezo
should have filed a suit for recovery of possession and not for the issuance of a writ of
demolition. A writ of demolition can be granted only as an effect of a final judgment or
order, hence the spouses Cañezo‗s complaint should be dismissed. The spouses
Cañezo failed to specify the assessed value of the encroached portion of their property.
Because of this failure, the complaint lacked sufficient basis to constitute a cause of
action. Finally, the appellate court ruled that should there be a finding of encroachment
in the action for recovery of possession and that the encroachment was built in good
faith, the market value of the encroached portion should be proved to determine the
appropriate indemnity.

117
Issue:

Whether or not petitioners should have filed recovery of possession and not writ of
demolition

Ruling:

The present case, while inaccurately captioned as an action for a ―Writ of Demolition
with Damages is in reality an action to recover a parcel of land or an accion
reivindicatoria under Article 434 of the Civil Code. Accion reivindicatoria seeks the
recovery of ownership and includes the jus utendi and the jus fruendi brought in the
proper regional trial court. Accion reivindicatoria is an action whereby plaintiff alleges
ownership over a parcel of land and seeks recovery of its full possession. The spouses
Cañezo were able to establish their ownership of the encroached property. Aside from
testimonial evidence, the spouses Cañezo were also able to present documentary and
object evidence which consisted of photographs, transfer certificates of title, and a
relocation survey plan. The relocation survey plan also corroborated Elegio Cañezo‗s
testimony on the reason for the spouses Bautista‗s attitude regarding the encroached
property. The relocation survey plan showed that the spouses Bautista‗s property
encroached upon that of the spouses Cañezo by 0.97 centimeters, while the spouses
Bautista‗s property was encroached upon by 1.01 centimeters by another landowner.
The testimony and the relocation survey plan both show that the spouses Bautista were
aware of the encroachment upon their lot by the owner of Lot 15 and thus they made a
corresponding encroachment upon the lot of the spouses Cañezo. This awareness of
the two encroachments made the spouses Bautista builders in bad faith. The spouses
Cañezo are entitled to the issuance of a writ of demolition in their favor and against the
spouses Bautista, in accordance with Article 450 of the Civil Code.

118
MODESTO PALALI VS. JULIET AWISAN

G.R. No. 158385


February 12, 2010

Facts:

Respondent Juliet Awisan claimed to be the owner of a parcel of land in Sitio


Camambaey, Tapapan, Bauko, Mountain Province, allegedly consisting of 6.6698
hectares and covered by Tax Declaration No. 147 in her name. On March 7, 1994, she
filed an action for quieting of title against petitioner Modesto Palali, alleging that the
latter occupied and encroached on the northern portion of her property and
surreptitiously declared it in his name for tax purposes. We shall refer to this land
occupied by petitioner, which allegedly encroached on the northern portion of
respondent‗s 6.6698-hectare land, as the "subject property". Respondent prayed to be
declared the rightful owner of the northern portion, for the cancellation of petitioner‗s tax
declaration, and for the removal of petitioner and his improvements from the property.
Respondent‗s (Plaintiff‗s) Allegations.

According to respondent, the 6.6698 hectare land was originally owned by her father,
Cresencio Cadwising. The latter testified that he and his wife were able to consolidate
ownership over the land by declaring them from public land as well as by purchasing
from adjoining landowners. He admitted including in his tax declaration a communal
sacred lot (patpatayan) even if he did not acquire free patent title over the same. As for
the properties he bought, these were generally purchased without any documentation,
save for two.

Cadwising also claimed having introduced improvements on the subject property as


early as the 1960s.11 The 6.6698 hectare land was mortgaged to the Development
Bank of the Philippines (DBP), which acquired it in the foreclosure sale. DBP then sold
the land to one Tico Tibong, who eventually donated the same to respondent.

Petitioner‗s (Defendant‗s) Allegations- In his defense, petitioner denied the


encroachment and asserted ownership over the subject property. He maintained that he
and his ancestors or predecessors-in-interest have openly and continuously possessed
the subject land since time immemorial. He and his siblings were born on that land and,
at that time, the area around the house was already planted with bananas, alnos, and
coffee. When his mother died, he buried her in the lot beside the house in 1975; while
his father was buried near the same plot in 1993. His own home had been standing on
the property for the past 20 years. Petitioner insisted that during this entire time, no one
disturbed his ownership and possession thereof.

119
Sometime in 1974, petitioner declared the said land in his name for taxation purposes.
The said Tax Declaration indicates that the property consists of 200 square meters of
residential lot and 648 square meters of rootcrop land (or a total of 848 square meters).
After due trial, the Regional Trial Court of Bontoc, Mountain Province, Branch 35,
dismissed22 the complaint. Respondent appealed the trial court‗s decision to the CA,
which reversed the same. The CA found that petitioner failed to prove actual possession
of the entire 6.6698 hectare land, which the CA believed to be the subject of the case.
According to the appellate court, petitioner was only able to prove actual occupation of
the portion where his house was located and the area below where he had planted fruit-
bearing plants.

Issue:

The issue in this case is who between the parties has the better right to the subject
property.

Ruling:

Having gone over the parties‗ evidence before the trial court, we find adequate support
for the trial court‗s ruling in favor of petitioner. The CA erred in reversing the trial court‗s
findings, particularly because, as discussed above, such reversal was premised on the
CA‗s erroneous understanding of the subject property.

From their consistent, unwavering, and candid testimonies, we find that petitioner‗s
grandfather Mocnangan occupied the land during the pre-war era. He planted camote
on the property because this was the staple food at that time. He then gave the subject
property to his daughter Tammam, while he gave a separate one to his son Pacolan
Mocnangan. In the 1960s, Tammam and her husband Palalag cultivated the land, built
a cogon home, and started a family there. Palalag introduced terraces and, together
with his sons, built earth fences around the property. Palalag‗s family initially planted
bananas, coffee, and oranges; they later added avocadoes, persimmons, and
pineapples. When Tammam and Palalag died, their son, petitioner herein, buried them
in the subject property and continued cultivating the land. He also constructed a new
home.

All told, we hold that as between the petitioner and the respondent, it is the petitioner
who has the better claim or title to the subject property. While the respondent merely
relied on her tax declaration, petitioner was able to prove actual possession of the
subject property coupled with his tax declaration. We have ruled in several cases that

120
possession, when coupled with a tax declaration, is a weighty evidence of ownership.39
It certainly is more weighty and preponderant than a tax declaration alone.

The preponderance of evidence is therefore clearly in favor of petitioner, particularly


considering that, as the actual possessor under claim of ownership, he enjoys the
presumption of ownership.40 Moreover, settled is the principle that a party seeking to
recover real property must rely on the strength of her case rather than on the weakness
of the defense.41 The burden of proof rests on the party who asserts the affirmative of
an issue. For he who relies upon the existence of a fact should be called upon to prove
that fact. Having failed to discharge her burden to prove her affirmative allegations, we
find that the trial court rightfully dismissed respondent‗s complaint.

121
NELSON LAGAZO VS. GERALD B. SORIANO and GALILEO B. SORIANO

G.R. No. 170864


February 16, 2010

Facts:

On January 16, 2001, respondents filed with the Municipal Trial Court of Tabuk, Kalinga
(MTC), a complaint for Forcible Entry with Application for Termporary Restraining Order
and a Writ of Preliminary Injunction and Damages against petitioner.Respondents
claimed they were the owners of a parcel of land covered by Original Certificate of Title
No. P-665, Lot No. 816, Pls-93 with an area of 58,171 square meters. They allegedly
acquired the same by purchase from their grandfather, Arsenio Baac, on September 10,
1998, but even prior thereto, they were already allowed by Arsenio Baac to cultivate
said land. They paid real property taxes for said property from 1990 to 1998 and had
been in actual possession from that time. However, on January 6, 2001, herein
petitioner allegedly unlawfully entered the property by means of force, stealth, and
strategy and began cultivating the land for himself.

On the other hand, petitioner insisted in his Answer that he, together with his mother,
brothers, and sisters, were the lawful owners of the land in question, being the legal
heirs of Alfredo Lagazo, the registered owner thereof. They denied that the subject land
was sold to Arsenio Baac, alleging instead that the agreement between Alfredo Lagazo
and Arsenio Baac was merely one of mortgage. Petitioner, likewise maintained that he
and his co-heirs had always been in possession of the disputed land.They allegedly
tried several times to redeem the property, but Baac increased the redemption price
from P10,000.00 to P100,000.00. This prompted them to bring the matter before the
Barangay Lupon of Balong, Tabuk, Kalinga, but no agreement was reached.

Issue:

Who had prior physical possession of the disputed land.

Ruling:

The Court held that: Ejectment proceedings are summary proceedings intended to
provide an expeditious means of protecting actual possession or right to possession of
property. Title is not involved. The sole issue to be resolved is who is entitled to the
physical or material possession of the premises or possession de facto. Regardless of
the actual condition of the title to the property, the party in peaceable quiet possession
shall not be thrown out by a strong hand, violence or terror. Neither is the unlawful

122
withholding of property allowed. Courts will always uphold respect for prior possession.
Thus, a party who can prove prior possession can recover such possession even
against the owner himself. Whatever may be the character of his possession, if he has
in his favor prior possession in time, he has the security that entitles him to remain on
the property until a person with a better right lawfully ejects him. To repeat, the only
issue that the court has to settle in an ejectment suit is the right to physical possession.
On the other hand, what petitioner's evidence sought to establish was that he and his
co-heirs continued to be the owners of the land, as his predecessor never intended to
sell the property to Arsenio Baac, the true agreement being only one of a mortgage.
Petitioner never established the fact of his physical possession over the disputed land.
Ironically, the most telling pieces of evidence that doomed petitioner's case were the
testimonies of petitioner himself and his sister, Marina Nialga. Verily, the foregoing
leaves no doubt in our mind that it was only on January 6, 2001 that petitioner, believing
himself to be the lawful owner of the disputed land, entered the same, thereby
disturbing respondents' peaceful possession thereof.

123
HEIRS OF PEDRO LAURORA VS. STERLING TECHNOPARK and SP PROPERTIES

G.R. No. 146815


April 9, 2003

Facts:

The spouses Laurora alleged that they were the owners of Lot 1315-G SWD-40763 of
the Yaptinchay Estate with an area of 39,771 sqmeter located in Carmona, Cavite.
Pedro Laurora planted trees and has possessed the land up to the filing of the
complaint. On Sept. 15, 1997 , Sterling Technopark III and S.P. Properties, Inc. through
their Engineer Bernie Gtachalian bulldozed and uprooted the trees and plants, and with
the use of armed men and by means of threats and intimidation succeeded in forcibly
ejecting Laurora spouses. As a result of their dispossession , Laurora suffered actual
damages in the amount of P3M and P10k attorney‗s fees.

The Sterling and S.P. Properties that the Laurora were/are not the owner of the property
because they dispose of it sometime in 1976 as shown by legal documents. On Apr. 2,
1969, the Land Authority issued an order of award in favour of the Laurora to buy the
subject lot from the government. In a documents presented it was the Laurora who
requested the transfer of the lot to Juan Manaig and it was favourably acted upon by the
DAR AND Manaig as a transferee buyer paid the required amount as evidenced by the
official receipt. And the transfer was made between Laurora and Manaig through
Kasulatan ng Bilihang Tuluyan or Deed of Absolute Sale including all the improvements
in favour of Manaig. The Sale was approved by the DAR. Thereafter the heirs of Manaig
sold the lot to Golden Mile Resources Development which likewise sold it to S.P.
Properties, Inc.

The MCTC dismissed the complaint and was elevated to RTC reversed the MCTC
Ruling that was reversed by CA and reinstated the Ruling of MCTC that the petitioner
has no evidence to support the claim of the petitioner of the prior possession of the
property. The CA was convinced with the evidence the LAURORA already sold the
property with the approval of DAR and subsequent possession of the land constituted
usurpation., which could not be source of right to occupy it. Being planters in bad faith ,
they had no right to be reimbursed for improvements of the land, in accordance with
Article 449 of the Civil Code. Hence, the petition.

Issues:

Whether the respondents have a valid and legal right to forcibly eject the petioners?

124
Ruling:

The SC held that the only issue here is the material possession of a real property.
Granting that the petitioners illegally entered and occupied the property, the
respondents has np right to take the law in their own hands and forcibly eject the
occupants therfrom. Notwithstanding the actual condition of the title, a person in
possession cannot be ejected by force even by the owners. If such illegal manner of
ejectment is employed, the party who proves prior possession, can recover possession
even from the owners.

The availment of the aforementioned remedies is the legal alternative to prevent


breaches of peace and criminal disorder from the use of force by claimants out to gain
possession.

125
PHILIPPINE NATIONAL BANK VS. COURT OF APPEALS

G.R. No. L-57757


31 August 1987

Facts:

Donata Montemayor is the administrator of 30 parcels of land of her late husband


Clodualdo Vitug who died intestate. Several portions of such land (TCT-2289, TCT-
2887, and TCT-2888) were mortgaged to PNB as security for certain loans availed by
Salvador Vitug, Salvador Jaramilla and Pedro Bacani, respectively. The loans were
never paid so the bank foreclosed all the mortgaged properties. PNB as the highest
bidder, purchased the lots, and subsequently sold the same to the Vitugs and the
Fajardos.

Meanwhile, Donata executed a contract of lease for a lot covered by TCT-2887-R to her
sons Pragmacio and Maximo Vitug. After a few years, the same brothers filed an action
for partition and reconveyance with damages in the CFI Pampanga against the PNB,
the Vitugs, the Fajardos, and Marcelo Mendiola, the special administrator of Donata‗s
intestate estate. They claimed that the 30 parcels of land form part of the conjugal
property of the spouses Donata and Clodualdo and they claim a share interest of 2/11
of 1/2 thereof. They assailed the mortgage of said properties to the PNB and the
subsequent public auction. They invoked the Vitug vs. Montemayor case where the SC
ruled on the conjugal nature of the 30 parcels of land.

Issues:
(1) Can the PNB rely merely on the Torrens Certificate of Title covering Donata‗s
properties for the processing of the respective mortgage loan applications?
(2) Is the earlier action for reconveyance and partition concerning the 30 lots real
actions and binding upon the PNB by virtue of the Vitug vs. Montemayor decision?

Ruling:

(1) Yes. The PNB had sufficient reason to rely on the Torrens Certificate of Title of the
mortgaged properties. The SC ruled that in processing the loan applications, the PNB
had the right to rely upon the face of the certificate of title. Clearly, it appears that
Donata (a widow) owns the properties and the PNB had no reason to doubt her status
and ownership. The PNB also found no liens or encumbrances covering the properties.
The clean facts reasonably cancel the need to make further inquiry.

126
The Court applied the well-known rule in jurisdiction that a person has a right to rely
upon the face of the Torrens Certificate of Title when dealing with a registered land. It is
not necessary to inquire beyond its face, except when such person has an actual
knowledge of facts and circumstances that would prompt him to inquire further. The
Court ruled that a Torrens title ―concludes all controversy over ownership of the land
covered by a final degree of registration and upon such registration, the person is
assured of ownership without going to court or sitting ―at the veranda of his house‖ to
avoid the fear of losing his land.

(2) No. The SC maintained that although actions for recovery of real property and for
partition are real actions, they are actions in personam which are binding only upon the
persons who are parties thereto. The PNB is not a party in the cited case and is
therefore not bound by the decision. In addition, there is no showing that the PNB was
aware of the case decision when it extended the mortgage loan involving the subject
properties. The court settled that if the PNB knew that said properties were conjugal, it
would not have approved the mortgage application without securing the consent of the
its co-owners. Therefore, the PNB is considered as a purchaser for value in good faith
when it sold the foreclosed properties at a public auction. The complaint is dismissed.

127
SPS. RONALD HUTCHISON AND VALENTINE NAVALLE-HUTCHISO VS. ENRIQUE
M. BUSCAS

459 SCRA 214


May 26, 2005

Facts:

On October 1, 1987, petitioner spouses purchased from V.A. Development Enterprises,


Inc. a 76,207-sq. m. land in San Juan, Lubao, Pampanga. They occupied the land after
a title was issued in their names.

On August 22, 1989, one Juanita Arrastia, the owner of a lot adjacent to that of
petitioner spouses, sold a portion of her land to respondent. The transaction, covering
7,581 sq. m., was evidenced by a Quitclaim Deed in favor of respondent. Respondent
occupied 1,100 sq. m. of his land. However, he failed to register the portion of the lot in
his name and title to the property remained in Arrastias name.

Respondent commissioned geodetic engineer Narciso Manansala and his survey


revealed that 6,471 sq. m. thereof was occupied by petitioner spouses.
Respondent sent a demand letter to petitioner spouses to vacate the encroached area.
Petitioner spouses refused and insisted that it was part of their land. Thus, respondent
filed a complaint for unlawful detainer

After trial, the MTC ruled in favor of respondent. However, on appeal, the RTC
dismissed the case. It ruled that MTC had no jurisdiction over the subject matter as it is
a boundary dispute and the proper action should have been an accion reinvindicatoria
before the RTC. Consequently, respondent filed a case for accion reinvindicatoria
against petitioner spouses with the RTC of Guagua, Pampanga

After trial, the RTC dismissed the complaint for lack of merit. It ruled that respondents
Quitclaim Deed was not sufficient proof of ownership; that respondent failed to clearly
identify the property claimed as it was only marked with an X sign, and; that petitioner
spouses, as registered owners, are entitled to possession of the disputed lot. On
appeal, the Court of Appeals reversed the decision of the trial court. It ruled that
respondent is entitled to possession of the disputed area as he was able to prove his
claim of ownership and the identity of the subject land.

128
Issue:

Whether or not the survey conducted was a sufficient proof of the identity of the land?

Ruling:

In civil cases, the law requires that the party who alleges a fact and substantially asserts
the affirmative of the issue has the burden of proving it.[5] This evidentiary rule is based
on the principle that the suitor who relies upon the existence of a fact should be called
upon to prove it.

Article 434 of the New Civil Code provides that to successfully maintain an action to
recover the ownership of a real property, the person who claims a better right to it must
prove two (2) things: first, the identity of the land claimed, and; second, his title thereto.
In the case at bar, we find that respondent failed to establish these two (2) legal
requirements.

The first requisite: the identity of the land. In an accion reinvindicatoria, the person who
claims that he has a better right to the property must first fix the identity of the land he is
claiming by describing the location, area and boundaries thereof. Anent the second
requisite, i.e., the claimants title over the disputed area, the rule is that a party can claim
a right of ownership only over the parcel of land that was the object of the deed.
Respondent sought to prove these legal requisites by anchoring his claim on the
Quitclaim Deed over a portion of land which was executed by Arrastia in his favor.

Annex A of the Deed, where the entire lot of Arrastia was particularly described and
where the specific portion of the property sold to respondent was marked, was not
presented by respondent at the trial. Neither can the surveys of the lots of petitioner
spouses and respondent prove the identity of the contested area and respondents
ownership thereof.

129
RUDY LAO VS. JAIME LAO

G.R. No. 149599


May 16, 2005

Facts:

The spouses Julian Lao and Anita Lao had constructed a building on a parcel of land in
Balasan, Iloilo City, owned by Alfredo Alava and covered by Transfer Certificate of
Title.They then occupied and leased the same without any written agreement thereon.

On May 12, 1982, Alfredo Alava, as lessor, and Anita Lao, as lessee, executed a
Contract of Lease over the said property. However, the contract of lease was not filed
with the Office of the Register of Deeds; hence, was not annotated at the dorsal portion
of the said title. Aside from Anita Lao, petitioner Rudy Lao also leased another portion of
the same property where he put up his business. In fact, Anita Laos building was
adjacent to where the petitioner conducted his business. At that time, the petitioner
knew that Anita Lao and her husband were the owners of the said building. He also
knew that she had leased that portion of the property, and that respondent Jaime Lao,
their son, managed and maintained the building, as well as the business thereon.
In the meantime, on March 21, 1995, the petitioner purchased the property from Alava,
and was later issued TCT No. 152,097 in his name.

On July 14, 1997, the petitioner filed a Complaint for Unlawful Detainer against the
respondent. The petitioner alleged, inter alia, that the respondent had occupied a
portion of his property without any lease agreement and without paying any rentals
therefor, and that the same was only through his tolerance and generosity.

During the preliminary conference, the respondent admitted that he was in actual
possession of the property. For his part, the petitioner admitted that he had been renting
another portion of the same property from Alava for years, and that his business
establishment and that of Anita Laos were adjacent to each other. He also admitted that
Anita Lao had been renting the said portion of the property for years before he bought it.
MCTC rendered judgment in favor of the petitioner and against the respondent. The
RTC also affirmed the decision. On appeal to the CA, the decision of the RTC was
reversed.

Issue:

Whether or not unlawful detainer is proper?

130
Ruling:

No. The records in this case show that the respondent has been in possession of the
property subject of the complaint not by mere tolerance or generosity of the petitioner,
but as the manager of his mother, Anita Lao, who conducted her business in the
building/warehouse which stood on a portion of the property leased from Alava, the
former owner. Contrary to the petitioners claim, the respondents possession of the
property was in behalf of his mother, the lessee thereof, and not in his own right,
independently of that of his mother. The petitioner cannot feign ignorance of the
existence of the lease of the subject property by Anita Lao, the existence of the building
and her business thereon, and the fact that the respondent managed his mothers
building and business. It must be stressed that during the preliminary conference of the
parties before the MCTC, the petitioner admitted his knowledge of the foregoing facts.
While it is true that the contract of lease between Alava and Anita Lao was not filed in
the Office of the Register of Deeds and annotated at the dorsal portion of the petitioners
title over the property, nevertheless, the petitioner was bound by the terms and
conditions of the said contract of lease. The lease, in effect, became a part of the
contract of sale.

131
RENE GANILA VS. HON. COURT OF APPEALS

G.R. No. 150755


June 28, 2005

Facts:

Private respondent Violeta Herrera filed ejectment complaints before the MCTC. Private
respondent alleged that she owns Lot 1227 of the Cadastral Survey, with an area of
43,210 square meters; that she inherited the lot from her parents; and that she only
tolerated petitioners to construct residential houses or other improvements on certain
portions of the lot without rental. Sometime in September or October 1996, private
respondent demanded that the petitioners vacate the lot and remove their houses and
other improvements thereon. Petitioners refused, despite offer of money by way of
assistance to them.

In their Answers, eight of the petitioners claimed that Lot 1227 was formerly a shoreline
which they developed when they constructed their respective houses. Another eight
maintained that their houses stood on Lot 1229 of the Cadastral Survey of Jordan,
Guimaras. The other three asserted that Lot 1227 is a social forest area.

The MCTC rendered in favor of the plaintiff and ordered defendant to vacate the lot. The
decision however was revered by the RTC. The court ruled that Private respondents
position paper, affidavit and tax declaration supported her allegations. In addition, the
commissioners report and sketch plan showed that indeed petitioners occupy Lot 1227.
On the other hand, according to the RTC, the petitioners failed to present evidence
which would show that they are entitled to possess the lot.

The 19 petitioners, who were ordered to vacate the lot, filed a joint petition for review
with the Court of Appeals. The Court of Appeals, however, affirmed the factual findings
and conclusions arrived at by the trial courts and denied the amended petition for lack of
merit.

Issue:

Whether or not unlawful detainer is proper?

Ruling:

Yes. Petitioners insist that private respondent should have filed an action to recover
possession de jure, not a mere complaint for ejectment, for two reasons. One, they

132
possessed Lot 1227 in good faith for more than 30 years in the concept of owners. And
two, there was no withholding of possession since private respondent was not in prior
possession of the lot.

While petitioners assert that this case involves only deprivation of possession, they
confuse the remedy of an action for forcible entry with that of unlawful detainer. In
unlawful detainer, prior physical possession by the plaintiff is not necessary. It is enough
that plaintiff has a better right of possession. Actual, prior physical possession of a
property by a party is indispensable only in forcible entry cases. In unlawful detainer
cases, the defendant is necessarily in prior lawful possession of the property but his
possession eventually becomes unlawful upon termination or expiration of his right to
possess.

In this case for ejectment, private respondents allegations sufficiently present a case of
unlawful detainer. She alleged that (1) she owns Lot 1227; (2) she tolerated petitioners
to construct their houses thereon; (3) she withdrew her tolerance; and (4) petitioners
refused to heed her demand to vacate the lot. The Complaints were also filed within one
year from the date of her demand. The cause of action for unlawful detainer between
the parties springs from the failure of petitioners to vacate the lot upon lawful demand of
the private respondent. When they refused to vacate the lot after her demand,
petitioners continued possession became unlawful. Her complaint for ejectment against
respondent, to put it simply, is not without sufficient basis.

What really distinguishes an action for unlawful detainer from a possessory action
(accion publiciana) and from a reinvindicatory action (accion reinvindicatoria) is that the
first is limited to the question of possession de facto. An unlawful detainer suit (accion
interdictal) together with forcible entry are the two forms of an ejectment suit that may
be filed to recover possession of real property. Aside from the summary action of
ejectment, accion publiciana or the plenary action to recover the right of possession and
accion reinvindicatoria or the action to recover ownership which includes recovery of
possession, make up the three kinds of actions to judicially recover possession. Denied
for lack of merit.

133
RUBEN SANTOS VS. SPOUSES TONY AYON and MERCY AYON

G.R. No. 137013


May 6, 2005

Facts:

The petition alleges that on November 6, 1996, Ruben Santos, petitioner, filed with the
Municipal Trial Court in Cities a complaint for illegal detainer against spouses Tony and
Mercy Ayon.

Petitioner averred that he is the registered owner of three lots situated Davao City,
covered by Transfer Certificates of Title. Respondent spouses are the registered
owners of an adjacent parcel of land. The previous occupant of this property built a
building which straddled both the lots of the herein parties.

Petitioner further alleged in his complaint that in 1985, when he bought the three lots, he
informed respondents that the building occupies a portion of his land. However, he
allowed them to continue using the building. But in 1996, he needed the entire portion of
his lot, hence, he demanded that respondents demolish and remove the part of the
building encroaching his property and turn over to him their possession. But they
refused.

Respondents sought a dismissal of this case on the ground that the court has no
jurisdiction over it since there is no lessor-lessee relationship between the parties.
Respondents denied they were occupying petitioners property by mere tolerance,
claiming they own the contested portion and have been occupying the same long before
petitioner acquired his lots in 1985.

MTCC rendered its decision in favor of petitioner. The Regional Trial Court RTC
affirmed in toto the MTCC judgment. The Court of Appeals through a petition for review,
held that petitioners proper remedy should have been an accion publiciana before the
RTC, not an action for unlawful detainer.

Issue:

Whether the Court of Appeals committed a reversible error of law in holding that
petitioners complaint is within the competence of the RTC, not the MTCC?

134
Ruling:

Yes. It is an elementary rule that the jurisdiction of a court over the subject matter is
determined by the allegations of the complaint and cannot be made to depend upon the
defenses set up in the answer or pleadings filed by the defendant. This rule is no
different in an action for forcible entry or unlawful detainer. All actions for forcible entry
or unlawful detainer shall be filed with the proper Metropolitan Trial Courts, the
Municipal Trial Courts and the Municipal Circuit Trial Courts, which actions shall include
not only the plea for restoration of possession but also all claims for damages and costs
arising therefrom. The said courts are not divested of jurisdiction over such cases even
if the defendants therein raises the question of ownership over the litigated property in
his pleadings and the question of possession cannot be resolved without deciding the
issue of ownership.

Section 1, Rule 70 on forcible entry and unlawful detainer of the 1997 Rules of Civil
Procedure, as amended, reads:
Section 1. Who may institute proceedings, and when. Subject to the provisions of the
next succeeding section, a person deprived of the possession of any land or building by
force, intimidation, threat, strategy, or stealth, or a lessor, vendor, vendee, or other
person against whom the possession of any land or building is unlawfully withheld after
the expiration or termination of the right to hold possession, by virtue of any contract,
express or implied, or the legal representatives or assigns of any such lessor, vendor,
vendee or other person may, at any time within one (1) year after such unlawful
deprivation or withholding of possession, bring an action in the proper Municipal Trial
Court against the person or persons unlawfully withholding or depriving of possession,
or any person or persons claiming under them, for the restitution of such possession,
together with damages and costs.

Under section1, Rule 70 of the Rules of Civil Procedure, there are two entirely distinct
and different causes of action, to wit: (1) a case for forcible entry, which is an action to
recover possession of a property from the defendant whose occupation thereof is illegal
from the beginning as he acquired possession by force, intimidation, threat, strategy or
stealth; and (2) a case for unlawful detainer, which is an action for recovery of
possession from defendant whose possession of the property was inceptively lawful by
virtue of a contract (express or implied) with the plaintiff, but became illegal when he
continued his possession despite the termination of his right thereunder.

135
HEIRS OF DEMETRIO MELCHOR VS. JULIO MELCHOR

G.R. No. 150633


November 12, 2003
Facts:

Petitioners, who are the heirs of DEMETRIO MELCHOR, claim to be the owners, by
way of succession, of the subject property allegedly in possession of respondent JULIO
MELCHOR. The subject property is a portion of the twenty (20) hectares of land
registered in the name of PEDRO MELCHOR, evidenced by Original Certificate of Title
No.I-6020 of the Registry of Deeds for Isabela. The said property was purchased by the
late DEMETRIO MELCHOR from PEDRO MELCHOR, the deceased father of herein
respondent JULIO MELCHOR. Since February 14, 1947 up to the present, petitioners
further alleged that respondent has been occupying the subject property and has been
harvesting crops thereon and using it for grassing cows and carabaos.

A demand letter dated August 21, 1999 was allegedly sent by the petitioners to the
respondent, demanding him to vacate and surrender the said property, but the latter
refused. Thus, Petitioners filed against respondent a complaint for ejectment before the
MTC.

For his part, the defendant (now respondent) principally raised the matter of ownership
by alleging affirmative/special defenses, among others, that the parcel of land in
possession of the defendant is registered in the name of ANTONIA QUITERAS, the
deceased mother of the defendant, and that the same property is now owned by the
defendant and his three (3) sisters and one (1) brother, having inherited the same from
their late mother, ANTONIA QUITERAS.

The MTC rendered in favor of the respondent. It held that There is no allegation that
plaintiffs have been deprived of the possession of the land by force, intimidation, threat,
strategy or stealth. The decision was affirmed by the RTC.

The CA also sustained the RTC and further held that the proper remedy should have
been a plenary action for recovery of possession, not a summary action for ejectment.

Issue:
Whether or not the complaint sufficiently alleged a cause of action for unlawful detainer?

136
Ruling:

No. Petitioners maintain that while the Complaint does not support a cause of action for
forcible entry, the allegations therein certainly indicate one for unlawful detainer. They
add that they did not commit any jurisdictional infirmity in failing to allege prior physical
possession, because that fact is not an element of unlawful detainer. We do not agree.
Even if petitioners may be correct in saying that prior physical possession by the plaintiff
need not be alleged in an action for unlawful detainer, the absence of such possession
does not ipso facto make their Complaint sufficient to confer jurisdiction on the MTC.

In ejectment cases, the jurisdiction of the court is determined by the allegations of the
complaint. The test for determining the sufficiency of those allegations is whether,
admitting the facts alleged, the court can render a valid judgment in accordance with the
prayer of the plaintiff.

A review of the Second Amended Complaint of petitioners discloses these pertinent


allegations: the absolute owner of the subject land was their father, Demetrio Melchor,
who bought it on February 14, 1947 from respondents father, Pedro Melchor; being the
heirs of Demetrio Melchor, petitioners became the owners of the property by reason of
succession; as such, they sent a formal demand letter to respondent, who had been
using the property since February 14, 1947, for grazing cows and carabaos and for
planting crops; and in that letter, they asked him to vacate and surrender the property,
but he failed to do so.

It is clear from the foregoing that the allegations in the Complaint failed to constitute a
case for either forcible entry or unlawful detainer.
As correctly held by the appellate court, [f]orcible entry must be ruled out as there was
no allegation that the petitioners were denied possession of the subject property
through any of the means stated in Section 1, Rule 70 [of the Rules of Court].

Neither was unlawful detainer satisfactorily alleged. In determining the sufficiency of a


complaint therefor, it is not necessary to employ the terminology of the law. Not averred
in this case, however, were certain essential facts such as how entry was effected, or
how and when dispossession started. Petitioners merely alleged their ownership of the
land, which had supposedly been possessed by respondent since 1947. There was no
allegation showing that his possession of it was initially legal -- by virtue of a contract,
express or implied -- and that it became illegal after the expiration of his right to
possess.

137
PABLO D. ACAYLAR, JR., VS. DANILO G. HARAYO

G.R. No. 176995


July 30, 2008
Facts:

In his Complaint filed with the MTCC, and docketed as Civil Case No. 622, respondent
alleged that he acquired the subject property from the spouses Pablo Acaylar, Sr., and
Zoila Dangcalan Acaylar (the spouses Acaylar) by virtue of a Deed of Sale executed on
14 September 2004. On the same day, respondent took possession of the subject
property. On 19 September 2004, one of the spouses Acaylars sons, the petitioner,
using strategy, intimidation, threats and stealth, entered the subject property, cut the tall
grasses in the coconut plantation therein, gathered the fallen coconuts and other fruits,
and pastured his cows and other animals thereon.

In his Answer, petitioner countered that the subject property claimed by respondent is a
portion of the entire property owned by petitioners parents, the spouses Acaylar, with a
total area of 59,775 square meters. Petitioner is in possession of his parents‗ entire
property since 1979 as administrator thereof. He built his house on the property and
farmed the land.

After trial, the MTCC rendered a Decision on 28 March 2005, awarding to respondent
the possession of the subject property. The MTCC gave credence to respondents claim
that he took immediate possession of the subject property after the execution of the
Deed of Sale but was ousted therefrom by petitioner who invoked the alleged authority
granted to him by Zoila Acaylar as the administrator of the unsold portion of her and her
husband‗s property.

The RTC affirmed the award of possession in favor of respondent after finding that the
appealed MTCC Decision was based on facts and law on the matter.
The CA dismissed the Petition for Review on Certiorari. The court said that it should
have been a Petition for Review under Rule 42 of the Revised Rules of Court

Issue:

Whether or not forcible entry is the proper case?

Ruling:

Under section 1, Rule 70 of the Revised Rule of Court, there are two entirely distinct
and different causes of action, to wit: (1) a case for forcible entry, which is an action to

138
recover possession of a property from the defendant whose occupation thereof is illegal
from the beginning as he acquired possession by force, intimidation, threat, strategy or
stealth; and (2) a case for unlawful detainer, which is an action for recovery of
possession from defendant whose possession of the property was inceptively lawful by
virtue of a contract (express or implied) with the plaintiff, but became illegal when he
continued his possession despite the termination of his right thereunder.

The distinctions between the two forms of ejectment suits, are: first, in forcible entry, the
plaintiff must prove that he was in prior physical possession of the premises until he was
deprived thereof by the defendant, whereas, in unlawful detainer, the plaintiff need not
have been in prior physical possession; second, in forcible entry, the possession of the
land by the defendant is unlawful from the beginning as he acquires possession thereof
by force, intimidation, threat, strategy or stealth, while in unlawful detainer, the
possession of the defendant is inceptively lawful but it becomes illegal by reason of the
termination of his right to the possession of the property under his contract with the
plaintiff; third, in forcible entry, the law does not require a previous demand for the
defendant to vacate the premises, but in unlawful detainer, the plaintiff must first make
such demand, which is jurisdictional in nature.

It bears to stress that it is the nature of defendants entry into the land which determines
the cause of action, whether it is forcible entry or unlawful detainer. If the entry is illegal,
then the action which may be filed against the intruder is forcible entry. If, however, the
entry is legal but the possession thereafter becomes illegal, the case is unlawful
detainer.

In this case petitioner was in peaceful possession of the subject property prior to its sale
to respondent. Even if petitioner was not authorized to possess the subject property as
administrator, his possession was not opposed and was, thus, tolerated by his parents.
Since petitioner was in prior physical possession of the subject property, respondent
has no cause of action against petitioner for forcible entry. Neither can we treat
respondents case against petitioner as one for unlawful detainer absent the
jurisdictional requirement of demand to vacate made upon petitioner.

139
HEIRS OF ANACLETO B. NIETO, ET AL. VS. MUNICIPALITY OF MEYCAUAYAN,
BULACAN, REPRESENTED BY MAYOR EDUARDO ALARILLA

G.R. No. 150654


December 13, 2007

Facts:

Anacleto Nieto was the registered owner of a parcel of land, consisting of 3,882 square
meters, situated at Poblacion, Meycauayan, Bulacan The property is being used by
respondent which constructed an extension of the public market therein. Upon
Anacleto‗s death on July 26, 1993, his wife and their three children collated all the
documents pertaining to his estate. When petitioners failed to locate the owner‗s
duplicate copy they filed a petition for the issuance of a second owner‗s copy but they
discovered that the missing copy of the title was in the possession of the respondent.
Consequently, petitioners withdrew the petition and demanded from respondent the
return of property and the certificate of title. Respondent did not comply with petitioners‗
demand.

On December 28, 1994, petitioners filed a complaint for recovery of possession and
damages against respondent. They averred that, in 1966, respondent occupied the
subject property by making it appear that it would expropriate the same. Respondent
then used the land as a public market site and leased the stalls therein to several
persons without paying Anacleto Nieto the value of the land or rent therefor. Petitioners
prayed that respondent be ordered to surrender to them the owner‗s copy of TCT No. T-
24.055 (M), vacate the property, and pay them the rents thereon from 1966 until the
date of the filing of the complaint. In its Answer, respondent alleged that the property
was donated to it and that the action was already time-barred because 32 years had
elapsed since it possessed the property.

On August 1, 1995, the RTC rendered a Decision dismissing the complaint as well as
respondent‗s counterclaims for damages. For lack of proof, the RTC disregarded
respondent‗s claim that Anacleto Nieto donated the property to it in light of the fact that
the title remained in the name of Anacleto.

Issue:

Whether or not the action of petitioner to recover possession of the property is already
barred by prescription

140
Ruling:

An action to recover possession of a registered land never prescribes in view of the


provision of Section 44 of Act No. 496 to the effect that no title to registered land in
derogation of that of a registered owner shall be acquired by prescription or adverse
possession. It follows that an action by the registered owner to recover a real property
registered under the Torrens System does not prescribe.

In this case, the Court noted that the certificate of title in the name of Anacleto Nieto
was found in respondent‗s possession but there was no evidence that ownership of the
property was transferred to the municipality either through a donation or by
expropriation, or that any compensation was paid by respondent for the use of the
property. Anacleto allegedly surrendered the certificate of title to respondent upon the
belief that the property would be expropriated. Absent any showing that this certificate
of title was fraudulently obtained by respondent, it can be presumed that Anacleto
voluntarily delivered the same to respondent. Anacleto‗s delivery of the certificate of title
to respondent could, therefore, be taken to mean acquiescence to respondent‗s plan to
expropriate the property, or a tacit consent to the use of the property pending its
expropriation. The Court has consistently held that those who occupy the land of
another at the latter‗s tolerance or permission, without any contract between them, are
necessarily bound by an implied promise that the occupants will vacate the property
upon demand.

The Supreme Court granted the petition. The decision of the Regional Trial Court is
reversed and set aside. Respondent is Ordered (a) to vacate and surrender peaceful
possession of the property to petitioners, or pay the reasonable value of the property;
(b) to pay P1,716,000.00 as reasonable compensation for the use of the property from
1966 until the filing of the complaint and P10,000.00 monthly rental thereafter until it
vacates the property, with 12% interest from the filing of the complaint until fully paid;
and (c) to return to petitioners the duplicate copy of TCT No. T-24.055 (M).

141
GLORIA S. DY VS. MANDY COMMODITIES CO., INC.

G.R. No. 171842


July 22, 2009

Facts:

This case has its origin in the contract entered into by the National Government with the
Philippine National Bank (PNB) on 9 June 1978, wherein the former leased in favor of
the latter the 21,727-square meter government-owned land located at Binondo, Manila.
The lease was good for 25 years which commenced on 1 August 1978 and was to
expire on 31 July 2003, renewable for the same period upon agreement of both parties.
On 17 October 1994, PNB sublet a portion of the subject land consisting of 8,530.l6
square meters to respondent Mandy Commodities Co., Inc., for a period corresponding
to PNB's contract with the National Government. When the expiration of the subject
lease contract was approaching, then Department of Environment and Natural
Resources (DENR) Secretary Heherson Alvarez issued a Memorandum Order dated 6
May 2002 initially approving the renewal of PNB's lease for another 25 years. Later, in a
Final Endorsement Secretary Alvarez had a change of heart and approved the renewal
of the lease in favor of PNB and included respondent as one of the sub-lessees. This
Final Endorsement, though, did not last long as the then new DENR Secretary, Elisea
Gozun, issued a Memorandum withdrawing the lease contract with PNB and,
consequently calling off the sub-lease contract with the respondent.

Since the subject lease was about to expire, the Land Management Bureau (LMB),
informed PNB that a take over team was created to effect repossession of the subject
property and requested the PNB to turn it over to the DENR upon the termination of the
lease contract. In order to avert the eventual take over, PNB commenced a complaint
for Injunction with prayer for the issuance of a Temporary Restraining Order or Writ of
Preliminary Injunction and damages alleging that the contract of lease between it and
the National Government had already been renewed by virtue of the Final Endorsement
of then Secretary Alvarez; hence, PNB's possession of the disputed property must be
respected by the LMB.

In the meantime, LMB granted petitioner Gloria Dy a provisional permit to occupy the
subject realty. Equipped with the provisional permit from the LMB, petitioner was able to
enter and install her own guards in the premises of the property on 10 October 2003.
Petitioner also posted notices announcing that all the tenants therein should secure
from her an authorization to enter the same. On 15 October 2003, respondent Mandy
Commodoties, being the sub-lessee, reacted to petitioner's intrusion on the subject
property by filing a complaint for Damages with prayer for injunction.

142
Issue:

Whether or not petitioner‗s unlawful detainer case is proper

Ruling:

In the case under consideration, the Court of Appeals found that respondent as sub-
lessee of the PNB was acting within its prerogatives as possessor when it filed the
forcible entry suit against petitioner. From 1994 until the controversy arose, respondent
was in peaceful possession of the property in question. The Court of Appeals even
pointed out that even when the LMB gained possession of the property on 29 August
2003, respondent was allowed to continue business within the premises. In contrast,
petitioner's possession was predicated on the provisional permit issued to her by LMB
and the 28 August 2003 Order of the Pasay City RTC in the PNB Injunction Case. It
must be noted that the said order directing the take over of the disputed property was
declared void by the Court of Appeals, even when it denied the propriety of the issuance
of a TRO in the PNB Injunction Case.

Considering that the possession of petitioner was declared void, and bearing in mind
that the validity of petitioner's provisional permit to occupy the property is yet to be
settled in the PNB Injunction Case, still pending in the Pasay City RTC, petitioner's
occupation thereof is without legal authority. Simply put, petitioner has no right to
occupy the property. In contrast, respondent's right to occupy it remains intact, since the
records of the case are barren of any indication that the National Government or the
PNB made a formal demand on the respondent to vacate said property. The way things
stand, respondent, whose prior possession over the property remains intact, has the
better right over it. Thus, when it filed the instant forcible entry case against petitioner
who forcibly took possession thereof on 7 November 2003, respondent was just
exercising its right.

143
ATTRIBUTES OF
OWNERSHIP

Right of Accession

144
VILMA G. ARRIOLA AND ANTHONY RONALD G. ARRIOLA VS. JOHN NABOR C.
ARRIOLA

G.R. No. 177703


January 28, 2008
Facts:

John Nabor C. Arriola filed a special civil action with the Regional Trial Court (RTC)
against petitioners for judicial partition of the properties of decedent Fidel Arriola (the
decedent Fidel). Respondent is the son of decedent Fidel with his first wife Victoria C.
Calabia, while petitioner Anthony is the son of decedent Fidel with his second wife,
petitioner Vilma. On February 16, 2004, the RTC rendered a decision ordering the
partition of the parcel of land covered by Transfer Certificate of Title No. 383714 left by
the decedent Fidel.

As the parties failed to agree on how to partition among them the land, respondent
sought its sale through public auction. Accordingly, the RTC ordered the public auction
of the subject land. The public auction sale was scheduled on May 31, 2003 but it had
to be reset when petitioners refused to include in the auction the house (subject house)
standing on the subject land. This prompted respondent to file with the RTC a motion
praying that petitioners be declared in contempt. The RTC denied the motion for the
reason that petitioners were justified in refusing to have the subject house included in
the auction. In its November 30, 2006 decision, the CA granted the Petition for Certiorari
to proceed with the public auction sale of the subject lot, including the house
constructed thereon.

Issue:

Whether or not the public auction should include the subject house

Ruling:

Under the provisions of the Civil Code, the subject house is deemed part of the subject
land. As the CA stated, in general, the right to accession is automatic (ipso jure),
requiring no prior act on the part of the owner or the principal. So that even if the
improvements including the house were not alleged in the complaint for partition, they
are deemed included in the lot on which they stand, following the principle of accession.
Consequently, the lot subject of judicial partition in this case includes the house which is
permanently attached thereto, otherwise, it would be absurd to divide the principal, i.e.,
the lot, without dividing the house which is permanently attached thereto.

145
Under Article 494 of the Civil Code, may, at any time, demand the partition of the
subject house. Therefore, respondent's recourse to the partition of the subject house
cannot be hindered, least of all by the mere technical omission of said common property
from the complaint for partition. That said notwithstanding, while the Court treats the
subject house as part of the co-ownership of the parties, it stop short of authorizing its
actual partition by public auction at this time. The family home - consisting of the subject
house and lot on which it stands - cannot be partitioned at this time, even if it has
passed to the co-ownership of his heirs, the parties herein. Decedent Fidel died on
March 10, 2003.Thus, for 10 years from said date or until March 10, 2013, or for a
longer period, if there is still a minor beneficiary residing therein, the family home he
constituted cannot be partitioned, much less when no compelling reason exists for the
court to otherwise set aside the restriction and order the partition of the property.

146
MAGDALENA T. VILLASI VS. SPOUSES FILOMENO GARCIA AND ERMELINDA
HALILI-GARCIA

G.R. No. 190106


January 15, 2014

Facts:

Sometime in 1990, petitioner Magdalena T. Villasi engaged the services of respondent


Fil-Garcia Construction, Inc. (FGCI) to construct a seven-storey condominium building
For failure of Villasi to fully pay the contract price despite several demands, FGCI
initiated a suit for collection of sum of money. Villasi filed an answer specifically denying
the material allegations of the complaint. Finding that FGCI was able to preponderantly
establish by evidence its right to the unpaid accomplishment billings, the RTC rendered
a decision in FGCI‗s favor. Elevated on appeal, the Court of Appeals reversed the
disquisition of the RTC and ruled that an overpayment was made by Villasi and thereby
directed FGCI to return the amount that was paid in excess. To enforce her right as
prevailing party, Villasi filed a Motion for Execution of the Court of Appeals Decision,
which was favorably acted upon by the RTC.

To satisfy the judgment, the sheriff levied on a building located at No. 140 Kalayaan
Avenue, Quezon City, covered by Tax Declaration No. D-021-01458, and built in the
lots registered under Transfer Certificates of Title (TCT) Nos. 379193 and 379194.
While the building was declared for taxation purposes in the name of FGCI, the lots in
which it was erected were registered in the names of the Spouses Filomeno Garcia and
Ermelinda Halili-Garcia. To forestall the sale on execution, the Spouses Garcia filed an
Affidavit of Third Party Claim and a Motion to Set Aside Notice of Sale on Execution,
claiming that they are the lawful owners of the property which was erroneously levied
upon by the sheriff.

Issue:

Whether or not the subject lot and the building should be treated separately

Ruling:

A perusal of the record shows that, as the party asserting their title, the Spouses Garcia
failed to prove that they have a bona fide title to the building in question. Aside from
their postulation that as title holders of the land, the law presumes them to be owners of
the improvements built thereon, the Spouses Garcia were unable to adduce credible
evidence to prove their ownership of the property. In contrast, Villasi was able to

147
satisfactorily establish the ownership of FGCI thru the pieces of evidence she appended
to her opposition. Worthy to note is the fact that the building in litigation was declared for
taxation purposes in the name of FGCI and not in the Spouses Garcias. FGCI is also in
actual possession of the building and as the payment of taxes coupled with actual
possession of the land covered by tax declaration strongly supports a claim of
ownership. Quite significantly, all the court processes in an earlier collection suit
between FGCI and Villasi were served, thru the former‗s representative Filomeno
Garcia, at No. 140 Kalayaan Avenue, Quezon City, where the subject property is
located. This circumstance is consistent with the tax declaration in the name of FGCI.

148
ACCESSION
DISCRETA

149
REPUBLIC OF THE PHILIPPINES, REPRESENTED BY THE TOLL REGULATORY
BOARD (TRB) VS. HOLY TRINITY REALTY DEVELOPMENT CORP.

G.R. No. 172410


April 14, 2008

Facts:

On December 29, 2000, petitioner filed with the RTC a Consolidated Complaint for
Expropriation against landowners whose properties would be affected by the
construction, rehabilitation and expansion of the North Luzon Expressway. Respondent
Holy Trinity Realty and Development Corporation (HTRDC) was one of the affected
landowners. On March 18, 2002, TRB filed an Urgent Ex-Parte Motion for the issuance
of a Writ of Possession, manifesting that it deposited a sufficient amount to cover the
payment of 100% of the zonal value of the affected properties, in the total amount of
P28,406,700.00. TRB maintained that since it had already complied with the provisions
of Section 4 of Republic Act No. 8974 in relation to Section 2 of Rule 67 of the Rules of
Court, the issuance of the writ of possession becomes ministerial on the part of the
RTC. The RTC issued an Order for the Issuance of a Writ of Possession, as well as the
Writ of Possession itself. HTRDC thereafter moved for the reconsideration of the Order
of the RTC. On October 7, 2002, the Sheriff filed with the RTC a Report on Writ of
Possession stating, among other things, that since none of the landowners voluntarily
vacated the properties subject of the expropriation proceedings, the assistance of the
Philippine National Police (PNP) would be necessary in implementing the Writ of
Possession.

On March 3, 2003, HTRDC filed with the RTC a Motion to Withdraw Deposit, praying
that the respondent or its duly authorized representative be allowed to withdraw the
amount of P22,968,000.00, out of TRB's advance deposit. On 11 March 2004, the RTC
issued an Order resolving as follows the issue of ownership of the interest that had
accrued on the amount deposited by DPWH in its expropriation current account. TRB
filed a Motion for Reconsideration of the afore-quoted RTC Order, contending that the
payment of interest on money deposited and/or consigned for the purpose of securing a
writ of possession was sanctioned neither by law nor by jurisprudence.

Issue:

Whether or not the interest earnings from the expropriation account under the principle
of accession should be considered as fruits

150
Ruling:

Under Article 440 of the Civil Code, the right of accession is conferred by ownership of
the principal property:

Art. 440. The ownership of property gives the right by accession to everything which is
produced thereby, or which is incorporated or attached thereto, either naturally or
artificially.

The principal property in the case at bar is part of the deposited amount in the
expropriation account of DPWH which pertains particularly to HTRDC. Such amount,
determined to be P22,968,000.00 of the P28,406,700.00 total deposit, was already
ordered by the RTC to be released to HTRDC or its authorized representative. Since
the Court of Appeals found that the HTRDC is the owner of the deposited amount, then
the latter should also be entitled to the interest which accrued thereon. Nonetheless, the
Court finds it necessary to emphasize that HTRDC is determined to be the owner of
only a part of the amount deposited in the expropriation account, in the sum of
P22,968,000.00. Hence, it is entitled by right of accession to the interest that had
accrued to the said amount only.

151
OWNERSHIP
AS TO THE
FRUITS

152
EQUATORIAL REALTY DEVELOPMENT, INC. VS. MAYFAIR THEATER, INC.

G.R. No. 133879


November 21, 2001

Facts:

Carmelo & Bauermann, Inc. used to own a parcel of land, together with two 2-storey
buildings constructed thereon, located at Claro M. Recto Avenue, Manila. On June 1,
1967, Carmelo entered into a Contract of Lease with Mayfair Theater Inc. for a period of
20 years. The lease covered a portion of the second floor and mezzanine of a two-
storey building with about 1,610 square meters of floor area, which respondent used as
a movie house known as Maxim Theater. Two years later, on March 31, 1969, Mayfair
entered into a second Contract of Lease with Carmelo for the lease of another portion of
the latter's property. In that space, Mayfair put up another movie house known as
Miramar Theater. The Contract of Lease was likewise for a period of 20 years. Both
leases contained a provision granting Mayfair a right of first refusal to purchase the
subject properties. However, on July 30, 1978 - within the 20-year-lease term - the
subject properties were sold by Carmelo to Equatorial Realty Development, Inc. for the
total sum of P11,300,000, without their first being offered to Mayfair.

As a result of the sale of the subject properties to Equatorial, Mayfair filed a Complaint
before the Regional Trial Court for the annulment of the Deed of Absolute Sale between
Carmelo and Equatorial. The controversy reached the Supreme Court where it upheld
the rescission of the contract of sale. On April 25, 1997, Mayfair filed a Motion for
Execution, which the trial court granted. However, Carmelo could no longer be located.
Thus, following the order of execution of the trial court, Mayfair deposited with the clerk
of court a quo its payment to Carmelo in the sum of P11, 300,000 less P847,000 as
withholding tax. The lower court issued a Deed of Reconveyance in favor of Carmelo
and a Deed of Sale in favor of Mayfair. On the basis of these documents, the Registry of
Deeds of Manila cancelled Equatorial's titles and issued new Certificates of Title in the
name of Mayfair. Ruling on Equatorial's Petition for Certiorari and Prohibition contesting
the foregoing manner of execution, the CA in its Resolution of November 20, 1998,
explained that Mayfair had no right to deduct the P847,000 as withholding tax.

On September 18, 1997, barely five months after Mayfair had submitted its Motion for
Execution, Equatorial filed with the Regional Trial Court of Manila, Branch 8, an action
for the collection of a sum of money against Mayfair, claiming payment of rentals or
reasonable compensation for the defendant's use of the subject premises after its lease
contracts had expired. This action was the progenitor of the present case.

153
Issue:

Whether or not Equatorial is entitled to back rentals

Ruling:

From the peculiar facts of this case, it is clear that petitioner never took actual control
and possession of the property sold, in view of respondent's timely objection to the sale
and the continued actual possession of the property. The objection took the form of a
court action impugning the sale which, as we know, was rescinded by a judgment
rendered by this Court in the mother case. It has been held that the execution of a
contract of sale as a form of constructive delivery is a legal fiction. The rental payments
made by Mayfair should not be construed as a recognition of Equatorial as the new
owner. They were made merely to avoid imminent eviction. In short, the sale to
Equatorial may have been valid from inception, but it was judicially rescinded before it
could be consummated. Petitioner never acquired ownership, not because the sale was
void, as erroneously claimed by the trial court, but because the sale was not
consummated by a legally effective delivery of the property sold.

Furthermore, assuming for the sake of argument that there was valid delivery, petitioner
is not entitled to any benefits from the "rescinded" Deed of Absolute Sale because of its
bad faith. This being the law of the mother case decided in 1996, it may no longer be
changed because it has long become final and executory.

154
ACCESSION
INDUSTRIAL

155
ELVIRA ARANGOTE VS. SPOUSES MARTIN and LOURDES MAGLUNOB AND
ROMEO SALIDO

G.R. No. 178906


February 18, 2009

Facts:

Elvira Arangote acquired the subject parcel of land from Esperanza Maglunob, who is
grandaunt of respondents Martin Maglunob and Romeo Salido. In June 1986,
Esperenza executed an affidavit in which she renounced her rights, share and
participation in the land in favor of Elvira and her husband. It appears that the lot was
not exclusive property of Esperanza but also of the other heirs of Martin I whom she
represented in the partition agreement. Elvira and her husband, Ray constructed a
house on the land in 1989 and in1993, OCT was issued in her name by the DAR.
However, respondents with the help of hired persons entered the property and built a
wall behind and in front of Elvira‗s house. Elvira and Ray sued respondents for quieting
of title and declaration of ownership. Respondents averred that they were co-owners of
the land with Esperanza who allegedly inherited the land from Martin together with
Tomas and Inocencia (Martin 2‗sand Romeo‗s predecessor in interest). They argued
that Esperanza could not have validly waived her rights in favor of Elvira and Ray.
MCTC ruled for Elvira. RTC reversed MCTC and declared respondents lawful owners of
the land together with the other heirs of Martin I. Elvira went to the CA but the CA
affirmed the RTC decision. Before SC, Elvira argued that both RTC and CA erred in
declaring the affidavit of Esperanza void because it is a valid and binding proof of
transfer of ownership of the subject property as it was coupled with actual delivery.

Issue:

Whether or not the donation to Elvira and her husband is valid.

Ruling:

Supreme Court affirmed the decision of CA. SC ruled that the affidavit executed by
Esperanza wherein she renounced, relinquished and waived all her rights, share,
interest and participation in the subject property in favor of Elvira and Ray is in fact a
donation. Thus, it should have complied with the requirements of Article 749 of the Civil
Code.
A simple donation of real property to be valid:
1) Must be made in a public instrument;

156
2) It must be accepted, which must be in the same deed of donation or in a separate
public instrument;
3) If the acceptance is made in a separate instrument, the donor must be notified in
authentic form and the same must be noted in both instruments.

The affidavit executed by Esperanza relinquishing her rights, share and participation
over the property in favor of Elvira suffered from legal infirmities. In Sumipat vs. Banga,
the Court ruled that title to immovable does not pass from the donor to the done by
virtue of a Deed of Donation until and unless it has been accepted in a public instrument
and the donor duly notified thereof. In this case, the acceptance of the donation was not
made by Elviraeither in the same affidavit or in a separate public instrument. Neither
was there notice of acceptance given to the donor, therefore the donation is void.

157
TORBELA VS SPOUSES ROSARIO

G.R. No. 140528


December 7, 2011

Facts:

The issue is over a parcel of land inherited by the Torbela siblings from their parents.
They executed a deed of absolute quitclaim over the property in favor of Dr. Rosario.
Four days after, a TCT was issued in Dr. Rosario‗s name covering the property.
Another deed of absolute quitclaim was subsequently executed twelve days after by Dr.
Rosario acknowledging that he only borrowed the lot from the Torbela siblings and was
already returning the same. This deed was notarized but not immediately annotated.
Dr. Rosario used the land as mortgage for a loan he obtain through DBP for
P70,000.00. He used the proceeds of the loan to build a 4 storey building which was
initially used as a hospital but later converted into a commercial space. Part was leased
to PT&T and the rest to Rosario‗s sister who operated the Rose Inn Hotel and
Restaurant.

Dr. Rosario fully paid the loan from DBP and the mortgage was cancelled and ratified by
a notary public. However, Dr. Rosario took another loan from PNB. He later acquired a
third loan from Banco Filipino and bought out the loan from PNB cancelling the
mortgage with PNB. Rosario failed to pay their loan in Banco Filipino and the property
was extra-judicially foreclosed.

Meanwhile, back in 1965, the Torbela siblings sought to register their ownership over
the lot and to perfect their title but couldn‗t because the title was still with DBP. They
showed as proof the deed of absolute quitclaim presented executed by Rosario himself.
In 1986, they filed a civil case for recovery of ownership and possession and damages.
They tried to redeem the lot from Banco Filipino but failed. TCT was issued to Banco
FIilipino. The Torbela‗s claim they have right over the rents of the building through
accession because they are the land owners.

Issue:

Who has right over the improvements made on the lot and the rents thereof.

158
Ruling:

According to Art. 440, the accessory follows the principal. Ownership of property gives
the right by accession to everything which is produced thereby, or which is incorporated
or attached thereto, either naturally or artificially.

However, in the case at bar, both Torbela siblings and Rosario are deemed in bad faith.
The Torbelas knew Rosario built on the land and even allowed him to use the land to
obtain a loan from DBP. Rosario on the other hand consciously built on land he knew
was not his. They both had knowledge and did not oppose.

Art. 453 states that when both parties are in bad faith, the case shall be treated as
though both were in good faith thus the application of Art. 448.

Art. 448 allow the Land Owner 2 options in the case at bar. Either indemnify Rosario
and appropriate the lot to him or ask Rosario to buy the lot or the rent rate. This case
was remanded to the RTC for the Torbelas to make such decision.

159
FILOMENA R. BENEDICTO VS. ANTONIO VILLAFLORES

G.R. No. 185020


October 6, 2010

Facts:

In 1980, Maria Villaflores (Maria) sold a portion of Lot 2-A to her nephew, respondent
Antonio Villaflores (Antonio). Antonio then took possession of the portion sold to him
and constructed a house thereon. Twelve (12) years later, or on August 15, 1992, Maria
executed in favor of Antonio a Kasulatan ng Bilihang Tuluyan covering the entire Lot 2-
A. However, Antonio did not register the sale or pay the real property taxes for the
subject land.

On August 31, 1994, Maria sold the same Lot 2-A to Filomena, evidenced by a
Kasulatan ng Bilihang Tuluyan. Filomena registered the sale on September 6, 1994.
Since then Filomena paid the real property taxes for the subject parcel of land.

After trial, the RTC sustained Filomena‘s ownership who was the one who registered
the sale in good faith. It rejected Antonio‘s allegation of bad faith on the part of Filomena
because no sufficient evidence was adduced to prove it. This finding was affirmed by
the CA.

Issue:

Whether or not Antonio is a possessor in good faith.

Ruling:

Yes. The Court sustained the finding that Antonio is a builder in good faith.

Under Article 448, a landowner is given the option to either appropriate the
improvement as his own upon payment of the proper amount of indemnity, or sell the
land to the possessor in good faith. Anent to this, Article 546 provides that a builder in
good faith is entitled to full reimbursement for all the necessary and useful expenses
incurred; it also gives him right of retention until full reimbursement is made. The
objective of Article 546 of the Civil Code is to administer justice between the parties
involved. Guided by this precept, it is therefore the current market value of the
improvements which should be made the basis of reimbursement.

However, in spite of its finding of good faith on the part of Antonio by the RTC it did not
order the reimbursement of the necessary and useful expenses he incurred. Hence, the
CA correctly ordered the remand of the case for further proceedings.

160
LUCIANO BRIONES AND NELLY BRIONES VS. JOSE MACABAGDAL, ET. AL.

G.R. No. 150666


August 3, 2010

Facts:

Spouses Macabagdal purchased from Vergon a lot located in Vergonville Subdivision.


Sometime in 1984, spouses Briones, after obtaining the necessary building permit from
Vergon, started constructing on the same parcel of land. After learning of such, spouses
Macabagdal demanded sposes Briones to demolish the house and vacate the property.
Spouses Briones refused.
Spouses Macabagdal thus filed a case for recovery of ownership and possession of
said parcel of land. The RTC and CA ruled in favor of spouses Macabagdal, and
ordered spouses Briones to either vacate the property or to pay spouses Macabagdal
the prevailing price of the land.

Issue:

Whether or not spouses Briones were builders in good faith, thus, must not bear the
damage alone.

Ruling:

Yes. Since there was no evidence to show that spouses Briones were builders in bad
faith, they must be considered builders in good faith. Thus, the landowner is given the
option to appropriate the building by paying compensation or to oblige the builder to pay
the price of the land. Moreover, the builder in good faith is entitled to be reimbursed the
necessary and useful expenses they made of the subject land.

Thus, spouses Macabagdal cannot demand the removal of the building unless he first
exercises the option of appropriating such after payment to spouses Briones of the
proper compensation.

161
BARSTOWE PHILIPPINES CORPORATION VS. REPUBLIC OF THE PHILIPPINES

G.R. No. 133110


March 28, 2007

Facts:

This case involves the conflicting titles to the same parcels of land (subject lots) of
petitioner Barstowe Philippines Corporation (BPC) and the respondent Republic of the
Philippines (Republic). Due to the fire that gutted the Office of the Quezon City Register
of Deeds on 11 June 1988 and destroyed many certificates of title kept therein, Antonio
sought the administrative reconstitution of the original copies and owner‘s duplicate
copies of 2 TCTs. The Republic applied for administrative reconstitution of the same
with the LRA. It was then that the Republic came to know that another party had applied
for reconstitution which also covered the same lots. The RTC rendered judgment
declaring both BPC and Republic as buyers in good faith. But it upheld BPC‘s rights
over the republic since it was registered earlier. The CA ruled for the Republic.

Issue:

Who between BPC and the Republic has a better title over the subject lots?

Ruling:

Under Section 55 of the Land Registration Act, as amended by Section 53 of


Presidential Decree No. 1529, an original owner of registered land may seek the
annulment of a transfer thereof on the ground of fraud. However, such a remedy is
without prejudice to the rights of any innocent holder for value with a certificate of title. A
purchaser in good faith and for value is one who buys the property of another, without
notice that some other person has a right to or interest in such property, and pays a full
and fair price for the same at the time of such purchase or before he has notice of the
claim or interest of some other person in the property. It has been consistently ruled that
a forged deed can legally be the root of a valid title when an innocent purchaser for
value intervenes. A deed of sale executed by an impostor without the authority of the
owner of the land sold is a nullity, and registration will not validate what otherwise is an
invalid document. However, where the certificate of title was already transferred from
the name of the true owner to the forger and, while it remained that way, the land was
subsequently sold to an innocent purchaser, the vendee had the right to rely upon what
appeared in the certificate and, in the absence of anything to excite suspicion, was
under no obligation to look beyond the certificate and investigate the title of the vendor
appearing on the face of said certificate.

BPC cannot really claim that it was a purchaser in good faith which relied upon the face
of Servando‘s titles. It should be recalled that the Quezon City Register of Deeds caught
fire on 11 June 1988. Presumably, the original copies of TCTs were burnt in the said
fire. Servando‘s heirs sought the administrative reconstitution of the TCTs. There was

162
no showing that there were surviving owner‘s duplicate copies of TCTs. Without the
original copies and owner‘s duplicate copies of TCTs, BPC had to rely on the
reconstituted certificates. Under section 7 of Republic Act No. 26, "Reconstituted titles
shall have the same validity and legal effect as the originals thereof" unless the
reconstitution was made extrajudicially. In this case, TCTs were reconstituted
administratively, hence, extrajudicially. In contrast to the judicial reconstitution of a lost
certificate of title which is in rem, the administrative reconstitution is essentially ex-parte
and without notice.

The reconstituted certificates of title do not share the same indefeasible character of the
original certificates of title for the following reason –
x x x The nature of a reconstituted Transfer Certificate Of Title of registered land is
similar to that of a second Owner's Duplicate Transfer Certificate Of Title. Both are
issued, after the proper proceedings, on the representation of the registered owner that
the original of the said TCT or the original of the Owner's Duplicate TCT, respectively,
was lost and could not be located or found despite diligent efforts exerted for that
purpose. Both, therefore, are subsequent copies of the originals thereof. A cursory
examination of these subsequent copies would show that they are not the originals.
Anyone dealing with such copies are put on notice of such fact and thus warned to be
extra-careful. x x x.

The fact that the TCTs were reconstituted should have alerted BPC and its officers to
conduct an inquiry or investigation as might be necessary to acquaint themselves with
the defects in the titles of Servando. This Court cannot declare BPC an innocent
purchaser for value, and it acquired no better titles to the subject lots than its
predecessors-in-interest, Servando and Antonio. The general rule is that the State
cannot be put in estoppel by the mistakes or errors of its officials or agents.

However, like all general rules, this is also subject to exceptions, viz:
"Estoppels against the public are little favored. They should not be invoked except in
rare and unusual circumstances, and may not be invoked where they would operate to
defeat the effective operation of a policy adopted to protect the public. They must be
applied with circumspection and should be applied only in those special cases where
the interests of justice clearly require it. Nevertheless, the government must not be
allowed to deal dishonorably or capriciously with its citizens, and must not play an
ignoble part or do a shabby thing; and subject to limitations x x x the doctrine of
equitable estoppel may be invoked against public authorities as well as against private
individuals."

Significantly, the other private respondents – Spouses Santos, Spouses Calaguian,


Dela Fuente and Madaya – bought such "expanded" lots in good faith, relying on the
clean certificates of St. Jude, which had no notice of any flaw in them either. It is only
fair and reasonable to apply the equitable principle of estoppel by laches against the
government to avoid an injustice to the innocent purchasers for value.

163
SPOUSES NUGUID VS. COURT OF APPEALS

G.R. No. 151815


February 23, 2005

Facts:

Pedro P. Pecson owned a commercial lot on which he built a four-door two-storey


apartment building. For failure to pay realty taxes, the lot was sold at public auction by
the City Treasurer to Mamerto Nepomuceno, who in turn sold it for P103,000 to the
spouses Juan and Erlinda Nuguid.

Pecson challenged the validity of the auction sale before the RTC of Quezon City, the
RTC upheld the spouses‘ title but declared that the four-door two-storey apartment
building was not included in the auction sale. This was affirmed by the CA and by the
SC.

The Nuguids became the uncontested owners of commercial lot. The Nuguid spouses
moved for delivery of possession of the lot and the apartment building.

Issue:

Whether or not the Nuguids should reimburse Pecson for the benefits derived from the
apartment building.

Ruling:

Yes. Since petitioners opted to appropriate the improvement for themselves as early as
June 1993, when they applied for a writ of execution despite knowledge that the auction
sale did not include the apartment building, they could not benefit from the lot‘s
improvement, until they reimbursed the improver in full, based on the current market
value of the property.

Under Article 448, the landowner is given the option, either to appropriate the
improvement as his own upon payment of the proper amount of indemnity or to sell the
land to the possessor in good faith. Relatedly, Article 546 provides that a builder in good
faith is entitled to full reimbursement for all the necessary and useful expenses incurred;
it also gives him right of retention until full reimbursement is made.

The right of retention is considered as one of the measures devised by the law for the
protection of builders in good faith. Its object is to guarantee full and prompt
reimbursement as it permits the actual possessor to remain in possession while he has
not been reimbursed (by the person who defeated him in the case for possession of the
property) for those necessary expenses and useful improvements made by him on the
thing possessed.

164
Given the circumstances of the instant case where the builder in good faith has been
clearly denied his right of retention for almost half a decade, we find that the increased
award of rentals by the RTC was reasonable and equitable. The petitioners had reaped
all the benefits from the improvement introduced by the respondent during said period,
without paying any amount to the latter as reimbursement for his construction costs and
expenses. They should account and pay for such benefits.

165
PHILIPPINE NATIONAL BANK VS. DE JESUS

G.R. No. 149295


September 23, 2003

Facts:

It would appear that on 10 June 1995, respondent filed a complaint against petitioner
before the Regional Trial Court of Occidental Mindoro for recovery of ownership and
possession, with damages, over the questioned property. In his complaint, respondent
stated that he had acquired a parcel of land situated in Mamburao, Occidental Mindoro,
with an area of 1,144 square meters covered by TCT No. T-17197, and that on 26
March 1993, he had caused a verification survey of the property and discovered that the
northern portion of the lot was being encroached upon by a building of petitioner to the
extent of 124 square meters. Despite two letters of demand sent by respondent,
petitioner failed and refused to vacate the area.

Petitioner, in its answer, asserted that when it acquired the lot and the building
sometime in 1981 from then Mayor Bienvenido Ignacio, the encroachment already was
in existence and to remedy the situation, Mayor Ignacio offered to sell the area in
question (which then also belonged to Ignacio) to petitioner at P100.00 per square
meter which offer the latter claimed to have accepted. The sale, however, did not
materialize when, without the knowledge and consent of petitioner, Mayor Ignacio later
mortgaged the lot to the Development Bank of the Philippines. He also contends that he
is a builder in good faith.

Issue:

Whether or not being a builder in good faith matters under article 448.

Ruling:

Article 448, of the Civil Code refers to a piece of land whose ownership is claimed by
two or more parties, one of whom has built some works (or sown or planted something)
and not to a case where the owner of the land is the builder, sower, or planter who then
later loses ownership of the land by sale or otherwise for, elsewise stated, ―where the
true owner himself is the builder of works on his own land, the issue of good faith or bad
faith is entirely irrelevant.‖

166
PLEASANTVILLE DEVELOPMENT CORPORATION VS. COURT OF APPEALS

G.R. No. 79688


February 1, 1996

Facts:

Edith Robillo purchased from Pleasantville Development Corporation, herein petitioner a


parcel of land at Pleasantville Subdivision, Bacolod City. The property was designated
as Lot 9, Phase II. In 1975, herein respondent Eldred Jardinico bought the said subject
lot from the former purchaser. Eldred later discovered that the property he purchased
had improvements introduced therein by respondent Wilson Kee. Kee on the other hand
bought on installments Lot 8 of the same subdivision from C.T. Torres Enterprises, Inc.
(CTTEI) which is the exclusive real estate agent of the petitioner. Under the contract
Kee was allowed to take possession of the property even before full payment of the
price. CTTEI through an employee, Zenaida Octaviano accompanied Kee‘s wife
Donabelle to inspect Lot No. 8. Octaviano however mistakenly pointed towards Lot 9.
Hence spouses Kee had their residence, an auto repair shop, a store and other
improvements constructed on the wrong lot. Upon discovery of the blunder both Kee
and Jardinico tried to reach an amicable settlement but they failed. Jardinico demanded
that the improvements be removed but as Kee refused, Jardinico filed a complaint for
ejectment with damages against Kee at the Municipal Trial Court in Cities (MTCC) of
Bacolod City. Kee filed a third-party complaint against herein petitioner and CTTEI. The
MTCC found that the error was attributable to CTTEI also since at present the contract
with Kee has rescinded for Kee‘s failure to pay installments. Kee no longer had any right
over the subject property and must pay rentals for its use. The Regional Trial Court
(RTC) of Bacolod City ruled that petitioner and CTTEI were not at fault or were not
negligent. It argued that Kee was a builder in bad faith. Even if assuming that he was in
good faith, he was no longer so and must pay rentals from the time that he was given
notice to vacate the lot. The Court of Appeals ruled that Kee was a builder in good faith
she was unaware of the mix-up when he constructed the improvements. It was in fact
due to the negligence and wrongful delivery of CTTEI which included its principal the
herein petitioner. It further ruled that the award of rental was without basis. Pending the
resolution of the case at the Court of Appeals Jardinico and Kee entered into a deed of
sale, wherein Lot 9 was sold to Kee. In the said deed a provision stating that regardless
of the outcome of the decision, such shall not be pursued by the parties and shall be
considered dismissed and without effect. The appellate court was not informed of this
deal.

Issue:

Whether or not a lot buyer who constructs improvements on the wrong property
erroneously delivered by the owner‘s agent, a builder in good faith.

167
Ruling:

Yes. Article 527 of the Civil Code provides the presumption that petitioner has the
burden of proving that Kee was a builder in bad faith. Kee may be made liable for the
violation of the contract with CTTEI but this may not be used as a basis of bad faith and
as a sufficient ground to negate the presumption of good faith. Jardinico is presently
only allowed to file a complaint for unlawful detainer. Good faith is based on the belief of
the builder that the land he is building on is his and his ignorance of any flaw or defect in
is title. Since at the time when Kee constructed his improvements on Lot 8, he was not
aware that it was actually Lot 9 that was delivered to him.

168
EVADEL REALTY AND DEVELOPMENT CORPORATION VS. SPOUSES SORIANO

G.R. No. 144291


April 20, 2001

Facts:

On April 12, 1996, the spouses Antero and Virginia Soriano (respondent spouses), as
sellers, entered into a Contract to Sell with Evadel Realty and Development Corporation
(petitioner), as buyer, over a parcel of land denominated as Lot 5536-C of the
Subdivision Plan of Lot 5536 covered by Transfer Certificate of Title No. 125062 which
was part of a huge tract of land known as the Imus Estate.

Respondent spouses successively sent demand letters to petitioner on February 14,


March 7, and April 24, 1997, to vacate the encroached area. Petitioner admitted
receiving the demand letters but refused to vacate the said area. Thus, on May 23,
1997, a complaint for accion reinvindicatoria was filed by respondent spouses against
petitioner with the Regional Trial Court, Branch 88 of Cavite City.

In its Answer, petitioner admitted the encroachment but claimed that it was a builder in
good faith since it merely relied on the boundaries pointed out by the representatives of
respondent spouses. Petitioner also argued that there was a novation of contract
because of the encroachment made by the national road on the property subject of the
contract by 1,647 square meters. On March 19, 1998, respondents filed a Motion for
Summary Judgment, alleging that there existed no genuine issue as to the material
facts of the case due to the admissions made by petitioner in its Answer.

In a Decision promulgated on April 20, 2001, this Court ruled in favor of respondents
and affirmed the Court of Appeals decision which held that the trial court did not err in
rendering a summary judgment in this case. However, on June 7, 2001, petitioner filed a
Motion for Reconsideration alleging therein that the Court committed a serious
misapprehension of facts and that it was not given its day in court and hence, denied
due process of law. Respondents, in turn, filed their Opposition to the Motion for
Reconsideration on June 15, 2001. In a Resolution issued on September 5, 2001, the
Court denied the said Motion for Reconsideration on the ground that the basic issues
have been passed upon in the Decision and no substantial arguments have been raised
in the said motion. Petitioner‘s Motion for Reconsideration was denied with finality.
After hearing the argument of both parties, the Court issued another Resolution dated
July 10, 2002, which stated that it shall allow the parties to explore the possibility of
entering into a compromise and to inform the court as soon as possible as to the efforts
exerted. The Court likewise declared therein that the case shall be deemed submitted
for resolution. On July 26, 2002, respondents Dr. Antero Soriano and the heirs of
Virginia Soriano manifested before the Court that they still have not received any
proposal for an amicable settlement from petitioner and in view thereof, they are no
longer interested in such an amicable settlement.

169
Issue: Whether or not the encroachment on the northern side of sold Lot 5536-C which
is the area in dispute and that such admissions are tantamount to an admission that
respondents have a rightful claim of ownership to the subject property warranting a
summary judgment in their favor.

Ruling:

No. As correctly pointed out by the trial court and the CA, petitioner already admitted in
its Amended Answer that the lot in dispute is covered by TCT No. T-769166 of
respondent spouses. With this admission, petitioner can no longer claim that it was a
builder in good faith. Good faith consists in the belief of the builder that the land he is
building on is his and his ignorance of any defect or flaw in his title. In this case, since
petitioner, by its own admission, had knowledge of respondent spouses title over the
subject lot; it was clearly in bad faith when it introduced improvements thereon.
Further, the contract to sell between petitioner and respondent spouses, the
genuineness and due execution thereof was admitted by petitioner, clearly delineated
the metes and bounds of the lot subject thereof. Attached to the said contract was a
graphic illustration of the lot purchased by petitioner including a technical description
thereof. Petitioner, as a real estate developer, is presumed to be experienced in its
business and ought to have sufficient technical expertise to correctly determine the
metes and bounds of the lands it acquires. Despite this, petitioner still introduced
improvements on the lot not covered by the contract to sell. Petitioners bad faith had
been duly established by the pleadings and there was thus no need to further conduct
any trial on the matter. Our ruling in Congregation of the Religious of the Virgin Mary vs.
Court of Appeals is particularly instructive.

170
TECNOGAS PHILIPPINES MANUFACTURING CORPORATION VS. COURT OF
APPEALS

G.R. No. 108894


February 10, 1997

Facts:

That plaintiff (herein petitioner) which is a corporation duly organized and existing under
and by virtue of Philippine laws is the registered owner of a parcel of land situated in
Barrio San Dionisio, Paranaque, Metro Manila known as Lot 4331-A (should be 4531-A)
of Lot 4531 of the Cadastral Survey of Paranaque, Metro Manila, covered by Transfer
Certificate of Title No. 409316 of the Registry of Deeds of the Province of Rizal; that
said land was purchased by plaintiff from Pariz Industries, Inc. in 1970, together with all
the buildings and improvements including the wall existing thereon; that the defendant
(herein private respondent) is the registered owner of a parcel of land known as Lot No.
4531-B of Lot 4531 of the Cadastral Survey of Paranaque, LRC (GLRO) Rec. No.
19645 covered by Transfer Certificate of Title No. 279838, of the Registry of Deeds for
the Province of Rizal; that said land which adjoins plaintiffs land was purchased by
defendant from a certain Enrile Antonio also in 1970; that in 1971, defendant purchased
another lot also adjoining plaintiffs land from a certain Miguel Rodriguez and the same
was registered in defendants name under Transfer Certificate of Title No. 31390, of the
Registry of Deeds for the Province of Rizal; that portions of the buildings and wall
bought by plaintiff together with the land from Pariz Industries are occupying a portion of
defendants adjoining land; that upon learning of the encroachment or occupation by its
buildings and wall of a portion of defendants land, plaintiff offered to buy from defendant
that particular portion of defendants land occupied by portions of its buildings and wall
with an area of 770 square meters, more or less, but defendant, however, refused the
offer.

In 1973, the parties entered into a private agreement before a certain Col. Rosales in
Malacanang, wherein plaintiff agreed to demolish the wall at the back portion of its land
thus giving to defendant possession of a portion of his land previously enclosed by
plaintiffs wall; that defendant later filed a complaint before the office of Municipal
Engineer of Paranaque, Metro Manila as well as before the Office of the Provincial
Fiscal of Rizal against plaintiff in connection with the encroachment or occupation by
plaintiffs buildings and walls of a portion of its land but said complaint did not prosper;
that defendant dug or caused to be dug a canal along plaintiffs wall, a portion of which
collapsed in June, 1980, and led to the filing by plaintiff of the supplemental complaint in
the above-entitled case and a separate criminal complaint for malicious mischief against
defendant and his wife which ultimately resulted into the conviction in court of
defendants wife for the crime of malicious mischief; that while trial of the case was in
progress, plaintiff filed in Court a formal proposal for settlement of the case but said
proposal, however, was ignored by defendant.

171
Issue:

Whether or not the respondent Court of Appeals erred in holding the petitioner a builder
in bad faith because it is presumed to know the metes and bounds of his property.

Ruling:

Yes. The two cases it relied upon do not support its main pronouncement that a
registered owner of land has presumptive knowledge of the metes and bounds of its
own land, and is therefore in bad faith if he mistakenly builds on an adjoining land.
Aside from the fact that those cases had factual moorings radically different from those
obtaining here, there is nothing in those cases which would suggest however remotely,
that bad faith is imputable to a registered owner of land when a part of his building
encroaches upon a neighbors land, simply because he is supposedly presumed to know
the boundaries of his land as described in his certificate of title. No such doctrinal
statement could have been made in those cases because such issue was not before
the Supreme Court. Quite the contrary, we have rejected such a theory in Co Tao vs.
Chico, where we held that unless one is versed in the science of surveying, no one can
determine the precise extent or location of his property by merely examining his paper
title.

172
SPOUSES REYNATO AND EDITHA LOPEZ VS. MARGARITA SARABIA

G.R. No. 140357


September 24, 2004

Facts:

Margarita Sarabia owned two (2) lots with a residential house built on one of the lots in
Poblacion, Kalibo, Aklan. Spouses Reynaldo and Editha Lopez were renting the second
floor of the house for P300.00 per month. On the other lot was a building rented by Dr.
Nilda Tambong for P600.00 a month, with two (2) boarders upstairs paying P440.00 a
month. Her outstanding loan balance as of March 1984 had already ballooned to about
P63,000.00.

The Spouses Lopez tried to convince Margarita that it was better to transfer the
mortgage to the DBP where interest rates were lower; Editha Lopez was a public school
teacher and the monthly amortization could easily be deducted from her salary. They
told Margarita that the PNB loan balance could be paid off from the proceeds of the loan
from the DBP, and the excess could be used for the construction of the rooms. In order
to facilitate the loan, it was, however, necessary that the property be in the name of the
Spouses Lopez.

Relying on the couple's good faith and assurances that they would religiously pay the
amortization, Margarita agreed to their proposition. A document was thus executed
denominated as "Assumption of Mortgage with Quitclaim." Sometime in October 1987,
Reynaldo Lopez approached Margarita and informed her that he needed P30,000.00 to
update their loan payments. Margarita gave him the amount as part of the refund to the
payment of the PNB loan. She expected Reynaldo to give her an official receipt from the
DBP, but did not receive any. Sensing something irregular, she went to the DBP to
inquire about the status of the loan. She was aghast to find out that the loan
amortization had not been paid and that her property was again in danger of being
foreclosed.

Margarita was constrained to file an action with the RTC against the Spouses Lopez for
annulment of document, specific performance and reconveyance with damages. The
trial court continued to rule that the Spouses Lopez were in bad faith, so whatever
improvements were made on the land were forfeited in favor of Margarita. The trial court
found the Spouses Lopez in bad faith and ordered the forfeiture of the improvements in
Margarita's favor. The CA disagreed with the trial court.

Issue:

Whether or not the spouses Lopez is in bad faith.

173
Ruling:

Yes. The Decision of the Court of Appeals is affirmed with the modification that
respondent Margarita Sarabia is directed to exercise, within thirty (30) days from the
finality of this decision, her option of either paying one-half of the value of the
improvements made on the land at that time they were made, or to demand the removal
by the petitioners of the improvements made on the subject property at their expense.

174
PROGRAMME INCORPORATED VS. PROVINCE OF BATAAN

G.R. No. 144635


June 26, 2006

Facts:

On May 14, 1986, BASECO granted petitioner a contract of lease over Piazza Hotel at a
monthly rental of P6,500 for three years, i.e., from January 1, 1986 to January 1, 1989,
subject to renewal by mutual agreement of the parties. After the expiration of the three-
year lease period, petitioner was allowed to continue operating the hotel on monthly
extensions of the lease. In April 1989, however, the Presidential Commission on Good
Government (PCGG) issued a sequestration order against BASECO pursuant to
Executive Order No. 1 of former President Corazon C. Aquino. Among the properties
provisionally seized and taken over was the lot on which Piazza Hotel stood.

On July 19, 1989, however, Piazza Hotel was sold at a public auction for non-payment
of taxes to respondent Province of Bataan. The title of the property was transferred to
respondent. BASECO‘s Transfer Certificate of Title (TCT) No. T-59631 was cancelled
and a new one, TCT No. T-128456, was issued to the Province of Bataan. On July 21,
1989, petitioner filed a complaint for preliminary injunction and collection of sum of
money against BASECO (Civil Case No. 129-ML). Respondent, as the new owner of
the property, filed a motion for leave to intervene on November 22, 1990. After its
motion was granted, respondent filed a complaint-in-intervention praying, inter alia, that
petitioner be ordered to vacate Piazza Hotel and Mariveles Lodge for lack of legal
interest.

During the pre-trial of the complaint-in-intervention, the parties agreed that the case7 be
tried on the sole issue of whether respondent province, as complainant-intervenor, was
the legitimate owner of the Piazza Hotel and Mariveles Lodge. On February 3, 1995,
after trial on the merits, the trial court rendered judgment in favor of respondent. The CA
affirmed the decision of the trial court.

Issue:

Whether or not the Province of Bataan is the owner of the Piazza Hotel and Marivelez
Lodge.

Ruling:

Yes. Time and again, we have ruled that factual matters are best evaluated by trial
courts which can scrutinize evidence and hear testimony presented and offered by the
parties (in this case, on the issue of ownership of the subject property). All the more
does this principle ring true in this petition since such factual determination by the RTC

175
was upheld by the CA. Only questions of law are the proper subject of a petition for
review on certiorari in this Court, unless any of the known exceptions is extant in this
case. There is none.

The evidence clearly established respondent‘s ownership of Piazza Hotel. First, the title
of the land on which Piazza Hotel stands was in the name of respondent. Second, Tax
Declaration No. 12782 was in the name of respondent as owner of Piazza Hotel. The
benefits granted to a possessor in good faith cannot be maintained by the lessee
against the lessor because, such benefits are intended to apply only to a case where
one builds or sows or plants on land which he believes himself to have a claim of title
and not to lands wherein one‘s only interest is that of a tenant under a rental contract,
otherwise, it would always be in the power of a tenant to improve his landlord out of his
property. Besides, as between lessor and lessee, the Code applies specific provisions
designed to cover their rights. Hence, the lessee cannot claim reimbursement, as a
matter of right, for useful improvements he has made on the property, nor can he assert
a right of retention until reimbursed. His only remedy is to remove the improvement if
the lessor does not choose to pay its value; but the court cannot give him the right to
buy the land.

176
FEDERICO GEMINIANO, ET. AL., VS. COURT OF APPEALS

G.R. No. 120303


July 24, 1996

Facts:

The subject parcel of land was originally owned by the petitioners' mother, Paulina
Amado vda. de Geminiano. On a portion of that lot stood the petitioners' unfinished
bungalow, which the petitioners sold in November 1978 to the private respondents, with
an alleged promise to sell to the latter that portion of the lot occupied by the house.
Subsequently, the petitioners' mother executed a contract of lease over a 126 square-
meter portion of the lot, including that portion on which the house stood, in favor of the
private respondents for a period of seven years commencing on 15 November 1978.
The private respondents then introduced additional improvements and registered the
house in their names. After the expiration of the lease contract in November 1985,
however, the petitioners' mother refused to accept the monthly rentals. It turned out that
the lot in question was the subject of a suit, which resulted in its acquisition by one
Maria Lee in 1972. In 1982, Lee sold the lot to Lily Salcedo, who in turn sold it in 1984
to the spouses Agustin and Ester Dionisio. On 14 February 1992, the Dionisio spouses
executed a Deed of Quitclaim over the said property in favor of the petitioners. As such,
the lot was registered in the latter's names. On 9 February 1993, the petitioners sent,
via registered mail, a letter addressed to private respondent Mary Nicolas demanding
that she vacate the premises and pay the rentals in arrears within twenty days from
notice. Upon failure of the private respondents to heed the demand, the petitioners filed
a complaint for unlawful detainer and damages.

Issue:

Whether or not petitioners can be compelled to reimburse private respondents the value
of the house and other improvements and allow the latter to retain the premises until
reimbursement was made.

Ruling:

The right to indemnity under Article 1678 of the Civil Code arises only if the lessor opts
to appropriate the improvements. Since the petitioners refused to exercise that option,
the private respondents cannot compel them to reimburse the one-half value of the
house and improvements. Neither can they retain the premises until reimbursement is
made. The private respondents' sole right then is to remove the improvements without
causing any more impairment upon the property leased than is necessary. It is
undisputed that the private respondents came into possession of the said lot by virtue of
a contract of lease executed by the petitioners' mother in their favor establishing
relationship as lessor and lessee and carries with it a recognition of the lessor's title.
The private respondents, as lessees are then estopped to deny their landlord's title, or

177
to assert a better title not only in themselves, but also in some third person while they
remain in possession of the leased premises and until they surrender possession to the
landlord. Being mere lessees, the private respondents knew that their occupation of the
premises would continue only for the life of the lease. Plainly, they cannot be
considered as possessors nor builders in good faith. Article 448 of the Civil Code, in
relation to Article 546, which allows full reimbursement of useful improvements and
retention of the premises until reimbursement is made, applies only to a possessor in
good faith. It does not apply where one's only interest is that of a lessee under a rental
contract.

178
SULO SA NAYON, INCORPORATED VS. NAYONG PILIPINO FOUNDATION

G.R. No. 170923


January 20, 2009

Facts:

Respondent Nayong Pilipino Foundation, a government-owned and controlled


corporation, is the owner of a parcel of land in Pasay City, known as the Nayong Pilipino
Complex. Petitioner Philippine Village Hotel, Inc. (PVHI), formerly called Sulo sa Nayon,
Inc and Jose Marcel E. Panlilio is its Senior Executive Vice President. On June 1, 1975,
respondent leased a portion of the Nayong Pilipino Complexto petitioner Sulo sa Nayon,
Inc. for the construction and operation of a hotel building, to be known as the Philippine
Village Hotel. The lease was for an initial period of 21 years, or until May 1996. It is
renewable for a period of 25 years under the same terms and conditions upon due
notice in writing to respondent of the intention to renew at least 6 months before its
expiration. Thus, on March 7, 1995, petitioners sent respondent a letter notifying the
latter of their intention to renew the contract for another 25 years. On July 4, 1995, the
parties executed a Voluntary Addendum to the Lease Agreement. The addendum was
signed by petitioner Jose Marcel E. Panlilio in his official capacity as Senior Executive
Vice President of the PVHI and by Chairman Alberto A. Lim of the Nayong Pilipino
Foundation. They agreed to the renewal of the contract for another 25 years, or until
2021. Beginning January 2001, petitioners defaulted in the payment of their monthly
rental. Respondent repeatedly demanded petitioners to pay the arrears and vacate the
premises. The last demand letter was sent on March 26, 2001. On September 5, 2001,
respondent filed a complaint for unlawful detainer.

Issue:

Whether or not petitioners are builders in good faith and are entitled to reimbursement
of improvements thereof.

Ruling:

In the case at bar, petitioners have no adverse claim or title to the land. In fact, as
lessees, they recognize that the respondent is the owner of the land. What petitioners
insist is that because of the improvements, which are of substantial value, that they
have introduced on the leased premises with the permission of respondent, they should
be considered builders in good faith who have the right to retain possession of the
property until reimbursement by respondent.

The introduction of valuable improvements on the leased premises does not give the
petitioners the right of retention and reimbursement which rightfully belongs to a builder
in good faith. Otherwise, such a situation would allow the lessee to easily improve the
lessor out of its property. We reiterate the doctrine that a lessee is neither a builder in

179
good faith nor in bad faith that would call for the application of Articles 448 and 546 of
the Civil Code. His rights are governed under Article 1678, where it provides that the
lessor has the option of paying one-half of the value of the improvements which the
lessee made in good faith, which are suitable for the use for which the lease is intended,
and which have not altered the form and substance of the land. On the other hand, the
lessee may remove the improvements should the lessor refuse to reimburse.

180
SERAFIN CHENG VS. SPOUSES DONINI

G.R. No. 167017


June 22, 2009

Facts:

The subject of this petition is an oral lease agreement that went sour. Petitioner Serafin
Cheng agreed to lease his property located at 479 Shaw Blvd., Mandaluyong City to
respondents, Spouses Vittorio and Ma. Helen Donini, who intended to put up a
restaurant thereon, hence, they agreed to a monthly rental. However, before
respondents business could take off and before any final lease agreement could be
drafted and signed, the parties began to have serious disagreements regarding its
terms and conditions. Petitioner thus wrote respondents demanding payment of the
deposit and rentals, and signifying that he had no intention to continue with the
agreement should respondents fail to pay. Respondents, however, ignoring petitioners
demand, continued to occupy the premises until their caretaker voluntarily surrendered
the property to petitioner.

Issue:

Whether or not petitioner can be compelled to reimburse respondent of the full value of
expenses for their alleged repairs and improvements of the leased premises and value
of ornamental expenses.

Ruling:

The relationship between petitioner and respondents was explicitly governed by the
Civil Code provisions on lease, which clearly provide for the rule on reimbursement of
useful improvements and ornamental expenses after termination of a lease agreement
under Article 1678. Under Article 1678 of the Civil Code, the lessor has the primary right
(or the first move) to reimburse the lessee for 50% of the value of the improvements at
the end of the lease. If the lessor refuses to make the reimbursement, the subsidiary
right of the lessee to remove the improvements, even though the principal thing suffers
damage, arises. Consequently, on petitioner rests the primary option to pay for one-half
of the value of the useful improvements. It is only when petitioner as lessor refuses to
make the reimbursement that respondents, as lessees, may remove the improvements.
Should petitioner refuse to exercise the option of paying for one-half of the value of the
improvements, he cannot be compelled to do so. It then lies on respondents to insist on
their subsidiary right to remove the improvements even though the principal thing
suffers damage but without causing any more impairment on the property leased than is
necessary. As regards the ornamental expenses, respondents are not entitled to
reimbursement. Article 1678 gives respondents the right to remove the ornaments
without damage to the principal thing. But if petitioner appropriates and retains said
ornaments, he shall pay for their value upon the termination of the lease.

181
SPOUSES JIMENEZ VS. PATRICIA, INCORPORATED

G.R. No. 134651


September 18, 2000

Facts:

Petitioners Virgilio and Josie Jimenez, spouses, are sub-lessees of a lot and building
owned by respondent Patricia, Inc., a Filipino corporation. The Jimenez spouses
subleased the property in 1980 from a certain Purisima Salazar who had been leasing
the property from PATRICIA since 1970. Sometime in 1995 Purisima Salazar
abandoned the property thus incurring back rentals dating back to January 1992.
Hence, by reason of her non-payment of the monthly rentals, her contract of lease with
PATRICIA was terminated. On 29 March 1995 PATRICIA sent a letter to the Jimenez
spouses informing them of the termination of the lease and demanding that they vacate
the premises within fifteen (15) days from notice since they had no existing lease
contract with it. But the spouses refused to leave. Thus, on 5 May 1995 PATRICIA filed
a complaint for unlawful detainer against the Jimenez spouses alleging, among others,
that the lessee Purisima Salazar subleased the premises to the Jimenezes; that
Purisima Salazar no longer occupied the premises; that this notwithstanding, the
Jimenez spouses continued to occupy the premises without any contract with
PATRICIA, its owner, hence, their stay was merely being tolerated by the latter; and,
that despite demands made upon them, they refused to vacate the premises thereby
unlawfully and illegally withholding the property to the damage and prejudice of
PATRICIA. In their Answer, the Jimenez spouses claimed that they occupied the
premises as sublessees of Purisima Salazar with the knowledge of PATRICIA; that the
building originally found on the lot was owned by Purisima Salazar which she sold to
them in 1984 with notice and without any objection from PATRICIA; that, when the
building was gutted by fire in 1987 they constructed a new house on the lot with the
knowledge and without any objection from PATRICIA; and, that PATRICIA never
collected any rental for the land but they nevertheless voluntarily paid rents.

Issue:

Whether or not the petitioners can recover full reimbursement of the value spent for the
construction of the house.

Ruling:

The status of petitioner spouses is akin to that of a lessee or a tenant whose term of
lease has expired but whose occupancy has continued by tolerance of the owner. A
person who occupies the land of another at the latter's forbearance or permission
without any contract between them is necessarily bound by an implied promise that he
will vacate upon demand failing which a summary action for ejectment is the proper
remedy against him.

182
As to the house built by petitioners on the property, this Court has previously ruled that
lessees, much less, sub-lessees, are not possessors or builders in good faith over
rented land because they know that their occupancy of the premises continues only
during the life of the lease, or sublease as the case may be; and, they cannot as a
matter of right recover the value of their improvements from the lessor, much less retain
the premises until they are reimbursed. Instead, their rights are governed by Art. 1678
of the Civil Code which allows reimbursement of lessees up to one-half (1/2) of the
value of their improvements if the lessor so elects. Thus, petitioners cannot recover full
reimbursement of the value spent for the construction of the house, but is limited only to
one-half (1/2) of its value at the election of the lessor. However, as PATRICIA has
manifested its lack of intention to do so, the Jimenez spouses have no recourse but to
remove the house at their own expense.

183
NATIONAL HOUSING AUTHORITY VS. GRACE BAPTIST CHURCH

G.R. No. 156437


March 1, 2004

Facts:

On June 13, 1986, respondent Grace Baptist Church wrote a letter to petitioner National
Housing Authority, manifesting its interest in acquiring some parcels of lands. In its
reply, petitioner informed respondent that the request was granted and my process
application to purchase said lots. Respondent then possessed said lands and
introduced improvements. Later on, the NHA‘s Board of Directors passed a resolution
approving the sale of the subject lots to respondent Church. The Church then tendered
to the NHA a manager‘s check purportedly in full payment of the subject properties. The
Church insisted that this was the price quoted to them by the NHA Field Office, as
shown by an unsigned piece of paper with a handwritten computation scribbled thereon.
Petitioner NHA returned the check, stating that the amount was insufficient considering
that the price of the properties have changed. The Church made several demands on
the NHA to accept their tender of payment, but the latter refused. Thus, the Church
instituted a complaint for specific performance and damages against the NHA with the
Regional Trial Court it rendered its decision that there was no contract of sale between
the parties. Upon appeal to the CA, CA affirmed the trial court‘s finding that there was
indeed no contract of sale between the parties. However, petitioner was ordered to
execute the sale of the lots to Grace Baptist Church. The appellate court ruled that the
NHA‘s resolution which earlier approved the sale of the subject lots to Grace Baptist
Church has not been revoked at any time and was therefore still in effect. As a result,
the NHA was estopped from fixing a different price for the subject properties.
Considering further that the Church had been occupying the subject lots and even
introduced improvements thereon, the Court of Appeals ruled that, in the interest of
equity, it should be allowed to purchase the subject properties.

Issue:

Whether or not the NHA be compelled to sell the subject lots to Grace Baptist Church.

Ruling:

Petitioner NHA is not estopped from selling the subject lots at a price equal to their fair
market value, even if it failed to expressly revoke its resolution. The principle of estoppel
does not operate against the Government for the act of its agents, or, as in this case,
their inaction.

Appearing that there is no dispute that this case involves an unperfected contract, the
Civil Law principles governing contracts should apply. The offer of the NHA to sell the
subject property, as embodied in its resolution, was similarly not accepted by the
respondent. Thus, the alleged contract involved in this case should be more accurately

184
denominated as inexistent. There being no concurrence of the offer and acceptance, it
did not pass the stage of generation to the point of perfection. Equity cannot give validity
to a void contract, and this rule should apply with equal force to inexistent contracts.
The Church, despite knowledge that its intended contract of sale with the NHA had not
been perfected, proceeded to introduce improvements on the disputed land. On the
other hand, the NHA knowingly granted the Church temporary use of the subject
properties and did not prevent the Church from making improvements thereon. Thus,
the Church and the NHA, who both acted in bad faith, shall be treated as if they were
both in good faith. In this connection, Article 448 of the Civil Code shall apply.

185
ACCESSION
NATURAL

186
FERNANDO ET. AL., VS. ACUNA ET. AL.

G.R. No. 161030


September 14, 2011

Facts:

A parcel of land was registered in the names of Spouses Jose Fernando and Lucila
Tinio and spouses Antonia Fernando and Felipe Galvez. When they died intestate, the
property remained undivided. The heirs and successors-in-interest, herein petitioners
(Jose and Zoilo Fernando, Norma Fernando Banares, Rosario Fernando Tangkencgo,
heirs of Tomas Fernando, heirs of Guillermo Fernando, heirs of Iluminada Fernando
and heirs of Germogena Fernando) failed to agree on the division of the subject
property. Thus, except for the heirs of Germogena Fernando, filed a Complaint for
partition against the heirs of Germogena Fernando praying that the subject property be
partitioned into 8 equal parts corresponding to the hereditary interest of each group of
heirs. In his Complaint in intervention, 1998, respondent Leon Acuna averred that the
portion of the property identified as Lot 1303 was already adjudicated the petitioners'
predecessor-in-interest. He likewise claimed the portion identified as Lot 1302 was also
already adjudicated to other people as well. TC found that Lot 1302 was already titled in
the names of third persons. With respect to Lot 1303 TC found out that the November
1929 decision was never executed and has already prescribed. TC ordered the
reversion of Lot 1303 to the ownership of spouses Jose Fernando and Lucila Tinio and
spouses Antonia Fernando and Felipe Galvez and allowed the partition of Lot 1303
among petitioners as successors-in-interest of said registered owners. Excluded from
the partition, however, were the portions of the property which petitioners admitted had
been sold or transferred to Ruperta Sto. Domingo Villasenor and respondent Acuna.
With respect to Sapang Bayan, RTC found that the same had not been alleged in the
pleadings nor raised as an issue during the pre-trial conference. Also, according to the
trial court, the parties failed to clearly show whether Sapang Bayan was previously a dry
portion of either Lot 1302 or Lot 1303. Neither was there any proof that Sapang Bayan
was a river that just dried up or that it was an accretion which the adjoining lots
gradually received from the effects of the current of water. It was likewise not
established who were the owners of the lots adjoining Sapang Bayan. The trial court
concluded that none of the parties had clearly and sufficiently established their claims
over Sapang Bay.

All the parties, except Acuna, elevated this case to the CA which REVERSED and SET
ASIDE the desicsion. Hence, plaintiffs and defendants in the court to SC for review.

Issue:

Whether or not the petitioners can validly claim the ownership of the Sapang Bayan.

187
Ruling:

NO. CA erred in ruling that the principle of accretion is applicable. the Court of Appeals
erred in ruling that the principle of accretion is applicable ("to the owners of lands
adjoining the banks of rivers belong the accretion which they gradually receive from the
effects of the current of the waters." Art. 457 CC) the character of the Sapang Bayan
property was not shown to be of the nature that is being referred to in the provision
which is an accretion known as alluvium. In fact the parties could not agree how Sapang
Bayan ncame about. Whether it was a gradual deposit received from the river current or
a dried-up creek bed connected to the main river could not be ascertained. Even
assuming that Sapang Bayan was a dried-up creek bed, under Article 420, paragraph 1
and Article 502, of the CC, rivers and their natural beds are property of public dominion.
In the absence of any provision of law vesting ownership of the dried-up river bed in
some other person, it must continue to belong to the State.

188
OFFICE OF THE CITY MAYOR OF PARAÑAQUE CITY VS. MARIO D. EBIO

G.R. No. 178411


June 23, 2010

Facts:

Respondents claim that they are the absolute owners of a parcel of land consisting of
406 square meters, more or less, covered by Tax Declaration Nos. 01027 and 01472 in
the name of respondent Mario D. Ebio. Said land was an accretion of Cut-cut creek.
Respondents assert that the original occupant and possessor of the said parcel of land
was their great grandfather, Jose Vitalez. Sometime in 1930, Jose gave the land to his
son, Pedro Vitalez. From then on, Pedro continuously and exclusively occupied and
possessed the said lot. In 1966, after executing an affidavit declaring possession and
occupancy, Pedro was able to obtain a tax declaration over the said property in his
name. Since then, respondents have been religiously paying real property taxes for the
said property. On March 30, 1999, the Office of the Sangguniang Barangay of Vitalez
passed Resolution No. 08, series of 1999 seeking assistance from the City Government
of Parañaque for the construction of an access road along Cut-cut Creek located in the
said barangay. The proposed road, projected to be eight (8) meters wide and sixty (60)
meters long, will run from Urma Drive to the main road of Vitalez Compound traversing
the lot occupied by the respondents. On March 28, 2005, City Administrator Noli Aldip
sent a letter to the respondents ordering them to vacate the area within the next thirty
(30) days, or be physically evicted from the said property.

Issue:

Who owns the said lands adjoining Cutcut Creek particularly Road Lot No. 8 and the
accreted portion beside it?

Ruling:

Art. 457. To the owners of lands adjoining the banks of rivers belongs the accretion
which they gradually receive from the effects of the current of the waters. It is therefore
explicit from the foregoing provisions that alluvial deposits along the banks of a creek do
not form part of the public domain as the alluvial property automatically belongs to the
owner of the estate to which it may have been added. The only restriction provided for
by law is that the owner of the adjoining property must register the same under the
Torrens system; otherwise, the alluvial property may be subject to acquisition through
prescription by third persons. In contrast, properties of public dominion cannot be
acquired by prescription. No matter how long the possession of the properties has been,
there can be no prescription against the State regarding property of public domain.
Even a city or municipality cannot acquire them by prescription as against the State.
From these findings of fact by both the trial court and the Court of Appeals, only one
conclusion can be made: that for more than thirty (30) years, neither Guaranteed

189
Homes, Inc. nor the local government of Parañaque in its corporate or private capacity
sought to register the accreted portion. Undoubtedly, respondents are deemed to have
acquired ownership over the subject property through prescription. Respondents can
assert such right despite the fact that they have yet to register their title over the said lot.
It must be remembered that the purpose of land registration is not the acquisition of
lands, but only the registration of title which the applicant already possessed over the
land. Registration was never intended as a means of acquiring ownership. A decree of
registration merely confirms, but does not confer, ownership.

190
DIONISIA P. BAGAIPO VS. COURT OF APPEALS

G.R. No. 116290


December 8, 2000

Facts:

Petitioner Dionisia P. Bagaipo is the registered owner of Lot No. 415, a 146,900 square
meter agricultural land situated in Ma-a, Davao City while Respondent Leonor Lozano is
the owner of a registered parcel of land located across and opposite the southeast
portion of petitioner‘s lot facing the Davao River.On May 26, 1989, Bagaipo filed a
complaint for Recovery of Possession with Mandatory Writ of Preliminary Injunction and
Damages against Lozano for:(1) the surrender of possession by Lozano of a certain
portion of land measuring 29,162 square meters which is supposedly included in the
area belonging to Bagaipo under TCT No. T-15757; and(2) the recovery of a land area
measuring 37,901 square meters which Bagaipo allegedly lost when the Davao River
traversed her property. Bagaipo contended that as a result of a change in course of the
said river, her property became divided into three lots, namely: Lots 415-A, the area
presently occupied by Bagaipo, 415-B, which cut across Bagaipo‘s land was taken up
by the new course of the Davao River and 415-C, the land presently located across the
river and parallel to Bagaipo‘s property. The trial court concluded that the applicable law
is Article 457 of the New Civil Code and not Art. 461and dismissed the complaint. On
appeal, the Court of Appeals affirmed the decision of the trial court, hence the present
case.

Issue: Whether the land is owned by Bagaipo due to the changing of the river‘s course
or by Lozano by the principle of accretion.

Ruling:

The trial court and the appellate court both found that the decrease in land area was
brought about by erosion and not a change in the river‘s course. The decrease in
petitioner‘s land area and the corresponding expansion of respondent‘s property were
the combined effect of erosion and accretion respectively. Art. 461 of the Civil Code is
inapplicable. Petitioner cannot claim ownership over the old abandoned riverbed
because the same is inexistent. The riverbed‘s former location cannot even be
pinpointed with particularity since the movement of the Davao River took place
gradually over an unspecified period of time, up to the present. The rule is well-settled
that accretion benefits a riparian owner when the following requisites are present: 1)
That the deposit be gradual and imperceptible; 2) That it resulted from the effects of the
current of the water; and 3) That the land where accretion takes place is adjacent to the
bank of the river. These requisites were sufficiently proven in favor of respondents. In
the absence of evidence that the change in the course of the river was sudden or that it
occurred through avulsion, the presumption is that the change was gradual and was
caused by alluvium and erosion.

191
HEIRS OF EMILIANO NAVARRO VS. INTERMEDIATE APPELLATE COURT

G.R. No. 68166


February 12, 1997

Facts:

On October 3, 1946, Sinforoso Pascual, filed an application for foreshore lease covering
a tract of foreshore land in Sibocon, Balanga, Bataan, having an area of approximately
seventeen (17) hectares. This application was denied on January 15, 1953 and so was
his motion for reconsideration. Subsequently, petitioners' predecessor-in-interest,
Emiliano Navarro, filed a fishpond application with the Bureau of Fisheries covering
twenty five (25) hectares of foreshore land also in Sibocon, Balanga, Bataan. Initially,
such application was denied by the Director of Fisheries on the ground that the property
formed part of the public domain. Upon motion for reconsideration, the Director of
Fisheries, on May 27, 1958, gave due course to his application but only to the extent of
seven (7) hectares of the property as may be certified by the Bureau of Forestry as
suitable for fishpond purposes. The Municipal Council of Balanga, Bataan, had opposed
Emiliano Navarro's application. Aggrieved by the decision of the Director of Fisheries, it
appealed to the Secretary of Natural Resources who, however, affirmed the grant. The
then Executive Secretary, acting in behalf of the President of the Philippines, similarly
affirmed the grant.

Sometime in the early part of 1960, Sinforoso Pascual filed an application to register
and confirm his title to a parcel of land, situated in Sibocon, Balanga, Bataan, with an
area of 146,611 square meters. Pascual claimed that this land is an accretion to his
property, situated in Barrio Puerto Rivas, Balanga, Bataan. It is bounded on the eastern
side by the Talisay River, on the western side by the Bulacan River, and on the northern
side by the Manila Bay. The Talisay River as well as the Bulacan River flow
downstream and meet at the Manila Bay thereby depositing sand and silt on Pascual's
property resulting in an accretion thereon. Sinforoso Pascual claimed the accretion as
the riparian owner.

On March 25, 1960, the Director of Lands, represented by the Assistant Solicitor
General, filed an opposition thereto stating that neither Pascual nor his predecessors-in-
interest possessed has a sufficient title to the subject property, the same being a portion
of the public domain and, therefore, it belongs to the Republic of the Philippines. The
Director of Forestry, through the Provincial Fiscal, similarly opposed Pascual's
application for the same reason as that advanced by the Director of Lands. Later on,
however, the Director of Lands withdrew his opposition and the Director of Forestry
becomes the sole oppositor.

The case was decided adversely against Pascual. Thus, Pascual appealed to the Court
of First Instance (now Regional Trial Court) of Balanga, Bataan, rendered judgment
finding the subject property to be foreshore land and, being a part of the public domain,
it cannot be the subject of land registration proceedings. On appeal, the respondent

192
court reversed the findings of the court a quo and granted the petition for registration of
the subject property. Hence, this petition.

Issue:

Whether or not the accretion taking place on property adjacent to the sea can be
registered under the Torrens system.

Ruling:

No. The court held that the land in dispute is a land of the public domain. The claimed of
ownership of Pascual under Article 457 of the Civil Code is misplaced because if there‘s
any land to be claimed, it should be land adjacent to the rivers of Talisay and Bulacan.
The law is clear on this. Accretion of land along the river bank may be registered. This is
not the case of accretion of land on the property adjacent to Manila Bay. Furthermore,
Manila Bay is a sea. Accretion on a sea bank is foreshore land and the applicable law is
not Article 457 of the Civil Code but Article 4 of the Spanish Law of waters of 1866. This
law, while old, holds that accretion along sea shore cannot be registered as it remains
public domain unless abandoned by government for public use and declared as private
property capable of alienation.

193
DESAMPARADO VDA. DENAZARENO ET. AL., VS. COURT OF APPEALS

G.R. No. 98045


June 26, 1996

Facts:

The subject of this controversy is a parcel of land situated in Telegrapo, Puntod,


Cagayan de Oro City. Said land was formed as a result of sawdust dumped into the
dried-up Balacanas Creek and along the banks of the Cagayan River. Antonio
Nazareno caused the approval by the Bureau of Lands of the survey plan designated as
Plan Csd-106-00571 with a view to perfecting his title over the accretion area being
claimed by him. It was protested by private respondents hence an investigation was
conducted and the investigator recommended the survey plan no. MSI-10-06-000571-D
be cancelled. Upon the denial of the claim of Antonio, petitioners Desamparado Vda de
Nazareno et. al filed a case before RTC for annulment of order of investigation by
respondent Gillera, report and recommendation by respondent Labis, decision by
respondednt Hilario, order by respondent Ignacio affirming the decision of respondent
Palad. RTC and CA ruled against the petitioners. Hence, the petition.

Issue:

Whether or not the subject land is public land.

Ruling:

Yes. In the case at bar, the subject land was the direct result of the dumping of sawdust
by the Sun Valley Lumber Co. consequent to its sawmill operations. Even if this Court
were to take into consideration petitioners' submission that the accretion site was the
result of the late Antonio Nazareno's labor consisting in the dumping of boulders, soil
and other filling materials into the Balacanas Creek and Cagayan River bounding his
land, the same would still be part of the public domain.

Having determined that the subject land is public land, a fortiori, the Bureau of Lands,
as well as the Office of the Secretary of Agriculture and Natural Resources have
jurisdiction over the same in accordance with the Public Land Law. Accordingly, the
court a quo dismissed petitioners' complaint for non-exhaustion of administrative
remedies which ruling the Court of Appeals affirmed. Wherefore, the petition is
dismissed.

194
SPOUSES CACHOPERO VS. RACHEL CELESTIAL

G.R. No. 146754


March 21, 2012

Facts:

Petitioner Jesse Cachopero, married to co-petitioner Bema Cachopero (spouses


Cachopero), is the younger brother of respondent Rachel Celestial (Celestial). Celestial
owned an old residential house (old house) situated on a lot which was formerly part of
the Salunayan Creek that became dry as a result of the construction of an irrigation
canal by the National Irrigation Administration.

On July 21, 1989, Celestial filed an Ejectment case against the spouses Cachopero
alleging that the spouses had been living in her house for free and out of tolerance
since 1973.

Cachopero was trying to obtain a Miscellaneous Sales Application (MSA) to the


Department of Environment and Natural Resources (DENR) alleging that he had been
the owner of that land whereon he built a house and other improvements. However,
Celestial protests that she has preferential right over the land because it is adjacent to
and is the only outlet from her house. According to the Bureau of Land, the land in
dispute was a creek and is therefore outside the commerce of man. The first MSA was
denied by the Municipal Trial Court (MTC) prompting Cachopero to obtain another MSA
which was granted by the DENR. Due to conflicting interests of the parties, the land in
dispute must be sold in a public auction.

Issue:

Whether or not the parties can claim ownership over the subject land.

Ruling:

NO. A dried up creek is property of public dominion and not susceptible to acquisitive
prescription. Since property of public dominion is outside the commerce of man and not
susceptible to private appropriation and acquisitive prescription, the adverse possession
which may be the basis of a grant of title in the confirmation of an imperfect title refers
only to alienable or disposable portions of the public domain. It is only after the
Government has declared the land to be alienable and disposable agricultural land that
the year of entry, cultivation and exclusive and adverse possession can be counted for
purposes of an imperfect title. Therefore, celestial cannot claim ownership over the
property based on adverse possession.

195
MAXIMO JAGUALING, ET. AL., VS. COURT OF APPEALS

G.R. No. 94283


March 4, 1991

Facts:

The parties to this case dispute the ownership of a certain parcel of land located in Sta.
Cruz, Tagoloan, Misamis Oriental with an area of 16,452 square meters, more or less,
forming part of an island in a non-navigable river.

Private respondents filed with the Regional Trial Court of Misamis Oriental an action to
quiet title and/or remove a cloud over the property in question against petitioners
claiming that she inherited the land from his father and acquired sole ownership of the
property by virtue of a Deed of Extra Judicial Partition with sale. Defendants, on the
other hand, asserted that they are the real owners of the land in litigation containing an
area of 18,000 square meters more or less. During the typhoon Ineng in 1964 the river
control was washed away causing the formation of an island, which is now the land in
litigation.

On 17 July 1987 the trial court dismissed the complaint for failure of private respondents
as plaintiffs therein to establish by preponderance of evidence their claim of ownership
over the land in litigation. The court found that the island is a delta forming part of the
river bed which the government may use to reroute, redirect or control the course of the
Tagoloan River. Accordingly, it held that it was outside the commerce of man and part
of the public domain, citing Article 420 of the Civil Code. As such it cannot be registered
under the land registration law or be acquired by prescription. The trial court, however,
recognized the validity of petitioners' possession and gave them preferential rights to
use and enjoy the property.

Issue:

Between the one who has actual possession of an island that forms in a non-navigable
and non-flotable river and the owner of the land along the margin nearest the island,
who has the better right thereto?

Ruling:

It is well-settled that lands formed by accretion belong to the riparian owner. This
preferential right is, under Article 465, also granted the owners of the land located in the
margin nearest the formed island for the reason that they are in the best position to
cultivate and attend to the exploitation of the same. In fact, no specific act of possession
over the accretion is required. If, however, the riparian owner fails to assert his claim
thereof, the same may yield to the adverse possession of third parties, as indeed even
accretion to land titled under the Torrens system must itself still be registered.

196
Petitioners may therefore, acquire said property by adverse possession for the required
plumber of years under the doctrine of acquisitive prescription. Their possession cannot
be considered in good faith, however, because they are presumed to have notice of the
status of private respondents as riparian owners who have the preferential right to the
island as recognized and accorded by law; they may claim ignorance of the law,
specifically Article 465 of the Civil Code, but such is not, under Articles 3 and 526 of the
same code, an adequate and valid defense to support their claim of good faith. Hence,
not qualifying as possessors in good faith, they may acquire ownership over the island
only through uninterrupted adverse possession for a period of thirty years. By their own
admission, petitioners have been in possession of the property for only about fifteen
years. Thus, by this token and under the theory adopted by petitioners, the island
cannot be adjudicated in their favor.

The court ruled that the petition is not technically an action in rem or an action in
personam, but characterized as quasi in rem which is an action in personam concerning
real property. Thus, the judgment in proceedings of this nature is conclusive only
between the parties and does not bind the State or the other riparian owners who may
have an interest over the island involved herein.

197
CYNTHIA CRUZ KHEMANI VS. THE HEIRS OF ANASTACIO TRINIDAD

G.R. No. 147340


December 13, 2007

Facts:

Petitioner Cynthia Cruz Khemani is the registered owner of Lot No. 107. Respondents
are claiming ownership alleging that they and their predecessors-in-interest, Spouses
Anastacio and Francisca Trinidad, have openly, peacefully, publicly and adversely
possessed the Disputed Property in the concept of owner since 1950.

Lot No. 107 and Lot Nos. 108 and 109, constitute Lot No. 355 which was part of the
public domain. On July 10, 1950, Lot No. 355 with an original area of 1,500 square
meters was awarded to Jesus M. Larrabaster by the National Land Settlement
Administration (NLSA) who subsequently sold his rights and interests over the said
property to Jose B. Peña (Peña) on June 29, 1956.

Thereafter, the original area of Lot No. 355 which was 1,500 square meters increased to
3,616.93 square meters due to accretion. Peña then requested the Bureau of Lands
(BOL) to adjust the area of the lot awarded to him but the BOL denied the request on
the ground that the accretion belonged to the government.

In the case of Assistant Executive Secretary for Legal Affairs of the Office of the
President v. Court of appeals, the Supreme Court rejected Mendoza‘s claim over Lot
No. 107 and found the Miscellaneous Sales Application without legal force and effect
since the object thereof was no longer public land. Thus, Peña‘s right of ownership over
the entire area of Lot No. 355, which consists of Lot Nos. 107, 108 and 109, was
affirmed. Peña‘s Heirs were awarded a patent by the Department of Environment and
Natural Resources (DENR)and an OCT was issued in their name.

On January 27, 1994, respondents filed with the RTC a verified complaint arguing that
they have been in open, peaceful, public and adverse possession of the Disputed
Property in the concept of owner since 1950; that the patent and original certificate of
title were fraudulently issued in favor of the Peña Heirs; and that their action for review
of decree of registration and/or reconveyance is not barred by the Court‘s ruling in
Assistant Executive Secretary.

Issue:

Whether or not respondent‘s right to file an action for review of decree is barred.

Ruling:

No. Respondents‘ action for review of decree of registration is sanctioned under Section
32 of Presidential Decree No. 1529, which provides that a person deprived of his land

198
through actual fraud may institute an action to reopen or review a decree of registration
within one year from entry of such decree.

In this case, the patent was issued in favor of the Peña Heirs on September 20, 1993.
Respondents filed Civil Case No. 1122 for "Review of Decree of Registration and/or
Reconveyance with Prayer for Issuance of Writ of Preliminary Prohibitory Injunction and
Temporary Restraining Order" on January 27, 1994, or well within the prescribed one-
year period. Likewise, records show that TCT No. 58976 under petitioner‘s name bears
a Notice of Lis Pendens. Thus, it cannot be said that petitioner is an innocent purchaser
for value as she was well aware of respondents‘ claim over the Disputed Property.
Further, even assuming arguendo that respondents filed their action after one year, they
may still be entitled to relief. An aggrieved party may file an action for reconveyance
based on implied or constructive trust, which prescribes in ten years from the date of the
issuance of the certificate of title over the property provided that the property has not
been acquired by an innocent purchaser for value.

After the court‘s ruling in the Assistant Executive Secretary case in 1989, the BOL
issued a Patent on September 20, 1993 in favor of the Peña Heirs which became the
basis for the issuance of OCT No. P-33658 covering Lot No. 107. However, as held in
the Assistant Executive Secretary case, Lot No. 107 – as accretions to the original lot
(Lot No. 355) awarded to Larrabaster on July 10, 1950 – "no longer belonged to the
Government[,] the subdivision thereof by the Bureau of Lands into three lots (Lot No.
107, Lot No. 108 and Lot No. 109), as well as the allocation of said lots to two other
individuals, was beyond the scope of its authority." As a result, while Lot No. 107 may
no longer be acquired under the provisions of the Public Land Act, it does not absolutely
foreclose the possibility that, as a private property, a portion thereof (the Disputed
Property) may have been acquired by respondents through acquisitive prescription
under the Civil Code. These matters, however, are the proper subject of a separate
action should one be filed subject, of course, to such claims and defenses that either
party may have under relevant laws.

The court ruled that it would be premature to order the dismissal of respondents‘
complaint as they have yet to be given an opportunity to substantiate their claims.
Noting that respondents are in actual physical possession of the Disputed Property up
to this date, and the fact of their physical possession over many years is not disputed by
petitioner. Under the circumstances, it would be more in keeping with the standards of
fairness to have a full-blown trial where the evidentiary matters are threshed out.

199
EULOGIO AGUSTIN, ET. AL., VS. INTERMEDIATE APPELLATE COURT

G.R. Nos. L-66075-76


July 5, 1990

Facts:

The Cagayan River separates the towns of Solana on the west and Tuguegarao on the
east in the province of Cagayan. As the years went by, the Cagayan River moved
gradually eastward, depositing silt on the western bank. The shifting of the river and the
siltation continued until 1968. Private respondents, Maria Melad and Pablo Binuyag are
among those who are occupying the western bank of the Cagayan River while on the
eastern bank is owned by petitioner Eulogio Agustin. From 1919 to 1968, the Cagayan
river has eroded the lands on the eastern bank including Agustin‘s Lot depositing
alluvium on the land possessed by Pablo Binuyag. In 1968, after a typhoon which
caused a big flood, the Cagayan River changed its course and returned it to its 1919
bed and it cut through the lands of respondents whose lands were transferred on the
eastern side. To cultivate the lands they had to cross the river. When they were
cultivating said lands, (they were planting corn) Agustin accompanied by the mayor and
some policemen claimed the land and drove them away. So Melad and Binuyag filed
separate complaints for recovery of their lots and its accretions. The Trial Court held
ordered Agustin et. al to vacate the lands and return them to respondents. On appeal,
the IAC affirmed in toto the judgment thus the case at bar.

Issue:

Whether or not private respondents own the accretion and such ownership is not
affected by the sudden and abrupt change in the course of the Cagayan River when it
reverted to its old bed.

Ruling:

The court ruled in the positive.

Art. 457 states that the owner of the lands adjoining river banks own the accretion which
they gradually receive from the effects of the currents of the waters. Accretion benefits a
riparian owner provided that these elements are present: 1) deposit be gradual and
imperceptible 2) it resulted from the effects of the current of the water and 3) the land is
adjacent to the river bank. When the River moved from 1919 to 1968, there was
alluvium deposited and it was gradual and imperceptible.

Accretion benefits the riparian owner because these lands are exposed to floods and
other damage due to the destructive force of the waters, and if by virtue of law they are
subject to encumbrances and various kinds of easements, it is only just that such risks
or dangers should in some way be compensated by the right of accretion. Also,
respondent‘s ownership over said lots was not removed when due to the sudden and

200
abrupt change in the course of the river; their accretions were transferred to the other
side. Art. 459 states when the current of a river x x x segregates from an estate on its
bank a known portion of land and transfers it to another estate, the owner of segregated
portion retains ownership provided he removes the same w/in 2 years. And Art. 463
states that whenever the current of a river divides itself into branches, leaving a piece of
land or part thereof isolated, the owner of the land retains ownership. He also retains it if
a portion of land is separated from the estate by the current.

201
MARCELINO C. AGNE VS. THE DIRECTOR OF LANDS

G.R. No. L-40399


February 6, 1990

Facts:

The land subject matter of the case was originally covered by Free Patent 23263 issued
on 17 April 1937 in the name of Herminigildo Agpoon. On 21 May 1937, pursuant to the
said patent, the Register of Deeds of Pangasinan issued to said Herminigildo Agpoon
OCT 2370. Presentacion Agpoon Gascon inherited the said parcel of land upon the
death of her father, Herminigildo, and was issued TCT 32209 on 6 April 1960.
Presentacion declared the said land for taxation purposes in her name under TD 11506
and taxes were paid thereon in her name. On 13 April 1971, spouses Joaquin and
Presentacion Gascon filed Civil Case U-2286 in the then CFI Pangasinan for recovery
of possession and damages against Marcelino C. Agne, et al. Their complaint states
that they are the registered owners under TCT 32209 of the parcel of land which is now
in the possession of Agne, et.al.; that during the Japanese occupation, the latter, taking
advantage of the abnormal conditions then obtaining, took possession of said land by
means of fraud, stealth, strategy and intimidation; that Gascon repeatedly demanded
the surrender of the physical possession of said property but the latter refused. Agne,
et.al. alleged that the land in question was formerly a part of the river bed of the Agno-
Chico River; that in the year 1920, a big flood occurred which caused the said river to
change its course and abandon its original bed; that by virtue of the provisions of Article
370 of the Spanish Civil Code which was then the law in force, Agne, et.al., by operation
of law, became the owners by accession or accretion of the respective aliquot parts of
said river bed bordering their properties; that since 1920, they and their predecessors in
interest occupied and exercised dominion openly and adversely over said portion of the
abandoned river bed in question abutting their respective riparian lands continuously up
to the present to the exclusion of all other persons, particularly Herminigildo Agpoon;
that they have introduced improvements thereon by constructing irrigation canals and
planting trees and agricultural crops thereon and converted the land into a productive
area.

On 6 March 1974, while the above case was still pending, Agne, et.al. filed a complaint
against Director of Lands and spouses Agpoon with the former CFI Pangasinan for
annulment of title, reconveyance of and/or action to clear title to a parcel of land, which
action was docketed as Civil Case U-2649. Agne, et. al. alleged in their said complaint
that the land in question, which was formerly a portion of the bed of Agno-Chico river
which was abandoned as a result of the big flood in 1920, belongs to them pursuant to
the provision of Article 370 of the old Civil Code; that it was only on 13 April 1971, when
spouses filed a complaint against them, that they found out that the said land was
granted by the Government to Herminigildo Agpoon under Free Patent 23263, pursuant
to which OCT 2370 was issued in the latter‘s name; and that the said patent and
subsequent titles issued pursuant thereto are null and void since the said land, an
abandoned river bed, is of private ownership and, therefore, cannot be the subject of a

202
public land grant. On 21 June 1974, the trial court rendered a decision in Civil Case U-
2286, ordering Agne, et.al. to surrender physical possession of land in question.

Issue:

Who is the owner of the property?

Ruling:

The Supreme Court reversed and set aside the assailed decision of IAC in AC-GR CV
60388-R and the questioned order of dismissal of the trial court in its Civil Case 2649,
and rendered judgment ordering the Gascon to reconvey the parcel of land to Agne,
et.al.The court ruled that Agne owns the property. Riparian owners ipso facto owners of
abandoned river . The acquisition of ownership is automatic. There need be no act on
the part of the riparian owners to subject the accession to their ownership, as it is
subject thereto ipso jure from the moment the mode of acquisition becomes evident,
without the need of any formal act of acquisition. Such abandoned river bed had fallen
to the private ownership of the owner of the riparian land even without any formal act of
his will and any unauthorized occupant thereof will be considered as a trespasser. The
right in re to the principal is likewise a right in re to the accessory, as it is a mode of
acquisition provided by law, as the result of the right of accretion. Since the accessory
follows the nature of the principal, there need not be any tendency to the thing or
manifestation of the purpose to subject it to our ownership, as it is subject thereto ipso
jure from the moment the mode of acquisition becomes evident. The right of the owner
of land to additions thereto by accretion has been said to rest in the law of nature, and
to be analogous to the right of the owner of a tree to its fruits, and the owner of flocks
and herds to their natural increase. In the present case, Agne, et.al. became owners of
aliquot portions of said abandoned river bed as early as 1920, when the Agno River
changed its course, without the necessity of any action or exercise of possession on
their part, it being an admitted fact that the land in dispute, prior to its registration, was
an abandoned bed of the Agno River and that Agne, et. al. are the riparian owners of
the lands adjoining the said bed.

Moreover, imprescriptibility does not apply as title based on public grant. The failure of
Agne et.al. to register the accretion in their names and declare it for purposes of
taxation did not divest it of its character as a private property. Although an accretion to
registered land is not automatically registered and therefore not entitled or subject to the
protection of imprescriptibility enjoyed by registered property under the Torrens system.
The said rule is not applicable to the case since the title claimed by spouses Gascon is
not based on acquisitive prescription but is anchored on a public grant from the
Government, which presupposes that it was inceptively a public land. Ownership over
the accession is governed by the Civil Code. The court further stated, the President of
the Philippines or his alter ego, the Director of Lands, has no authority to grant a free
patent for land that has ceased to be a public land and has passed to private ownership,
and a title so issued is null and void.

203
LEOGARIO RONQUILLO, ET AL. VS. JOSE ROCO, ET AL.

G.R. No. L-10619


February 28, 1958

Facts:

The plaintiffs alleged that they have been in the continuous and uninterrupted use of a
road or passage way which traversed the land of the defendants and their predecessors
in interest, in going to Igualdad Street and the market place of Naga City, from their
residential land and back, for more than 20 years. Plaintiffs further claim that defendants
have long recognized and respected the private legal easement of road right of way of
the former. On May 12, 1953, the defendants and their men constructed a chapel in the
middle of the said right of way which, accordingly has impeded, obstructed and
disturbed the continuous exercise of the rights of the plaintiffs over said right of way. On
July 10, 1954 defendants planted wooden posts, fenced with barbed wire and closed
hermitically the road passage way and their right of way against the plaintiff‘s protests
and opposition. This prevented them from going to or coming from their homes to
Igualdad Street and the public market of the City of Naga.

Issue:

Whether or not an easement of right of way can be acquired through prescription

Ruling:

An easement of right of way though it may be apparent is, nevertheless, discontinuous


or intermittent and, therefore, cannot be acquired through prescription, but only by virtue
of a title. Under the New Civil Code, easements may be continuous discontinuous
(intermittent),apparent or non-apparent, discontinuous being those used at more or less
long intervals and which depend upon acts of man (Articles 615). Continuous and
apparent easements are acquired either, by title or prescription, continuous non-
apparent easements and discontinuous ones whether apparent or not, may be acquired
only by virtue of a title (Articles 620 and622). Under the provisions of the Civil Code,
particularly the articles thereof aforecited, it would therefore appear that the easement
of right of way may not be acquired through prescription. Even Article 1959 of the Old
Civil Code providing for prescription of ownership and other real rights in real property,
excludes there from the exception established by Article 539, referring to discontinuous
easements, such as, easement of right of way. (Bargayo vs. Camumot, 40 Phil., 857,
867)

204
HILARIO VS. CITY OF MANILA

G.R. No. L-19570


April 27, 1967

Facts:

Dr. Jose Hilario was the registered owner of a large tract of land around 49 hectares in
area (Barrio Guinayang, San Mateo, Rizal). Upon his death this property was inherited
by his son, Jose Hilario, Jr., to whom a new certificate of title was issued. During the
lifetime of plaintiff‘s father, the Hilario estate was bounded on the western side by the
San Mateo River. To prevent its entry into the land, a bamboo and lumber post dike or
ditch was constructed on the northwestern side. This was further fortified by a stonewall
built on the northern side. For years, these safeguards served their purpose. However,
in 1937, a great and extraordinary flood occurred which inundated the entire place
including the neighboring barrios and municipalities. The River destroyed the dike on
the northwest, left its original bed and meandered into the Hilario estate, segregating
from the rest thereof a lenticular piece of land. The disputed area is on the eastern side
of this lenticular strip which now stands between the old riverbed site and the new
course. In 1945, the US Army opened a sand and gravel plant within the premises, and
started scraping, excavating and extracting soil, gravel and sand from the nearby areas
along the River. The operations eventually extended northward into the strip of land.
Consequently, a claim for damages was filed with the US War Department by Luis
Hidalgo, the then administrator of Dr. Hilario‘s estate. The US Army paid. In 1947, the
plant was turned over to herein defendants-appellants and appellee who took over its
operations.

On 22 October 22, 1949, plaintiff filed his complaint for injunction and damages against
the defendants City Engineer of Manila, District Engineer of Rizal, the Director of Public
Works, and Engr. Busuego, the Engineer-in-charge of the plant. Subsequently, the
Bureau of Mines and Atty. Maximo Calalang were respectively allowed to join the
litigation as intervenors; as per issue of fees and penalties for materials (sand and
gravel) extracted.

On 14 March 1954, defendants filed a petition for injunction against plaintiff and
intervenor Calalang in the same case, alleging that the latter have fenced off the
disputed area in contravention of an agreement had between the latter and the Director
of Public Works wherein the defendants were allowed to continue their operations but
subject to the final outcome of the pending suit.

On 21 December 1956, the lower court rendered its decision, ordering the City of Manila
and Director of Public Works to pay Hilario in solidum the sum of P376,989.60 as cost
of gravel and sand extracted from the plaintiff‘s land, plus costs; and ordering the
Provincial Treasurer of Rizal to reimburse intervenor Calalang of P36.80 representing
gravel fees illegally collected. None of the parties litigants seemed satisfied with this
decision and they all sought a reconsideration of the same.

205
On August 30, 1957, the lower court resolved the motions to reconsider with an order,
holding that the 2/5 portion of the area in controversy to Hilario, and dismissing the case
against the Bureau of Public Works insofar as money claims are concerned without
prejudice to Hilario taking action against proper party in such claim. Hilario and Calalang
filed a second motion for reconsideration, which the lower court denied.

Hence, the appeal.

The Supreme Court set aside the decision and orders appealed from, and entered
another judgment to the effect that the City of Manila and the Director of Public Works,
and his agent and employees, are absolved of liability from extracting materials from
subject property (of public domain); and the portion within the strip of land question
declared not part of public domain and confirmed as part of Hilario‘s private property.
No Costs.

Issue:

Whether or not the new civil code shall apply to the present case

Ruling:

Since the change in the course of the River took place in 1937, long before the present
Civil Code took effect,the question before Us should be determined in accordance with
the provisions of the old Civil Code and those of the Law of Waters of August 3, 1866.
We agree with defendants that under the cited laws, all riverbanks are of public
ownership — including those formed when a river leaves its old bed and opens a new
course through a private estate. Art. 339 of the old Civil Code is very clear. Without any
qualifications, it provides: Property of public ownership is —1. That devoted to public
use, such as roads, canals, rivers, torrents, ports and bridges constructed by the State,
riverbanks, shores, roadsteads, and that of a similar character;

Moreover, as correctly contended by defendants, the riverbank is part of the riverbed.


Art. 73 of the Law of Waters which defines the phrase "banks of a river" provides:
By the phrase "banks of a river" is understood those lateral strips or zones of its bed
which are washed by the stream only during such high floods as do not cause
inundations.

The use the of words "of its bed [de sus alveos]" clearly indicates the intent of the law to
consider the banks — for all legal purposes — as part of the riverbed. The lower court
also ruled — correctly — that the banks of the River are paint of its bed.20 Since
undeniably all beds of rivers are of public ownership, it follows that the banks, which
form part of them, are also of public ownership.

Plaintiff's contention that Arts. 70 and 73 of the Law of Waters cannot apply because
Art. 312 of the old Civil Code mentions only the new bed but omits the banks, and that
said articles only apply to natural — meaning original — bed and banks is untenable.

206
Art. 70, which define beds of rivers and creeks, provide: The natural bed or channel of a
creek or river is the ground covered by its waters during the highest [ordinary] floods.

Art. 372 of the old Civil Code which provides that —Whenever a navigable or floatable
river changes its course from natural causes and opens a new bedthrough a private
estate, the new bed shall be of public ownership, but the owner of the estate shall
recover it in the event that the waters leave it dry again either naturally or as the result
of any work legally authorized for this purpose.

It did not have to mention the banks because it was unnecessary. The nature of the
banks always follows that of the bed and the running waters of the river. A river is a
compound concept consisting of three elements: (1) the running waters, (2) the bed and
(3) the banks. All these constitute the river.

The public nature of riverbanks still obtained only by implication. But with the
promulgation of the Civil Code of 1889, this fact was finally made explicit in Art. 339
thereof. Riverbanks were declared as public property since they were destined for
public use. And the first paragraph of Art. 36 of the Law of Waters of 1879 was
substantially reenacted in Art. 553 of the Code.36 Hence, this article must also be
understood not as authorizing the private acquisition of riverbanks but only as
recognizing the vested titles of riparian owners who already owned the banks.
The authority, then, for the private ownership of the banks is neither the old Civil Code
nor the Law of Waters of 1866 but the Siete Partidas. Unfortunately, plaintiff cannot
invoke it. Law 6, Title 28, Partida 3, which provides for private ownership of banks,
ceased to be of force in this jurisdiction as of 1871 yet when the Law of Waters of
August 3, 1866, took effect. Since the change in the course of the River took place in
1937, the new banks which were formed could not have been subjected to the
provisions of the Siete Partidas which had already been superseded by then.

207
QUIETING OF TITLE

208
RESIDENTS OF LOWER ATAB, ET. AL. VS. STA. MONICA INDUSTRIAL &
DEVELOPMENT CORPORATION

G.R. No.198878
October 15, 2014

Facts:

In May 2001, petitioners – residents of Lower Atab & Teachers‘ Village, Sto. Tomas
Proper Barangay, Baguio City – filed a civil case for quieting of title with damages
against respondent Sta. Monica Industrial and DevelopmentCorporation. The case was
docketed as Civil Case No. 4946-R and assigned to Branch 59 of the Baguio RTC.6
The Complaint7 in said case essentially alleged that petitioners are successors and
transferees-in-interest of Torres, the supposed owner of an unregistered parcel of land
in Baguio City (the subject property, consisting of 177,778 square meters) which Torres
possessed and declared for tax purposes in 1918; that they are in possession of the
subject property in the concept of owner, declared their respective lots and homes for
tax purposes, and paid the real estate taxes thereon; that in May 2000, respondent
began to erect a fence on the subject property, claiming that it is the owner of a large
portion thereof8 by virtue of Transfer Certificate of Title No. T-631849 (TCT No. T-
63184); that said TCT No. T-63184 is null and void, as it was derived from Original
Certificate of Title No. O-281 (OCT No. O-281), which was declared void pursuant to
Presidential Decree No. 127110 (PD 1271) and in the decided case of Republic v.
Marcos;11 and that TCT No. T-63184 is a cloud upon their title and interests and should
therefore be cancelled. Petitioners thus prayed that respondent‘s TCT No. T-63184 be
surrendered and cancelled; that actual, moral and exemplary damages, attorney‘s fees,
legal expenses, and costs be awarded in their favor; and finally, that injunctive relief be
issued against respondent to prevent it from selling the subject property.

Issue:

Whether or not petitioners have equitable title

Ruling:

For an action to quiet title to prosper, two indispensable requisites must be present,
namely: "(1) the plaintiff or complainant has a legal or an equitable title to or interest in
the real property subject of the action; and (2) the deed, claim, encumbrance, or
proceeding claimed to be casting cloud on his title must be shown to be in fact invalid or
inoperative despite its prima facie appearance of validity or legal efficacy."

"Legal title denotes registered ownership, while equitable title means beneficial
ownership."

Beneficial ownership has been defined as ownership recognized by law and capable of
being enforced in the courts at the suit of the beneficial owner. Black‘s Law Dictionary

209
indicates that the term is used in two senses: first, to indicate the interest of a
beneficiary in trust property (also called "equitable ownership"); and second, to refer to
the power of a corporate shareholder to buy or sell the shares, though the shareholder
is not registered in the corporation‘s books as the owner. Usually, beneficial ownership
is distinguished from naked ownership, which is the enjoyment of all the benefits and
privileges of ownership, as against possession of the bare title to property.

Petitioners do not have legal or equitable title to the subject property. Evidently, there
are no certificates of title in their respective names. And by their own admission in their
pleadings, specifically in their pre-trial brief and memorandum before the trial court, they
acknowledged that they applied for the purchase of the property from the government,
through townsite sales applications coursed through the DENR. In their Petition before
this Court, they particularly prayed that TCT No. T-63184 be nullified in order that the
said title would not hinder the approval of their townsite sales applications pending with
the DENR.Thus, petitioners admitted that they are not the owners of the subject
property; the same constitutes state or government land which they would like to
acquire by purchase. It would have been different if they were directly claiming the
property as their own as a result of acquisitive prescription, which would then give them
the requisite equitable title. By stating that they were in the process of applying to
purchase the subject property from the government, they admitted that they had no
such equitable title, at the very least, which should allow them to prosecute a case for
quieting of title.

In short, petitioners recognize that legal and equitable title to the subject property lies in
the State. Thus, as to them, quieting of title is not an available remedy.

Lands within the Baguio Townsite Reservation are public land. Laws and decrees such
as PD 1271 were passed recognizing ownership acquired by individuals over portions of
the Baguio Townsite Reservation, but evidently, those who do not fall within the
coverage of said laws and decrees – the petitioners included – cannot claim ownership
over property falling within the said reservation. This explains why they have pending
applications to purchase the portions of the subject property which they occupy; they
have no legal or equitable claim to the same, unless ownership by acquisitive
prescription is specifically authorized with respect to such lands, in which case they may
prove their adverse possession, if so. As far as this case is concerned, the extent of
petitioners‘ possession has not been sufficiently shown, and by their application to
purchase the subject property, it appears that they are not claiming the same through
acquisitive prescription.

210
HEIRS OF PACIFICO POCDO VS. ARSENIA AVILA AND EMELINDA CHUA

G.R. No. 199146


March 19, 2014

Facts:

In June 2000, Pacifico Pocdo, who was later substituted by his heirs upon his death,
filed a complaint to quiet title over a 1,728–square meter property (disputed property)
located in Camp 7, Baguio City, and covered by Tax Declaration 96–06008–106641.
Pacifico claimed that the disputed property is part of Lot 43, TS–39, which originally
belonged to Pacifico‘s father, Pocdo Pool. The disputed property is allegedly different
from the one–hectare portion alloted to Polon Pocdo, the predecessor–in–interest of the
defendants Arsenia Avila and Emelinda Chua, in a partition made by the heirs of Pocdo
Pool. Pacifico alleged that the defendants unlawfully claimed the disputed property,
which belonged to Pacifico.

Issue:

Whether or not petitioners have equitable title over the disputed land to apply for
quieting of title.

Ruling:

In an action for quieting of title, the complainant is seeking for ―an adjudication that a
claim of title or interest in property adverse to the claimant is invalid, to free him from the
danger of hostile claim, and to remove a cloud upon or quiet title to land where stale or
unenforceable claims or demands exist.‖ Under Articles 476 and 477 of the Civil Code,
the two indispensable requisites in an action to quiet title are: (1) that the plaintiff has a
legal or equitable title to or interest in the real property subject of the action; and (2) that
there is a cloud on his title by reason of any instrument, record, deed, claim,
encumbrance or proceeding, which must be shown to be in fact invalid or inoperative
despite its prima facie appearance of validity.

In this case, petitioners, claiming to be owners of the disputed property, allege that
respondents are unlawfully claiming the disputed property by using void documents,
namely the ―Catulagan‖ and the Deed of Waiver of Rights. However, the records reveal
that petitioners do not have legal or equitable title over the disputed property, which
forms part of Lot 43, a public land within the Baguio Townsite Reservation. It is clear
from the facts of the case that petitioners‘ predecessors–in–interest, the heirs of Pocdo
Pool, were not even granted a Certificate of Ancestral Land Claim over Lot 43, which
remains public land. Thus, the trial court had no other recourse but to dismiss the case.
There is no more need to discuss the other issues raised since these are intrinsically
linked to petitioners‘ action to quiet title.

211
TEOFISTO ONO, ETC. VS. VICENTE LIM

G.R. No. 154270


March 9, 2010

Facts:

1992, Lim filed in RTC Cebu a petition for reconstitution of the owner's duplicate copy of
OCT, alleging that the same OCT was lost during World War 2 by his mother, Luisa.
This land was located in Balamban, Cebu which was sold to Luisa by spouses Ono.
Although the deed evidencing the sale was lost, the only legitimate son of Ono had
executed a notarized document in favor of Luisa denominated as confirmation of the
sale which was duly filed in Provincial Assessor's Office of Cebu.

Now, Spouses Ono's successors-in-interest opposed Lim's petition contending that they
had the certificate of title of the land.

Lim then converted the petition into a complaint for quieting of title, averring that they
had been in actual possession of the property since 1973, cultivating and developing it,
enjoying its fruits and paying taxes corresponding to it.

The other party claimed that the land was never sold to Luisa, and that the confirmation
by the legitimate son was fabricated, the signature not being authentic.

RTC ruled in favor of Lim. CA affirmed the RTC. The CA ruled that the action for
quieting of title was not a collateral, but a direct attack on the title; and that the Lims'
undisturbed possession had given them a continuing right to seek the aid of the courts
to determine the nature of the adverse claim of a third party and its effect on their own
title.

Issues:

1. Whether or not the validity of the OCT could be collaterally attacked through an
ordinary civil action to quiet title; and
2. Whether or not the ownership over registered land could be lost by prescription,
laches, or adverse possession;

Ruling:

Petition has no merit.

1. Action for cancellation of title is not an attack on the title. The attack is direct
when the objective is to annul or set aside such judgment, or enjoin its
enforcement. On the other hand, the attack is indirect or collateral when, in an
action to obtain a different relief, an attack on the judgment is nevertheless made
as an incident thereof.

212
2. Prescription was not relevant. Prescription, in general, is a mode of acquiring or
losing ownership and other real rights through the lapse of time in the manner
and under the conditions laid down by law. However, prescription was not
relevant to the determination of the dispute herein, considering that Lim did not
base his right of ownership on an adverse possession over a certain period. He
insisted herein, instead, that title to the land had been voluntarily transferred by
the registered owners themselves to Luisa, his predecessor-in-interest.

213
INOCENCIO Y. LUCASAN VS. PHILIPPINE DEPOSIT INSURANCE CORPORATION

G.R. No. 176929


July 4, 2008

Facts:

Petitioner Inocencio Y. Lucasan (Lucasan) and his wife Julianita Sorbito (now
deceased) were the owners of Lot Nos. 1500-A and 229-E situated in Bacolod City,
respectively covered by TCT Nos. T-68115 and T-13816.

On August 3, 1972, Pacific Banking Corporation (PBC) extended a P5,000.00 loan to


Lucasan, with Carlos Benares as his co-maker. Lucasan and Benares failed to pay the
loan when it became due and demandable. Consequently, PBC filed a collection case
with the RTC of Bacolod City.

On April 30, 1979, the RTC rendered a decision ordering Lucasan and Benares to
jointly and severally pay PBCP7,199.99 with interest at 14% per annum computed from
February 7, 1979, until the full payment of the obligation. Lucasan failed to pay the
monetary award; thus, to satisfy the judgment, the RTC issued a writ of execution
directing the sheriff to effect a levy on the properties owned by Lucasan and sell the
same at public auction.

On May 13, 1981, the lots were sold at public auction and were awarded to PBC as the
highest bidder. A certificate of sale was executed in its favor. Neither PNB nor RPB, the
mortgagees, assailed the auction sale.

Lucasan, as well as the mortgagee banks, PNB and RPB, did not redeem the properties
within the redemption period. Nevertheless, PBC did not file a petition for consolidation
of ownership.

In January 1997, Lucasan, through counsel, wrote a letter to the Philippine Deposit
Insurance Corporation (PDIC) seeking the cancellation of the certificate of sale and
offering to pay PBC‘s claim against Lucasan.

On August 13, 2001, PDIC denied Lucasan‘s request for the cancellation of the
certificate of sale.

Lucasan then filed a petition denominated as declaratory relief with the RTC of Bacolod
City. He sought confirmation of his rights provided in the second paragraph of Section 1,
Rule 63 of the Rules of Court in relation to Section 75 of Presidential Decree (P.D.) No.
1529. Lucasan also pleaded for the lifting and/or cancellation of the notice of embargo
and the certificate of sale annotated on TCT Nos. T-68115 and T-13816, and offered to
pay P100,000.00 or such amount as may be determined by the RTC, as consideration
for the cancellation.

214
PDIC moved to dismiss the complaint for lack of cause of action. Lucasan opposed the
motion.He countered that the subject properties were still in his possession, and neither
PBC nor PDIC instituted an action for consolidation of ownership. Since the certificate of
title was still in his name, he contended that he could pursue all legal and equitable
remedies, including those provided for in Section 1, Rule 63 of the Rules of Court to
reacquire the properties. He also claimed that PDIC‘s policy of disposing the subject
properties through public bidding at the appraised value of P2,900,300.00 was unjust,
capricious and arbitrary, considering that the judgment debt amounted only to
P7,199.99 with interest at 14% per annum. Lucasan urged the RTC to apply the liberal
construction of the redemption laws stressed in Cometa v. Court of Appeals. The court
dismissed the case. Petitioner appealed but the CA affirmed in toto the RTC‘s decision.

Issue:

Whether or not quieting of title may be availed by the petitioner on the case at bar.

Ruling:

Quieting of title is a common law remedy for the removal of any cloud of doubt or
uncertainty with respect to real property. The Civil Code authorizes the said remedy in
the following language:

ART. 476. Whenever there is a cloud on title to real property or any interest therein, by
reason of any instrument, record, claim, encumbrance or proceeding which is
apparently valid or effective but is in truth and in fact invalid, ineffective, voidable, or
unenforceable, and may be prejudicial to said title, an action may be brought to remove
such cloud or to quiet the title.
An action may also be brought to prevent a cloud from being cast upon title to real
property or any interest therein.

ART. 477. The plaintiff must have legal or equitable title to, or interest in the real
property which is the subject-matter of the action. He need not be in possession of said
property.

To avail of the remedy of quieting of title, two (2) indispensable requisites must concur,
namely: (1) the plaintiff or complainant has a legal or an equitable title to or interest in
the real property subject of the action; and (2) the deed, claim, encumbrance or
proceeding claimed to be casting a cloud on his title must be shown to be in fact invalid
or inoperative despite its prima facie appearance of validity or legal efficacy. Stated
differently, the plaintiff must show that he has a legal or at least an equitable title over
the real property in dispute, and that some deed or proceeding beclouds its validity or
efficacy.

Unfortunately, the foregoing requisites are wanting in this case. Admittedly, the subject
parcels of land were levied upon by virtue of a writ of execution issued in Civil Case No.
12188. On May 13, 1981, a public auction of the subject parcels of land was held and

215
the lots were awarded to PBC as the highest bidder. A certificate of sale in favor of PBC
was issued on the same day, and was registered and annotated on TCT Nos. T-68115
and T-13816 as Entry No. 112552 on June 5, 1981.

Under the 1964 Rules of Court, which were in effect at that time, the judgment debtor or
redemptioner had the right to redeem the property from PBC within twelve (12) months
from the registration of the certificate of sale. With the expiration of the twelve-month
period of redemption and no redemption having been made, as in this case, the
judgment debtor or the redemptioner lost whatever right he had over the land in
question.

Lucasan no longer possess any legal or equitable title to or interest over the subject
parcels of land; hence, he cannot validly maintain an action for quieting of title.
Lucasan‘s right to redeem the subject properties had elapsed on June 5, 1982.

216
CO-OWNERSHIP

217
SPOUSES MARCOS VS. HEIRS OF ISIDRO BANGI

G.R. No. 185745


October 15, 2014

Facts:

On June 26, 1998, heirs of Isidro Bangi and Genoveva Diccion filed with the RTC a
complain for annulment of documents, cancellation of TCT, restoration of OCT and
recovery of ownership plus damages against Spouses Dominador and Gloria Marcos.
Spouses Jose Dilla and Pacita Dilla, Ceasaria Alap, and spouses Emilio Sumajit and
Zenaida Sumajit were also impleaded.

Respondents‘ parents,Isidro and Genoveva, bought 1/3 portion of a land in Pangasinan


from Eusebio Bangi as evidenced by a DOAS (Nov 5, 1943).The OCT was registered in
the name of Alipio Bangi, Eusebio‘s father. After the sale, Isidro and Genoveva took
possession of the land until death, and then respondents took possession of the same.

Sometime in 1998, they learned that the title of the property, including the portion sold
to their parents, was transferred to petitioner Dominador, etc. through a DOAS (Aug 10,
1995) supposedly executed by Alipio. Respondents claimed that this DOAS was a
forgery since Alipio died in 1918 while Ramona (wife of Alipio) died in 1957. Through
that DOAS, TCT was issued to Dominador, etc. Another DOAS (Nov 21, 1995) was
executed and TCT issued over same property in favor of petitioners. Respondents also
claimed forgery in that DOAS.

Petitioners claimed that they are owners of the property including the 1/3 portion sold by
Eusebio to Isidro. Land was originally owned by Alipio; after his death, his children
(Eusebio, Espedita and Jose) inherited the property; Espedita and Jose executed a
Deed of Extrajudicial Partition with Quitclaim wherein they waived their rights over the
property in favor of Eusebio‘s children

They further claimed that Eusebio could not have sold the 1/3 portion since he acquired
the land by virtue of a donation propter nuptias from Alipio when he got married, and
such donation was fictitious or is void since it was not registered. 5. Respondents +
Caesaria and Spouses Emilio and Zenaida entered into a compromise agreement
wherein they acknowledged the rights of respondents over the property and admitted
the existence of the 1/3 portion. 6. RTC held in favor of respondents and declared the
two later Deeds null and void. It upheld the DOAS over the 1/3 portion executed by
Eusebio in favor of Isidro. CA affirmed RTC and held that while the claim of donation
propter nuptias was not sufficiently established, Eusebio, at the time he executed the
DOAS in favor of the Isidro, already owned the subject property, having inherited the
same from his father Alipio who died in 1918.

218
Further, the CA did not give credence to the Deed of Extrajudicial Partition with
Quitclaim purportedly executed by Espedita and Jose Bangi since it appears to have
been caused to be executed by the petitioners as a mere afterthought.

Issue:

Whether the heirs of Alipio had already effected a partition of his estate prior to the sale
of the 1/3 portion of the property to the spouses.

Ruling:

Petition denied. CA Decision affirmed.

The determination of whether the heirs of Alipio had already partitioned his estate prior
to the sale of the 1/3 of the subject property on November 5, 1943 necessarily requires
an examination of the probative value of the evidence presented by the parties; the
doubt arises on the truth or falsity of the allegations of the parties. Still, even if the
petition falls under any of the exceptions justifying a factual review of the findings of the
appellate court, the petition cannot prosper.

Partition is the separation, division and assignment of a thing held in common among
those to whom it may belong. Every act which is intended to put an end to in division
among co-heirs and legatees or devisees is deemed to be a partition. Partition may be
inferred from circumstances sufficiently strong to support the presumption. Thus, after a
long possession in severalty, a deed of partition may be presumed.

On general principle, independent and in spite of the statute of frauds, courts of equity
have enforced oral partition when it has been completely or partly performed.
Regardless of whether a parol partition or agreement to partition is valid and
enforceable at law, equity will in proper cases, where the parol partition has actually
been consummated by the taking of possession in severalty and the exercise of
ownership by the parties of the respective portions set off to each, recognize and
enforce such parol partition and the rights of the parties thereunder. It has been held in
cases involving an oral partition under which the parties went into possession, exercised
acts of ownership, or otherwise partly performed the partition agreement, that equity will
confirm such partition and in a proper case decree title in accordance with the
possession in severalty.

Evidence presented by the parties show that after the death of Alipio, his heirs had
orally partitioned the estate, including the property, which was assigned to Eusebio.
When Eusebio executed the DOAS in favour of Isidro, Eusebio already acquired interest
in the property through succession from Alipio, his father.

Such interest extends to the entire property, as can be gleaned from the testimony of
Gloria herself, who said that her father Eusebio owned the entire lot because his
siblings Espedita and Jose already had their share from other properties. 5. That there

219
was no written memorandum of the partition among Alipio Bangi‘s heirs cannot detract
from appellee‘s cause.

It has been ruled that oral partition is effective when the parties have consummated it by
the taking of possession in severalty and the exercise of ownership of the respective
portions set off to each. Here, it is obvious that Eusebio took possession of his share
and exercised ownership over it. 6. Considering that Eusebio already owned the subject
property at the time he sold the 1/3 portion thereof to the spouses, having been
assigned the same pursuant to the oral partition of the estate of Alipio effected by his
heirs, the lower courts correctly nullified the 2 DOAS.

220
RAUL V. ARAMBULO VS. GENARO NOLASCO

G.R. No. 189420


March 26, 2014

Facts:

Petitioners and Iraida Arambulo Nolasco (Iraida) are co–owners of two (2) parcels of
land. When Iraida passed away, she was succeeded by her husband, respondent
Genaro Nolasco. Petitioners filed a petition for relief under Article 491 of the Civil Code
alleging that all of the co–owners, except for respondents, have authorized petitioners to
sell their respective shares to the subject properties; that only respondents are
withholding their consent to the sale of their shares; that in case the sale pushes
through, their mother and siblings will get their respective 1/9 share of the proceeds of
the sale, while respondents will get ¼ share each of the 1/9 share of Iraida; that the sale
of subject properties constitutes alteration; and that under Article 491 of the Civil Code,
if one or more co–owners shall withhold their consent to the alterations in the thing
owned in common, the courts may afford adequate relief.

Issue:

Whether or not the respondents, as co–owners, can be compelled by the court to give
their consent to the sale of their shares in the co–owned properties

Ruling:

No, Petitioners filed the case on the submission that Article 491 covers the petition and
grants the relief prayed for, which is to compel the respondent co–owners to agree to
the sale of the co–owned property. We have to remove the issue out of the coverage of
Article 491. It does not apply to the problem arising out of the proposed sale of the
property co–owned by the parties in this case.

Article 493 dictates that each one of the parties herein as co–owners with full ownership
of their parts can sell their fully owned part. The sale by the petitioners of their parts
shall not affect the full ownership by the respondents of the part that belongs to them.
Their part which petitioners will sell shall be that which may be apportioned to them in
the division upon the termination of the co–ownership. With the full ownership of the
respondents remaining unaffected by petitioners‘ sale of their parts, the nature of the
property, as co–owned, likewise stays. In lieu of the petitioners, their vendees shall be
co–owners with the respondents. Co–owners such as respondents have over their part,
the right of full and absolute ownership. Such right is the same as that of individual
owners which is not diminished by the fact that the entire property is co–owned with
others. That part, which ideally belongs to them, or their mental portion, may be
disposed of as they please independent of the decision of their co–owners. So we rule
in this case. The respondents cannot be ordered to sell their portion of the co–owned
properties.

221
TEODORO S. TEODORO VS. DANILO ESPINO, ET. AL.

G.R. No. 189248


February 5, 2014

Facts:

Genaro had five children: Santiago; Maria, from whom respondents descended and
trace their claim of ownership and right of possession; Petra, Mariano, Teodoro
Teodoro‘s father; and Ana. Genaro and his children are all deceased. Respondents‘
respective parents are first cousins of Teodoro Teodoro. All parties are collateral
relatives of Petra Teodoro: Teodoro Teodoro is her nephew while respondents are her
grandnephews and grandnieces, descendants of Petra‘s sister, Maria Teodoro. Of all
Genaro‘s children, only Petra occupied the subject property, living at the ancestral
house. Genaro‘s other children were bequeathed, and stayed at, a different property
within the same locality, still from the estate of their father.

After Petra‘s death, her purported will, a holographic will, was probated in a Special
Proceedings, asserting ownership, devised the subject property to Teodoro Teodoro.
Respondents, who resided at portions of Lot No. 2476 that surround the subject
property on which the ancestral house previously stood, erected a fence on the
surrounding portion, barricaded its frontage, and put up a sign thereat, effectively
dispossessing Teodoro Teodoro of the property bequeathed to him by Petra. After
Teodoro Teodoro‘s demand for respondents to vacate the subject property went
unheeded, he filed the complaint for forcible entry against respondents.

After trial, the MTC dismissed the complaint, stating that the subject lot still forms part of
the estate of the late Genaro Teodoro. In the absence of an actual and approved
partition plan among his heirs, the subject lot remains part of the Genaro Teodoro‘s
estate. Since his children Santiago, Maria, Petra, Maraino and Ana are all deceased,
their children or grandchildren by right of representation have the right to inherit from
their ancestor. The RTC reversed the MTC‘s ruling wherein again, was reversed by the
CA. it ruled that Teodoro Teodoro failed to discharge the burden of proof that he had
prior actual physical possession of the subject property before it was barricaded by
respondents to warrant the institution of the forcible entry suit.

Issue:

Whether or not the act of respondents in barricading the frontage of the portion of Lot
No. 2476 on which stood the ancestral house occupied by Petra amounted to Teodoro
Teodoro‘s unlawful dispossession thereof through the forcible entry of respondents.

Ruling:

In the sense that Teodoro Teodoro has not proven exclusive ownership, the MTC was
right. But exclusive ownership of Lot No. 2476 or a portion thereof is not in this case

222
required of Teodoro Teodoro for him to be entitled to possession. Co-ownership, the
finding of both the MTC at first instance and by the RTC on appeal, is sufficient. The
pertinent provisions of the Civil Code state:

Art. 484. There is co-ownership whenever the ownership of an undivided thing or right
belongs to different persons.

Art. 1078. When there are two or more heirs, the whole estate of the decedent is, before
its partition, owned in common by such heirs, subject to the payment of debts of the
deceased.

Certainly, and as found by the trial courts, the whole of Lot No. 2476 including the
portion now litigated is, owing to the fact that it has remained registered in the name of
Genaro who is the common ancestor of both parties herein, co-owned property. All or
both Teodoro Teodoro and respondents are entitled to exercise the right of possession
as co-owners.

Neither party can exclude the other from possession. Although the property remains
unpartitioned, the respondents in fact possess specific areas. Teodoro Teodoro can
likewise point to a specific area, which is that which was possessed by Petra. Teodoro
Teodoro cannot be dispossessed of such area, not only by virtue of Petra's bequeathal
in his favor but also because of his own right of possession that comes from his co-
ownership of the property. As the RTC concluded, petitioners, as heirs substituting
Teodoro Teodoro in this suit, should be restored in the lawful possession of the disputed
area.

223
ANTIPOLO INING VS. LEONARDO R. VEGA

G.R. No. 174727


August 12, 2013

Facts:

Leon Roldan (Leon), married to Rafaela Menez (Rafaela), is the owner of a 3,120-
square meter parcel of land (subject property) in Kalibo, Aklan. Leon and Rafaela died
without issue. Leon was survived by his siblings Romana Roldan (Romana) and
Gregoria Roldan Ining (Gregoria), who are now both deceased. Romana was survived
by her daughter Anunciacion Vega and grandson, herein respondent Leonardo R. Vega
(Leonardo) (also both deceased). Leonardo in turn is survived by his wife Lourdes and
children Restonilo I. Vega, Crispulo M. Vega, Milbuena Vega-Restituto and Lenard
Vega, the substituted respondents.

Gregoria, on the other hand, was survived by her six children. In short, herein
petitioners, except for Ramon Tresvalles (Tresvalles) and Roberto Tajonera (Tajonera),
are Gregoria‘s grandchildren or spouses thereof (Gregoria‘s heirs). Tresvalles and
Tajonera are transferees of the said property. In 1997, acting on the claim that one-half
of subject property belonged to him as Romana‘s surviving heir, Leonardo filed with the
Regional Trial Court (RTC) of Kalibo, Aklan Civil Case No. 5275 for partition, recovery
of ownership and possession, with damages, against Gregoria‘s heirs. In their Answer
with counterclaim, Gregoria‘s heirs (through son Antipolo) claimed that Leonardo had no
cause of action against them; that they have become the sole owners of the subject
property through Lucimo Sr. who acquired the same in good faith by sale from Juan
Enriquez (Enriquez), who in turn acquired the same from Leon, and Leonardo was
aware of this fact.

Issue:

Whether or not the appellate court committed grave abuse of discretion in reversing the
decision of the trial court on the ground that Lucimo Francisco repudiated the co-
ownership only on February 9, 1979.

Ruling:

No, the Court of Appeals is correct based on the following reasons:


1. The finding that Leon did not sell the property to Lucimo Sr. had long been
settled and had become final for failure of petitioners to appeal. Thus, the
property remained part of Leon‘s estate.
2. Leon died without issue; his heirs are his siblings Romana and Gregoria.
3. Gregoria‘s and Romana‘s heirs are co-owners of the subject property. No
prescription shall run in favor of one of the co-heirs against the others so long as
he expressly or impliedly recognizes the co-ownership.

224
4. For prescription to set in, the repudiation must be done by a co-owner. The CA
held that prescription began to run against Leonardo only in 1979 – or even in
1980 – when it has been made sufficiently clear to him that Lucimo Sr. has
renounced the co-ownership and has claimed sole ownership over the property.
The CA thus concluded that the filing of Civil Case No. 5275 in 1997, or just less
than 20 years counted from 1979, is clearly within the period prescribed under
Article 1141. Lucimo Sr. is not a co-owner of the property. Indeed, he is not an
heir of Gregoria; he is merely Antipolo‘s son-in-law, being married to Antipolo‘s
daughter Teodora. One who is merely related by affinity to the decedent does not
inherit from the latter and cannot become a co-owner of the decedent‘s property.
Consequently, he cannot

225
REY CASTIGADOR CATEDRILLA VS. MARIO AND MARGIE LAURON

G.R. No. 179011


April 15, 2013

Facts:

Catedrilla filed with MTC a Complaint for ejectment against the spouses Lauron. The
Laurons, through the tolerance of the heirs of Lilia, constructed a residential building of
string materials on the northwest portion of Lot 5 covering an area of one hundred
square meters; that they were demanded to vacate the premises and even exerted
earnest efforts to compromise with them but the same was unavailing; and that
petitioner reiterated the demand on respondents to vacate the subject lot on January
15, 2003, but respondents continued to unlawfully withhold such possession.
Respondents claimed that petitioner had no cause of action. MTC rendered decision
ordering defendants to vacate the lot in question and restore possession to the plaintiff.
Respondents filed an appeal with the RTC which affirmed the appealed decision with
modifications. Dissatisfied, respondents filed with the CA petition for review. The CA
reversed and set aside. Consequently, the complaint for ejectment of the respondent is
dismissed. Hence, this petition for review.

Issue:

Whether or not petitioner can file the action for ejectment without impleading his co-
owners.

Ruling:

Yes. Petitioners can file the action for ejectment without impleading his co-owners. In
sum, in suits to recover properties, all co-owners are real parties in interest. However,
pursuant to Article 487 of the Civil Code and the relevant jurisprudence, any one of
them may bring an action, any kind of action for the recovery of co-owned properties.
Therefore, only one of the co-owners, namely the co-owner who filed the suit for the
recovery of the co-owned property, is an indispensable party thereto. The other co-
owners are not indispensable parties. They are not even necessary parties, for a
complete relief can be afforded in the suit even without their participation, since the suit
is presumed to have been filed for the benefit of all co-owners. In this case, although
petitioner alone filed the complaint for unlawful detainer, he stated in the complaint that
he is one of the heirs of the late Lilia Castigador, his mother, who inherited the subject
lot, from her parents. Petitioner did not claim exclusive ownership of the subject lot, but
he filed the complaint for the purpose of recovering its possession which would redound
to the benefit of the co-owners. Since petitioner recognized the existence of a co-
ownership, he, as a co-owner, can bring the action without the necessity of joining all
the other co-owners as co-plaintiffs.

226
CAROLINA VDA. DE FIGURACION ET. AL. VS. EMILIA FIGURACION-GERILLA

G.R. No. 151334


February 13, 2013

Facts:

The parties are the heirs of Leandro Figuracion (Leandro) who died intestate in May
1958. Petitioner Carolina is the surviving spouse. Subject of the dispute are two parcels
of land both situated in Urdaneta, Pangasinan, which were acquired by Leandro during
his lifetime. Leandro executed a Deed of Quitclaim over the real properties in favor of
his six children. Their shares, however, were not delineated with particularity because
spouses Leandro and Carolina reserved the lots and its fruits for their expenses. Soon
thereafter or on December 11, 1962, petitioner Carolina executed an Affidavit of Self-
Adjudication 9 adjudicating unto herself the entire Lot No. 707 as the sole and exclusive
heir of her deceased parents, Eulalio and Faustina.10 On the same date, Carolina also
executed a Deed of Absolute Sale 11 over Lot No. 707 in favor of petitioners Hilaria and
Felipa, who in turn immediately caused the cancellation of OCT No. 15867 and the
issuance of TCT No. 42244 in their names. In 1971, Emilia and her family went to the
United States and returned to the Philippines only in 1981. She built a house on the
eastern half of Lot No. 707. The legal debacle of the Figuracions started in 1994 when
Hilaria and her agents threatened to demolish the house of Emilia. The matter was
initially brought before the Katarungang Pambarangay, but no amicable settlement was
reached by the parties. Respondent Emilia appealed the CA‘s decision to the Court. In a
Decision promulgated on August 22, 2006, the Court denied the appeal, concurring with
the CA‘s ruling that a partition of Lot Nos. 2299 and 705 would be inappropriate
considering that: (1) the ownership of Lot No. 705 is still in dispute; and (2) there are still
unresolved issues as to the expenses chargeable to the estate of Leandro.

Issue:

Whether or not the Affidavit of Self-Adjudication executed by Carolina prejudiced the


share of Agripina.

Ruling:

The Affidavit of Self-Adjudication executed by Carolina did not prejudice the share of
Agripina because it is not legally possible for one to adjudicate unto himself an entire
property he was not the sole owner of. A co-owner cannot alienate the shares of her
other co-owners – nemo dat qui non habet. Hence, Lot No. 707 was a co-owned
property of Agripina and Carolina. As co-owners, each of them had full ownership of her
part and of the fruits and benefits pertaining thereto. Each of them also had the right to
alienate the lot but only in so far as the extent of her portion was affected. A co-owner is
entitled to sell his undivided share; hence, a sale of the entire property by one co-owner
without the consent of the other co-owners is not null and void and only the rights of the
co-owner/seller are transferred, thereby making the buyer a co-owner of the property.

227
SPOUSES PASCUAL, ET. AL. vS. SPOUSES BALLESTEROS

G.R. No. 186269


February 15, 2012

Facts:

The subject property is owned by the following persons, with the extent of their
respective shares over the same: (1) the spouses Albino and Margarita Corazon
Mariano, 330 square meters; (2) Angela Melchor (Angela), 466.5 square meters; and
(3) the spouses Melecio and Victoria Melchor (Spouses Melchor), 796.5 square meters.
Upon the death of the Spouses Melchor, their share in the subject property was
inherited by their daughter Lorenza Melchor Ballesteros (Lorenza). Subsequently,
Lorenza and her husband Antonio Ballesteros (respondents) acquired the share of
Angela in the subject property by virtue of an Affidavit of Extrajudicial Settlement with
Absolute Sale dated October 1, 1986.

On August 11, 2000, Margarita, then already widowed, together with her children, sold
their share in the subject property to Spouses Pascual and Francisco. Subsequently,
Spouses Pascual and Francisco caused the cancellation of TCT No. 30375 and, thus,
TCT No. T-32522 was then issued in their names together with Angela and Spouses
Melchor.

Consequently, the respondents, claiming that they did not receive any written notice of
the said sale in favor of Spouses Pascual and Francisco, filed with the Regional Trial
Court (RTC) of Laoag City a Complaint for legal redemption against the petitioners. The
respondents claimed that they are entitled to redeem the portion of the subject property
sold to Spouses Pascual and Francisco being co-owners of the same.

Issue:

Whether the respondents herein and the predecessors-in-interest of the petitioners are
co-owners of the subject property who have the right of redemption under Article 1620
of the Civil Code.

Ruling:

Yes. The 30-day period given to the respondents within which to exercise their right of
redemption has not commenced in view of the absence of a written notice. Verily,
despite the respondents‘ actual knowledge of the sale to the respondents, a written
notice is still mandatory and indispensable for purposes of the commencement of the
30-day period within which to exercise the right of redemption.

Article 1623 of the Civil Code succinctly provides that:

228
The right of legal pre-emption or redemption shall not be exercised except within thirty
days from the notice in writing by the prospective vendor, or by the vendor, as the case
may be. The deed of sale shall not be recorded in the Registry of Property, unless
accompanied by an affidavit of the vendor that he has given written notice thereof to all
possible redemptioners.

The right of redemption of co-owners excludes that of adjoining owners.

229
NENITA GONZALES, ET. AL. VS. MARIANO BUGAAY, ET. AL.

G.R. No. 173008


February 22, 2012

Facts:

Petitioner filed an Amended Complaint for Partition and Annulment of Documents with
Damages. After due proceedings, the RTC rendered a Decision.

Respondents filed a motion for reconsideration and/or new trial from the said Decision.
The RTC issued an Order which reads: "in the event that within a period of one (1)
month from today, they have not yet settled the case, it is understood that the motion for
reconsideration and/or new trial is submitted for resolution without any further hearing."
Without resolving the foregoing motion, the RTC, noting the failure of the parties to
submit a project of partition, issued a writ of execution giving them a period of 15 days
within which to submit their nominees for commissioner, who will partition the subject
estate.

Subsequently, the RTC discovered the pendency of the motion for reconsideration
and/or new trial and set the same for hearing. The RTC granted respondents' motion for
reconsideration and/or new trial for the specific "purpose of receiving and offering for
admission the documents referred to by the [respondents]."

However, instead of presenting the documents adverted to, consisting of the documents
sought to be annulled, respondents demurred to petitioners' evidence on which the RTC
denied as well as respondents' motion for reconsideration.

Issue:

Whether or not the filing of a demurrer to evidence after a Decision had been rendered
in the case is proper.

Ruling:

No. In passing upon the sufficiency of the evidence raised in a demurrer, the court is
merely required to ascertain whether there is competent or sufficient proof to sustain the
judgment. Being considered a motion to dismiss, thus, a demurrer to evidence must
clearly be filed before the court renders its judgment. In this case, respondents
demurred to petitioners' evidence after the RTC promulgated its Decision. While
respondents' motion for reconsideration and/or new trial was granted, it was for the sole
purpose of receiving and offering for admission the documents not presented at the trial.
As respondents never complied with the directive but instead filed a demurrer to
evidence, their motion should be deemed abandoned. Consequently, the RTC's original
Decision stands.

230
JOSE FERNANDO, JR., ET. AL. VS. LEON ACUNA, ET. AL.

G.R. No. 161030


September 14, 2011

Facts:

At the heart of this controversy is a parcel of land covered by OCT No. RO-487 (997)
registered in the names of Jose A. Fernando, married to Lucila Tinio, and Antonia A.
Fernando, married to Felipe Galvez. When they died intestate, the property remained
undivided. Petitioners herein are the heirs and successors-in-interest of the deceased
registered owners.

Thus, petitioners, except for the heirs of Germogena Fernando, filed a Complaint for
partition against the heirs of Germogena Fernando. In the Complaint, plaintiffs alleged,
among others, that they and defendants are common descendants and compulsory
heirs of the late spouses Jose A. Fernando and Lucila Tinio, and the late spouses
Antonia A. Fernando and Felipe Galvez. They further claimed that their predecessors-
in-interest died intestate and without instructions as to the disposition of the property left
by them covered by OCT No. RO-487 (997). There being no settlement, the heirs are
asking for their rightful and lawful share because they wish to build up their homes or
set up their business in the respective portions that will be allotted to them. In sum, they
prayed that the subject property be partitioned into eight equal parts, corresponding to
the hereditary interest of each group of heirs.

Issue:

Whether or not petitioners‘ ascendants held the property erroneously titled in their
names under an implied trust for the benefit of the true owners.

Ruling:

Yes. Article 1456 of the Civil Code provides: If property is acquired through mistake or
fraud, the person obtaining it is, by force of law, considered a trustee of an implied trust
for the benefit of the person from whom the property comes. The party thus aggrieved
has the right to recover his or their title over the property by way of reconveyance while
the same has not yet passed to an innocent purchaser for value. As was held in
Medizabel v. Apao, the essence of an action for reconveyance is that the certificate of
title is respected as incontrovertible. What is sought is the transfer of the property, in
this case its title, which has been wrongfully or erroneously registered in another
person's name, to its rightful owner or to one with a better right. It is settled in
jurisprudence that mere issuance of the certificate of title in the name of any person
does not foreclose the possibility that the real property may be under co-ownership with
persons not named in the certificate or that the registrant may only be a trustee or that
other parties may have acquired interest subsequent to the issuance of the certificate of
title.

231
JULITA ROMBAUA PANGANIBAN, ET. AL. VS. JULITA S. OAMIL

G.R. No. 149313


January 22, 2008

Facts:

Julita Oamil, herein respondent, filed a complaint for specific performance with
damages with the Regional Trial Court of Olongapo City, praying that Partenio
Rombaua (Partenio) be ordered to execute a final deed of sale over the parcel of land
which was the subject of a prior "Agreement to Sell" executed by and between them.
There are two portions of the subject property in contention: one consisting of 204.5
square meters facing 21stStreet (the 21st St. portion), and another consisting of 204.5
square meters facing Canda Street (the Canda St. portion). Petitioners and their father
Partenio are acknowledged co-owners of the subject property to the following extent:
one-half to Partenio as his conjugal share, and one-sixth each of the remaining half to
petitioners and Partenio as the surviving heirs of Juliana.

Petitioners filed a verified petition for relief from the decision of the trial court, grounded
on the following: 1) that Partenio‘s conjugal share in the property, and that of petitioners
as well, are being litigated in a judicial partition proceeding (the partition case) which is
pending with the Court of Appeals, hence the trial court may not yet render a decision
disposing of a definite area of the subject property in respondent‘s favor; and, (2) that
petitioners were unjustly deprived of the opportunity to protect and defend their interest
in court because, notwithstanding that they are indispensable parties to the case (being
co-owners of the subject property), they were not impleaded in Civil Case No. 140-0-93.

Issue:

Whether or not petitioners can intervene in the proceedings in Civil Case No. 140-0-93
in order to protect their rights as co-owners of the subject property.

Ruling:

Yes. Under a co-ownership, the ownership of an undivided thing or right belongs to


different persons. During the existence of the co-ownership, no individual can claim title
to any definite portion of the community property until the partition thereof; and prior to
the partition, all that the co-owner has is an ideal or abstract quota or proportionate
share in the entire land or thing. Before partition in a co-ownership, every co-owner has
the absolute ownership of his undivided interest in the common property. The co-owner
is free to alienate, assign or mortgage this undivided interest, except as to purely
personal rights. The effect of any such transfer is limited to the portion which may be
awarded to him upon the partition of the property.

Under Article 497 of the Civil Code, in the event of a division or partition of property
owned in common, assignees of one or more of the co-owners may take part in the

232
division of the thing owned in common and object to its being effected without their
concurrence. But they cannot impugn any partition already executed, unless there has
been fraud, or in case it was made notwithstanding a formal opposition presented to
prevent it, without prejudice to the right of the debtor or assignor to maintain its validity.

233
SPOUSES DEL CAMPO VS. BERNARDA FERNANDEZ ABESIA

G.R. No. L-49219


April 15, 1988

Facts:

An action for partition was filed by plaintiffs in the CFI of Cebu. Plaintiffs and defendants
are co-owners pro-indiviso of subject lot in the proportion of and 1/3 shares each,
respectively. The Id commissioner appointed by the trial court conducted a survey,
prepared a sketch plan and submitted a report to the trial court on May 29, 1976,
recommending that the property be divided into two lots: Lot 1161-A with an area of 30
square meters for plaintiffs and Lot No. 1161-B with an area of 15 square meters for the
defendants. The houses of plaintiffs and defendants were surveyed and shown on the
sketch plan. The house of defendants occupied the portion with an area of 5 square
meters of Lot 1161-A of plaintiffs. The parties manifested their conformity to the report
and asked the trial court to finally settle and adjudicate who among the parties should
take possession of the 5 square meters of the land in question.
The trial court ruled that Article 448 of the Civil Code is inapplicable and the rules on co-
ownership are more relevant over the encroaching structure and the land on which it
was built.

Issue:

Whether or not the rights of a builder in good faith under Art. 448 of the Civil Code apply
to defendants-appellants with respect to that part of their house occupying a portion of
the lot assigned to plaintiffs-appellees.

Ruling:

When, as in this case, the co-ownership is terminated by the partition and it appears
that the house of defendants overlaps or occupies a portion of 5 square meters of the
land pertaining to plaintiffs which the defendants obviously built in good faith, then the
provisions of Article 448 of the new Civil Code should apply. Manresa and Navarro
Amandi agree that the said provision of the Civil Code may apply even when there was
co-ownership if good faith has been established. Applying the aforesaid provision of the
Civil Code, the plaintiffs have the right to appropriate said portion of the house of
defendants upon payment of indemnity to defendants as provided for in Article 546 of
the Civil Code. Otherwise, the plaintiffs may oblige the defendants to pay the price of
the land occupied by their house. However, if the price asked for is considerably much
more than the value of the portion of the house of defendants built thereon, then the
latter cannot be obliged to buy the land. The defendants shall then pay the reasonable
rent to the plaintiff upon such terms and conditions that they may agree. In case of
disagreement, the trial court shall fix the terms thereof. Of course, defendants may
demolish or remove the said portion of their house, at their own expense, if they so
decide.

234
TITAN CONSTRUCTION VS. DAVID

G.R. No. 169548


March 15, 2010

Facts:

Manuel A. David, Sr. (Manuel) and Martha S. David (Martha) were married on March
25, 1957. In 1970, the spouses acquired a 602 square meter lot located at White Plains,
Quezon City, which was registered in the name of MARTHA S. DAVID, of legal age,
Filipino, married to Manuel A. David and covered by Transfer Certificate of Title (TCT)
No. 156043 issued by the Register of Deeds of Quezon City. In 1976, the spouses
separated de facto, and no longer communicated with each other.

Sometime in March 1995, Manuel discovered that Martha had previously sold the
property to Titan Construction Corporation (Titan) for P1,500,000.00 through a Deed of
Sale dated April 24, 1995, and that TCT No. 156043 had been cancelled and replaced
by TCT No. 130129 in the name of Titan.

Thus, on March 13, 1996, Manuel filed a Complaint for Annulment of Contract and
Recovenyance against Titan before the RTC of Quezon City. Manuel alleged that the
sale executed by Martha in favor of Titan was without his knowledge and consent, and
therefore void. He prayed that the Deed of Sale and TCT No. 130129 be invalidated,
that the property be reconveyed to the spouses, and that a new title be issued in their
names.

In its Answer with Counterclaim, Titan claimed that it was a buyer in good faith and for
value because it relied on a Special Power of Attorney (SPA) dated January 4, 1995
signed by Manuel which authorized Martha to dispose of the property on behalf of the
spouses. Titan thus prayed for the dismissal of the complaint.

In his unverified Reply, Manuel claimed that the SPA was spurious, and that the
signature purporting to be his was a forgery; hence, Martha was wholly without authority
to sell the property.

Subsequently, Manuel filed a Motion for Leave to File Amended Complaint which was
granted by the trial court. Thus, on October 15, 1996, Manuel filed an Amended
Complaint impleading Martha as a co-defendant in the proceedings. However, despite
personal service of summons upon Martha, she failed to file an Answer. Thus, she was
declared in default. Trial then ensued.

Issue:

Whether or not the property in dispute is considered as part of the conjugal property.

235
Ruling:

The Civil Code of the Philippines, the law in force at the time of the celebration of the
marriage between Martha and Manuel in 1957, provides all property of the marriage is
presumed to belong to the conjugal partnership, unless it is proved that it pertains
exclusively to the husband or to the wife. This includes property which is acquired by
onerous title during the marriage at the expense of the common fund, whether the
acquisition is for the partnership, or for only one of the spouses. The court is not
persuaded by Titan‘s arguments that the property was Martha‘s exclusive property
because Manuel failed to present before the RTC any proof of his income in 1970,
hence he could not have had the financial capacity to contribute to the purchase of the
property in 1970; and that Manuel admitted that it was Martha who concluded the
original purchase of the property. In consonance with its ruling in Spouses Castro v.
Miat, Manuel was not required to prove that the property was acquired with funds of the
partnership. Rather, the presumption applies even when the manner in which the
property was acquired does not appear. Here, we find that Titan failed to overturn the
presumption that the property, purchased during the spouses‘ marriage, was part of the
conjugal partnership. Since the property was undoubtedly part of the conjugal
partnership, the sale to Titan required the consent of both spouses. Article 165 of the
Civil Code expressly provides that ―the husband is the administrator of the conjugal
partnership‖. Likewise, Article 172 of the Civil Code ordains that ―(t)he wife cannot bind
the conjugal partnership without the husband‘s consent, except in cases provided by
law‖.

236
AGUIRRE, ET AL. VS. COURT OF APPEALS, ET AL.

G.R. No. 122249


January 29, 2004

Facts:

Leocadio Medrano and his first wife Emilia owned a piece of land. After the death of
Emilia, Leocadio married his second wife Miguela. When Leocadio died, all his heirs
agreed that Sixto Medrano, a child of the first marriage, should manage and administer
the said property. After Sixto died, his heirs learned that he had executed an Affidavit of
Transfer of Real Property in which he falsely stated that he was the only heir of
Leocadio. It turned out that while Sixto were still alive, he sold a portion of the subject
land tp Tiburcio Balitaan and another portion to Maria Bacong, Maria Bacong later sold
the said portion to Rosendo Bacong. Petitioners, all heirs of Leocadio who were
affected by the sale demanded reconveyance of the portions sold by Sixto but the 3
vendees refused. Resultantly, petitioners filed a suit against them seeking the nullity of
the documents and partition thereof. The vendees contended that they acquired the
property under the valid deed of sale and petitioners‘ cause of action was barred by
laches and prescription. Tiburcio also contended that he is an innocent purchaser for
value.

Issue:

Whether or not there was a valid sale between Sixto Medrano and the three purchases
considering the fact that it was made without the consent of the co-owners.

Ruling:

Under Article 493 of the New Civil Code, a sale by a co-owner of the whole property as
his will affect only his own share but not those of the other co-owners who did not
consent to the sale). The provision clearly provides that the sale or other disposition
affects only the seller‘s share, and the transferee gets only what corresponds to his
grantor‘s share in the partition of the property owned in common. Since a co-owner is
entitled to sell his undivided share, a sale of the entire property by one co-owner without
the consent of the other co-owner is not null and void; only the rights of the co-owner-
seller are transferred, thereby making the buyer a co-owner of the property. It is clear
therefore that the deed of sale executed by Sixto in favor of Tiburcio Balitaan is a valid
conveyance only insofar as the share of Sixto in the co-ownership is concerned. Acts
which may be considered adverse to strangers may not be considered adverse in so far
as co-owners are concerned. A mere silent possession by a co-owner, his receipts of
rentals, fruits or profits from the property, the erection of buildings and fences and
planting of trees thereon, and the payment of land taxes, cannot serve as proof of
exclusive ownership, if it is not borne out by clear and convincing evidence that he
exercised such acts of possession which unequivocally constituted an ouster or
deprivation of the rights of the other co-owners. Thus, in order that a co-owner‘s

237
possession may be deemed adverse to the cestui que trust or the other co-owners, the
following elements must concur: (1) that he has performed unequivocal acts of
repudiation amounting to an ouster of the cestui que trust or the other co-owners; (2)
that such positive acts of repudiation have been known to the cestui que trust or the
other co-owners; and (3) that the evidence thereon must be clear and convincing.

Tested against these guidelines, the respondents failed to present competent evidence
that the acts of Sixto adversely and clearly repudiate the existing co-ownership among
the heirs of Leocadio Medrano. Respondent‘s reliance on the tax declaration in the
name of Sixto Medrano is unworthy of credit since we have held on several occasions
that tax declarations by themselves do not conclusively prove title to land. Further,
respondents failed to show that the Affidavit executed by Sixto to the effect that he is
the sole owner of the subject property was known or made known to the other co-heirs
of Leocadio Medrano.

238
IGLESIA NI KRISTO VS. PONFERRADA

G.R. No. 168943


December 27, 2006

Facts:

On October 24, 2001, Alicia, Alfredo, Roberto, Enrique and Susan, all surnamed
Santos, and Sonia Santos-Wallin, represented by Enrique G. Santos, filed a
complaint for Quieting of Title and/or Accion Reinvindicatoria before the Regional
Trial Court (RTC) of Quezon City against the Iglesia Ni Cristo (INC), defendant
therein.

Plaintiffs alleged therein that, during his lifetime, Enrique Santos was the owner
of a 936-square-meter parcel of land located in Tandang Sora, Quezon City.

He had been in possession of the owner’s duplicate of said title and had been in
continuous, open, adverse and peaceful possession of the property. He died on
February 9, 1970 and was survived by his wife, Alicia Santos, and other plaintiffs,
who were their children. Thereafter, plaintiffs took peaceful and adverse
possession of the property, and of the owner’s duplicate of said title.

When the Office of the Register of Deeds of Quezon City was burned on June 11,
1988, the original copy of said title was burned as well. The Register of Deeds had
the title reconstituted based on the owner’s duplicate.

Sometime in February 1996, plaintiffs learned that defendant was claiming


ownership over the property based on a TCT issued on September 18, 1984 under
the name of the Philippine National Bank, which allegedly cancelled TCT No.
252070 in the names of the spouses Marcos and Romana dela Cruz. They insisted
that TCT Nos. 321744, 320898 and 252070 were not among the titles issued by the
Register of Deeds of Quezon City and even if the Register of Deeds issued said
titles, it was contrary to law. Enrique Santos, during his lifetime, and his heirs,
after his death, never encumbered or disposed the property. In 1996, plaintiffs
had the property fenced but defendant deprived them of the final use and
enjoyment of their property.

As gleaned from the caption of the complaint, plaintiffs appear to be the heirs of
Enrique Santos, represented by Enrique G. Santos. The latter signed the
Verification and Certificate of Non-Forum Shopping alone. (i.e. only his name and
signature appeared on the verification and certificate.)

Defendant moved to dismiss plaintiffs’ complaint on the following grounds: (1)


plaintiffs failed to faithfully comply with the procedural requirements set forth in
Section 5, Rule 7 of the 1997 Rules of Civil Procedure; (2) the action (either
Quieting of Title or Accion Reinvindicatoria) had prescribed, the same having

239
been filed only on October 24, 2001 beyond the statutory ten-year period therefor;
and (3) that the complaint is defective in many respects.
In their Comment on the motion, plaintiffs averred that the relationship of a co-
owner to the other co-owners is fiduciary in character; thus, anyone of them
could effectively act for another for the benefit of the property without need for an
authorization. Consequently, Enrique Santos had the authority to represent the
other heirs as plaintiffs and to sign the verification and certification against forum
shopping.

In its reply, defendant averred that absent any authority from his co-heirs,
Enrique Santos must implead them as plaintiffs as they are indispensable parties.
In response, plaintiffs aver that a co-owner of a property can execute an action
for quieting of title without impleading the other co-owners.

The trial court issued an Order denying defendant’s motion to dismiss. It declared
that since Enrique Santos was one of the heirs, his signature in the verification
and certification constitutes substantial compliance with the Rules. The court
cited the ruling of this Court in Dar v. Alonzo-Legasto. The court, likewise, held
that prescription had not set in and that failure to state the address of plaintiffs in
the complaint does not warrant the dismissal of the complaint.

Defendant filed a motion for reconsideration, which the court likewise denied in
an Order dated July 10, 2002. Unsatisfied, defendant, as petitioner, filed a Petition
for Certiorari and Prohibition with Prayer for the Issuance of a Temporary
Restraining Order and/or Preliminary Injunction before the CA.

The CA affirmed the RTC decision.

Issue:

Whether a verification issued by only one of the plaintiffs is sufficient enough to


render the same valid

Ruling:

DENIED. The purpose of verification is simply to secure an assurance that the


allegations of the petition (or complaint) have been made in good faith; or are true
and correct, not merely speculative. This requirement is simply a condition
affecting the form of pleadings, and noncompliance therewith does not
necessarily render it fatally defective. Indeed, verification is only a formal, not a
jurisdictional requirement.

It is noteworthy that the Court applied the rule on substantial compliance


because of the commonality of interest of all the parties with respect to the
subject of the controversy in many past similar cases.

240
The CA did not err in affirming the application of the rule on substantial
compliance. In the instant case, the property involved is a 936-square-meter real
property. Both parties have their respective TCTs over the property. Respondents
herein who are plaintiffs in the case below have a common interest over the
property being the heirs of the late Enrique Santos, the alleged registered owner
of the subject property as shown in one of the TCTs. As such heirs, they are
considered co-owners pro indiviso of the whole property since no specific
portion yet has been adjudicated to any of the heirs. Consequently, as one of the
heirs and principal party, the lone signature of Enrique G. Santos in the
verification and certification is sufficient for the RTC to take cognizance of the
case. The commonality of their interest gave Enrique G. Santos the authority to
inform the RTC on behalf of the other plaintiffs therein that they have not
commenced any action or claim involving the same issues in another court or
tribunal, and that there is no other pending action or claim in another court or
tribunal involving the same issues. Hence, the RTC correctly denied the motion to
dismiss filed by petitioner.
Considering that at stake in the present case is the ownership and possession
over a prime property in Quezon City, the apparent merit of the substantive
aspects of the case should be deemed as a special circumstance or compelling
reason to allow the relaxation of the rule.Indeed, this Court strictly applied the
rules on verification and certification against forum shopping in cases where the
commonality of interest between or among the parties is wanting.

Anent the issue of the authority of Enrique G. Santos to represent his co-heirs/co-
plaintiffs, we find no necessity to show such authority. Respondents herein are
co-owners of the subject property. As such co-owners, each of the heirs may
properly bring an action for ejectment, forcible entry and detainer, or any kind of
action for the recovery of possession of the subject properties. Thus, a co-owner
may bring such an action, even without joining all the other co-owners as co-
plaintiffs, because the suit is deemed to be instituted for the benefit of all.

We uphold the validity of the complaint because of the following circumstances:


(1) the caption of the instant case is Heirs of Enrique Santos v. Iglesia ni
Cristo; (2) the opening statement of the complaint states that plaintiffs are the
heirs of Enrique Santos and likewise names the particular heirs of the latter who
instituted the complaint below; (3) the case involves a property owned by the
predecessor-in-interest of plaintiffs therein; and (4) the verification signed by
Enrique G. Santos clearly states that he is one of the children of the late Enrique
Santos and that he represents the heirs of said Enrique Santos.

241
SPOUSES MENDOZA VS. MARIA CORONEL

G.R. No. 156402


February 13, 2006

Facts:

Respondent Maria Coronel is one of the co-owners of Lots 3250 and 3251 located at
Sagrada Familia, Hagonoy, Bulacan. Petitioners, spouses Alfredo and Rosario
Mendoza, occupied said lots upon tolerance of respondent and her co-owners without
paying any rent. When respondent demanded that petitioners vacate the premises, the
latter refused. Thus, on December 27, 2000, respondent filed a case before the MTC of
Hagonoy, Bulacan for unlawful detainer against petitioners. The MTC ruled in favor of
respondent, ordering petitioners to vacate the disputed lots.

Petitioners appealed to the RTC of Malolos, Bulacan which ruled in their favor. It
annulled and set aside the appealed decision of the MTC. It held that the co-owners of
the subject lot should have been impleaded as indispensable parties.

On appeal, the CA reversed and set aside the ruling of the RTC and revived the
decision of the MTC dated May 29, 2001. Petitioner then appealed to the Supreme
Court.

Issue:

Whether or not a co-owner is allowed to bring an action for ejectment, which covers all
kinds of actions for the recovery of possession, including forcible entry and unlawful
detainer, without the necessity of joining all the other co-owners as co-plaintiffs.

Ruling:

The Supreme Court affirmed the decision of the Lower Court which grants defendant‘s
request for the reconveyance of the lot occupied by the petitioner and for the latter to
vacate the property occupied. The Supreme Court affirms the Appellate Court decision
that a co-owner can maintain an action in ejectment without joining all the other co-
owners, the latter being indispensable parties. Article 487 of the Civil Code provides that
any one of the co-owners may bring an action in ejectment. Article 487 is a departure
from the rule laid down in the case of Palarca v. Baguisi which held that an action for
ejectment must be brought by all the co-owners. As explained by Tolentino, the law now
allows a co-owner to bring an action for ejectment, which covers all kinds of actions for
the recovery of possession, including forcible entry and unlawful detainer, without the
necessity of joining all the other co-owners as co-plaintiffs, because the suit is deemed
to be instituted for the benefit of all.

The Court also reject petitioner‘s claim that Juanito Coronel, Attorney-in-fact of Maria
Coronel, one of the co-owners of the lots in dispute is not authorized to file the

242
ejectment suit. The court maintains that an attorney-in-fact of the co-owner does not
need authority from all the co-owners. He needs authority only from the co-owner
instituting the ejectment suit.

Execution of the certification against forum shopping by an attorney-in-fact is also not a


violation of the requirement that the parties must personally sign the same. The
attorney-in-fact, who has authority to file with Special Power of Attorney, is a party to the
ejectment suit. As a representative of the owner in an ejectment suit, he is authorized to
institute the proceedings.

243
DE GUIA VS. COURT OF APPEALS

G.R. No. 120864


October 8, 2003

Facts:

Two parcels of land covering a fishpond equally owned by Primitiva Lejano and Lorenza
Araniego. The one half undivided portion owned by Araniego was later purchased by
plaintiff from his father Teofilo Abejo, the only heir of the original owner, the husband of
Araniego. Prior to this sale, the whole fishpond was leased by the heirs of Primitiva
Lejano with the knowledge and consent of Teofilo Abejo in favor of De Guia. De Guia
continues to possess the entire fishpond and derived income therein despite the
expiration of the lease contract and several demands to vacate by Teofilo Abejo and by
his successor-in-interest, Jose Abejo.

Abejo filed a complaint for recovery of possession with damages against De Guia. The
trial court rendered a decision in favor of plaintiff. The Court of Appeals affirmed the
decision of the trial court on the ground that De Guia‘s appeal is without merit.

Issue:

Whether or not a co-owner validly lease his undivided interest to a third party.

Ruling:

Yes. Under Article 484 of the Civil Code, there is co-ownership whenever the ownership
of an undivided thing or right belongs to different persons. A co-owner of an undivided
parcel of land is an owner of the whole, and over the whole he exercises the right of
dominion, but he is at the same time the owner of a portion which is truly abstract. On
the other hand, there is no co-ownership when the different portions owned by different
people are already concretely determined and separately identifiable, even if not yet
technically described.

Any co-owner may file an action under Article 487 not only against a third person, but
also against another co-owner who takes exclusive possession and asserts exclusive
ownership of the property.

244
VILLANUEVA VS. COURT OF APPEALS

G.R. No. 143286


April 14, 2004

Facts:

In 1926, Eusebia Retuya was lawfully married to Nicolas Retuya. Out of the lawful
wedlock, they begot 5 children. During their marriage, they acquired real properties and
all improvements, which generated income.

In 1945, Nicolas cohabited with Pacita Villanueva wherein Procopio Villanueva was their
illegitimate child. Nicolas then was the only person receiving the income from the
properties. Pacita had no occupation nor properties of her own which she could derive
income.

In 1985, Nicolas suffered stroke resulting to him incapacitated to talk and walk. From
this incident on, it was Procopio who was receiving the income. Natividad Retuya, one
the five children, negotiated with procopio but no settlement was reached even after the
intervention of the barangay captain.

When the case reached the trial court, Natividad testified that the parcel of land covered
by tax declaration marked Exhibit T was the property bought by her father from Adriano
Marababol for at the time of purchase of the property, defendant Pacita Villanueva had
no means of livelihood. The trial court rendered a decision in favor of Eusebia. Upon
appeal, the CA affirmed the trial court‘s ruling. In sum, the properties must be rendered
in favor of Eusebia because Article 116 of the Family Code provides that propertied
acquired during the marriage is presumed conjugal. The burden is on Pacita to rebut the
presumption, who failed however, to do so.

In addition, Pacita‘s claim that she bought Lot No. 152 was also unfounded as found by
the court a quo.

Issues:

1. Whether or not Eusebia failed to claim that the properties are conjugal
2. Whether or not the defense by respondents of prescription and laches can be
validly claimed on appeal
3. Whether or not the subject properties are conjugal

Ruling:

1. No, she did not fail. It was very clear from the first sentence of the second
paragraph of the complaint that Eusebia alleged that the properties were
conjugal. ―2. The plaintiff Eusebia Retuya and defendant Nicolas Retuya are
husband and wife and conjugal owners of real properties and all

245
improvements…‖ In fact, the same was restated and repleaded throughout the
complaint.
2. No, it cannot be. The respondents are mistaken in relying in section 1 of rule 9
because this applies only to defenses which are not raised in a motion to dismiss
or in an answer. In the present case, the respondents were able to raise their
defense of prescription and laches in their answer. However, the same cannot
be appreciated since they were not included among the issues during pre-trial.
The non-inclusion of this defense in the pre-trial order barred its consideration
during the trial. Pre-trial is primarily intended to insure that the parties properly
raise all issues necessary to dispose of a case. The parties must disclose during
pre-trial all issues they intend to raise during the trial, except those involving
privileged or impeaching matters. Indeed, the Court may review matters for the
first time on appeal only in meritorious situations. The case at present does not
present an exceptional circumstance. They are now bound by the stipulations
they made at pre-trial.
3. Yes, they are conjugal properties. The family code explicitly mandates that it
shall have retroactive effect. Thus, if the properties are acquired during the
marriage, they are presumed conjugal (Art. 116). This presumption however
requires that the properties must first be proven to have been acquired during the
marriage. This pre-requisite is satisfied in this case as attested by the tax
declarations along with the testimonies of Eusebia‘s witnesses. Absence any
sufficient reason to doubt the correctness of the findings of the courts a quo will
not warrant a reversal thereof, which is the case.

Lot 152 is conjugal since it was bought during the marriage of Nicolas and Eusebia.
They were married in 1926. Nicolas and his subsequent wife, Pacita were married in
1996. The lot was bought in 1957.

Now since all the properties are presumed conjugal, it was the burden of petitioners to
prove the contrary, which they utterly failed to do so.

The deed of sale, TCT and tax declaration of lot 152 may have been under the name of
Pacita but this was just a scheme Nicholas employed to deprive Eusebia of the
properties. In fact, in a prior civil case, it was testified by Marababol, the seller of lot 152,
that the buyer is not Pacita but Nicolas.

The other tax declarations may have been under the sole name of Nicolas, but this does
not mean that they are his exclusive properties. However, tax declarations are
insufficient to overturn the presumption in Art. 116. In fact, the same provision provides
that the presumption remains even if the property is registered in the name of one or
both of the spouses.

Nicolas may have intentionally misrepresented himself as single in some other


documents in order to exclude Eusebia, but whether a property is conjugal or not is
determined by law and not by will of one of the spouses.

246
Moreover, the cohabitation of a spouse with another person, even for a long period,
does not sever the tie of a subsisting previous marriage. Otherwise, the law would be
giving a stamp of approval to an act that is both illegal and immoral. What petitioners fail
to grasp is that Nicolas and Pacitas cohabitation cannot work to the detriment of
Eusebia, the legal spouse. The marriage of Nicolas and Eusebia continued to exist
regardless of the fact that Nicolas was already living with Pacita. Hence, all property
acquired from 7 October 1926, the date of Nicolas and Eusebias marriage, until 23
November 1996, the date of Eusebias death, are still presumed conjugal. Petitioners
have neither claimed nor proved that any of the subject properties was acquired outside
or beyond this period.

247
AVILA VS. BARABAT

G.R. No. 141993


March 17, 2006

Facts:

Upon the death of Anunciacion Bahena vda. de Nemeño, the ownership of the subject
property was transferred to her five children who now built their respective houses
thereon. Spouses Barabat leased a portion of the house of Avila, one of the children.
Avila relocated to Cagayan De Oro but came back to sell her house. But since no one
from her siblings showed interest, she then offered the house to respondent who agreed
upon the same, evidenced by a private document.

Respondents were confronted by Adlawan (another heir) who informed the former that
they have until March 1982 to stay in Avila‘s place because the latter is buying the
property. Thereafter, the respondents received a letter from atty. Alo informing them that
Avila sold the house to Adlawan. In view of this, respondents demanded Avila to
execute a public document which the latter refused to do.

The RTC, in a quieting of title, ruled in favor of respondents which the appellate court
affirmed.

Petitioners argue that they paid for the realty taxes of the property and there was gross
inadequacy of consideration making the transaction an equitable mortgage under
articles 1602,1604. They also argue that they have been denied of their right of
redemption.

Issues:
1. Whether or not the transaction was an equitable mortgage and not an absolute
sale
2. Whether or not the respondents have the legal right of redemption

Ruling:
1. No, the transaction is an absolute sale. For articles 1602 and 1604 to apply, (1)
the parties entered into a contract denominated as contract of sale and (2) their
intention was to secure an existing old debt by way of mortgage. However, as
found by the court a quo, the contract was a contract of sale and Avila intended a
sale or to absolutely convey the property to respondents. Moreover, it was
actually the respondents who took over the payment of real property taxes.
Finally, that there was gross inadequacy of consideration has no basis.
Petitioners failed to introduce evidence to that effect.
2. No, they do not have. The right to redeem would have existed had there been co-
ownership but there was none. By their own admission, petitioners were no
longer co-owners when the property was sold to respondents because there was
a partition that extinguished said co-ownership.

248
AGUILAR VS. AGUILAR

G.R. No. 141613


December 16, 2005

Facts:

Senen and Virgilio purchased a house and lot for their father with the written agreement
that theirs shares in the house and lot would be equal and that Senen would live with
their father provided he pays the SSS remaining obligation of former owners. When the
father died, Virgilio demanded Senen to vacate and the house be sold, the proceeds
thereof be divided among them but Senen refused to comply.
Virgilio filed a complaint with the RTC which rendered a decision recognizing the co-
ownership, ordering the sale of the house, and for Senen to vacate the premises. The
CA reversed the trial court‘s decision. This Court reinstated the trial court‘s decision.
Meanwhile, Senen filed an action for legal redemption alleging that while he was aware
that Virgilio sold his share to Angel, he (Senen) was not furnished of the written notice
of sale. The trial court dismissed the case on the ground of laches when it took 7 years
for Senen to assert his right. The CA affirmed.

Issue:

Whether or not Senen‘s right of redemption is now barred by laches?

Ruling:

Yes, Senen is now barred by laches.

Right of legal redemption may be exercised when: (1) there is a co-ownership; (2) one
of the co-owners sold his share to third persons; (3) the sale was made before the
partition; (4) it must exercised within thirty days from the time that he was notified in
writing by the vendee or by co-owner; and (5) the vendee must be reimbursed for the
price.

In this case, petitioner is wrong in saying that his right to redeem has not yet prescribed
since there was no written notice of the sale to him. Indeed, there was no written notice
given to him but he was full aware of the existence of the sale. A co-owner with actual
notice of the sale is not entitled to a written notice for such would be superfluous.
Petitioner has actual knowledge of the sale of Virgilio‘s share to Angel in 1989. Senen
had thirty days therefrom to exercise his right of redemption but he did not take any
action. He waited seven years before filing the complaint. Definitely, such unexplained
delay is tantamount to laches. His right is no longer available to him.

249
FERNANDEZ VS. TARUN

G.R. No. 143868


November 14, 2002

Facts:

Four brothers Fernandez together with their uncle, co-owned a fishpond. Their uncle
died thus enlarging the brothers‘ share. Two of them sold their shares to spouses
Tarun. By virtue of the Deed of Extrajudicial Partition, one of the brothers, Angel,
exchange his share on the other fishpond to shares of the co-owners, making Angel and
spouses Tarun as co-owners now of the property Lot No. 2991. After the death of
Angel, the spouses Tarun still demanded partition but was rejected by the petitioners-
heirs.

The RTC rendered in favor of the petitioners holding that they were entitled to redeem
the property they sold to respondents. The CA reversed.

Issues:

1. Whether or not petitioners are entitled to exercise their right of legal redemption
2. Whether or not the deed of extra judicial partition is void and inefficacious

Ruling:

1. No, petitioners-heirs are not entitled to the right of legal redemption. This right
only applies when a portion is sold to a stranger or not a co-owner. However it
was very clear that respondents herein are co-owners. The sale is not void
absent the notice in writing to the co-owners of the sale. Nowhere in the law
would state that absent the notice, the sale would be void. Moreover, while the
law requires that the notice must be written, it did not however, require for a
specific form. Angel was deemed notified by the execution and signing of the
Deed of Extrajudicial Partition and Exchange of Shares.
2. We are not convinced. It is a long-established doctrine that the law will not relieve
parties from the effects of an unwise, foolish or disastrous agreement they
entered into with all the required formalities and with full awareness of what they
were doing. Courts have no power to relieve them from obligations they
voluntarily assumed, simply because their contracts turn out to be disastrous
deals or unwise investments. petitioners herein are bound by the extrajudicial
partition, because contracts not only take effect between the parties, but also
extend to their assigns and heirs.

250
MONTEROSO VS. COURT OF APPEALS

G.R. No. 105608


April 30, 2008

Facts:

In 1906, Don Fabian married Soledad Doldol. Out of this marriage were born Soledad,
Reygula, Benjamin, and Tirso. On April 8, 1927, Soledad Doldol Monteroso passed
away. A little over a year later, Don Fabian contracted a second marriage with Sofia
Pendejito. From this union were born Florenda, Reynato, Alberto, and Fabian, Jr. Don
Fabian filed an intestate proceeding for the estate of his deceased first wife to avoid
disputes over the inheritance of his children from his first marriage. Land: Parcels F-1 to
F-8 (First marriage) and Parcels S-1 to S-4 (Second marriage) The partition in SP No.
309 covered Parcels F-1 to F-5, and adjudicated to Don Fabian the whole of Parcels F-
1, F-2, and F-3, and one-half of Parcel F-5, while the intestate estate of Soledad D.
Monteroso comprised the whole of Parcel F-4 and one-half of Parcel F-5. The intestate
estate of Soledad D. Monteroso was partitioned and distributed to her four children in
equal shares. On October 26, 1948, Don Fabian also passed away. On July 28, 1969,
the children of the late Benjamin D. Monteroso, filed with the RTC a Complaint for
Recovery of Property with Damages against their uncle, Tirso D. Monteroso.

As the heirs of Benjamin alleged in their complaint, their uncle, Tirso, was entrusted with
one-fourth portion of Parcel F-4 as part of the share from the estate of Soledad D.
Monteroso allotted to their father. However, their uncle refused to surrender and deliver
the same when they demanded such delivery upon their reaching the majority age.
Tirso countered that the portion pertaining to Benjamin was never entrusted to him; it
was in the possession of their sister, Soledad Monteroso-Cagampang, who was not
entitled to any share in Parcel F-4, having previously opted to exchange her share in
said property for another parcel of land, i.e. Parcel F-7, then being occupied by her
Tirso, in turn, filed a Complaint for Partition and Damages with Receivership, involving
12 parcels of land against his stepmother, Pendejito, and all his full and half-siblings
and/or their representatives.

Issue:

Whether or not the invocation of prescription deems to admit co-ownership?

Ruling:

Yes. Petitioners are mistaken; their error flows from compartmentalizing what the CA
wrote. The aforecited portion of the CAs decision should not have been taken in
isolation. It should have been read in the context of the appellate courts disquisition on
the matter of Tirso being a co-owner of the subject undivided properties whose rights
thereto, as a compulsory heir, accrued at the moment of death of Don Fabian, vis--vis

251
the defense of acquisitive prescription foisted by the Cagampang spouses. For clarity,
we reproduce the pertinent portion of the assailed decision:
―Nor do we find any merit in the third. From the allegation in the Complaint in Civil Case
No. 1332 as well as from the arguments advanced by the parties on the issues raised
therein, this Court is convinced that therein plaintiff Tirso Monterosos principal cause of
action is unmistakably one for partition which by its very nature is imprescriptible and
cannot be barred by laches x x x. The only exception to the rule on the imprescriptibility
of an action for partition is provided in a case where the co-ownership of the properties
sought to be partitioned had been properly repudiated by a co-owner at which instance
the remedy available to the aggrieved heirs lies not in action for partition but for
reconveyance which is subject to the rules on extinctive prescription. By invoking the
benefits of prescription in their favor, the Cagampang spouses are deemed to have
admitted the existence of a co-ownership among the heirs of Fabian Monteroso, Sr.
over the properties forming the decedents estate.‖

From the foregoing disquisition, what the appellate court tried to convey is clear and
simple: partition is the proper remedy available to Tirso who is a co-owner of the subject
properties by virtue of his being a compulsory heir, like siblings Soledad, Reygula, and
Benjamin, of Don Fabian. The right to seek partition is imprescriptible and cannot be
barred by laches. Consequently, acquisitive prescription or laches does not lie in favor
of the Cagampang spouses and against Tirso, the general rule being that prescription
does not run against a co-owner or co-heir. The only exception to the imprescriptibility
of an action for partition against a co-owner is when a co-owner repudiates the co-
ownership. Thus, the appellate court ruled that by invoking extinctive prescription as a
defense, the lone exception against imprescriptibility of action by a co-owner, the
Cagampang spouses are deemed to have contextually recognized the co-ownership of
Tirso and must have repudiated such co-ownership in order for acquisitive prescription
to set in. Taking off from that premise, the appellate court then proceeded to tackle the
issue of repudiation by the Cagampang spouses. Therefore, we hold that the appellate
court did not err in finding that the Cagampang spouses are effectively barred from
invoking prescription, given that the subject properties are conjugal properties of the
decedent, Don Fabian, which cannot be subjected to acquisitive prescription, the
necessary consequence of recognizing the co-ownership stake of other legal heirs.

252
LEONORA B. CRUZ VS. TEOFILA M. CATAPANG

G.R. No. 164110


February 12, 2008

Facts:

Petitioner Leonora B. Cruz, Luz Cruz and Norma Maligaya are co-owners of a parcel of
land. Sometime in 1992, respondent Teofilo built a house on the said land with the
consent of Norma Maligaya. In 1995, petitioner visited the land and was surprised to
see a respondent‘s house intruding unto a portion of the co-owned property. She made
several demands upon respondent to demolish the intruding structure and vacate the
portion encroaching on their property. Respondent refused thus petitioner filed a
complaint for forcible entry. MCTC and RTC ruled in favor of the petitioner. CA on the
other hand reversed and set aside the appealed ruling of the lower courts. Hence,
petition of review.

Issue:

Whether or not the consent of one co-owner will warrant the dismissal of the forcible
entry case filed by another co-owner against the person who was given the consent to
construct a house on the co-owned property.

Ruling:

No. Articles 486 and 491 of the Civil Code provide:

Art. 486. Each co-owner may use the thing owned in common, provided he does so in
accordance with the purpose for which it is intended and in such a way as not to injure
the interest of the co-ownership or prevent the other co-owners from using it according
to their rights. The purpose of the co-ownership may be changed by agreement,
express or implied.

Art. 491. None of the co-owners shall, without the consent of the others, make
alterations in the thing owned in common, even though benefits for all would result
therefrom. However, if the withholding of the consent by one or more of the co-owners
is clearly prejudicial to the common interest, the courts may afford adequate relief.
Article 486 states each co-owner may use the thing owned in common provided he
does so in accordance with the purpose for which it is intended and in such a way as
not to injure the interest of the co-ownership or prevent the other co-owners from using
it according to their rights. Giving consent to a third person to construct a house on the
co-owned property will injure the interest of the co-ownership and prevent other co-
owners from using the property in accordance with their rights.

Under Article 491, none of the co-owners shall, without the consent of the others, make
alterations in the thing owned in common. It necessarily follows that none of the co-

253
owners can, without the consent of the other co-owners, validly consent to the making
of an alteration by another person, such as respondent, in the thing owned in common.
Alterations include any act of strict dominion or ownership and any encumbrance or
disposition has been held implicitly to be an act of alteration.[19] The construction of a
house on the co-owned property is an act of dominion. Therefore, it is an alteration
falling under Article 491 of the Civil Code. There being no consent from all co-owners,
respondent had no right to construct her house on the co-owned property.

Consent of only one co-owner will not warrant the dismissal of the complaint for forcible
entry filed against the builder. The consent given by Norma Maligaya in the absence of
the consent of petitioner and Luz Cruz did not vest upon respondent any right to enter
into the co-owned property. Her entry into the property still falls under the classification
through strategy or stealth. Wherefore, petition is granted.

254
REPUBLIC OF THE PHILIPPINES VS. HEIRS OF DIGNOS-SORONO

G.R. No. 171571


March 24, 2008

Facts:

The heirs of Tito Dignos sold the entire two lots to Civil Aeronautics Administration
(CAA) via public instrument entitled to Extrajudicial Settlement and Sale without the
knowledge of the respondents whose predecessors-in-interest were adjudicates of the
rest of the portion of the two lots. In 1996, CAA‘s successor-in-interest, the Mactan
Cebu International Airport Authority (MCIAA), erected a security fence traversing lot no.
2316 and relocated a number of families, who had built their dwellings. Respondents
asked the agents of the MCIAA to cease giving third persons permission to occupy the
lots but the same was ignored. Respondents filed a complaint for quieting of title, legal
redemption with prayer for writ of a preliminary injunction against MCIAA before the
RTC. Republic of the Philippines, represented by MCIAA claimed that through
acquisitive prescription they already acquired the said land or acquired ownership
thereof by extraordinary prescription. The RTC and CA ruled in favor of the
respondents. Hence, petition of certiorari.

Issue:

Whether or not respondents are precluded from recovering disputed lots due to the prior
sale to the Republic and by estoppel and laches.

Ruling:

No. The petition fails. Article 493 of the Civil Code provides: Each co-owner shall have
the full ownership of his part and of the fruits and benefits pertaining thereto, and he
may therefore alienate, assign or mortgage it, and even substitute another person in its
enjoyment, except when personal rights are involved. But the effect of the alienation of
the mortgage, with respect to the co-owners, shall be limited to the portion which may
be allotted to him in the division upon the termination of the co-ownership. A co-owner is
entitled to sell his undivided share, a sale of the entire property by one co-owner without
the consent of the other co-owners is not null and void. However, only the rights of the
co-owner-seller are transferred, thereby making the buyer a co-owner of the property.
Wherefore, the petition is denied.

255
CELESTINO BALUS VS. SATURNINO & LEONARDA BALUS VDA. DE CALUNOD

G.R. No. 168970


January 15, 2010

Facts:

Herein the petitioner and respondents are siblings. Before their father died who is Rufo,
the latter had a loan and the security of it was the land. Rufo failed to pay his loan and
as a result the mortgaged property was forclosed and subsequently sold to the Bank.
The property was not redeemed within the period as prescribed by law. In 1989,
petitioner and respondents executed an Extrajudicial Settlement of Estates adjudicating
to each of them a specific one-third portion of the subject property consisting of 10,246
square meters. The Extrajudicial Settlement also contained provisions wherein the
parties admitted knowledge of the fact that their father mortgaged the subject property
to the Bank and that they intended to redeem the same at the soonest possible time. In
1995, a deed of sale of registered land was executed by the Bank in favor of
respondents. Meanwhile, the petitioner continued possession thereof. Respondents
filed a complaint for recovery of possession and damages against the petitioner. The
RTC ruled in favor of petitioner but the CA reversed the decision. Hence, petition of
certiorari.

Issue:

Whether or not co-ownership still exist between petitioner and respondents.

Ruling:

No. Petitioner and respondents are arguing on the wrong premise that, at the time of the
execution of the Extrajudicial Settlement, the subject property formed part of the estate
of their deceased father to which they may lay claim as his heirs.

At the outset, it bears to emphasize that there is no dispute with respect to the fact that
the subject property was exclusively owned by petitioner and respondents' father, Rufo,
at the time that it was mortgaged in 1979. This was stipulated by the parties during the
hearing conducted by the trial court on October 28, 1996. Evidence shows that a
Definite Deed of Salewas issued in favor of the Bank on January 25, 1984, after the
period of redemption expired. There is neither any dispute that a new title was issued in
the Bank's name before Rufo died on July 6, 1984. Hence, there is no question that the
Bank acquired exclusive ownership of the contested lot during the lifetime of Rufo.
The rights to a person's succession are transmitted from the moment of his death. In
addition, the inheritance of a person consists of the property and transmissible rights
and obligations existing at the time of his death, as well as those which have accrued
thereto since the opening of the succession. In the present case, since Rufo lost
ownership of the subject property during his lifetime, it only follows that at the time of his
death, the disputed parcel of land no longer formed part of his estate to which his heirs

256
may lay claim. Stated differently, petitioner and respondents never inherited the subject
lot from their father. Petitioner and respondents, therefore, were wrong in assuming that
they became co-owners of the subject lot. Thus, any issue arising from the supposed
right of petitioner as co-owner of the contested parcel of land is negated by the fact that,
in the eyes of the law, the disputed lot did not pass into the hands of petitioner and
respondents as compulsory heirs of Rufo at any given point in time. Wherefore, the
petition is denied.

257
ELY QUILATAN ET. AL VS. HEIRS OF LORENZO QUILATAN

G.R. No. 183059


August 28, 2009

Facts:

On August 15, 1999, petitioners Ely Quilatan and Rosvida Quilatan-Elias filed Civil Case
No. 67367 for nullification of Tax Declaration Nos. D-014-00330 and D-014-00204 and
Partition of the Estate of the late Pedro Quilatan with damages against respondent heirs
of Lorenzo Quilatan. They claim that during his lifetime, Pedro Quilatan owned two
parcels of land covered by Tax Declaration Nos. 1680 and 2301, both located in Taguig,
Metro Manila; that sometime in 1998, they discovered that said tax declarations were
cancelled without their knowledge and new ones were issued, to wit: Tax Declaration
No. D-014-00204 and D-014-00330, under the names of Spouses Lorenzo Quilatan and
Anita Lizertiquez as owners thereof. On June 22, 2004, the trial court rendered its
decision declaring as void the cancellation of Tax Declaration Nos. 1680 and 2301. At
the same time, it ordered the partition of the subject properties into three equal shares
among the heirs of Francisco, Ciriaco and Lorenzo, all surnamed Quilatan. On appeal,
the Court of Appeals reversed without prejudice the decision of the trial court on the
ground that petitioners failed to implead other co-heirs who are indispensable parties to
the case. Thus, the judgment of the trial court was null and void for want of jurisdiction.
Petitioners filed a motion for reconsideration but it was denied. Hence, petition of
certiorari.

Issue:

Whether or not order of dismissal and re-filing of the case in order to implead
indispensable parties invite multiplicity of suit since the second action would be a
repetition of the first action.

Ruling:

No. On the issue of multiplicity of suits, the Court of Appeals correctly ordered the
dismissal of Civil Case No. 67367 without prejudice for want of jurisdiction. The
dismissal could have been avoided had petitioners, instead of merely stating in their
complaint the unimpleaded indispensable parties, joined them as parties to the case in
order to have a complete and final determination of the action. As aptly observed by the
appellate court:Indeed, a perusal of the records will show that plaintiffs-appellees did
not implead their other co-heirs, either as plaintiffs or defendants in the case. Their
complaint squarely stated that Pedro Quilatan had three children, namely, Ciriaco
Quilatan, Francisco Quilatan, and Lorenzo Quilatan, who are now all deceased. Ciriaco
Quilatan is survived by his children, namely, Purita Santos, Rosita Reyes, Renato
Quilatan, Danilo Quilatan, and Carlito Quilatan. Defendants-appellants are the children
of Lorenzo Quilatan. The plaintiffs-appellees, along with Solita Trapsi and Rolando
Quilatan, are the children of Francisco Quilatan. However, Purita Santos, Rosita Reyes,

258
Renato Quilatan, Danilo Quilatan, Carlito Quilatan, Solita Trapsi, and Rolando Quilatan
were not joined as parties in the instant case. In fine, the absence of an indispensable
party renders all subsequent actions of the court null and void for want of authority to
act, not only as to the absent parties but even as to those present. Wherefore, the
petition s denied.

259
REILLO ET.AL. VS. SAN JOSE

G.R. No. 166393


June 18, 2009

Facts:

Petitioner Reillo et. al specifically Ma. Teresa Pion a.k.a Ma. Teresa S.J. Fernando
successfully caused the cancellation of the title under the name of spouses Quiterio San
Jose and Antonina Espiritu Santo and was issued a new title under her name only to the
prejudice of respondents‘ rights who were also the successors-in-interest of said
spouses. Respondents assailed the validity of Extrajudicial Settlement of Estate Among
Heirs with Waiver of Rights and to cancel the title under the name of Ma. Teresa and
ask of partition the land co-owned by the petitioners and respondents. The RTC and CA
ruled in favor of San Jose. Hence, petition of certiorari.

Issue:

Whether or not RTC should have simply give directive order to pay docket fees and
should not make it as a ground for automatic dismissal of the case.

Ruling:

The rule regarding the payment of docket fees upon the filing of the initiatory pleading is
not without exception. It has been held that if the filing of the initiatory pleading is not
accompanied by payment of docket fees, the court may allow payment of the fee within
reasonable time but in no case beyond the applicable prescriptive or reglementary
period.

It is apparent from the arguments of the defendants-appellants that they are blaming the
trial court for their omission to pay the docket fees. It is, however, our opinion that the
defendants-appellants cannot pass on to the trial court the performance of a positive
duty imposed upon them by the law. It should be noted that their omission to file the
docket fees was raised as one of the grounds to dismiss the counter petition for
partition. The defendants-appellants opposed the said motion without, however, offering
an answer to the said ground raised by the plaintiffs-appellees. In fact, during the period
the motion was being heard by the trial court, the defendants-appellants never paid the
docket fees for their petition so that it could have at least brought to the attention of the
trial court their payment of the docket fees although belatedly done. They did not even
ask the trial court for time within which to pay the docket fees for their petition. When the
trial court ruled to dismiss the petition of the defendants-appellants, the latter did not, in
their motion for reconsideration, ask the trial court to reconsider the dismissal of their
petition by paying the required docket fees, neither did they ask for time within which to
pay their docket fees. In other words, the trial court could have issued an order allowing
the defendants-appellants a period to pay the docket fees for their petition if the
defendants-appellants made such manifestation. What is apparent from the factual

260
circumstances of the case is that the defendants-appellants have been neglectful in
complying with this positive duty imposed upon them by law as plaintiffs of the counter
petition for partition. Because of their omission to comply with their duty, no grave error
was committed by the trial court in dismissing the defendants-appellants counter petition
for partition.

261
FELICIDAD DADIZON, ET AL. VS. SOCORRO BERNADAS

G.R. No. 172367


June 5, 2009

Facts:

Petitioners and respondents are the children and representatives of the deceased
children of the late Diosdado Bernadas, Sr. who died intestate on February 1, 1977,
leaving in co-ownership with his then surviving spouse, Eustaquia Bernadas (who died
on May 26, 2000), several parcels of agricultural and residential land situated in Naval,
Biliran.

On May 14, 1999, respondents filed a Complaint against petitioners to compel the
partition of the one-half (1/2) conjugal share of the properties left by their late father
(subject properties) based on the Deed of Extrajudicial Partition dated February 24,
1996. Respondents alleged that petitioner Felicidad Dadizon was in possession of the
subject properties and refused to heed their demands to cause the partition of the
same. On the other hand, petitioners averred that the Deed of Extrajudicial Partition
dated February 24, 1996, which respondents sought to enforce, was revoked by the
Deed of Extrajudicial Partition dated February 10, 1999. They argued that certain
parcels of land included in respondents‘ complaint had long been disposed of or
extrajudicially partitioned by them. They further claimed that certain parcels of land
listed in the Deed of Extrajudicial Partition dated February 24, 1996 as sold to
respondent Socorro Bernadas could not go to the latter, since the alleged sales were
under annulment in a civil case pending before the RTC of Naval, Biliran, a case filed by
their mother, Eustaquia Bernadas, to revoke the sales of her one-half (1/2) conjugal
share on the grounds of lack of consideration, fraud and lack of consent.

During trial, on June 13, 2000, both parties manifested that in view of the death of their
mother, Eustaquia Bernadas, they have an ongoing negotiation for the extrajudicial
partition of the subject properties to end their differences once and for all. In the next
scheduled hearing, on November 15, 2000, the counsel of respondents asked for
postponement on the ground that he was in the process of soliciting the signatures of
other heirs to complete a compromise agreement. On January 30, 2001, the counsel of
respondents filed a Project of Partition.

On July 23, 2001, the RTC issued an Order approving the Project of Partition dated
October 23, 2000. Petitioners filed a Motion for Reconsideration of the said Order, but
the same was denied by the RTC. On appeal, the CA affirmed the RTC‘s decision.
Petitioners filed a Motion for Reconsideration of the assailed decision, but the same was
denied by the CA in its Resolution dated March 15, 2006. Hence, this Petition.

262
Issue:

Whether or not the CA erred when it affirmed the Order dated September 5, 2001 of the
RTC.

Ruling:

Yes. The court held that there are two stages in every action for partition under Rule 69
of the Rules of Court. The first stage is the determination of whether or not a co-
ownership in fact exists and a partition is proper (i.e., not otherwise legally proscribed)
and may be made by voluntary agreement of all the parties interested in the property.
The second stage commences when it appears that "the parties are unable to agree
upon the partition" directed by the court. In that event, partition shall be done for the
parties by the court with the assistance of not more than three (3) commissioners.

A careful study of the records of this case reveals that the RTC departed from the
foregoing procedure mandated by Rule 69. In its Order dated July 23, 2001, the RTC
noted that both parties filed the Project of Partition dated October 23, 2000 that it
approved. In its Order dated September 5, 2001 denying petitioners‘ motion for
reconsideration, the RTC reiterated that both parties filed the same. However, the
records show that the Project of Partition dated October 23, 2000 was filed only by
respondents‘ counsel, and that the same was not signed by the respondents or all of the
parties.

Even if petitioners did manifest in open court to the RTC that they have already agreed
with the respondents on the manner of partition of the subject properties, what is
material is that only the respondents filed the Project of Partition dated October 23,
2000 and that the same did not bear the signatures of petitioners because only a
document signed by all of the parties can signify that they agree on a partition. Hence,
the RTC had no authority to approve the Project of Partition dated October 23, 2000,
which did not bear all of the signatures of the parties, on the premise that they had all
agreed to the same. Likewise, the failure to file any comment or suggestion as to
manner of distribution of the subject properties does not justify the RTC‘s non-
observance of the procedure mandated by Rule 69. When the parties were unable to
submit the signed Project of Partition despite being ordered to do so, the RTC should
have ordered the appointment of commissioners to make the partition as mandated by
Section 3, Rule 69. Therefore, in partition proceedings, reference to commissioners is
required as a procedural step in the action and is not discretionary on the part of the
court.

263
SPOUSES LITA DE LEON & FELIX RIO TARROSA VS. ANITA B. DE LEON, ET. AL.

G.R. No. 185063


July 23, 2009

Facts:

On July 20, 1965, Bonifacio O. De Leon, then single, and the People‘s Homesite and
Housing Corporation (PHHC) entered into a Conditional Contract to Sell for the
purchase on installment of a 191.30 square-meter lot situated in Fairview, Quezon City.
Subsequently, on April 24, 1968, Bonifacio married Anita de Leon in a civil rite officiated
by the Municipal Mayor of Zaragosa, Nueva Ecija. To this union were born Danilo and
Vilma. Following the full payment of the cost price for the lot thus purchased, PHHC
executed, on June 22, 1970, a Final Deed of Sale in favor of Bonifacio. Accordingly,
Transfer Certificate of Title (TCT) No. 173677 was issued on February 24, 1972 in the
name of Bonifacio, "single." Subsequently, Bonifacio, for PhP 19,000, sold the subject
lot to her sister, Lita, and husband Felix Rio Tarrosa (Tarrosas), petitioners herein. The
conveying Deed of Sale dated January 12, 1974 (Deed of Sale) did not bear the written
consent and signature of Anita. Thereafter, or on May 23, 1977, Bonifacio and Anita
renewed their vows in a church wedding at St. John the Baptist Parish in San Juan,
Manila. On February 29, 1996, Bonifacio died.

Three months later, the Tarrosas registered the Deed of Sale and had TCT No. 173677
canceled. They secured the issuance in their names of TCT No. N-173911 from the
Quezon City Register of Deeds.

Getting wind of the cancellation of their father‘s title and the issuance of TCT No. N-
173911, Danilo and Vilma filed on May 19, 2003 a Notice of Adverse Claim before the
Register of Deeds of Quezon City to protect their rights over the subject property. Very
much later, Anita, Danilo, and Vilma filed a reconveyance suit before the RTC in
Quezon City. In their complaint, Anita and her children alleged, among other things, that
fraud attended the execution of the Deed of Sale and that subsequent acts of Bonifacio
would show that he was still the owner of the parcel of land. The Tarrosas, in their
Answer with Compulsory Counterclaim, averred that the lot Bonifacio sold to them was
his exclusive property inasmuch as he was still single when he acquired it from PHHC.
As further alleged, they were not aware of the supposed marriage between Bonifacio
and Anita at the time of the execution of the Deed of Sale.

On October 4, 2006, the RTC, on the finding that the lot in question was the conjugal
property of Bonifacio and Anita, rendered judgment in favor of Anita and her children.
On appeal, the CA rendered a decision affirmatory of that of the RTC. Hence, this
petition.

Issue:

Whether or not the subject property belongs to conjugal partnership.

264
Ruling:

No. The court held that Article 160 of the 1950 Civil Code, the governing provision in
effect at the time Bonifacio and Anita contracted marriage, provides that all property of
the marriage is presumed to belong to the conjugal partnership unless it is proved that it
pertains exclusively to the husband or the wife. In the case at bar, ownership over what
was once a PHHC lot and covered by the PHHC-Bonifacio Conditional Contract to Sell
was only transferred during the marriage of Bonifacio and Anita. It is well settled that a
conditional sale is akin, if not equivalent, to a contract to sell. In both types of contract,
the efficacy or obligatory force of the vendor‘s obligation to transfer title is subordinated
to the happening of a future and uncertain event, usually the full payment of the
purchase price, so that if the suspensive condition does not take place, the parties
would stand as if the conditional obligation had never existed. In other words, in a
contract to sell ownership is retained by the seller and is not passed to the buyer until
full payment of the price, unlike in a contract of sale where title passes upon delivery of
the thing sold.Such is the situation obtaining in the instant case. The conditional
contract to sell executed by and between Bonifacio and PHHC on July 20, 1965
provided that ownership over and title to the property will vest on Bonifacio only upon
execution of the final deed of sale which, in turn, will be effected upon payment of the
full purchase price

Evidently, title to the property in question only passed to Bonifacio after he had fully paid
the purchase price on June 22, 1970. This full payment, to stress, was made more than
two (2) years after his marriage to Anita on April 24, 1968. In net effect, the property
was acquired during the existence of the marriage; as such, ownership to the property
is, by law, presumed to belong to the conjugal partnership.

265
POSSESSION

266
ROLANDO S. ABADILLA, JR. VS. SPS. BONIFACIO P. OBRERO

G.R. No. 199448


November 12, 2014

Facts:

Complainant Sps. Bonifacio P. Obrero and Bernabela N. Obrero initiated a case for
forcible entry against defendant Rolando S. Abadilla, Jr. Complainants claimed that they
are the registered owners of the land in question based on a TCT registered under the
name. They claimed they were in possession thereof based on improvements erected
therein utilized for residential and business purposes prior to the alleged acts of
Respondent who forcible fenced the perimeter of the land with barbed wire.

By way of defense, defendant claimed that the land was sold by complainants to his late
father as evidenced by a Deed of Absolute Sale. Being one of the heirs, he is one of the
owners thereof. In fact, they left a caretaker to oversee the land. Despite the sale,
complainants supposedly attempted to remove the fence and even built concrete
structures on the land using it for dwelling purposes.

Ruling:

The defendant was held liable. In an ejectment case, title is not involved as the sole
issue is the determination of who is entitled to the physical or material possession of the
premises or possession de facto. ―Thus, where the parties to an ejectment case raise
the issue of ownership, the courts may pass upon that issue but only to determine who
between the parties has the better right to possess the property. As such, any
adjudication of the ownership issue is not final and binding; it is only provisional, and not
a bar to an action between the same parties involving title to the property.‖

Here, both parties anchor their right of possession on ownership. Between a Deed of
Absolute Sale and a TCT, it is the TCT which must prevail. ―A certificate of title is
evidence of indefeasible and incontrovertible title to the property in favor of the person
whose name appears therein. ‗[A] title issued under the Torrens system is entitled to all
the attributes of property ownership, which necessarily includes possession.‘ Hence, as
holders of the Torrens title over the subject land, the respondents are entitled to its
possession.

Further, in the Deed of Absolute Sale, no sale was perfected as the parties failed to
agree on the purchase price. Thus, defendants claim of possession ―had no sufficient
basis and it cannot overthrow the attribute of possession attached to the respondents‘
certificate of title.‖

267
Regarding the criminal case involving the Quitclaim, it is immaterial to this ejectment
case. ―Questions on the validity of a Torrens title are outside the jurisdiction and
competence of the trial court in ejectment proceedings which are limited only to the
determination of physical possession. This is in consonance with the settled doctrine
that questions relating to the validity of a certificate of title during ejectment proceedings
are deemed and proscribed as collateral attack to such title. A Torrens certificate of title
cannot be the subject of collateral attack. The title represented by the certificate cannot
be changed, altered, modified, enlarged, or diminished except in a direct proceeding.
Thus, issues as to the validity of the respondents‘ title can only be definitively resolved
in a direct proceeding for cancellation of title before the RTCs.‖

268
HEIRS OF BIENVENIDO & ARACELI TANYAG VS. SALOME E. GABRIEL, ET. AL.

G.R. No. 175763


April 11, 2012

Facts:

Subject of controversy are two adjacent parcels of land located at Ruhale, Barangay
Calzada, Municipality of Taguig (now part of Pasig City, Metro Manila). The first parcel
("Lot 1") with an area of 686 square meters was originally declared in the name of Jose
Gabriel under Tax Declaration (TD) Nos. 1603 and 6425 issued for the years 1949 and
1966, while the second parcel ("Lot 2") consisting of 147 square meters was originally
declared in the name of Agueda Dinguinbayan under TD Nos. 6418 and 9676 issued for
the years 1966 and 1967. For several years, these lands lined with bamboo plants
remained undeveloped and uninhabited.

Petitioners claimed that Lot 1 was owned by Benita Gabriel, sister of Jose Gabriel, as
part of her inheritance as declared by her in a 1944 notarized instrument ("Affidavit of
Sale") whereby she sold the said property to spouses Gabriel Sulit and Cornelia Sanga.
Lot 1 allegedly came into the possession of Benita Gabriel‘s own daughter, Florencia
Gabriel Sulit, when her father-in-law Gabriel Sulit gave it to her as part of inheritance of
his son, Eliseo Sulit who was Florencia‘s husband. Florencia Sulit sold the same lot to
Bienvenido S. Tanyag, father of petitioners, as evidenced by a notarized deed of sale
dated October 14, 1964. Petitioners then took possession of the property, paid the real
estate taxes due on the land and declared the same for tax purposes.

As to Lot 2, petitioners averred that it was sold by Agueda Dinguinbayan to Araceli


Tanyag under Deed of Sale executed on October 22, 1968. Thereupon, petitioners took
possession of said property and declared the same for tax purposes. Petitioners
claimed to have continuously, publicly, notoriously and adversely occupied both Lots 1
and 2 through their caretaker Juana Quinones; they fenced the premises and
introduced improvements on the land. On March 20, 2000, petitioners instituted in a civil
case alleging that respondents never occupied the whole 686 square meters of Lot 1
and fraudulently caused the inclusion of Lot 2 such that Lot 1 consisting of 686 square
meters originally declared in the name of Jose Gabriel was increased to 1,763 square
meters. On the other hand, respondents asserted that petitioners have no cause of
action against them for they have not established their ownership over the subject
property covered by a Torrens title in respondents‘ name.

In its decision, the trial court dismissed the complaint as well as the counterclaim,
holding that petitioners failed to establish ownership of the subject property and finding
the respondents to be the declared owners and legal possessors. It likewise ruled that
petitioners were unable to prove by preponderance of evidence that respondents
acquired title over the property through fraud and deceit. Petitioners appealed to the CA
which affirmed the trial court‘s ruling. Petitioners‘ motion for reconsideration was
likewise denied by the CA. Hence, this petition.

269
Issue:

Whether or not the petitioners acquired the property through acquisitive prescription.

Held:

Yes. The court held that the CA was mistaken in concluding that petitioners have not
acquired any right over the subject property simply because they failed to establish
Benita Gabriel‘s title over said property. The appellate court ignored petitioners‘
evidence of possession that complies with the legal requirements of acquiring
ownership by prescription.

Acquisitive prescription is a mode of acquiring ownership by a possessor through the


requisite lapse of time. In order to ripen into ownership, possession must be in the
concept of an owner, public, peaceful and uninterrupted. Possession is open when it is
patent, visible, apparent, notorious and not clandestine. It is continuous when
uninterrupted, unbroken and not intermittent or occasional; exclusive when the adverse
possessor can show exclusive dominion over the land and an appropriation of it to his
own use and benefit; and notorious when it is so conspicuous that it is generally known
and talked of by the public or the people in the neighborhood. The party who asserts
ownership by adverse possession must prove the presence of the essential elements of
acquisitive prescription.

In this case, it was only in 1979 that respondents began to assert a claim over the
property by securing a tax declaration in the name of Jose Gabriel albeit over a bigger
area than that originally declared. In 1998, they finally obtained an original certificate of
title covering the entire 1,763 square meters which included Lot 1. Thus, from 1969 until
the filing of this complaint by the petitioners in March 2000, the latter have been in
continuous, public and adverse possession of the subject land for 31 years. Having
possessed the property for the period and in the character required by law as sufficient
for extraordinary acquisitive prescription, petitioners have indeed acquired ownership
over the subject property. Such right cannot be defeated by respondents‘ acts of
declaring again the property for tax purposes in 1979 and obtaining a Torrens certificate
of title in their name in 1998.

270
CELERINO E. MERCADO VS. BELEN ESPINOCILLA & FERDINAND ESPINOCILLA

G.R. No. 184109


February 1, 2012

Facts:

Doroteo Espinocilla owned a parcel of land, Lot No. 552, with an area of 570 sq. m.,
located at Magsaysay Avenue, Zone 5, Bulan, Sorsogon. After he died, his five children,
Salvacion, Aspren, Isabel, Macario, and Dionisia divided Lot No. 552 equally among
themselves. Later, Dionisia died without issue ahead of her four siblings, and Macario
took possession of Dionisia‘s share. In an affidavit of transfer of real property dated
November 1, 1948, Macario claimed that Dionisia had donated her share to him in May
1945. Thereafter, on August 9, 1977, Macario and his daughters Betty Gullaba and
Saida Gabelo sold 225 sq. m. to his son Roger Espinocilla, husband of respondent
Belen Espinocilla and father of respondent Ferdinand Espinocilla. On March 8, 1985,
Roger Espinocilla sold 114 sq. m. to Caridad Atienza. Per actual survey of Lot No. 552,
respondent Belen Espinocilla occupies 109 sq. m., Caridad Atienza occupies 120 sq.
m., Caroline Yu occupies 209 sq. m., and petitioner, Salvacion's son, occupies 132 sq.
m.

Petitioner sued the respondents to recover two portions: an area of 28.5 sq. m. which
he bought from Aspren and another 28.5 sq. m. which allegedly belonged to him but
was occupied by Macario‘s house. His claim has since been modified to an alleged
encroachment of only 39 sq. m. that he claims must be returned to him. He avers that
he is entitled to own and possess 171 sq. m. of Lot No. 552, having inherited 142.5 sq.
m. from his mother Salvacion and bought 28.5 sq. m. from his aunt Aspren. According
to him, his mother‘s inheritance is 142.5 sq. m., that is, 114 sq. m. from Doroteo plus
28.5 sq. m. from Dionisia. Since the area he occupies is only 132 sq. m., he claims that
respondents encroach on his share by 39 sq. m. On the other hand, the respondents
agree that Doroteo‘s five children each inherited 114 sq. m. of Lot No. 552. However,
Macario‘s share increased when he received Dionisia‘s share. Macario‘s increased
share was then sold to his son Roger, respondents‘ husband and father. Respondents
claim that they rightfully possess the land they occupy by virtue of acquisitive
prescription and that there is no basis for petitioner‘s claim of encroachment.

On May 15, 2006, the Regional Trial Court (RTC) ruled in favor of petitioner. On appeal,
the CA reversed the RTC decision and dismissed petitioner‘s complaint on the ground
that extraordinary acquisitive prescription has already set in in favor of respondents.
Hence, this petition.

Issue:

Whether or not petitioner‘s action to recover the subject portion is barred by


prescription.

271
Ruling:

Yes. The court held that prescription, as a mode of acquiring ownership and other real
rights over immovable property, is concerned with lapse of time in the manner and
under conditions laid down by law, namely, that the possession should be in the
concept of an owner, public, peaceful, uninterrupted, and adverse. Acquisitive
prescription of real rights may be ordinary or extraordinary. Ordinary acquisitive
prescription requires possession in good faith and with just title for 10 years. In
extraordinary prescription, ownership and other real rights over immovable property are
acquired through uninterrupted adverse possession for 30 years without need of title or
of good faith. In this case, petitioner himself admits the adverse nature of respondents‘
possession with his assertion that Macario‘s fraudulent acquisition of Dionisia‘s share
created a constructive trust. In a constructive trust, there is neither a promise nor any
fiduciary relation to speak of and the so-called trustee (Macario) neither accepts any
trust nor intends holding the property for the beneficiary (Salvacion, Aspren, Isabel).
The relation of trustee and cestui que trust does not in fact exist, and the holding of a
constructive trust is for the trustee himself, and therefore, at all times adverse.
Prescription may supervene even if the trustee does not repudiate the relationship.
Therefore, Petitioner‘s action for recovery of possession having been filed 55 years after
Macario occupied Dionisia‘s share was barred by extinctive prescription.

272
JULITA V. IMUAN, ET. AL. VS. JUANITO CERENO, ET. AL.

G.R. No. 167995


September 11, 2009

Facts:

Pablo contracted two marriages and all his children on both are already dead. The
petitioners in the case are his grandchildren while the respondent is the husband of his
daughter from his second marriage. Juana, Pablo‘s second wife, together with her
children continued to be in possession of the parcel of land owned by Pablo after his
death. A joint affidavit was executed attesting that Pablo ceded the property in favor of
Juana in the occasion of their marriage but the document was lost. Juana sold said
parcel of land to the respondent which was registered in the register of deeds. The land
area sold to respondents was divided by a barangay road. They built a house on one
side and planted fruit-bearing trees on the other side. It is on the latter‘s side where the
petitioners took possession and built a nipa hut thereon. An ejectment case was filed by
the respondents against petitioners but was later dismissed when the petitioners left the
area.

Petitioners now brought an action for reconveyance, damages, and annulment of deed
of sale by Juana to the respondents. They contend that it was through their tolerance
that Juana and her children constructed their house on the lot in dispute, that Pablo
have not partitioned among his heirs his property and the sale made by Juana to
respondents are null and void. Respondents invoke the ground that when Pablo married
Juana the property was his exclusive property and donated such through propter
nuptias when they married. Thus Juana, being the owner of said lot, validly made the
sale to respondents who immediately took possession over the land and paid its realty
tax. MTC ruled in favor of the petitioners however upon appeal CA ruled in favor of the
respondents as it held that the respondents are in peaceful possession of said lot for 29
years which suffice to meet the requirement of 10-year period of open, public, and
adverse possession in the concept of owner that the law on prescription requires. It
ruled that petitioners are barred by latches from claiming ownership of the disputed
property. Hence, this petition.

Issue:

Whether or not the petitioners are barred by latches and prescription in claiming their
share of the property.

Ruling:

Yes. The court held that the respondents have acquired the disputed property by
acquisitive prescription. Prescription is another mode of acquiring ownership and other
real rights over immovable property and is concerned with a lapse of time laid down by
law where possession should be in the concept of an owner, public, peaceful,

273
uninterrupted, and adverse. Possession is open when it is patent, visible, apparent, and
notorious. It is continuous when uninterrupted, unbroken and not intermittent or
occasional; exclusive when the adverse possessor can show exclusive dominion over
the land and an appropriation of it to his own use and benefit; and notorious when it is
so conspicuous that it is generally known and talked of by the public or the people in the
neighborhood. The party who asserts ownership by adverse possession must prove the
presence of the essential elements of acquisitive prescription. Ordinary acquisitive
prescription requires possession in good faith and a just title in 10 years while
extraordinary acquisitive prescription involves uninterrupted adverse possession for 30
years without the need for good title and good faith.

Respondents immediately took possession of the property after buying it and diligently
paid its realty tax. Even if the petitioners saw respondents built a house thereon and
planted fruit- bearing trees, they did not raise objection on the respondent‘s possession.
Their inaction further made them guilty of latches since they live merely 100 meters
away from the property to know of the respondent‘s possession of said land. They only
filed an action for reconveyance 29 years after the respondent‘s peaceful possession
over the property, the 10-year prescription period for ordinary acquisitive prescription
has already lapsed. Thus, the respondents are the rightful owner of the land in dispute.

274
SEMIRARA COAL CORPORATION VS. HGL DEVELOPMENT CORPORATION

G.R. No. 166854


December 6, 2006

Facts:

Sometime in 1999, Semirara‘s representatives approached HGL and requested for


permission to allow Semirara‘s trucks and other equipment to pass through the property
covered by the FLGLA. HGL granted the request on condition that Semirara‘s use
would not violate the FLGLA in any way. However, Semirara erected several buildings
for Semirara‘s administrative offices and employees residences without HGLs
permission. Semirara also conducted blasting and excavation; constructed an access
road to petitioners minesite; and maintained a stockyard for the coal it extracted from its
mines. Thus, the land was greatly damaged, causing the decimation of HGLs cattle.
HGL wrote Semirara demanding full disclosure of the latter‘s activities on the subject
land as well as prohibiting them from constructing any improvements without HGLs
permission. Semirara ignored the demand and continued with its activities.

The DENR unilaterally cancelled FLGLA No. 184 and ordered HGL to vacate the
premises. The DENR found that HGL failed to pay the annual rental and surcharges
from 1986 to 1999 and to submit the required Grazing Reports or pay the corresponding
penalty for non-submission thereof. HGL filed a letter of reconsideration which was
denied by DENR. The DENR stated that it had coordinated with the DOE, which had
jurisdiction over coal or coal deposits and coal-bearing lands, and was informed that
coal deposits were very likely to exist in Sitios Bobog and Pontod. Hence, unless it
could be proved that coal deposits were not present, HGLs request had to be denied.
HGL filed a complaint against the DENR for specific performance and damages with
prayer for a temporary restraining order and/or writ of preliminary injunction. HGL had
also filed a complaint against Semirara for Recovery of Possession and Damages with
Prayer for TRO and/or Writ of Preliminary Mandatory Injunction.

Issue:

Whether or not the Writ of Preliminary Mandatory Injunction shall be granted.

Held:

Yes. Under Article 539 of the New Civil Code, a lawful possessor is entitled to be
respected in his possession and any disturbance of possession is a ground for the
issuance of a writ of preliminary mandatory injunction to restore the possession. With
the enactment of Article 539, the plaintiff is now allowed to avail of a writ of preliminary
mandatory injunction to restore him in his possession during the pendency of his action
to recover possession. It is likewise established that a writ of mandatory injunction is
granted upon a showing that (a) the invasion of the right is material and substantial; (b)

275
the right of complainant is clear and unmistakable; and (c) there is an urgent and
permanent necessity for the writ to prevent serious damage.

In the instant case, it is clear that as holder of a pasture lease agreement under FLGLA
No. 184, HGL has a clear and unmistakable right to the possession of the subject
property. Under the FLGLA, HGL has the right to the lawful possession of the subject
property for a period of 25 years. As lawful possessor, HGL is therefore entitled to
protection of its possession of the subject property and any disturbance of its
possession is a valid ground for the issuance of a writ of preliminary mandatory
injunction in its favor. The right of HGL to the possession of the property is confirmed by
Samirara itself when it sought permission from HGL to use the subject property in 1999.
In contrast to HGL‘s clear legal right to use and possess the subject property,
Samirara‘s possession was merely by tolerance of HGL and only because HGL
permitted Samirara to use a portion of the subject property so that the latter could gain
easier access to its mining area in the Panaan Coal Reserve.

276
SOLEDAD CALICDAN VS. SILVERIO CENDANA

G.R. No. 155080


February 5, 2004

Facts:

The land in question was formerly owned by Sixto Calicdan, who died intestate and was
survived by his wife, Fermina, and three children, namely, Soledad, Jose and Benigno,
all surnamed Calicdan. Fermina executed a deed of donation inter vivos whereby she
conveyed the land to respondent Silverio Cendaña, who immediately entered into
possession of the land, built a fence around the land and constructed a two-storey
residential house thereon, where he resided until his death. Soledad, through her legal
guardian Guadalupe Castillo, filed a complaint for Recovery of Ownership, Possession
and Damages against Silverio, alleging that the donation was void; that the latter took
advantage of her incompetence in acquiring the land; and that she merely tolerated
Silverio‘s possession of the land as well as the construction of his house thereon.
Silverio alleged, by way of affirmative defenses, that the land was donated to him by
Fermina; and that he had been publicly, peacefully, continuously, and adversely in
possession of the land for a period of 45 years.

The RTC ruled in favor of Soledad. On Appeal, the CA reversed the RTC‘s decision
holding that the donation was valid. Hence, the petition.

Issue:

Whether or not the donation in favor of Silverio was valid.

Ruling:

No. The Supreme Court held that the donation of the land was void because Fermina
was not the owner thereof, considering that it was inherited by Sixto from his parents.
Thus, the land was not part of the conjugal property of the spouses Sixto and Fermina
Calicdan, because under the Spanish Civil Code, the law applicable when Sixto died in
1941, the surviving spouse had a right of usufruct only over the estate of the deceased
spouse. However, notwithstanding the invalidity of the donation, the Court found that
respondent has become the rightful owner of the land by extraordinary acquisitive
prescription.

277
LINA LUBOS VS. MARITES GALUPO

G.R. No. 139136


January 16, 2002

Facts:

The subject of the case is a parcel of land with an area of 10.8224 hectares, more or
less, located in Sitio Naganaga, Barrio Cababtuan, Municipality of Pambujan, Northern
Samar, originally tax declared in the name of Victoriana Dulay. The plaintiffs claim that
in a private instrument written in Spanish entitled Escritura de Compra y Venta,
Victoriana Dulay and her son Restituto sold the property to Juan Galupo. On the death
of Juan Galupo, the same was inherited by his son Mansueto Galupo, Sr. Mansueto
Galupo, Sr. died in 1981. The plaintiffs are his children out of two marriages. The
plaintiffs complain that in 1984, they discovered the land to be occupied by the tenant
farmers of Lina Lubos. They found that the original Tax Declaration had been re-
assessed by Lina Abalon Lubos, who obtained a new Tax Declaration in her name. On
November 16, 1990, Lina Abalon Lubos sold a portion of the land to Alicio Poldo,
married to Teresita Poldo. The plaintiffs sought the annulment of the sale, and a
declaration that they are the lawful owners of the land.

On the other hand, Lina Lubos contends that the land was originally owned by
Victoriana Dulay alone, who is her great grandmother. Victoriana Dulay sold the
property to her father Juan Abalon. Her father possessed the property for over thirty
years until 1975, when he sold the same to her. Subsequently, she sold a portion of the
property to the spouses Poldo, who claim to be purchasers in good faith. The trial court
ruled in favor of the plaintiffs. On appeal, the CA affirmed the RTC‘s decision in toto.

Hence, the petition.

Issue:

Whether or not respondents may still file an action to recover realty.

Ruling:

No. The Court held that real actions over immovables prescribe after thirty years. To
determine if the respondents may still file an action to recover realty it becomes
necessary to determine if Lubos has acquired the land by acquisitive prescription.
Acquisitive prescription of dominion and other real rights may be ordinary or
extraordinary, depending on whether the property is possessed in good faith and with
just title for the time fixed by law.

Articles 1134 and 1137 of the Civil Code fixed the periods of possession, which provide:
Art. 1134. Ownership and other real rights over immovable property are acquired by
ordinary prescription through possession of ten years.

278
Art. 1137. Ownership and other real rights over immovables also prescribe through
uninterrupted adverse possession thereof for thirty years, without need of title or of good
faith.

For the purposes of prescription, there is just title when the adverse claimant came into
possession of the property through one of the modes recognized by law for the
acquisition of ownership or other real rights, but the grantor was not the owner or could
not transmit any right. On the other hand, good faith consists in the reasonable belief
that the person from whom the possessor received the thing was its owner but could not
transmit the ownership thereof.

The contract entered into between Lubos and her father, Juan Abalon, was fictitious.
Consequently, petitioner has not acquired a just title to the property.

279
ERNANDA MENDOZA CEQUEÑA VS. HONORATA MENDOZA BOLANTE

G.R. No. 137944


April 6, 2000

Facts:

Prior to 1954 a parcel of land located in the Province of Rizal was originally declared for
taxation purposes of the father, Sinfroso Mendoza, of Bolante who also is the present
occupant of the land. Subsequently the said tax declaration was cancelled and declared
under the name of Sinfroso's brother, Margarito Mendoza. On October 15, 1975,
Bolante and another brother of the petitioners, Miguel, had a dispute over the ownership
of the land during the cadastral survey of the same.

Trial Court rendered that the rightful owner and possessor of the land is the petitioners.
The Court of Appeals reversed the decision on the ground that the affidavits presented
during trial had not been sufficiently established. Moreover, the appellate court held that
the probative value of petitioners' tax receipts and declarations paled in comparison with
respondent's proof of ownership of the disputed parcel. Actual, physical, exclusive and
continuous possession by Bolante since 1985 indeed gave her a better title under
Article 538 of the Civil Code. Hence, the petition.

Issue:

Whether or not Bolante is the lawful owner and possessor of the parcel of land.

Ruling:

Yes. The Court held that when Bolante came of age in 1948, she paid realty taxes for
the years 1932-1948. Margarito declared the lot for taxation in his name in 1953 and
paid its realty taxes beginning 1952. When he died, Miguel continued cultivating the
land. Bolante and her mother were living on the land, which was being tilled by Miguel
until 1985 when he was physically ousted by the Bolante.

Based on Article 538 of the Civil Code, Bolante is the preferred possessor because,
benefiting from her father's tax declaration of the subject lot since 1926, she has been in
possession thereof for a longer period. On the other hand, Cequena's father acquired
joint possession only in 1952.

Ownership of immovable property is acquired by ordinary prescription through


possession for ten years. Being the sole heir of her father, Bolante showed through his
tax receipt that she had been in possession of the land for more than ten years since
1932. When her father died in 1930, she continued to reside there with her mother.
When she got married, she and her husband engaged in kaingin inside the disputed lot
for their livelihood. Bolante's possession was not disturbed until 1953 when Cequena's
father claimed the land. But by then, her possession, which was in the concept of owner

280
— public, peaceful, and uninterrupted— had already ripened into ownership.
Furthermore she herself, after her father's demise, declared and paid realty taxes for the
disputed land. Tax receipts and declarations of ownership for taxation, when coupled
with proof of actual possession of the property, can be the basis of a claim for
ownership through prescription.

In contrast, the Cequenas, despite thirty-two years of farming the subject land, did not
acquire ownership. It is settled that ownership cannot be acquired by mere occupation.
Unless coupled with the element of hostility toward the true owner, occupation and use,
however long, will not confer title by prescription or adverse possession. Moreover, the
petitioners cannot claim that their possession was public, peaceful and uninterrupted.
Although their father and brother arguably acquired ownership through extraordinary
prescription because of their adverse possession for thirty-two years, this supposed-
ownership cannot extend to the entire disputed lot, but must be limited to the portion
that they actually farmed.

281
PHILIPPINE NATIONAL BANK VS. COURT OF APPEALS

G.R. No. 135219


January 17, 2002

Facts:

Spouses Godofredo and Wilma Monsod obtained a loan in the amount of P120,000.00
from Philippine National Bank. To secure their loan, the Monsods mortgaged to PNB a
parcel of land. Due to Monsods‘ failure to pay their loan obligation, PNB extrajudicially
foreclosed the mortgage. At the auction sale of real property, PNB was declared the
highest bidder. A certificate of sale was issued in favor of PNB. Upon expiration of the
redemption period, ownership of the property was consolidated in PNB.

On June 23, 1992, PNB filed an "Ex-Parte Petition for the Issuance of Writ of
Possession". PNB‘s representative testified that the foreclosed property is occupied by
one Ernesto Austria. According to PNB, Mr. Austria was invited by the bank to a
conference to discuss the ownership of the foreclosed lot, however, he did not honor the
bank‘s invitation. The trial court granted PNB‘s petition and a writ of possession was
issued.

Ernesto and Loreto Quintana Austria filed a "Motion for Intervention and to Recall and/or
Stop the Enforcement of the Writ of Possession." The Austrias alleged that they are the
actual occupants of the subject lot, which they purportedly bought from the Monsods as
early as 1974. They claimed that the foreclosed property was enclosed within a
concrete fence and formed part of their family compound. PNB allegedly knew of this
fact even before it granted the loan to the Monsods, because the bank‘s credit
investigators were advised of the same when they inspected the property in the summer
of 1976. Consequently, the Austrias maintained that the issuance of the possessory writ
ex parte was improper, since it will deprive them of their property without due process.
Hence, PNB filed the instant petition

Issue:

Whether or not an ex-parte writ of possession issued pursuant to Act No. 3135, as
amended, can be enforced against a third person who is in actual possession of the
foreclosed property and who is not in privity with the debtor/mortgagor.

Ruling:

No. The court held that the writ of possession can only be implemented against the
debtor/mortgagor and his successors-in-interest. Since respondents acquired their
rights as owners of the property by virtue of a sale made to them by the Monsods prior
to the bank‘s mortgage lien, respondents cannot be dispossessed therefrom without
due notice and hearing, through the simple expedient of an ex-parte possessory writ.

282
Under applicable laws and jurisprudence, they cannot be ejected from the property by
means of an ex-parte writ of possession.

The operative provision under Act No. 3135, as amended, is Section 6, which states:
Sec. 6. Redemption. – In all cases in which an extrajudicial sale is made under the
special power hereinbefore referred to, the debtor, his successors in interest or any
person having a lien on the property subsequent to the mortgage or deed of trust under
which the property is sold, may redeem the same at any time within the term of one
year from and after the date of the sale; and such redemption shall be governed by the
provisions of section four hundred and sixty-four to four hundred and sixty-six, inclusive,
of the Code of Civil Procedure, in so far as these are not inconsistent with the provisions
of this Act.

A third person in possession of an extrajudicially foreclosed realty, who claims a right


superior to that of the original mortgagor, will have no opportunity to be heard on his
claim in a proceeding of this nature. It stands to reason, therefore, that such third
person may not be dispossessed on the strength of a mere ex-parte possessory writ,
since to do so would be tantamount to his summary ejectment, in violation of the basic
tenets of due process.

283
CARMEN CANIZA VS. COURT OF APPEALS

G.R. No. 110427


February 24, 1997

Facts:

Cañiza was the owner of a house and lot at No. 61 Tobias St., Quezon City. On
September 17, 1990, her guardian Amparo Evangelista commenced a suit in MeTC to
eject spouses Estrada from said premises. It was alleged that Cañiza was the absolute
owner of the property in question, covered by TCT 27147; that out of kindness, she
allowed spouses Estrada, et. al., to temporary reside in her house, rent-free; that
plaintiff already had need of the house on account of her advanced age and failing
health; that plaintiff, through guardian, asked the Estradas verbally and writing to vacate
the house but the latter refused; that the Estradas unlawfully deprived plaintiff of her
possession. In their Answer with Counterclaim, the defendants declared that they had
been living in Cañiza's house since the 1960's; that in consideration of their faithful
service they had been considered by Cañiza as her own family, and the latter had in
fact executed a holographic will on September 4, 1988 by which she "bequeathed" to
the Estradas the house and lot in question. MeTC ruled in favor of plaintiff. RTC
however reversed the decision holding that "action by which the issue of defendants'
possession should be resolved is accion publiciana, the obtaining factual and legal
situation . . demanding adjudication by such plenary action for recovery of possession
cognizable in the first instance by the Regional Trial Court." On appeal, CA affirmed
RTC‘s decision in toto.

Issue:

Whether or not an ejectment action is the appropriate judicial remedy for recovery of
possession of the property in dispute.

Ruling:
Yes. Cañiza's act of allowing the Estradas to occupy her house, rent-free, did not create
a permanent and indefeasible right of possession in the latter's favor. Common sense,
and the most rudimentary sense of fairness clearly require that that act of liberality be
implicitly, but no less certainly, accompanied by the necessary burden on the Estradas
of returning the house to Cañiza upon her demand. More than once has this Court
adjudged that a person who occupies the land of another at the latter's tolerance or
permission without any contract between them is necessarily bound by an implied
promise that he will vacate upon demand, failing which a summary action for ejectment
is the proper remedy against him. The situation is not much different from that of a
tenant whose lease expires but who continues in occupancy by tolerance of the owner,
in which case there is deemed to be an unlawful deprivation or withholding of
possession as of the date of the demand to vacate. In other words, one whose stay is
merely tolerated becomes a deforciant illegally occupying the land or property the
moment he is required to leave.

284
HABAGAT GRILL VS. DMC – URBAN PROPERTY DEVELOPER, INC.

G.R. No. 155110


March 31, 2005

Facts:

On June 11, 1981, David M. Consunji, Inc. acquired and became the owner of a
residential lot situated in Matina, Davao City and covered by TCT No. T-82338. On June
13, 1981, David M. Consunji, Inc. transferred said lot to its sister company, the DMC
Urban Property Developers, Inc. Alleging that Louie Biraogo forcibly entered said lot
and built thereon the Habagat Grill in December, 1993, DMC filed on March 28, 1994 a
Complaint for Forcible Entry against Habagat Grill and/or Louie Biraogo in the MTC of
Davao. Biraogo replied denying all allegations. MTC dismissed the case on the ground
of lack of jurisdiction and lack of cause of action. DMC appealed to the RTC which
affirmed the appealed decision. Motion for reconsideration was filed but was denied.
They appealed to the CA. The CA faulted petitioner for not presenting any other
documentary evidence to establish the date of Habagat Grill‘s construction. It added
that the court of origin had improperly adjudged the subject property as part of the
public domain. The appellate court explained that the lower court could take cognizance
of Presidential Proclamation No. 20, but not of the situational relation between the
property covered by the Proclamation and the land in question. The CA further criticized
petitioner for not presenting any evidence to show the basis of the latter‘s alleged
authority to build Habagat Grill on the property. Hence, this Petition.

Issue:

Whether or not there was prior possession on the part of respondent‘s predecessor-in-
interest.

Ruling:

Petitioner avers that respondent failed to prove that the latter‘s predecessor-in-interest
had prior possession of the property. Conversely, respondent alleges that its
predecessor was in prior physical possession of the property as the registered owner
thereof since June 11, 1981. Again, we rule for respondent. For one to be considered in
possession, one need not have actual or physical occupation of every square inch of
the property at all times. In the present case, prior possession of the lot by respondent‘s
predecessor was sufficiently proven by evidence of the execution and registration of
public instruments and by the fact that the lot was subject to its will from then until
December 1, 1993, when petitioner unlawfully entered the premises and deprived the
former of possession thereof. Hence, petition is denied and the decision of the CA is
affirmed.

285
USUFRUCT

286
PEDRO SEPULVEDA, SR. VS. ATTY. PACIFICO S. PELAEZ

G.R. No. 152195


January 31, 2005

Facts:

On December 6, 1972, private respondent filed a complaint against his granduncle


Pedro Sepulveda, with the CFI of Cebu, for the recovery of certain parcels of land. In his
complaint, the private respondent alleged that his mother Dulce died intestate on March
2, 1944, and aside from himself, was survived by her husband Rodolfo Pelaez and her
mother Carlota Sepulveda. Dulce‘s grandfather Vicente Sepulveda died intestate on
October 25, 1920, and Dulce was then only about four years old. According to the
private respondent, his grandmother Carlota repeatedly demanded the delivery of her
mother‘s share in the eleven parcels of land, but Pedro Sepulveda, Sr. who by then was
the Municipal Mayor of Tudela, refused to do so. Dulce, likewise, later demanded the
delivery of her share in the eleven parcels of land, but Pedro Sepulveda, Sr. still
refused, claiming that he needed to continue to possess the property to reap the
produce therefrom which he used for the payment of the realty taxes on the subject
properties. The private respondent alleged that he himself demanded the delivery of his
mother‘s share in the subject properties on so many occasions, the last of which was in
1972, to no avail. The private respondent further narrated that his granduncle executed
an affidavit on November 28, 1961, stating that he was the sole heir of Dionisia when
she died intestate, when, in fact, the latter was survived by her three sons, Santiago,
Pedro and Vicente. Pedro Sepulveda, Sr. also executed a Deed of Absolute Sale5 on
July 24, 1968 over the property covered by T.D. No. 19804 in favor of the City of Danao
for P7,492.00. According to the private respondent, his granduncle received this amount
without his knowledge. The trial court ruled that the private respondent‘s action for
reconveyance based on constructive trust had not yet prescribed when the complaint
was filed; that he was entitled to a share in the proceeds of the sale of the property to
Danao City; and that the partition of the subject property among the adjudicatees
thereof was in order. The petitioner appealed the decision to the CA, which affirmed the
appealed decision with modification. Hence, this petition.

Issue:

Whether or not a spouse can have a share in the usufruct in the estate of the deceased
spouse.

Ruling:

Yes. Under articles 807 and 834 of the old Civil Code the surviving spouse is a forced
heir and entitled to a share in usufruct in the estate of the deceased spouse equal to
that which by way of legitime corresponds or belongs to each of the legitimate children
or descendants who have not been bettered or have not received any share in the one-
third share destined for betterment. The right of the surviving spouse to have a share in

287
usufruct in the estate of the deceased spouse is provided by law of which such spouse
cannot be deprived and which cannot be ignored. Of course, the spouse may waive it
but the waiver must be express. Thus, all the co-heirs and persons having an interest in
the property are indispensable parties; as such, an action for partition will not lie without
the joinder of the said parties.

288
NATIONAL HOUSING AUTHORITY VS. GRACE BAPTIST CHURCH

G.R. No. 156437


March 1, 2004

Facts:

On June 13, 1986, the church wrote a letter to petitioner NHA, manifesting its interest in
acquiring Lots 4 and 17 of the General Mariano Alvarez Resettlement Project in Cavite.
The request was thereby granted. Respondent entered into possession of the lots and
introduced improvements thereon. On February 22, 1991, NHA‘s board of directors
passed a resolution approving sale of the lots to the church. The church was duly
informed of such. On April 8, 1991, the Church tendered to the NHA a manager‘s check
in the amount of P55,350.00, purportedly in full payment of the subject properties. The
Church insisted that this was the price quoted to them by the NHA Field Office, as
shown by an unsigned piece of paper with a handwritten computation scribbled
thereon.10Petitioner NHA returned the check, stating that the amount was insufficient
considering that the price of the properties have changed. The Church made several
demands on the NHA to accept their tender of payment, but the latter refused. Thus, the
Church instituted a complaint for specific performance and damages against the NHA
with the RTC. The RTC rendered a judgment: 1. Ordering the defendant to reimburse to
the plaintiff the amount of P4,290.00 representing the overpayment made for Lots 1, 2,
3, 18, 19 and 20; 2. Declaring that there was no perfected contract of sale with respect
to Lots 4 and 17 and ordering the plaintiff to return possession of the property to the
defendant and to pay the latter reasonable rental for the use of the property at P200.00
per month computed from the time it took possession thereof until finally vacated. On
appeal, CA affirmed RTC‘s findings. Hence, petition for review.

Issue:
Whether or not there was mutual consent by the parties.

Ruling:
In the case at bar, the offer of the NHA to sell the subject property, as embodied in
Resolution No. 2126, was similarly not accepted by the respondent. Thus, the alleged
contract involved in this case should be more accurately denominated as inexistent.
There being no concurrence of the offer and acceptance, it did not pass the stage of
generation to the point of perfection. As such, it is without force and effect from the very
beginning or from its incipiency, as if it had never been entered into, and hence, cannot
be validated either by lapse of time or ratification. Equity cannot give validity to a void
contract, and this rule should apply with equal force to inexistent contracts. We note
from the records, however, that the Church, despite knowledge that its intended
contract of sale with the NHA had not been perfected, proceeded to introduce
improvements on the disputed land. On the other hand, the NHA knowingly granted the
Church temporary use of the subject properties and did not prevent the Church from
making improvements thereon. Thus, the respondent Church and the NHA, who both
acted in bad faith, shall be treated as if they were both in good faith.

289
NUISANCE

290
EMILIO GANCAYCO VS. CITY GOVERNMENT OF QUEZON CITY & MMDA

G.R. No. 177807


October 11, 2011

Facts:

In the early 1950s, retired Justice Gancayco bought a parcel of land located at EDSA
covered by TCT No. RT114558. On 27 March 1956, the Quezon City Council issued
Ordinance No. 2904 which required the relevant property owner to construct an arcade
with a width of 4.50 meters and height of 5.00 meters along EDSA, from the north side
of Santolan Road to one lot after Liberty Avenue, and from one lot before Central
Boulevard to the Botocan transmission line. The ordinance was amended several times.
The ordinance covered the property of Justice Gancayco. Subsequently, sometime in
1965, Justice Gancayco sought the exemption of a two-storey building being
constructed on his property from the application of Ordinance 2904 that he be exempted
from constructing an arcade on his property. The City Council acted favorably on
Justice Gancayco. On April 28, 2003, MMDA sent a notice of demolition to Justice
Gancayco alleging that a portion of his building violated the National Building Code in
relation to Ordinance 2904 and gave him 15 days to clear the portion of the building that
was supposed to be an arcade along EDSA. Justice Gancayco did not comply with the
notice. Soon after the lapse of the fifteen days, the MMDA proceeded to demolish the
party wall, or what was referred to as the "wing walls," of the ground floor structure. The
records of the present case are not entirely clear on the extent of the demolition;
nevertheless, the fact of demolition was not disputed. At the time of the demolition, the
affected portion of the building was being used as a restaurant. On 29 May 2003,
Justice Gancayco filed a Petition with prayer for a temporary restraining order and/or
writ of preliminary injunction before the RTC which rendered decision in favor of Justice
Gancayco. It held that the questioned ordinance was unconstitutional, ruling that it
allowed the taking of private property for public use without just compensation. The RTC
said that because 67.5 square meters out of Justice Gancayco‘s 375 square meters of
property were being taken without compensation for the public‘s benefit, the ordinance
was confiscatory and oppressive. Decision was appealed. The CA upheld the validity of
the Ordinance. Hence, this petition.

Issue:

Whether or not the ―wing walls‖ of the building are nuisances per se.

Ruling:

No. The fact that in 1966 the City Council gave Justice Gancayco an exemption from
constructing an arcade is an indication that the wing walls of the building are not
nuisances per se. The wing walls do not per se immediately and adversely affect the
safety of persons and property. The fact that an ordinance may declare a structure
illegal does not necessarily make that structure a nuisance. Clearly, when Justice

291
Gancayco was given a permit to construct the building, the city council or the city
engineer did not consider the building, or its demolished portion, to be a threat to the
safety of persons and property. This fact alone should have warned the MMDA against
summarily demolishing the structure. Neither does the MMDA have the power to
declare a thing a nuisance. Only courts of law have the power to determine whether a
thing is a nuisance.

292
GUILLERMO M. TELMO VS. LUCIANO M. BUSTAMANTE

G.R. No. 182567


July 13, 2009

Facts:

Bustamante filed a complaint against Guillermo Telmo and Elizalde Telmo before the
Office of the Deputy Ombudsman for Luzon alleging that Bustamante is a co-owner of a
real property of Lot 952-A while the Telmos are owners of Lot 952-B and 952-C,
respectively, located at the back of Bustamante‘s lot. When Butamante‘s lot was
transgressed by the construction of the Noveleta-Naic-Tagaytay Road, he offered the
remaining lot for sale to the Telmos but the Telmos refused.

The complaint also alleged that Bustamante caused the resurvey of Lot 952-A in the
presence of the Telmos and it showed that the Telmos encroached upon his lot.
Guillermo then uttered that as long as he is the municipal engineer of Naic, Cavite, they
would not be able to build anything on their lot and he would not give them any building
permit. Bustamante put up concrete poles on his lot but the Telmos allegedly destroyed
them. Bustamante‘s relatives went to Brgy. Chairman Consumo to report the destruction
of the concrete poles but was told that he would not record the same.

Bustamante complained that he and his co-owners did not receive any just
compensation from the government when it took a portion of their property and that they
could not enjoy the remaining lot due to the abusive, illegal, and unjust acts of the
Telmos and Consumo.

Guillermo Telmo denied having uttered the words attributed to him and claimed that he
only performed his official duties. Guillermo also allege that Bustamante insisted on
enclosing with barbed wire and concrete posts the lot belonging to the national
government. Cosumo also denied collusion with Guillermo in not recording the incident.
Elizalde Telmo denied having encroached Bustamante‘s property and he was a mere
onlooker to the altercation between Guillermo and Bustamante.

The Office of the Deputy Ombudsman for Luzon found Guillermo and Consumo
administratively liable but dismissed the charge against Elizalde for lack of jurisdiction
over his person, he being a private individual.

Issue:

Whether or not the concrete posts erected by Bustamante were a public nuisance,
particularly nuisance per se

293
Ruling:

No. A nuisance per se is that which affects the immediate safety of persons and
property and may be summarily abated under the undefined law of necessity.

Evidently, the concrete posts summarily removed by Guillermo did not at all pose a
hazard to the safety of persons and properties, which would have necessitated
immediate and summary abatement. What they did, was to pose an inconvenience to
the public by blocking the free passage of people to and from the national road.

294
CITY OF MANILA VS. LAGUIO

G.R. No. 118127


April 12, 2005

Facts:

City Mayor Lim signed into law Ordinance No. 7783 entitled ―An Ordinance Prohibiting
the Establishment or Operation of Businesses Providing Certain Forms of Amusement,
Entertainment, Services and Facilities in the Ermita-Malate Area, Prescribing Penalties
for Violation thereof, and for other Purposes‖.

The Malate Tourist Development Corporation (MTDC) is a corporation engaged in the


business of operating hotels, motels, hostels and lodging houses. The MTDC filed a
Petition for Declaratory Relief with Prayer for a Writ of Preliminary Injunction and/or
Temporary Restraining Order praying that the Ordinance, insofar as it includes motels
and inns as among the prohibited establishments, be declared unconstitutional. The
MTDC argued that the Ordinance erroneously included in its enumeration of prohibited
establishments, motels and inns considering that these were not establishments for
amusement or entertainment and they do not disturb the community, annoy the
inhabitants, or adversely affect the social and moral welfare of the community.

Judge Laguio rendered a decision, enjoining the petitioners from implementing the
Ordinance and declared the Ordinance null and void.

Issue:

Whether or not the Ordinance can punish motels as a nuisance per se.

Ruling:

No. The problem, the Ordinance needs to be pointed out, is not the establishment,
which by its nature cannot be said to be injurious to the health or comfort of the
community and which in itself is amoral, but the deplorable human activity that may
occur within its premises. While a motel may be used as a venue for immoral sexual
activity, it cannot for that reason alone be punished. It cannot be classified as a house
of ill-repute or as a nuisance per se on a mere likelihood or a naked assumption. If that
were so and if that were allowed, then the Ermita-Malate area would not only be purged
of its supposed social ills, it would be extinguished of its soul as well as every human
activity, reprehensible or not, in its every nook and cranny would be laid bare to the
estimation of the authorities.

The Ordinance seeks to legislate morality but fails to address the core issues of
morality. Try as the Ordinance may to shape morality, it should not foster the illusion
that it can make a moral man out of it because immorality is not a thing, a building or
establishment; it is in the hearts of men. The City Council instead should regulate

295
human conduct that occurs inside the establishments, but not to the detriment of liberty
and privacy which are covenants, premiums and blessings of democracy.

Also, a nuisance per se, or one which affects the immediate safety of persons and
property and may be summarily abated under the undefined law of necessity. It cannot
be said that motels are injurious to the rights of property, health or comfort of the
community. It is a legitimate business. If it be a nuisance per accidens, it may be so
proven in a hearing conducted for that purpose. A motel is not per se a nuisance
warranting its summary abatement without judicial intervention.

296
JAIME S. PEREZ VS. SPOUSES FORTUNITO L. MADRONA & YOLANDA B. PANTE

G.R. No. 184478


March 21, 2012

Facts:

Spouses Fortunito Madrona and Yolanda Pante are registered owners of a registered
property where they built their house and enclosed it with a concrete fence and steel
gate. The spouses then received a letter from Perez alleging that their fence extended
to the sidewalk and they have 7 days to remove the structure. Madrona, in his answer,
stated that the letter contained an accusation libelous in nature and a false accusation
since their fence did not extend to the sidewalk.

The spouses received another letter requesting them to provide the office of Perez a
copy of the relocation survey on the property but the spouses did not oblige. More than
a year later, the spouses received the another letter with the same contents as the first
letter giving the spouses 10 days to remove the structure allegedly protruding to the
sidewalk. This prompted the spouses to file a complaint for injunction.

The RTC rendered a decision in favor of the spouses, permanently enjoining Perez from
performing any act which would tend to destroy or demolish the perimeter fence and
steel gate of the spouses‘ property. The RTC also ruled that there is no showing that
spouses‘ fence is a nuisance per se and presents an immediate danger to the
community‘s welfare, nor is there basis for the claim of Perez that the fence has
encroached on the sidewalk as to justify its summary demolition.

Issue:

Whether or not the spouses‘ fence is a nuisance per se.

Ruling:

No. Spouses‘ fence is not a nuisance per se. By its nature, it is not injurious to the
health or comfort of the community. It was built primarily to secure the property of the
spouses and prevent intruders from entering it. And as correctly pointed out by the
spouses, the sidewalk still exists. If Perez believes that the spouses‘ fence indeed
encroaches on the sidewalk, it may be so proven in a hearing conducted for that
purpose. Not being a nuisance per se, but at most a nuisance per accidens, its
summary abatement without judicial intervention is unwarranted. If Perez indeed found
the spouses‘ fence to have encroached on the sidewalk, his remedy is not to demolish
the same summarily after respondents failed to heed his request to remove it. Instead,
he should go to court and prove respondents‘ supposed violations in the construction of
the concrete fence. Indeed, unless a thing is a nuisance per se, it may not be abated
summarily without judicial intervention.

297
LUCENA GRAND CENTRAL TERMINAL, INC. VS. JAC LINER, INC.

G.R. No. 148339


February 23, 2005

Facts:

JAC Liner, Inc., a common carrier operating buses which ply various routes to and from
Lucena City, assailed, via a petition for prohibition and injunction before the RTC, City
Ordinances Nos. 1631 (An Ordinance Granting the Lucena Grand Central Terinal, Inc.,
a Franchise to Condtruct, Finance, Establish, Operate and Maintain a Common Bus-
Jeepney Terminal Facility in the City of Lucena) and 1778 (An Ordinance Regulating the
Entrance to the City of Lucena of all Buses, Mini-Buses and Out-of-Town Passenger
Jeepneys and for this Purpose, Amending Ordinance No. 1420, Series of 1993, and
Ordinance No. 1557, Series of 1995) as unconstitutional.

These ordinances, by granting an exclusive franchise for 25 years, renewable for


another 25 years, to one entity for the construction and operation of one common bus
and jeepneys terminal facility in Lucena City, to be located outside the city proper, were
professedly aimed towards alleviating the traffic congestion alleged to have been
caused by the existence of various bus and jeepney terminals within the city.

The Lucena Grand Central Terminal, Inc., claiming legal interest as the grantee of the
exclusive franchise for the operation of the common terminal, was allowed to intervene
in the petition before the trial court.

The RTC rendered a decision declaring Sec. 4(c) of Ordinance No. 1631 to the effect
that the City Government shall not grant any third party any privilege and/or concession
to operate a bus, mini-bus and/or jeepney terminal, as illegal and ultra vires, and
Ordinance No. 1778 as null and void. The appellate court affirmed the challenged
orders of the trial court.

Issue:

Whether or not the terminals are nuisances.

Ruling:

No. The terminals are not public nuisances as petitioner argues. For their operation is a
legitimate business which, by itself, cannot be said to be injurious to the rights of
property, health, or comfort of the community. But even assuming that terminals are
nuisances due to their alleged indirect effects upon the flow of traffic, at most they are
nuisance per accidens, not per se. Unless a thing is nuisance per se, however, it may
not be abated via an ordinance, without judicial proceedings, as was done in the case at
bar.

298
PEDRO J. VELASCO VS. MANILA ELECTRIC CO.

G.R. No. L-18390


August 6, 1971

Facts:

Velasco bought in 1948 from the PHHC 3 adjoining lots in Diliman, Quezon City, within
an area zoned out as a first district by the Quezon City Council. Subsequently, Velasco
sold 2 lots to the Meralco, but retained the third lot, which was farthest from the street-
corner, whereon he built his house.

In September 1953, Meralco started the construction of the sub-station and finished it in
November, without prior building permit or authority. It was constructed 10 to 20 meters
from Velasco‘s house. Meralco built a stone and cement wall at the sides along the
streets but along the side adjoining Velasco‘s property it put up a sawale wall but later
changed it to an interlink wire fence.

It is undisputed that a sound unceasingly emanates from the substation. Velasco


contents that the sound constitutes an actionable nuisance because since 1954 it
disturbed the concentration and sleep of Velasco and impaired his health and lowered
the value of his property. He sought a judicial decree for the abatement of the nuisance
and asked that he be declared entitled to recover compensatory, moral and other
damages.

The lower court dismissed Velasco‘s claim, finding that the sound of substation was
unavoidable and did not constitute nuisance.

Issue:

Whether or not the sound from the substation constitute nuisance.

Ruling:

Yes. The conclusion must be that, contrary to the finding of the trial court, the noise
continuously emitted, day and night, constitutes an actionable nuisance for which the
Velasco is entitled to relief, by requiring Meralco to adopt the necessary measures to
deaden or reduce the sound at Velasco's house, by replacing the interlink wire fence
with a partition made of sound absorbent material, since the relocation of the substation
is manifestly impracticable and would be prejudicial to the customers of the Electric
Company who are being serviced from the substation.

Technical charts submitted in evidence show the following intensity levels in decibels of
some familiar sounds: average residence: 40; average office: 55; average automobile,
15 feet: 70; noisiest spot at Niagara Falls: 92 (Exhibit "11- B"); average dwelling: 35;
quiet office: 40; average office: 50; conversation: 60; pneumatic rock drill: 130 (Exhibit

299
"12"); quiet home — average living room: 40; home ventilation fan, outside sound of
good home air conditioner or automobile at 50 feet: 70 (Exhibit "15-A"). Thus the
impartial and objective evidence points to the sound emitted by Meralco's substation
transformers being of much higher level than the ambient sound of the locality.

300
HIDALGO ENTERPRISES, INC. VS. GUILLERMO BALANDAN, ET. AL.

G.R. No. L-3422


June 13, 1952

Facts:

The petitioner Hidalgo Enterprises, Inc. "was the owner of an ice-plant factory in the City
of San Pablo, Laguna, in whose premises were installed two tanks full of water, nine
feet deep, for cooling purposes of its engine. While the factory compound was
surrounded with fence, the tanks themselves were not provided with any kind of fence
or top covers. The edges of the tanks were barely a foot high from the surface of the
ground. Through the wide gate entrance, which is continually open, motor vehicles
hauling ice and persons buying said commodity passed, and anyone could easily enter
the said factory, as he pleased. There was no guard assigned on the gate. At about
noon of April 16, 1948, plaintiff's son, Mario Balandan, a boy barely 8 years old, while
playing with and in company of other boys of his age entered the factory premises
through the gate, to take a bath in one of said tanks; and while thus bathing, Mario sank
to the bottom of the tank, only to be fished out later, already a cadaver, having been
died of "asphyxia secondary to drowning." The Court of Appeals, and the Court of First
Instance of Laguna, took the view that the petitioner maintained an attractive nuisance
(the tanks), and neglected to adopt the necessary precautions to avoid accidents to
persons entering its premises. It applied the doctrine of attractive nuisance, of American
origin.

Issue:

Whether or not a swimming pool or water tank an instrumentality or appliance likely to


attract the little children in play and therefore be considered as an attractive nuisance.

Ruling:

No, Hidalgo Enterprises, Inc. is absolved from liability. The attractive nuisance doctrine
generally is not applicable to bodies of water, artificial as well as natural, in the absence
of some unusual condition or artificial feature other than the mere water and its location.
The Court held that Nature has created streams, lakes and pools which attract children.
Lurking in their waters is always the danger of drowning. Against this danger children
are early instructed so that they are sufficiently presumed to know the danger; and if the
owner of private property creates an artificial pool on his own property, merely
duplicating the work of nature without adding any new danger is not liable because of
having created an "attractive nuisance. Therefore, as petitioner's tanks are not classified
as attractive nuisance, the question whether the petitioner had taken reasonable
precautions becomes immaterial.

301
JOSE "PEPITO" TIMONER VS. PEOPLE OF THE PHILIPPINES

G.R. No. L-62050


November 25, 1983

Facts:

At about 10:00 in the evening of December 13, 1971, petitioner, then Mayor of Daet,
Camarines Norte, accompanied by two uniformed policemen, Samuel Morena and
Ernesto Quibral, and six laborers, arrived in front of the stalls along Maharlika highway,
the main thoroughfare of the same town. Upon orders of petitioner, these laborers
proceeded to nail together rough lumber slabs to fence off the stalls which protruded
into the sidewalk of the Maharlika highway. Among the structures thus barricaded were
the barbershop of Pascual Dayaon, the complaining witness and the store belonging to
one Lourdes Pia-Rebustillos. These establishments had been recommended for closure
by the Municipal Health Officer, Dra. Alegre, for non-compliance with certain health and
sanitation requirements. Thereafter, petitioner filed a complaint in the Court of First
Instance of Camarines Norte against Lourdes Pia-Rebustillos and others for judicial
abatement of their stalls. The complaintalleged that these stalls constituted public
nuisances as well as nuisances per se. Dayaon was never able to reopen his
barbershop business. On appeal, the Court of Appeals affirmed in full the judgment of
the trial court.

Issue:

Whether or not the sealing off of complainant Dayaon's barbershop was done in
abatement of a public nuisance and, therefore, under lawful authority.

Ruling:

Yes, the Court declared that the structures subject of the complaint as well as those
occupied by the impleaded defendants are nuisances per se and therefore orders the
defendants to demolish the stall and vacate the premises immediately .The Court found
merit in this contention. Unquestionably, the barbershop in question did constitute a
public nuisance Under the facts of the case, as well as the law in point, there is no
semblance of any legality or right that exists in favor of the defendants to build a stall
and conduct their business in a sidewalk, especially in a highway where it does not only
constitute a menace to the health of the general public passing through the street and
also of the unsanitary condition that is bred therein as well as the unsightly and ugly
structures in the said place.

302
RESTITUTO CALMA VS. COURT OF APPEALS

G.R. No. 78447


August 17, 1989

Facts:

Sometime in August 1975, the spouses Restituto and Pilar Calma purchased a lot in
respondent Pleasantville subdivision in Bacolod City. In 1976, they built a house on said
lot and established residence therein. Fabian and Nenita Ong also purchased from
PLEASANTVILLE a lot fronting that of the Calma spouses sometime in the years 1979-
1980, and constructed their own buildings where they resided and conducted their
business. On 25 April 1981, petitioner Calma wrote the president of the Association of
Residents of City Heights, Inc. (ARCHI) complaining that the compound of the Ongs
was being utilized as a lumber yard and that a "loathsome noise and nervous
developing sound" emanating therefrom disturbed him and his family. The president of
the association, in his reply, stated that the association's board had referred the matter
to Fabian Ong who had already taken immediate action on petitioner's complaint but.
the measures taken by the association and Fabian Ong were found to be unsatisfactory,
petitioner on 17 June 1981 wrote and asked PLEASANTVILLE, as its duty and
obligation, to abate the nuisance emanating from the compound of the Ong family.
Failing to get an answer, the Calma spouses filed a complaint for damages against the
Ong spouses and PLEASANTVILLE on 28 July 1981 before the Court of First Instance
alleging that were it not for PLEASANTVILLE's act of selling the lot to the Ongs and its
failure to exercise its right to cause the demolition of the alleged illegal constructions,
the nuisance could not have existed and petitioner and his family would not have
sustained damage.

Issue:

Whether or not the property of the Ong spouses could be abated.

Ruling:

No, the petition was DENIED for lack of merit there being no cogent reasons to reverse
the decision of the Court of Appeals. But all is not lost for petitioner and his family. As
mentioned in the case, there is a pending civil instituted by petitioner, where the alleged
breach of warranty, coupled with PLEASANTVILLE's inaction, is the primary basis for
the complaint for abatement and damages. Here he can prove the existence of the
warranty and show how it was breached. It is also in this case where the determination
of whether or not the activities conducted in the property of the Ong spouses or the
structures thereat constitute a nuisance will have to be made. Also herein is the proper
forum where, following another theory, it could be determined whether the Contract to
Sell (assuming that the contract between PLEASANTVILLE and the Ongs is similar)
establishes an enforceable obligation in favor of third parties, i.e., other lot-buyers in the
subdivision.

303
CONCEPCION PARAYNO VS. JOSE JOVELLANOS

G.R. No. 148408


July 14, 2006

Facts:

Petitioner was the owner of a gasoline filling station in Calasiao, Pangasinan. In 1989,
some residents of Calasiao petitioned the Sangguniang Bayan (SB) of said municipality
for the closure or transfer of the station to another location. The Sangguniang Bayan
recommended to the Mayor the closure or transfer of location of petitioner's gasoline
station. It declared in Resolution No. 50: that the existing gasoline station is a blatant
violation and disregard of law. Petitioner moved for the reconsideration of the SB
resolution but it was denied. Hence, she filed a special civil action for prohibition and
mandamus with the Regional Trial Court of Dagupan City, it was denied by the trial
court the CA dismissed the petition

Issue:

Could the petitioner's business be considered a nuisance which respondent municipality


could summarily abate in the guise of exercising its police powers?

Ruling:

The Court held no. Respondent Municipality of Calasiao was directed to cease and
desist from enforcing Resolution No. 50 against petitioner insofar as it seeks to close
down or transfer her gasoline station to another location. The abatement of a nuisance
without judicial proceedings is possible only if it is a nuisance per se. A gas station is
not a nuisance per se or one affecting the immediate safety of persons and property
cannot be closed down or transferred summarily to another location. Hence, the Board
is inclined to believe that the project being hazardous to life and property is more
perceived than factual on the basis that even the Fire Station Commander, after
studying the plans and specifications of the subject proposed construction,
recommended on 20 January 1989, "to build such buildings.

304
VOLUNTARY
EASEMENT

305
PILAR DEVELOPMENT CORPORATION VS. RAMON DUMADAG

GR No. 194336
March 11, 2013

Facts:

The petitioner owned Pillar Village Subdivision at Las Piñas where the respondents
allegedly built their shanties without thepetitioner‘s knowledge or consent. Thus, a
Complaint for accion publiciana was filed against the respondents. The respondents
denied the material allegations of the Complaint asserting that it‘s the local government
and not the petitioner, which has jurisdiction and authority over them. RTC dismissed
the complaint saying that the land in question is situated on the sloping area leading
down a creek and within the three-meter legal easement and thus, it‘s considered as
public property and part of public dominion under Article 502 of the New Civil Code.
With this, only the local government of Las Pinas City could insititute an action for
recovery of possession or ownership. CA dismissed the case but noted that the proper
party to seek recovery of the property is not the City of Las Pinas but the Republic of the
Philippines, through the OSG pursuant to Section 101 of the Commonwealth Act (C.A.)
No. 141 otherwise known as the Public Land Act.

Issue:
Whether the land in question is part of public property

Ruling:
Petitioner used Article 630 of the Civil Code as it provides the general rule that the
owner of the estate retains the ownership of theportion of the easement established,
Article 635 says that ―all matters concerning easements established for public or
communaluse shall be governed by the special laws and regulations relating thereto.‖
The applicable special laws are DENR A.O. No. 99021 dated June 11, 1999 which
prescribed the guidelines for the implementation of P.D. Nos. 705 and 1067 which was
issued for biodiversity preservation, P.D. 1216 and P.D. 1067 or The Water Code of the
Philippines all of which states that such 3 meter allowance is reserved for public use.
Therefore, it cannot be denied that the subject land is public property. In relation to this,
the Court held that respondents have no better right to the property as the petitioners
because it is public land. Both the OSG and the local government of Las Pinas City,
may file an action depending on the purpose sought to be achieved. The former shall be
responsible in case of action for reversion under C.A. 141, while the latter may also
bring an action to enforce the relevant provisions of Republic Act No. 7279 (Urban
Development and Housing Act of 1992). Under RA No. 7279, all LGUs are mandated to
evict and demolish persons or entities occupying dangerous areas including riverbanks.
It also obliges the LGUS to strictly observe resettlement procedures and prohibition
against new illegal structures in Sections 29 and 30 respectively. Else, there will be
administrative and criminal liability.‖ The Court suggests that petitioner should file an
action for mandamus to compel the local government of Las Piñas City to enforce with
reasonable dispatch the eviction of respondents under R.A. 7279.

306
BICOL AGRO-INDUSTRIAL PRODUCERS COOPERATIVE, INC. VS. OBIAS, ET AL.

G.R. No. 172077


Ocotober 9, 2009

Facts:

The Bicol Sugar Development Corporation (BISUDECO) was established at


Himaao,Pili, Camarines Sur. In the same year, BISUDECO constructed a road ("the
disputed road") –measuring approximately 7 meters wide and 2.9 kilometers long. The
disputed road was used by BISUDECO in hauling and transporting sugarcane to and
from its mill site (Pensumil) and has thus become indispensable to its sugar milling
operations. Respondents unjustifiably barricaded the disputed road by placing
bamboos, woods, placards and stones across it, preventing petitioner‘s and the other
sugar planter‘s vehicles from passing through the disputed road, thereby causing
serious damage and prejudice to petitioner. Petitioner alleged that BISUDECO
constructed the disputed road pursuant to an agreement with the owners of the
rice fields the road traversed. The agreement provides that BISUDECO shall employ
the children and relatives of the landowners in exchange for the
construction of the road on their properties. Petitioner contends that through
prolonged and continuous use of the disputed road, BISUDECO acquired a right of way
over the properties of the landowners, which right of way in turn was acquired by it
when it bought BISUDECO‘sassets. Petitioner prayed that respondents be permanently
ordered to restrain from barricading the disputed road and from obstructing its free
passage. Respondents denied having entered into an agreement with BISUDECO
regarding the construction and the use of the disputed road. They alleged that
BISUDECO, surreptitiously and without their knowledge and consent, constructed the
disputed road on their properties and has since then intermittently and
discontinuously used the disputed road for hauling sugarcane despite their
repeated protests. Respondents claimed they tolerated BISUDECO in the
construction and the use of the road since BISUDECO was a government-owned and
controlled corporation, and the entire country was then under Martial Law. Respondents
likewise denied that the road has become a public road, since no public funds were
used for its construction and maintenance. The RTC ruled that petitioner failed to
present any concrete evidence to prove that there was an agreement between
BISUDECO and respondents for the construction of the disputed road. Moreover, it held
that petitioner did not acquire the same by prescription. The CA affirmed the finding of
the RTC that there was no conclusive proof to sufficiently establish the existence of an
agreement between BISUDECO and respondents regarding the construction of the
disputed road. Moreover, the CA also declared that an easement of right of way is
discontinuous and as such cannot be acquired by prescription.

307
Issue:

Whether or not the principles of prescription, laches and estoppels is applicable in this
case

Ruling:

No. In order for petitioner to acquire the disputed road as an easement of right-of-way,
it was incumbent upon petitioner to show its right by title or by an agreement with the
owners of the lands that said road traversed. Easement or servitude is an encumbrance
imposed upon an immovable for the benefit of another immovable belonging to a
different owner. By its creation, easement is established either by law (in which
case it is a legal easement) or by will of the parties (a voluntary
easement). In terms of use, easement may either be continuous or
discontinuous. The easement of right of way – the privilege of persons or a
particular class of persons to pass over another‘s land, usually through one particular
path or linen– is characterized as a discontinuous easement because its use
is in intervals and depends on the act of man. Because of this character , an
easement of a right of way may only be acquired by virtue of a title. Article 622 of the
New Civil Code is the applicable law in the case at bar, viz: Art. 622. Continuous non-
apparent easements, and discontinuous ones, whether apparent ornot, may be
acquired only by virtue of a title. It is clear that the plaintiff failed to present any concrete
evidence to prove that there was such an agreement between BISUDECO and
defendants. The lower court correctly disbelieved the plaintiffs-appellants‘
contention that an agreement existed because there is simply no direct evidence to
support this allegation. BAPCI submitted purely circumstantial evidence that are not
sufficiently adequate as basis for the inference than an agreement existed. By
themselves, the circumstances the plaintiffs-appellants cited – i.e., the employment
of sixteen (16) relatives of the defendants-appellants; the defendants-
appellants‘ unjustified silence; the fact that the existence of the agreement is known to
everyone, etc. – are events susceptible of diverse interpretations and do not necessarily
lead to BAPCI‘s desired conclusion.

It is already well-established that a right of way is discontinuous and, as such, cannot be


acquired by prescription. In the case at bar, the presence of railroad tracks for the
passage of petitioner‘s trains denotes the existence of an apparent but discontinuous
easement of right and way.

The positive mandate of Article 622 of the Civil Code – the statutory provision requiring
titile as basis for the acquisition of an easement of a right of way – precludes te
application of the equitable principle of laches.

308
SPOUSES SABIO VS. THE INTERNATIONAL CORPORATE BANK, INC.

G.R. No. 132709


September 4, 2001

Facts:

The object of the controversy is a portion of a vast tract of land measuring


approximately 152,454 square meters, located at Tindig na Manga, Almanza, Las Pias
City. This estate was registered in the name of Las Pias Ventures, Incorporated. In the
early 1970s, the said property was the subject of several land registration, as well as
civil, cases. On May 25, 1973, the spouses Gerardo and Emma Ledonio sold to the
spouses Camilo and Ma. Marlene Sabio (the petitioners) 119,429 square meters. For
this purpose, a deed of assignment with assumption of mortgage was later executed by
the Ledonio spouses in favor of the Sabio couple on November 23, 1981. Similarly,
while the subject property was still the object of several pending cases, the International
Corporate Bank, Inc. (or Interbank) acquired from the Trans-Resource Management
and Development Corporation all of the latters rights to the subject property by virtue of
a deed of assignment. Sometime thereafter, or on March 6, 1985, the Sabios and
Interbank settled their opposing claims by entering into a Memorandum of Agreement
(or MOA) whereby the Sabios assigned, conveyed and transferred all their rights over
the parcel covered by TCT No. 65162 to Interbank. Plaintiffs alleged that defendants,
particularly Ayala Group, failed to comply with their commitments and obligations in the
MOA. Hence, plaintiffs have been prevented from utilizing for productive purposes the
land.

Issue:

Whether or not the defendants had the obligation to clear the subject 58,000 square
meter portion of all occupants and to fence the said premises, before conveyance of the
property can be considered as full compliance with the obligation imposed upon the
defendants under the MOA.

Ruling:
The judgment appealed from is affirmed with the modification. Petitioners contention
that respondents never acquired ownership over the subject property since the latter
was never in possession of the subject property nor was the property ever delivered is
totally without merit. Under the aforementioned Article 1498, the mere execution of the
deed of conveyance in a public document is equivalent to the delivery of the property.
Since the execution of the deed of conveyance is deemed equivalent to delivery, prior
physical delivery or possession is not legally required. It is well-established that
ownership and possession are two entirely different legal concepts. Just as possession
is not a definite proof of ownership, neither is non-possession inconsistent with
ownership. Thus, it is of no legal consequence that respondents were never in actual

309
possession or occupation of the subject property. They, nevertheless, perfected and
completed ownership and title to the subject property.

310
EDILBERTO ALCANTARA VS. CORNELIO B. RETA, JR.

G.R. No. 136996


December 14, 2001

Facts:

Edilberto Alcantara, Florencio Villarmia, Policarpio Obregon, Ricardo Roble, Escolastica


Ondong, Esteban Rallos, Henry Sesbino, Sergio Sesbino, Manuel Centeno, Renato
Cruz, Marcelo Ceneza, Buenaventura Ondong and Benjamin Halasan claimed that they
were tenants or lessees of the land located in Barangay Sasa, Davao City, covered by
Transfer Certificate of Title No. T-72594, owned by Reta which has been converted by
him into a commercial center. Reta is threatening to eject them from the land. They
assert that they have the right of first refusal to purchase the land in accordance with
Section 3(g) of PD no. 1517 since they are legitimate tenants or lessees thereof. They
also claimed that the amicable settlement executed between Reta and Ricardo Roble
was void ab initio for being violative of Presidential Decree No. 1517.

On the other hand, Reta claimed that the land is beyond the ambit of Presidential
Decree No. 1517 since it has not been proclaimed as an Urban Land Reform Zone; that
the applicable law is Batas Pambansa Blg. 25 for failure of the plaintiffs to pay the
rentals for the use of the land; and that the amicable settlement between him and
Ricardo Roble was translated to the latter and fully explained in his own dialect.
The trial court rendered a decision dismissing the complaint and ordering the plaintiffs to
pay Reta certain sums representing rentals that had remained unpaid. Plaintiffs
appealed the decision to the Court of Appeals which affirmed the trial court‘s decision.

Issue:

Whether petitioners have the right of first refusal under Presidential Decree No. 1517.

Ruling:

No. Petitioners do not have the right of first refusal. The area involved has not been
proclaimed an Urban Land Reform Zone. In fact, Alcantara et. al. filed a petition with the
National Housing Authority requesting that the land they were occupying be declared as
an ULRZ. On May 27, 1986, the request was referred to Mr. Jose L. Atienza, General
Manager, National Housing Authority, for appropriate action. The request was further
referred to acting mayor Zafiro Respicio, Davao City, as per 2nd Indorsement dated July
1, 1986. Clearly, the request to have the land proclaimed as an ULRZ would not be
necessary if the property was an ULRZ.

Presidential Decree No. 1517, otherwise known as "The Urban Land Reform Act,"
pertains to areas proclaimed as Urban Land Reform Zones.11 Consequently,
petitioners cannot claim any right under the said law since the land involved is not an

311
ULRZ. To be able to qualify and avail oneself of the rights and privileges granted by the
said decree, one must be: (1) a legitimate tenant of the land for ten (10) years or more;
(2) must have built his home on the land by contract; and, (3) has resided continuously
for the last ten (10) years. Obviously, those who do not fall within the said category
cannot be considered "legitimate tenants" and, therefore, not entitled to the right of first
refusal to purchase the property should the owner of the land decide to sell the same at
a reasonable price within a reasonable time.

Respondent Reta allowed petitioner Ricardo Roble to use sixty-two (62) coconut trees
for P186 from where he gathered tuba. This arrangement would show that it is a
usufruct and not a lease. Usufruct gives a right to enjoy the property of another with the
obligation of preserving its form and substance, unless the title constituting it or the law
otherwise provides.

Petitioner Roble was allowed to construct his house on the land because it would
facilitate his gathering of tuba. This would be in the nature of a personal easement
under Article 614 of the Civil Code. Whether the amicable settlement is valid or not, the
conclusion would still be the same since the agreement was one of usufruct and not of
lease. Thus, petitioner Roble is not a legitimate tenant as defined by Presidential
Decree No. 1517.

312
NATIONAL IRRIGATION ADMINISTRATION VS. COURT OF APPEALS

G.R. No. 114348


September 20, 2000

Facts:

A free patent over three (3) hectares of land, situated in the province of Cagayan was
issued in the name of Vicente Manglapus, and registered under OCT No. P-24814. The
land was granted subject to the following proviso expressly stated in the title: "... it shall
not be subject to any encumbrance whatsoever in favor of any corporation, association
or partnership except with the consent of the grantee and the approval of the Secretary
of Agriculture and Natural Resources and solely for educational, religious or charitable
purposes or for a right of way; and subject finally to all conditions and public easements
and servitudes recognized and prescribed by law especially those mentioned in
sections 109, 110, 111, 112, 113 and 114 of Commonwealth Act No. 141 as
amended..."

Subsequently, respondent Manglapus acquired the lot from Vicente Manglapus by


absolute sale. Sometime in 1982, NIA was to construct canals in Amulung, Cagayan
and Alcala, Cagayan. NIA then entered a portion of Manglapus' land and made diggings
and fillings thereon. Manglapus filed a complaint for damages against NIA.

Issue:

Whether or not the NIA should pay Manglapus just compensation for the taking of a
portion of his property for use as easement of a right of way.

Ruling:

No. The transfer certificate of title contains such a reservation. It states that title to the
land shall be: ". . . subject to the provisions of said Land Registration Act and the Public
Land Act, as well as those of Mining Laws, if the land is mineral, and subject, further to
such conditions contained in the original title as may be subsisting."

Under the Original Certificate of Title, there was a reservation and condition that the
land is subject to "to all conditions and public easements and servitudes recognized and
prescribed by law especially those mentioned in Sections 109, 110, 111, 112, 113 and
114, Commonwealth Act No. 141, as amended." This reservation, unlike the other
provisos imposed on the grant, was not limited by any time period and thus is a
subsisting condition.

Section 112, Commonwealth Act No. 141, provides that lands granted by patent, "shall
further be subject to a right of way sot exceeding twenty meters in width for public

313
highways, railroads, irrigation ditches, aqueducts, telegraphs and telephone lines, and
similar works..."
The canal NIA constructed was only eleven meters in width. This is well within the limit
provided by law. Manglapus has therefore no cause to complain.

Article 619 of the Civil Code provides that, "Easements are established either by law or
by the will of the owners. The former are called legal and the latter voluntary
easements." In the present case, we find and declare that a legal easement of a right-
of-way exists in favor of the government.

The land was originally public land, and awarded to respondent Manglapus by free
patent. The ruling would be otherwise if the land were originally private property, in
which case, just compensation must be paid for the taking of a part thereof for public
use as an easement of a right of way.

314
The land was originally public land, and awarded to respondent Manglapus by free
patent. The ruling would be otherwise if the land were originally private property, in
which case, just compensation must be paid for the taking of a part thereof for public
use as an easement of a right of way.

315
CAMARINES NORTE ELECTRIC COOPERATIVE, INC. VS. COURT OF APPEALS

G.R. No. 109338


November 20, 2000

Facts:

Conrad Leviste filed a complaint for collection of a sum of money and foreclosure of
mortgage against Philippine Smelter Corporation. For failure to file an answer to the
complaint, the trial court declared PSC in default and allowed plaintiff Leviste to present
evidence ex-parte. The trial court rendered a decision in favor of plaintiff.

When the decision became final and executory, the trial court issued a writ of execution
and respondent sheriff Eduardo Moreno levied upon two parcels of land and were sold
at public auction in favor of Vines Realty Corporation. The Clerk of Court, as ex-officio
Provincial Sheriff, issued a Certificate of Sale. Vines Realty moved for the issuance of a
writ of possession over said property.

A copy of the writ of possession was served on petitioner as owner of the power lines
standing on certain portions of the subject property. Later, Vines Realty filed an
amended motion for an order of demolition and removal of improvements on the subject
land. Among the improvements for removal were the power lines and electric posts
belonging to petitioner. Petitioner opposed the motion on the ground that petitioner was
not a party to the case and therefore not bound by the judgment of the trial court and
that it had subsisting right-of-way agreements over said property.

Issue:

Whether or not petitioner is entitled to retain possession of the power lines located in
the land sold at public auction as a result of extra-judicial foreclosure of mortgage.

Ruling:

Yes. The acquisition of an easement of a right-of-way falls within the purview of the
power of eminent domain. Such conclusion finds support in easements of right-of-way
where the Supreme Court sustained the award of just compensation for private property
condemned for public use. However, a simple right-of-way easement transmits no
rights, except the easement. Vines Realty retains full ownership and it is not totally
deprived of the use of the land. It can continue doing what it wants to do with the land,
except those that would result in contact with the wires.

The acquisition of this easement, nevertheless, is not gratis. Considering the nature and
effect of the installation power lines, the limitations on the use of the land for an
indefinite period deprives private respondents of its ordinary use. For these reasons,
Vines Realty is entitled to payment of just compensation.

316
The most basic tenet of due process is the right to be heard. A court denies a party due
process if it renders its orders without giving such party an opportunity to present its
evidence. The court held that petitioner was denied due process. Petitioner could have
negated private respondents claims by showing the absence of legal or factual basis
therefor if only the trial court in the exercise of justice and equity reset the hearing
instead of proceeding with the trial and issuing an order of demolition on the same day.
It is incumbent upon the trial court to receive evidence on petitioner‘s right over the
property to be demolished.

A courts writ of demolition cannot prevail over the easement of a right-of-way which falls
within the power of eminent domain. Private respondents are ordered to restore or
restitute petitioner‘s electric posts and power lines or otherwise indemnify petitioner for
the cost of the restoration thereof. Private respondents are permanently prohibited from
disturbing or interfering with the operation and maintenance of the business of
petitioner.

317
BRYAN U. VILLANUEVA VS. HON. TIRSO D.C. VELASCO

G.R. No. 130845


November 27, 2000

Facts:

Petitioner Bryan Villanueva bought a parcel of land in QC from Pacific Banking


Corporation which it acquired from the spouses Maximo and Justina Gabriel. When he
bought it, there was a small house on its southeastern portion. It occupies one meter of
two meter wide easement of right of way the Gabriel spouses granted to Espinolas,
predecessors-in-interest of the private respondents, in a contract of easement of right of
way. Amongst others, the contract provides that the easement‘s purpose is to have an
outlet to Tandang Sora which is the nearest public road and the least burdensome
(Espinolas‘s property being the dominant estate and Gabriel spouses‘ as the servient
estate.) It was also provided in the contract that the easement ―shall be binding to the
successors, assigns without prejudice in cases of sale of subject property that will
warrant the circumstances.‖

The private respondents were able to acquire a writ of demolition on the house
obstructing the easement against the Spouses Gabriel. The petitioner filed a third party
claim with prayer to quash the writ saying that he was not made a party to the civil case
and that the writ of demolition should not prosper since the easement which is meant to
protect was not annotated in the petitioner‘s title.

CA ruled in favor of the private respondents saying that the easement exists even
though it was not annotated in the Torrens title because servitudes are inseparable from
the estate to which they actively or passively belong; and that Villanueva is bound by
the contact of easement, not only as a voluntary easement but as a legal easement.

Issue:

Whether or not the easement on the property binds petitioner.

Ruling:

YES. A legal easement is mandated by law, and continues to exist unless its removal is
provided for in a title of conveyance or the sign of the easement is removed before the
execution of the conveyance conformably with Art 647 in accordance with Article 617 of
the Civil Code.

Essential requisites for an easement to be compulsory are: (1) the dominant estate is
surrounded by other immovables and has no adequate outlet to a public highway; (2)
proper indemnity has been paid; (3) the isolation was not due to acts of the proprietor of
the dominant estate; (4) the right of way claimed is at point least prejudicial to the

318
servient estate; and (5) to the extent consistent with the foregoing rule, where the
distance from the dominant estate to a public highway may be the shortest.

The existence of the easement has been established by the lower courts and the same
has become conclusive to the SC. The small house occupying one meter of the two-
meter wide easement obstructs the entry of private respondent‘s cement mixer and
motor vehicle. Accordingly, the petitioner has to demolish the house to make way for the
easement.

319
SPOUSES STA. MARIA VS. COURT OF APPEALS

G.R. No. 127549


January 28, 1998

Facts:

Plaintiff spouses Arsenio and Roslynn Fajardo are the registered owners of a piece of
land, Lot No. 124, in Bulacan. Plaintiff's aforesaid Lot 124 is surrounded by Lot 1, a
fishpond, on the northeast portion thereof; by Lot 126, owned by Florentino Cruz, on the
southeast portion; by Lot 6-a and a portion of Lot 6-b owned respectively by Spouses
Cesar and Raquel Sta. Maria and Florcerfida Sta. Maria, on the southwest; and by Lot
122, owned by the Jacinto family, on the northwest.

On February 17, 1992, Fajardo filed a complaint against defendants Sta. Maria for the
establishment of an easement of right of way. Plaintiffs alleged that their lot, Lot 124, is
surrounded by properties belonging to other persons, including those of the defendants;
that since plaintiffs have no adequate outlet to the provincial road, an easement of a
right of way passing through either of the alternative defendants' properties which are
directly abutting the provincial road would be plaintiffs' only convenient, direct and
shortest access to and from the provincial road; that plaintiffs' predecessors-in-interest
have been passing through the properties of defendants in going to and from their lot;
that defendants' mother even promised plaintiffs' predecessors-in-interest to grant the
latter an easement of right of way as she acknowledged the absence of an access from
their property to the road; and that alternative defendants, despite plaintiffs' request for
a right of way and referral of the dispute to the barangay officials, refused to grant them
an easement. Thus, plaintiffs prayed that an easement of right of way on the lots of
defendants be established in their favor. They also prayed for damages, attorney's fees
and costs of suit.

RTC and CA both found for Fajardo and granted the easement of right of way. On
appeal, the Sta. Maria's allege that Fajardo failed to prove that it was not their own
actions which caused their lot to be enclosed or cut-off from the road.

Issue:

Whether or not the plaintiffs failed to prove the third requisite or that the isolation was
not caused by the plaintiffs themselves.

Ruling:

The plaintiffs sufficiently proved that they did not by themselves cause the isolation. As
to the third requisite, we explicitly pointed out; thus: "Neither have the private
respondents been able to show that the isolation of their property was not due to their
personal or their predecessors-in-interest's own acts." In the instant case, the Court of
Appeals have found the existence of the requisites. The petitioners, however, insist that

320
private respondents' predecessors-in-interest have, through their own acts of
constructing concrete fences at the back and on the right side of the property, isolated
their property from the public highway. The contention does not impress because even
without the fences private respondents' property remains landlocked by neighboring
estates belonging to different owners.

Again, for an estate to be entitled to a compulsory servitude of right of way under the
Civil Code, to wit: 1. the dominant estate is surrounded by other immovables and has no
adequate outlet to a public highway (Art. 649, par. 1); 2. there is payment of proper
indemnity (Art. 649, par. 1); 3. the isolation is not due to the acts of the proprietor of the
dominant estate (Art. 649, last par.); and 4. the right of way claimed is at the point least
prejudicial to the servient estate; and insofar as consistent with this rule, where the
distance from the dominant estate to a public highway may be the shortest (Art. 650).

321
REMMAN ENTERPRISES, INC. VS. COURT OF APPEALS

G.R. No. 107671


February 26, 1997

Facts:

Spouses Paulino and Purification Ochoa filed a complaint before the Regional Trial
Court of Lipa City against Remman Enterprises, Inc., for abatement of nuisance and
damages. After trial on the merits, said court rendered judgment in favor of spouses
Ochoa and against petitioner, ordering the latter to "stop and desist from draining their
waste matter, solid and liquid, to the estate of the plaintiffs x x x." The Court of Appeals
affirmed the trial court's decision, and the petition to this Court was denied.
Subsequently, a writ of execution of the decision was issued by the trial court. However,
on March 26, 1990, the spouses Ochoa filed another complaint before the same trial
court against Remman Enterprises, this time for indirect contempt. They alleged that a
portion of their estate was still being flooded with wastes coming from petitioner's hog
farm, in defiance of the final and executory order of the court directing it to stop and
desist from draining its waste matter into the Ochoa estate.

A hearing was held on may 18, 1990, wherein petitioner denied the allegations of the
complaint. In view of the conflicting claims of the parties, the trial court ordered an
ocular inspection on the properties of the parties. The branch clerk of court was
authorized by the court to conduct the ocular inspection and was directed to submit a
report immediately upon termination thereof. The ocular inspection was conducted on
the same day in the presence of both parties and their respective counsel.

Thereafter, said clerk of court reported his findings to the trial court, on the basis of
which the court issued its order dated June 15, 1990.

The Court of Appeals affirmed the trial court's orders, finding them to be "in accordance
with law and evidence." Petitioner's motion for reconsideration was denied. Hence, the
present recourse.

Issue:

Whether or not the petitioner may be held liable for indirect contempt after a single
hearing and on the basis of an ocular inspection report which was not furnished the
parties nor set for hearing.

Ruling:

Yes. The Supreme said that, there is no question that disobedience or resistance to a
lawful writ, process, order or judgment of a court or injunction granted by a court or
judge constitute indirect contempt punishable under Rule 71 of the Rules of Court. What

322
is put in issue here is the validity of the proceedings that found petitioner liable for such
misconduct.

The real character of the proceedings in contempt cases is to be determined by the


relief sought or by the dominant purpose. The proceedings are to be regarded as
criminal when the purpose is primarily punishment, and civil when the purpose is
primarily compensatory or remedial.

In general, criminal contempt proceedings should be conducted in accordance with the


principles and rules applicable to criminal cases, in so far as such procedure is
consistent with the summary nature of contempt proceedings. Strict rules that govern
criminal prosecutions apply to a prosecution for criminal contempt; the accused is to be
afforded many of the protections provided in regular criminal cases; and proceedings
under statutes governing them are to be strictly construed. However, criminal
proceedings are not required to take any particular form so long as the substantial rights
of the accused are preserved.

Civil contempt proceedings, on the other hand, are generally held to be remedial and
civil in nature; that is, for the enforcement of some duty, and essentially a remedy
resorted to, to preserve and enforce the rights of a private party to an action and to
compel obedience to a judgment or decree intended to benefit such a party litigant. The
rules of procedure governing criminal contempt proceedings, or criminal prosecutions,
ordinarily are inapplicable to civil contempt proceedings.

What the law prohibits is not the absence of previous notice but the absolute absence
thereof and the lack of opportunity to be heard. (Tajonera vs. Lamoroza, 110 SCRA 438
[1981])"

In the instant case, a written charge of indirect contempt was duly filed by the spouses
Ochoa before the Regional Trial Court of Lipa City. This is not contested by petitioner.
Acting on the complaint, the trial court issued an order requiring the defendant to "show
cause/explain why a judgment of contempt should not be rendered against it." A hearing
for the purpose was originally scheduled on May 11, 1990 which, upon motion of herein
petitioner, was reset to May 18, 1990. On the latter date, as petitioner admits in its
petition, it "vehemently denied the accusations in the motion for contempt". We can
draw no other conclusion than that a hearing was conducted and petitioner was heard in
its defenses in court.

Further, after the trial court promulgated its final order on June 15, 1990, and the
spouses Ochoa filed an omnibus motion for its reconsideration, petitioner did not raise
the question of not having been furnished a copy of the commissioner's report. No
mention thereof was made in its opposition to the omnibus motion. Neither did it do so
in its rejoinder to movants reply. It is only an afterthought of petitioner to raise on appeal
the alleged, though unsubstantiated, procedural defect.

323
COSTABELLA CORPORATION VS. COURT OF APPEALS, ET. AL.

G.R. No. 80511


January 25, 1991

Facts:

Petitioners owned a lot wherein they started constructing their beach hotel. Before such
construction, the private respondent, in going to and from their respective properties
and the provincial road, passed through a passageway which traversed the petitioner‘s
property. As a result of the construction, this passageway, including the alternative
route, was obstructed. Private respondent filed for injunction plus damages. In the same
complaint the private respondents also alleged that the petitioner had constructed a dike
on the beach fronting the latter‘s property without the necessary permit, obstructing the
passage of the residents and local fishermen, and trapping debris of flotsam on the
beach. The private respondent also claim that the have acquired the right of way
through prescription. They prayed for the re-opening of the ―ancient road right of way‖
(what they called the supposed easement in this case) and the destruction of the dike.
Petitioner answered by saying that their predecessor in interest‘s act of allowing them to
pass was gratuitous and in fact, they were just tolerating the use of the private
respondents. CA ruled in favor of the private respondents.

Issues:

1. Whether or not easement of right and way can be acquired through prescription?
2. Whether or not the private respondents had acquired an easement of right of way
in the form of a passageway, on the petitioner‘s property?

Ruling:

1. No. Easement of right of way is discontinuous thus it cannot be subject to


acquisitive prescription.
2. No. one may validly claim an easement of right of way when he has proven the:
(1) the dominant estate is surrounded by other immovables and has no adequate
outlet to a public highway; (2) proper indemnity has been paid; (3) the isolation
was not due to acts of the proprietor of the dominant estate; (4) the right of way
claimed is at point least prejudicial to the servient estate. The private respondent
failed to prove that there is no adequate outlet from their respective properties to
a public highway; in fact the lower court confirmed that there is another outlet for
the private respondents to the main road (yet they ruled in favor of the private
respondents). Apparently, the CA lost sight of the fact that the convenience of the
dominant estate was never a gauge for the grant of compulsory right of way.
There must be a real necessity and not mere convenience for the dominant
estate to acquire such easement. Also, the private respondents made no mention

324
of their intention to indemnify the petitioners. The SC also clarified that ―least
prejudicial‖ prevails over ―shortest distance‖ (so shortest distance isn‘t
necessarily the best choice.)

325
TOMAS ENCARNACION VS. COURT OF APPEALS

G.R. No. 77628


March 11, 1991

Facts:

Petitioner Tomas Encarnacion and private respondent Heirs of the late Aniceta Magsino
Viuda de Sagun are the owners of two adjacent estates situated in Buco, Talisay,
Batangas *** Petitioner owns the dominant estate which has an area of 2,590 square
meters and bounded on the North by Eusebio de Sagun and Mamerto Magsino, on the
south by Taal Lake, on the East by Felino Matienso and on the West by Pedro
Matienzo. Private respondents co-own the 405-square-meter servient estate which is
bounded on the North by the National Highway (Laurel Talisay Highway), on the South
by Tomas Encarnacion, on the East by Mamerto Magsino and on the West by Felipe de
Sagun. In other words, the servient estate stands between the dominant estate and the
national road.

Prior to 1960, when the servient estate was not yet enclosed with a concrete fence,
persons going to the national highway just crossed the servient estate at no particular
point. However, in 1960 when private respondents constructed a fence around the
servient estate, a roadpath measuring 25 meters long and about a meter wide was
constituted to provide access to the highway. One-half meter width of the path was
taken from the servient estate and the other one-half meter portion was taken from
another lot owned by Mamerto Magsino. No compensation was asked and none was
given for the portions constituting the pathway.

It was also about that time that petitioner started his plant nursery business on his land
where he also had his abode. He would use said pathway as passage to the highway
for his family and for his customers. Petitioner‘s plant nursery business through sheer
hard work flourished and with that, it became more and more difficult for petitioner to
haul the plants and garden soil to and from the nursery and the highway with the use of
pushcarts. In January, 1984, petitioner was able to buy an owner-type jeep which he
could use for transporting his plants. However, that jeep could not pass through the
roadpath and so he approached the servient estate owners (Aniceta Vda. de Sagun and
Elena Romero Vda. de Sagun) and requested that they sell to him one and one-half (1
1/2) meters of their property to be added to the existing pathway so as to allow passage
for his jeepney. To his utter consternation, his request was turned down by the two
widows and further attempts at negotiation proved futile. Petitioner then instituted an
action before the Regional Trial Court of Batangas, Branch 6 (Tanauan) to seek the
issuance of a writ of easement of a right of way over an additional width of at least two
(2) meters over the De Saguns‘ 405-square-meter parcel of land.

During the trial, the attention of the lower court was called to the existence of another
exit to the highway, only eighty (80) meters away from the dominant estate. On
December 2, 1985, the lower court rendered judgment dismissing petitioner‘s complaint.

326
It ruled: "It is clear, therefore, that plaintiff at present has two outlets to the highway:
one, through the defendants‘ land on a one meter wide passageway, which is bounded
on both sides by concrete walls and second, through the dried river bed eighty meters
away. The plaintiff has an adequate outlet to the highway through the dried river bed
where his jeep could pass.

"The reasons given for his claim that the one-meter passageway through defendants‘
land be widened to two and one-half meters to allow the passage of his jeep, destroying
in the process one of the concrete fences and decreasing defendants‘ already small
parcel to only about 332.5 square meters, just because it is nearer to the highway by 25
meters compared to the second access of 80 meters or a difference of only 65 meters
and that passage through defendants‘ land is more convenient for his (plaintiff‘s)
business and family use are not among the conditions specified by Article 649 of the
Civil Code to entitle the plaintiff to a right of way for the passage of his jeep through
defendant‘s land." On appeal, the Court of Appeals affirmed the decision of the trial
court on January 28, 1987 and rejected petitioner‘s claim for an additional easement.

Issue:

Whether or not the petitioner is entitled to a widening of an already existing


easement of right-of-way.

Ruling:

Yes, Petitioner Tomas Encarnacion is entitled to an additional easement of right


of way of twenty-five (25) meters long by one and one-half (11/2) meters wide over
the servient estate or a total area of 62.5 square meters after payment of the
proper indemnity.
Generally, the right of way may be demanded: (1) when there is absolutely no
access to a public highway, and (2) when, even if there is one, it is difficult or
dangerous to use or is grossly insufficient. In the present case, the river bed
route is traversed by a semi-concrete bridge and there is neither ingress nor
egress from the highway. For the jeep to reach the level of the highway, it must
literally jump four (4) to five (5) meters up. Moreover, during the rainy season, the
river bed is impassable due to the floods. Thus, it can only be used at certain
times of the year. With the inherent disadvantages of the river bed which make
passage difficult, if not impossible, it is if there were no outlets at all.

Where a private property has no access to a public road, it has the right of
easement over adjacent servient estates as a matter of law.

With the non-availability of the dried river bed as an alternative route to the
highway, we transfer our attention to the existing pathway which straddles the
adjoining properties of the De Sagun heirs and Mamerto Magsino.

327
Article 651 of the Civil Code provides that" (t)he width of the easement of right of way
shall be that which is sufficient for the needs of the dominant estate, and may
accordingly be changed from time to time." This is taken to mean that under the law, it is
the needs of the dominant property which ultimately determine the width of the passage.
And these needs may vary from time to time. When petitioner started out as a plant
nursery operator, he and his family could easily make do with a few pushcarts to tow the
plants to the national highway. But the business grew and with it the need for the use of
modern means of conveyance or transport. Manual hauling of plants and garden soil
and use of pushcarts have become extremely cumbersome and physically taxing. To
force petitioner to leave his jeepney in the highway, exposed to the elements and to the
risk of theft simply because it could not pass through the improvised pathway, is sheer
pigheadedness on the part of the servient estate and can only be counter-productive for
all the people concerned. Petitioner should not be denied a passageway wide enough to
accomodate his jeepney since that is a reasonable and necessary aspect of the plant
nursery business.

We are well aware that an additional one and one-half (11/2) meters in the width of the
pathway will reduce the servient estate to only about 342.5 square meters. But
petitioner has expressed willingness to exchange an equivalent portion of his land to
compensate private respondents for their loss. Perhaps, it would be well for
respondents to take the offer of petitioner seriously. 5 But unless and until that option is
considered, the law decrees that petitioner must indemnify the owners of the servient
estate including Mamerto Magsino from whose adjoining lot 1/2 meter was taken to
constitute the original path several years ago. Since the easement to be established in
favor of petitioner is of a continuous and permanent nature, the indemnity shall consist
of the value of the land occupied and the amount of the damage caused to the servient
estate pursuant to Article 649 of the Civil Code which states in part: ―Art. 649. The
owner, or any person who by virtue of a real right may cultivate or use any immovable,
which is surrounded by other immovables pertaining to other persons and without
adequate outlet to a public highway, is entitled to demand a right of way through the
neighboring estates, after payment of the proper indemnity.

"Should this easement be established in such a manner that its use may be continuous
for all the needs of the dominant estate, establishing a permanent passage, the
indemnity shell consist of the value of the land occupied and the amount of the damage
caused to the servient estate.

328
ELISEO FAJARDO, JR., and MARISSA FAJARDO VS. FREEDOM TO BUILD, INC.

G.R. No. 134692


August 1, 2000

Facts:

Freedom To Build, Incorporated, an owner-developer and seller of low-cost housing,


sold to petitioner-spouses, a house and lot designated Lot No. 33, Block 14, of the De la
Costa Homes in Barangka, Marikina, Metro Manila. The Contract to Sell executed
between the parties, contained a Restrictive Covenant providing certain prohibitions, to
wit: "Easements. For the good of the entire community, the homeowner must observe a
two-meter easement in front. No structure of any kind (store, garage, bodega, etc.) may
be built on the front easement. "Upward expansion. A second storey is not prohibited.
But the second storey expansion must be placed above the back portion of the house
and should not extend forward beyond the apex of the original building. "Front
expansion: 2nd Storey: No unit may be extended in the front beyond the line as
designed and implemented by the developer in the 60 sq. m. unit. In other words, the
2nd floor expansion, in front, is 6 meters back from the front property line and 4 meters
back from the front wall of the house, just as provided in the 60 sq. m. units."

The above restrictions were also contained in Transfer Certificate of Title No. N-115384
covering the lot issued in the name of petitioner-spouses.

The controversy arose when petitioners, despite repeated warnings from respondent,
extended the roof of their house to the property line and expanded the second floor of
their house to a point directly above the original front wall. Respondent filed before the
Regional Trial Court, National Capital Judicial Region, Branch 261, Pasig City, an action
to demolish the unauthorized structures.

After trial, judgment was rendered against petitioners. On appeal to it, the Court of
Appeals affirmed the decision of the trial court.

Issue:
Whether or not the Contract to Sell executed between the parties, contained a
Restrictive Covenant providing certain prohibitions is valid

Ruling:Yes. Restrictive covenants are not, strictly speaking, synonymous with


easements. While it may be correct to state that restrictive covenants on the use of land
or the location or character of buildings or other structures thereon may broadly be said
to create easements or rights, it can also be contended that such covenants, being
limitations on the manner in which one may use his own property, do not result in true
easements, but a case of servitudes (burden), sometimes characterized to be negative
easements or reciprocal negative easements. Negative easement is the most common

329
easement created by covenant or agreement whose effect is to preclude the owner of
the land from doing an act, which, if no easement existed, he would be entitled to do.
Courts which generally view restrictive covenants with disfavor for being a restriction on
the use of one's property, have, nevertheless, sustained them where the covenants are
reasonable, not contrary to public policy, or to law, and not in restraint of trade. Subject
to these limitations, courts enforce restrictions to the same extent that will lend judicial
sanction to any other valid contractual relationship. In general, frontline restrictions on
constructions have been held to be valid stipulations.

The provisions in a restrictive covenant prescribing the type of the building to be erected
are crafted not solely for the purpose of creating easements, generally of light and view,
nor as a restriction as to the type of construction, but may also be aimed as a check on
the subsequent uses of the building conformably with what the developer originally
might have intended the stipulations to be.

There appears to be no cogent reasons for not upholding restrictive covenants aimed to
promote aesthetics, health, and privacy or to prevent overcrowding.

The argument then of petitioners that expansion is necessary in order to accommodate


the individual families of their two children must fail for like reason. Nor can petitioners
claim good faith; the restrictive covenants are explicitly written in the Contract To Sell
and annotated at the back of the Transfer Certificate of Title.

Thus, a developer of a subdivision can enforce restrictions, even as against remote


grantees of lots, only if he retains part of the land. There would have been merit in the
argument of petitioners - that respondent, having relinquished ownership of the
subdivision to the homeowners, is precluded from claiming any right or interest on the
same property - had not the homeowners' association, confirmed by its board of
directors, allowed respondent to enforce the provisions of the restrictive covenant.

330
SPOUSES DE LA CRUZ VS. OLGA RAMISCAL

G.R. No. 137882


February 4, 2005

Facts:

Respondent Olga Ramiscal is the registered owner of a parcel of land located at the
corner of 18th Avenue and Boni Serrano Avenue, Murphy, Quezon City, covered by
Transfer Certificate of Title (TCT) No. 300302 of the Register of Deeds for Quezon City.
Petitioners SPS. ELIZABETH and ALFREDO DE LA CRUZ are occupants of a parcel of
land, with an area of eighty-five (85) square meters, located at the back of Ramiscals
property, and covered by TCT No. RT-56958 (100547) in the name of Concepcion de la
Pea, mother of petitioner Alfredo de la Cruz.

The subject matter of this case is a 1.10-meter wide by 12.60-meter long strip of land
owned by respondent which is being used by petitioners as their pathway to and from
18th Avenue, the nearest public highway from their property. Petitioners had enclosed
the same with a gate, fence, and roof.

In 1976, respondent leased her property, including the building thereon, to Phil. Orient
Motors. Phil. Orient Motors also owned a property adjacent to that of respondents. In
1995, Phil. Orient Motors sold its property to San Benito Realty. After the sale, Engr.
Rafael Madrid prepared a relocation survey and location plan for both contiguous
properties of respondent and San Benito Realty. It was only then that respondent
discovered that the aforementioned pathway being occupied by petitioners is part of her
property.

Through her lawyer, respondent immediately demanded that petitioners demolish the
structure constructed by them on said pathway without her knowledge and consent. As
her letter dated 18 February 1995 addressed to petitioners went unheeded, the former
referred the matter to the Barangay for conciliation proceedings, but the parties arrived
at no settlement. Hence, respondent filed this complaint with the RTC in Civil Case No.
Q-95-25159, seeking the demolition of the structure allegedly illegally constructed by
petitioners on her property. Respondent asserted in her complaint that petitioners have
an existing right of way to a public highway other than the current one they are using,
which she owns. She prayed for the payment of damages.

In support of the complaint, respondent presented TCT No. RT-56958 (100547)


covering the property denominated as Lot 1-B in the name of Concepcion de la Pea,
mother of petitioner herein Alfredo de la Cruz. The aforesaid TCT reveals that a portion
of Lot 1-B, consisting of 85 square meters and denominated as Lot 1-B-2, is being
occupied by petitioners. To prove that petitioners have an existing right of way to a
public highway other than the pathway which respondent owns, the latter adduced in
evidence a copy of the plan of a subdivision survey for Concepcion de la Pea and
Felicidad Manalo prepared in 1965 and subdivision plan for Concepcion de la Pea

331
prepared in 1990. These documents establish an existing 1.50-meter wide alley,
identified as Lot 1-B-1, on the lot of Concepcion de la Pea, which serves as
passageway from the lot being occupied by petitioners (Lot 1-B-2), to Boni Serrano
Avenue.

On 31 July 1997, the RTC handed down a decision, giving probative weight to the
evidence adduced by respondent. The decretal portion enunciates:
Plaintiffs claim for moral damages must be denied as no evidence in support thereof
was presented at all by her. Consequently, plaintiff is not entitled to exemplary
damages. However, for having been compelled to file this suit and incur expenses to
protect her interest, plaintiff is entitled to an attorneys fees in the amount of P10,000.00.
WHEREFORE, premises considered, judgment is hereby rendered in favor of the
plaintiff and ordering the defendants to demolish the structure built by them along the
pathway on the eastern side of plaintiffs property towards 18th Avenue, Murphy,
Quezon City and to pay [the] plaintiff the amount of P10,000.00 as and by way of
attorneys fees.

The Court of Appeals dismissed the appeal filed by petitioners from the RTC decision
for failure to file brief within the reglementary period

Issue:
Whether or not the petitioners are entitled to a voluntary or legal easement of right of
way.

Ruling:

No. An easement or servitude is a real right, constituted on the corporeal immovable


property of another, by virtue of which the owner has to refrain from doing, or must allow
someone to do, something on his property, for the benefit of another thing or persons.
The statutory basis for this right is Article 613, in connection with Article 619, of the Civil
Code, which states that, ― Art. 613. An easement or servitude is an encumbrance
imposed upon an immovable for the benefit of another immovable belonging to a
different owner.‖

The immovable in favor of which the easement is established is called the dominant
estate; that which is subject thereto, the servient estate.

Art. 619. Easements are established either by law or by the will of the owners. The
former are called legal and the latter voluntary easements.

Did respondent voluntarily accord petitioners a right of way? The Supreme Court ruled
in the negative. Petitioners failed to show by competent evidence other than their bare
claim that they and their tenants, spouses Manuel and Cecilia Bondoc and Carmelino
Masangkay, entered into an agreement with respondent, through her foreman, Mang
Puling, to use the pathway to 18th Avenue, which would be reciprocated with an
equivalent 1.50-meter wide easement by the owner of another adjacent estate. The

332
hands of this Court are tied from giving credence to petitioners self-serving claim that
such right of way was voluntarily given them by respondent for the following reasons:
First, petitioners were unable to produce any shred of document evidencing such
agreement. The Civil Code is clear that any transaction involving the sale or disposition
of real property must be in writing. Thus, the dearth of corroborative evidence opens
doubts on the veracity of the naked assertion of petitioners that indeed the subject
easement of right of way was a voluntary grant from respondent. Second, as admitted
by the petitioners, it was only the foreman, Mang Puling, who talked with them regarding
said pathway on the northern side of respondents property. Thus, petitioner Elizabeth
de la Cruz testified that she did not talk to respondent regarding the arrangement
proposed to them by Mang Puling despite the fact that she often saw respondent. It is,
therefore, foolhardy for petitioners to believe that the alleged foreman of respondent had
the authority to bind the respondent relating to the easement of right of way. Third, their
explanation that said Mang Puling submitted said agreement to the Quezon City
Engineers Office, in connection with the application for a building permit but said office
could no longer produce a copy thereof, does not inspire belief. As correctly pointed out
by the trial court, petitioners should have requested a subpoena duces tecum from said
court to compel the Quezon City Engineers Office to produce said document or to prove
that such document is indeed not available.

The fact that the perimeter wall of the building on respondents property was constructed
at a distance of 1.10 meters away from the property line, does not by itself bolster the
veracity of petitioners story that there was indeed such an agreement. Further, as noted
by the trial court, it was Atty. Federico R. Onandia, counsel of Phil. Orient Motors, who
wrote petitioners on 25 August 1994 advising them that his client would close the
pathway along 18th Avenue, thereby implying that it was Phil. Orient Motors,
respondents lessee, which tolerated petitioners use of said pathway.

The conferment of a legal easement of right of way under Article 649 is subject to proof
of the following requisites: (1) it is surrounded by other immovables and has no
adequate outlet to a public highway; (2) payment of proper indemnity; (3) the isolation is
not the result of its own acts; (4) the right of way claimed is at the point least prejudicial
to the servient estate; and (5) to the extent consistent with the foregoing rule, where the
distance from the dominant estate to a public highway may be the shortest. The first
three requisites are not obtaining in the instant case.

Article 649 of the Civil Code provides that the easement of right of way is not
compulsory if the isolation of the immovable is due to the proprietors own acts. To allow
defendants access to plaintiffs property towards 18th Avenue simply because it is a
shorter route to a public highway, despite the fact that a road right of way, which is even
wider, although longer, was in fact provided for them by Concepcion de la Pea towards
Boni Serrano Avenue would ignore what jurisprudence has consistently maintained
through the years regarding an easement of right of way, that mere convenience for the
dominant estate is not enough to serve as its basis. To justify the imposition of this
servitude, there must be a real, not a fictitious or artificial necessity for it. In Francisco
vs. Intermediate Appellate Court, 177 SCRA 527, it was likewise held that a person who

333
had been granted an access to the public highway through an adjacent estate cannot
claim a similar easement in an alternative location if such existing easement was
rendered unusable by the owners own act of isolating his property from a public
highway, such as what Concepcion de la Pea allegedly did to her property by
constructing houses on the 1.50 meter wide alley leading to Boni Serrano Avenue. And,
if it were true that defendants had already bought Lot 1-B-2, the portion occupied by
them, from Concepcion de la Pea, then the latter is obliged to grant defendants a right
of way without indemnity.

Under the above-quoted Article 649 of the Civil Code, it is the owner, or any person who
by virtue of a real right may cultivate or use any immovable surrounded by other
immovable pertaining to other persons, who is entitled to demand a right of way through
the neighboring estates. In this case, petitioners fell short of proving that they are the
owners of the supposed dominant estate. Nor were they able to prove that they possess
a real right to use such property. The petitioners claim to have acquired their property,
denominated as Lot 1-B-2, from Concepcion de la Pea, mother of defendant Alfredo de
la Cruz, who owns Lot 1-B-3, an adjacent lot. However, as earlier noted, the trial court
found that the title to both lots is still registered in the name of Concepcion de la Pea
under TCT No. RT-56958 (100547). Neither were petitioners able to produce the Deed
of Sale evidencing their alleged purchase of the property from de la Pea. Hence, by the
bulk of evidence, de la Pea, not petitioners, is the real party-in-interest to claim a right of
way although, as explained earlier, any action to demand a right of way from de la Peas
part will not lie inasmuch as by her own acts of building houses in the area allotted for a
pathway in her property, she had caused the isolation of her property from any access
to a public highway.

334
ENEDINA PRESLEY VS. BEL-AIR VILLAGE ASSOCIATION, INC.

G.R. No. 86774


August 21, 1991

Facts:

A complaint for specific performance and damages with preliminary injunction was filed
by plaintiff-appellee, Bel-Air Village Association, Inc. (BAVA for short) against Teofilo
Almendras and Rollo Almendras (now both deceased and substituted by defendant-
appellant Enedina Presley) for violation of the Deed Restrictions of Bel-Air Subdivision
that the subject house and lot shall be used only for residential and not for commercial
purposes and for non-payment of association dues to plaintiff BAVA amounting to
P3,803.55. The Almendrases were at the time of the filing of the action the registered
owners of a house and lot located at 102 Jupiter Street, Bel-Air Village, Makati, Metro
Manila. As such registered owners, they were members of plaintiff BAVA pursuant to
the Deed Restrictions annotated in their title over the property in question and
defendant Presley, as lessee of the property, is the owner and operator of 'Hot Pan de
Sal Store' located in the same address. At the time the Almendrases bought their
property in question from Makati Development Corporation, the Deed Restrictions was
already annotated in their title providing (among others) 'that the lot must be used only
for residential purpose. When BAVA came to know of the existence of the 'Pan de sal'
store, it sent a letter to the defendants asking them to desist from operating the store.
Under the existing Deed Restrictions aforesaid, the entire Bel-Air Subdivision is
classified as a purely residential area, particularly Jupiter Road which is owned by and
registered in the name of BAVA.

Issue:

Whether or not conditions annotated on the Torrens title can be impaired if necessary to
reconcile with the legitimate exercise of police power.

Ruling:

The court held that they are valid and can be enforced against the petitioner. However,
these contractual stipulations on the use of the land even if said conditions are
annotated on the torrens title can be impaired if necessary to reconcile with the
legitimate exercise of police power. We reiterate the Court's pronouncements in the
Sangalang case which are quite clear: It is not that we are saying that restrictive
easements, especially the easements herein in question, are invalid or ineffective. As
far as the Bel-Air subdivision itself is concerned, certainly, they are valid and
enforceable. But they are, like all contracts, subject to the overriding demands, needs,
and interests of the greater number as the State may determine in the legitimate
exercise of police power. Our jurisdiction guarantees sanctity of contract and is said to
be the 'law between the contracting parties, (Civil Code, supra, art. 1159) but while it is
so, it cannot contravene the law, morals, good customs, public order, or public policy.

335
Above all, it cannot be raised as a deterrent to police power, designed precisely to
promote health, safety, peace, and enhance the common good, at the expense of
contractual rights, whenever necessary.

336
SOLID MANILA CORPORATION VS. BIO HONG TRADING CO.

G.R. No. 90596


April 8, 1991

Facts:

Solid Manila Corp. is the owner of a parcel of land located in Ermita. The same lies in
the vicinity of another parcel registered under Bio Hong Trading whose title came from a
prior owner. In the deed of sale between Bio Hong and the vendor, 900 sqm of the lot
was reserved as an easement of way. The construction of the private alley was
annotated on Bio Hong‘s title stating among other things "(6) That the alley shall remain
open at all times, and no obstructions whatsoever shall be placed thereon; and (7) that
the owner of the lot on which the alley has been constructed shall allow the public to
use the same, and allow the City to lay pipes for sewer and drainage purposes, and
shall not act (sic) for any indemnity for the use thereof. The petitioner claims that ever
since, it made use of the above private alley and maintained and contributed to its
upkeep, until sometime in 1983, when, and over its protests, the private respondent
constructed steel gates that precluded unhampered use. On December 6, 1984, the
petitioner commenced suit for injunction against the private respondent, to have the
gates removed and to allow full access to the easement. The trial court ordered Bi Hong
to open the gates but the latter argued that the easement has been extinguished by
merger in the same person of the dominant and servient estates upon the purchase of
the property from its former owner.

Issues:

1. Whether or not easements may be alienated (sold) from the tenement or


mortgaged separately
2. Whether or not the easement had been extinguished by merger.

Ruling:

1. The sale included the alley. The court rejected Solid‘s contention that the alley
was not included in the sale. It was included but there was a limitation on its use-
the easement. As a mere right of way, it cannot be separated from the tenement
and maintain an independent existence. Even though Bio Hong acquired
ownership over the property –– including the disputed alley –– as a result of the
conveyance, it did not acquire the right to close that alley or otherwise put up
obstructions thereon and thus prevent the public from using it, because as a
servitude, the alley is supposed to be open to the public.
2. No genuine merger took place as a consequence of the sale in favor of the
private respondent corporation. According to the Civil Code, a merger exists
when ownership of the dominant and servient estates is consolidated in the same
person. Merger requires full ownership of both estates. Note that the servitude in
question is a personal servitude (established for the benefit of a community, or of

337
one or more persons to whom the encumbered estate does not belong). In a
personal servitude, there is therefore no "owner of a dominant tenement" to
speak of, and the easement pertains to persons without a dominant estate, in this
case, the public at large. Thus, merger could not have been possible.

338
DONATION

339
REPUBLIC OF THE PHILIPPINES VS. LEON SILIM and ILDEFONSA MANGUBAT

G.R. No. 140487


April 2, 2001

Facts:

On 17 December 1971, respondents, the Spouses Leon Silim and Ildefonsa Mangubat,
donated a 5,600 square meter parcel of land in favor of the Bureau of Public Schools,
Municipality of Malangas, Zamboanga del Sur (BPS). In the Deed of Donation,
respondents imposed the condition that the said property should be used exclusively
and forever for school purposes only.This donation was accepted by Gregorio Buendia,
the District Supervisor of BPS, through an Affidavit of Acceptance and/or Confirmation
of Donation.

Through a fund raising campaign spearheaded by the Parent-Teachers Association of


Barangay Kauswagan, a school building was constructed on the donated land.
However, the Bagong Lipunan school building that was supposed to be allocated for the
donated parcel of land in Barangay Kauswagan could not be released since the
government required that it be built upon a one (1) hectare parcel of land. To remedy
this predicament, Assistant School Division Superintendent of the Province of
Zamboanga del Sur, Sabdani Hadjirol, authorized District Supervisor Buendia to
officially transact for the exchange of the one-half (1/2) hectare old school site of
Kauswagan Elementary School to a new and suitable location which would fit the
specifications of the government. Pursuant to this, District Supervisor Buendia and
Teresita Palma entered into a Deed of Exchange whereby the donated lot was
exchanged with the bigger lot owned by the latter.

Consequently, the Bagong Lipunan school buildings were constructed on the new
school site and the school building previously erected on the donated lot was
dismantled and transferred to the new location. When respondent Leon Silim saw, to his
surprise, that Vice-Mayor Wilfredo Palma was constructing a house on the donated
land, he asked the latter why he was building a house on the property he donated to
BPS. Vice Mayor Wilfredo Palma replied that he is already the owner of the said
property. Respondent Leon Silim endeavored to stop the construction of the house on
the donated property but Vice-Mayor Wilfredo Palma advised him to just file a case in
court.

On February 10, 1982, respondents filed a Complaint for Revocation and Cancellation
of Conditional Donation, Annulment of Deed of Exchange and Recovery of Possession
and Ownership of Real Property with damages against Vice Mayor Wilfredo Palma,
Teresita Palma, District Supervisor Buendia and the BPS before the Regional Trial
Court of Pagadian City, Branch 21.

340
Issue:

Whether or not the donation null and void due to an invalid acceptance by the donee.

Ruling:

The court held that the donation is void. The purpose of the formal requirement for
acceptance of a donation is to ensure that such acceptance is duly communicated to
the donor. Thus, in Pajarillo vs. Intermediate Appellate Court, the Court held: There is
no question that the donation was accepted in a separate public instrument and that it
was duly communicated to the donors. Even the petitioners cannot deny this. But what
they do contend is that such acceptance was not "noted in both instruments," meaning
the extrajudicial partition itself and the instrument of acceptance, as required by the Civil
Code. That is perfectly true. There is nothing in either of the two instruments showing
that "authentic notice" of the acceptance was made by Salud to Juana and Felipe. And
while the first instrument contains the statement that "the donee does hereby accept this
donation and does hereby express her gratitude for the kindness and liberality of the
donor," the only signatories thereof were Felipe Balane and Juana Balane de Suterio.
That was in fact the reason for the separate instrument of acceptance signed by Salud a
month later.

341
DECS VS. JULIA DEL ROSARIO, ET. AL.

G.R. No. 146586


January 26, 2005

Facts:

Respondents filed before the trial court a complaint for Recovery of Possession against
petitioner Department of Education, Culture and Sports (DECS) over a parcel of land in
Kaypombo, Sta. Maria, Bulacan. Respondents alleged that the Kaypombo Primary
School Annex (KPPS) under DECS was occupying a portion of the Property through
respondents‘ tolerance, and that KPPS refused to vacate the premises despite their
valid demands to do so. In its Answer, DECS countered that KPPSs occupation of a
portion of the Property was with the express consent and approval of respondents
father, the late Isaias Del Rosario (Isaias), who donated a portion (Donated Site) of the
Property to the Municipality in 1959 for school site purposes.

After trial, the court dismissed the complaint, holding that the defense was able to prove
the due execution of the deed of donation and its acceptance, as well as the loss of the
same, in accordance with Rule 130, Sec. 4. Respondents appealed to the Court of
Appeals, which reversed the decision holding that the lower court erred in hastily
concluding that the loss of the document was sufficiently established when in fact, the
defendant did not look for it in the office of the Clerk of Court and the National Library.
Hence this petition.

Issue:

Whether or not the donation was properly proven, and Whether or not respondent‘s
claim was barred by laches.

Ruling:

Article 749 of the Civil Code states that in order that the donation of an immovable may
be valid, it must be made in a public document, specifying therein the property donated
and the value of the charges which the donee must satisfy. The acceptance may be
made in the same deed of donation or in a separate public document, but it shall not
take effect unless it is done during the lifetime of the donor. If the acceptance is made in
a separate instrument, the donor shall be notified thereof in an authentic form, and this
step shall be noted in both instruments. The best or primary evidence of a donation of
real property is an authentic copy of the deed of donation. In its absence, a party may
prove the donation by other competent or secondary evidence under the exceptions in
Section 3, Rule 130 of the Revised

In this case DECS did not introduce in evidence the municipal council Resolution
accepting the donation. There is also no proof that the donee communicated in writing
its acceptance to the donor aside from the circumstance that DECS constructed the

342
school during Isaias lifetime without objection on his part. There is absolutely no
showing that these steps were noted in both instruments. Further, there is inadequate
proof that DECS or the Municipality made a diligent search in the places where the
deed of donation may likely be found and that the search was unsuccessful.

After a party establishes the existence and due execution of the document, he must
prove that the document was lost or destroyed. Here, DECS allegedly made a search in
the municipal building and in the DECS Division Office. The copies of the deed of
donation furnished these offices were purportedly lost when these offices transferred to
new locations. However, as the Court of Appeals correctly pointed out, Judge Natividad
who claimed to have notarized the deed of donation failed to account for other copies of
the deed, which the law strictly enjoins him to record, and furnish to other designated
government offices.

Lastly, DECS raises for the first time before this Court the issue on whether
respondents claim is barred by the equitable defense of laches. DECS did not raise this
matter in the complaint or during the trial in the court below. DECS did not also raise
this matter in its appeal to the Court of Appeals. This Court cannot entertain this issue at
this late stage, for to do so would plainly violate the basic rule of fair play, justice and
due process. Hence, the petition was denied.

343
GONZALES VS. COURT OF APPEALS

G.R.No. L-37453
May 25, 1979

Facts:

Petitioner Rizalina Gonzales and Lutgarda Santiago (Private respondent) are the nieces
of the deceased Isabel Gabriel who died a widow. A will was thereafter submitted to
probate. The said will was typewritten, in Tagalog and appeared to have been executed
in April 1961 or two months prior to the death of Isabel. It consisted of 5 pages including
the attestation and acknowledgment, with the signature of testatrix on page 4 and the
left margin of all the pages.

Lutgarda was named as the universal heir and executor. The petitioner opposed the
probate.

The lower court denied the probate on the ground that the will was not executed and
attested in accordance with law on the issue of the competency and credibility of the
witnesses.

Issue:

Whether or not the credibility of the subscribing witnesses is material to the validity of a
will.

Ruling:

No. The law requires only that witnesses possess the qualifications under Art. 820
(NCC) and none of the disqualifications of Art. 802. There is no requirement that they
are of good standing or reputation in the community, for trustworthiness, honesty and
uprightness in order that his testimony is believed and accepted in court.

For the testimony to be credible, it is not mandatory that evidence be established on


record that the witnesses have good standing in the community. Competency is
distinguished from credibility, the former being determined by Art. 820 while the latter
does not require evidence of such good standing. Credibility depends on the convincing
weight of his testimony in court.

344
MARIA ALVAREZ VDA. DE DELGADO VS. COURT OF APPEALS

G.R. No. 125728


August 28, 2001

Facts:

During his lifetime, Carlos Delgado was the absolute owner of a parcel of land with an
area of 692,549 square meters, situated in the Municipality of Catarman, Samar. On
October 5, 1936, said Carlos Delgado granted and conveyed, by way of donation or gift
with quitclaim, all his rights, title, interest, claim and demand over a portion of said land
consisting of 165,000 square meters in favor of the Commonwealth of the Philippines or
its successors. Acceptance was made by then President Manuel L. Quezon in his
capacity as Commander-in-Chief of the Philippine Army.

The Deed of Donation states as reason or consideration the donor's desire to contribute
to the formation of the National Defense of the Philippines. It contained the following
condition:

The condition of this donation is, that the parcel of land above described shall be for the
exclusive benefit of the Commonwealth of the Philippines to be used as military
reservation for training cadres or for such other uses of the Philippine Army as the
Commander-in-Chief or Chief of Staff thereof may determine, provided that when the
Commonwealth of the Philippines no longer needs this parcel of land for any military
purposes, then said land shall automatically revert to the donor or its heirs or assigns.
The donee promptly occupied the donated land and constructed buildings thereon for
military purposes, such as a military training campsite. Further, after entering into
physical possession of the land and making the said improvements, the donee caused
the property and several others similarly donated to it to be surveyed, with a view to
having them all brought under the operation of the Torrens system and registered in the
name of the Commonwealth of the Philippines.

Issue:

Whether or not the petitioners‘ action for reconveyance is already barred by


prescription.

Ruling:

The court held that the causes of action which petitioners may have against the
respondent Republic, in our view, are already barred by prescription.

At the outset, we find that the case of Roman Catholic Archbishop of Manila vs. Court of
Appeals, 198 SCRA 300 (1991), provides a precedent in the resolution of the issue at
hand. It involved a donation by the Eusebio spouses as private respondents therein, of

345
a parcel of land, with an express provision for automatic reversion of the donated
property in case of a violation of the condition therein. This Court held that from parity of
reasons, the rules governing onerous donations are applicable to donations with a
resolutory condition. Although automatic reversion immediately happens upon a
violation of the condition and therefore no judicial action is necessary for such purpose,
still judicial intervention must be sought by the aggrieved party if only for the purpose of
determining the propriety of the rescission made.

Applying Article 1144 (1) of the Civil Code on prescription of actions based on a written
contract, the petitioners herein should have instituted the action for reconveyance within
10 years from the time the condition in the Deed of Donation was violated. The earliest
date the petitioners knew of the said violation of said condition was on July 4, 1946,
when the Republic, as successor of the Commonwealth of the Philippines, took over the
properties and diverted the property to uses other than that imposed by the donor.

346
CIRILA ARCABA VS. ERLINDA TABANCURA VDA. DE BATOCAEL, ET. AL.

G.R. No. 146683


November 22, 2001

Facts:

On January 16, 1956, Francisco Comille and his wife Zosima Montallana became the
registered owners of Lot No. 437-A. After the death of Zosima on October 3, 1980,
Francisco and his mother-in-law executed a deed of extrajudicial partition with waiver of
rights, in which the latter waived her share consisting of one-fourth (1/4) of the property
in favor of Francisco. Having no children to take care of him after his retirement,
Francisco asked his niece Leticia Bellosillo and her cousin, Luzviminda, and petitioner
Cirila Arcaba, then a widow, to take care of his house, as well as the store inside.

According to Leticia, Francisco and Cirila were lovers since they slept in the same room.
On the other hand, Erlinda Tabancura, another niece of Francisco claimed that the latter
told her that Cirila was his mistress. However, Cirila said she was a mere helper who
could enter the masters bedroom only when the old man asked her to and that
Francisco in any case was too old for her, also denying they ever had sexual
intercourse. When Leticia and Luzviminda got married, only Cirila was left to take care
of Francisco. Erlinda Tabancura testified that Francisco‘s sole source of income
consisted of rentals from his lot near the public streets, and that he did not pay Cirila a
regular cash wage as a househelper, though he provided the latter‘s family with food
and lodging.

In January 1991, few months before Francisco died, he executed a ―Deed of Donation
Inter Vivos‖ where he ceded a portion of Lot 437-A composed of 150 sq m., together
with his house to Cirila who accepted the same. The larger portion of 268 sq m. was left
under his name. This was made in consideration of the 10 year of faithful services of
Cirila. Atty. Lacaya notarized the deed and was later registered by Cirila as its absolute
owner. Francisco eventually died in October 1991 without any children. The decedent‘s
nephews and nieces and his heirs by intestate succession alleged that Cirila was the
common-law wife of Francisco.

Issue:

Whether or not the deed of donation inter vivos executed by Francisco in Arcaba‘s favor
was valid.

Ruling:

No, the donation inter vivos executed by Francisco is not valid. The Court held that
Cohabitation means more than sexual intercourse, especially when one of the parties is
already old and may no longer be interested in sex. At the very least, cohabitation is the

347
public assumption by a man and a woman of the marital relation, and dwelling together
as man and wife, thereby holding themselves out to the public as such.

Seigfredo Tabancura presented documents apparently signed by Cirila using the


surname Comille. Also, the fact that Cirila did not demand from Francisco a regular
cash wage is an indication that she was not simply a caregiver-employee, but
Francisco‘s common law spouse. She was, after all, entitled to a regular cash wage
under the law. It is difficult to believe that she stayed with Francisco and served him out
of pure beneficence.

Having proven by a preponderance of evidence that Cirila and Francisco lived together
as husband and wife without a valid marriage, the inescapable conclusion is that the
donation made by Francisco in favor of Cirila is void under Art. 87 of the Family Code.

348
RICKY QUILALA VS. GLICERIA ALCANTARA, ET. AL.

G.R. No. 132681


December 3, 2001

Facts:

On February 20, 1981, Catalina Quilala executed a ―Donation of Real Property Inter
Vivos" in favor of Violeta Quilala over a parcel of land located in Sta. Cruz, Manila,
containing an area of 94 square meters, and registered in her name under Transfer
Certificate of Title No. 17214 of the Register of Deeds for Manila. The first page
contains the deed of donation itself, and is signed on the bottom portion by Catalina
Quilala and Violeta Quilala, and two instrumental witnesses. The second page contains
the Acknowledgment, which states merely that Catalina Quilala personally appeared
before the notary public and acknowledged that the donation was her free and voluntary
act and deed. There appear on the left-hand margin of the second page the signatures
of Catalina Quilala and one of the witnesses, and on the right-hand margin the
signatures of Violeta Quilala and the other witness

On November 7, 1983, Catalina Quilala died. Violeta Quilala likewise died on May 22,
1984, while Petitioner Ricky Quilala alleges that he is the surviving son of Violeta
Quilala.

Meanwhile, respondents Gliceria Alcantara, Leonora Alcantara, Ines Reyes and Juan
Reyes, claiming to be Catalina‘s only surviving relatives within the fourth civil degree of
consanguinity, executed a deed of extrajudicial settlement of estate, dividing and
adjudicating unto themselves the above-described property. The trial court found that
the deed of donation, although signed by both Catalina and Violeta, was acknowledged
before a notary public only by the donor, Catalina. Consequently, there was no
acceptance by Violeta of the donation in a public instrument, thus rendering the
donation null and void.

Issue:

Whether or not the donation made by Catalina to Violeta valid.

Ruling:

Yes, the donation made by Catalina to Violeta is valid.

The second page of the deed of donation, on which the Acknowledgment appears, was
signed by the donor and one witness on the left-hand margin, and by the donee and the
other witness on the right-hand margin. Surely, the requirement under Section 112,
paragraph 2 of Presidential Decree No. 1529 that the contracting parties and their
witnesses should sign on the left-hand margin of the instrument is not absolute. The
intendment of the law merely is to ensure that each and every page of the instrument is

349
authenticated by the parties. The requirement is designed to avoid the falsification of the
contract after the same has already been duly executed by the parties. Hence, a
contracting party affixes his signature on each page of the instrument to certify that he is
agreeing to everything that is written thereon at the time of signing.

The specification of the location of the signature is merely directory. The fact that one of
the parties signs on the wrong side of the page does not invalidate the document. The
purpose of authenticating the page is served, and the requirement in the above-quoted
provision is deemed substantially complied with.

In the same vein, the lack of an acknowledgment by the donee before the notary public
does not also render the donation null and void. The instrument should be treated in its
entirety. It cannot be considered a private document in part and a public document in
another part. The fact that it was acknowledged before a notary public converts the
deed of donation in its entirety a public instrument. The fact that the donee was not
mentioned by the notary public in the acknowledgment is of no moment. To be sure, it is
the conveyance that should be acknowledged as a free and voluntary act. In any event,
the donee signed on the second page, which contains the Acknowledgment only. Her
acceptance, which is explicitly set forth on the first page of the notarized deed of
donation, was made in a public instrument.

350
APOLINARIA AUSTRIA-MAGAT VS. COURT OF APPEALS

G.R. No. 106755


February 1, 2002

Facts:

On December 17, 1975, Basilisa Comerciante, mother of petitioner Apolinaria Austria-


Magat and Florentino Lumubos, one of respondents, furnished a Deed of Donation to
donate her house and lot to her four children, including petitioner and respondent,
provided that the funeral expenses will be deducted from the total value of the lot before
it is to be divided among the children. The children signed to the same deed in
acceptance to the donation. That same day, they also signed into a notarized document
stating that the property and the document pertaining to the same will be under the
custody of Comerciante, the original owner for as long as she lives.

On February 6, 1979, Comerciante executed a Deed of Absolute Sale over the same
house and lot in favor of the petitioner, prompting the respondents to file an action
against the petitioner for the annulment of the deed of sale on September 21, 1983. The
lower court ruled in favor of the respondent, Apolinaria, stating that the donation is a
donation mortis causa based on the provision on the deed of donation that it would take
effect upon the death of the donor. Moreover, the provision stating that the donor
reserved the right to revoke the donation is a feature of a donation mortis causa which
must comply with the formalities of a will. However, the Court of Appeals reversed the
trial court decision, based on a provision in the deed of donation in question providing
for the irrevocability of the donation, stating that it is a characteristic of a donation inter
vivos.

Issue:

Whether or not the Court of Appeals is correct in saying that the donation in question is
a donation inter vivos.

Ruling:

The Court of Appeals committed no reversible error in its appealed Decision.

Significant to the resolution of this issue is the irrevocable character of the donation in
the case at bar. In Cuevas v. Cuevas, it was ruled that when the deed of donation
provides that the donor will not dispose or take away the property donated (thus making
the donation irrevocable), he in effect is making a donation inter vivos. He parts away
with his naked title but maintains beneficial ownership while he lives. It remains to be a
donation inter vivos despite an express provision that the donor continues to be in
possession and enjoyment of the donated property while he is alive.
In the Bonsato case, it was held that: What is most significant [in determining the type of
donation] is the absence of stipulation that the donor could revoke the donations; on the

351
contrary, the deeds expressly declare them to be ―irrevocable‖, a quality absolutely
incompatible with the idea of conveyances mortis causa where revocability is of the
essence of the act, to the extent that a testator cannot lawfully waive or restrict his right
of revocation.

In the case at bar, the express irrevocability of the same (hindi na mababawi) is the
distinctive standard that identifies that document as a donation inter vivos. The other
provisions therein which seemingly make the donation mortis causa do not go against
the irrevocable character of the subject donation.

352
HEIRS OF SEVILLA FLORENCIO VS. HEIRS OF SEVILLA DE LEON

G.R. No. 149570


March 12, 2004

Facts:

Teresa Sevilla de Leon, owned a residential lot with an area of 828 square meters
located in San Miguel, Bulacan, covered by Transfer Certificate of Title (TCT) No. T-
44349. In the 1960s, De Leon allowed the spouses Rosendo and Consuelo Florencio to
construct a house on the said property and stay therein without any rentals therefor. In
1996, De Leon, with the consent of her husband leased the land to Bienvenido Santos
as long as De Leon had an outstanding loan with Dev. Bank of QC but not to exceed
the period of 15 years. De Leon assigned her leasehold right in favor of the bank.
Thereafter, Santos constructed a house thereon. 1978, De Leon, then a widow, died
intestate. Her heirs allowed Florencio to continue staying in the property. In 1995, the
heirs of De Leon sent a letter to the heirs of Florencio demanding them to vacate the
property within 90 days. The heirs of Florencio refused to vacate.

The heirs of De Leon thereafter filed a complaint for ejectment. They alleged that they
were the pro-diviso owners of the land which they inherited from their mother. They
alleged also that their mother allowed Florencio and his family to occupy the property
without any compensation subject to the condition that they shall vacate the same upon
demand. The heirs of Florencio alleged that the plaintiffs had no cause of action against
them because Teresa De Leon donated the land in favor of their predecessor Rosendo
Florencio in 1976. The deed was witnessed by some people and was notarized.

The heirs of De Leon also filed a complaint to eject the heirs of Santos. But the heirs of
Santos refused to do so, alleging that they did not occupy the land by mere tolerance
but on the basis of contract of lease. For the heirs of Santos, only the heirs of Florencio
had the right to cause their eviction by reason of the deed of donation.

Issue:

Whether or not the heirs of Florencio who appears to be the donee under the
unregistered Deed of Donation, have a better right to the physical or material
possession of the property over the heirs of Teresa de Leon, the registered owners of
the property.

Ruling:

The petition has no merit. Under the New Civil Code, donation is one of the modes of
acquiring ownership. Among the attributes of ownership is the right to possess the
property. The essential elements of donation are as follows: (a) the essential reduction
of the patrimony of the donor; (b) the increase in the patrimony of the donee; and (c) the
intent to do an act of liberality or animus donandi. When applied to a donation of an

353
immovable property, the law further requires that the donation be made in a public
document and that the acceptance thereof be made in the same deed or in a separate
public instrument; in cases where the acceptance is made in a separate instrument, it is
mandated that the donor be notified thereof in an authentic form, to be noted in both
instruments.

In this case, the deed of donation, on its face, appears to bear all the essential
requisites of a valid donation inter vivos. With Teresa de Leon as the donor and
Rosendo Florencio as the donee, the deed of donation appears to have been notarized
by Notary Public Tirso Manguiat. On this premise, Florencio, and after his death, his
heirs, acquired ownership over the property although Certificate of Title No. T-44349
under the name of Teresa de Leon had not yet been cancelled.

However, there are cogent facts and circumstances of substance which engender
veritable doubts as to whether the petitioners have a better right of possession over the
property other than the respondents, the lawful heirs of the deceased registered owner
of the property, Teresa de Leon, based on the Deed of Donation: (1) The intention of
the donation was questioned because the donee did not have the copy of the duplicate
of title, or at the very least, they should have registered the donation. (2) Florencio failed
to inform the heirs of De Leon that De Leon executed the deed of donation in 1976 in
his favor. (3) Respondents consistently paid the real taxes. (4) Their possession of the
owner‘s duplicate of the title would have fortified their claim that indeed, De Leon had
intended to convey the property by donation to Florencio. (5) Petitioners failed to
adduce in evidence Atty. Manguiat‘s counter-affidavit to the said complaint, or, at the
very least, a separate affidavit explaining the facts and circumstances surrounding the
notarization of the deed of donation. (6) Mayor, at the time of the donation, did not
affixed his signature.

354
C-J YULO & SONS, INC. VS. ROMAN CATHOLIC BISHOP OF SAN PABLO, INC.

G.R. No. 133705


March 31, 2005

Facts:

On September 24, 1977, petitioner donated unto respondent a parcel of land at


Canlubang, Calamba, Laguna registered in its name under Transfer Certificate of Title
(TCT) No. T-82803, on the condition that it shall be used for the construction of a home
for the aged and infirm and for other charitable purposes, and that the Donee shall not
use the land for any other purpose except with prior written consent of the Donor or its
successor, otherwise the said land with all real improvements thereon shall revert in
trust to the Donor for prompt disposition in favor of some other charitable organization
that Donor may deem best suited to the care of the aged.

Thereafter, in 1980, the donee, for purposes of generating funds to build the perimeter
fence on the donated property and the construction of a nucleus building for the aged
and the infirm, leased a portion of the donated property to one Martin Gomez who
planted said portion with sugar cane, without prior written consent of the donor, and
which ended in 1985. 1986, a portion of the donated property was again leased by the
donee without prior written consent of the donor, this time to one Jose Bostre who used
the leased area as a ranch to protect the premises from vandals and for the
electrification of the nucleus building of the home for the aged and in the infirm, which
was named as ―Casa dela Merced.‖ After the termination of the Bostre lease
agreement, the donee, for the third time, leased a portion of the donated property to one
Rudy Caballes who used the leased area for fattening cattles for the purposes of
generating funds for the completion of ―Casa dela Merced,‖ without prior written consent
of the donor.

On September 20, 1990, pursuant to a board resolution, the donor, through its president
Miguel A. Yulo, addressed a letter to the donee informing the latter that it was revoking
the due to the donee‘s non-compliance with and material breach of the conditions
thereunder stipulated.

Issue:

Whether or not revocation is proper.

Ruling:

No, the revocation is not proper. In Republic vs. Silim, where the donor sought to revoke
the donation on the ground that the donee breached the condition to exclusively and
forever use the land for school purpose only, the Court ruled in favor of the donee and
held that ―the condition for the donation was not in any way violated when the lot
donated was exchanged with another one. The purpose for the donation remains the

355
same, which is for the establishment of a school. The exclusivity of the purpose was not
altered or affected. In fact, the exchange of the lot for a much bigger one was in
furtherance and enhancement of the purpose of the donation. The acquisition of the
bigger lot paved way for the release of funds for the construction of Bagong Lipunan
school building which could not be accommodated by the limited area of the donated
lot.‖

The three (3) lease contracts herein entered into by the donee were for the sole
purpose of pursuing the objective for which the donation was intended. In fact, such
lease was authorized by the donor by express provision in the deed of donation, albeit
the prior written consent therefor of the donor is needed. Hence, considering that the
donee‘s acts did not detract from the very purpose for which the donation was made but
precisely to achieve such purpose, a lack of prior written consent of the donor would
only constitute casual breach of the deed, which will not warrant the revocation of the
donation.

Besides, Court cannot consider the requirement of a prior written consent by the donor
for all contracts of lease to be entered into by the donee as an absolute ground for
revocation of the donation because such a condition, if not correlated with the purpose
of the donation, would constitute undue restriction of the donee‘s right of ownership over
the donated property.

356
MA. ESTELA MAGLASANG, ET. AL. VS. HEIRS OF CORAZON CABATINGAN

G.R. No. 131953


June 5, 2002

Facts:

Conchita Cabatingan executed in favor of her brother, petitioner Nicolas Cabatingan, a


"Deed of Conditional of Donation Inter Vivos for House and Lot covering one-half (½)
portion of the former's house and lot located at Cot-cot, Liloan, Cebu. Four (4) other
deeds of donation were subsequently executed by Conchita, bestowing upon petitioners
Nicolas, Merly S. Cabatingan and Estela C. Maglasang for two parcels of land. One of
the provisions in the deeds are as follows:

"That for and in consideration of the love and affection of the DONOR for the DONEE,
the DONOR does hereby, by these presents, transfer, convey, by way of donation, unto
the DONEE the above-described property, together with the buildings and all
improvements existing thereon, to become effective upon the death of the DONOR;
PROVIDED, HOWEVER, that in the event that the DONEE should die before the
DONOR, the present donation shall be deemed automatically rescinded and of no
further force and effect."

When Conchita died, and upon learning of the existence of the foregoing donations,
respondents filed an action to annul the said four (4) deeds of donation. Respondents
allege that petitioners, through their sinister machinations and strategies and taking
advantage of Conchita Cabatingan's fragile condition, caused the execution of the
deeds of donation, and, that the documents are void for failing to comply with the
provisions of the Civil Code regarding formalities of wills and testaments, considering
that these are donations mortis causa. Petitioners deny respondents' allegations
contending that Conchita Cabatingan freely, knowingly and voluntarily caused the
preparation of the instruments. The lower court ruled in favor of the respondents and
therefore the donations are null and void for failure to comply with the requirements.

Issue:

Whether or not the donations to the petitioners are donations mortis causa or inter
vivos.

Ruling:

Petitioners insist that the donations are inter vivos donations as these were made by the
late Conchita Cabatingan "in consideration of the love and affection of the donor" for the
donee, and there is nothing in the deeds which indicate that the donations were made in
consideration of Cabatingan's death. Petitioners' arguments are bereft of merit.
In determining whether a donation is one of mortis causa, the following characteristics
must be taken into account: (1) It conveys no title or ownership to the transferee before

357
the death of the transferor; or what amounts to the same thing, that the transferor
should retain the ownership (full or naked) and control of the property while alive; (2)
That before his death, the transfer should be revocable by the transferor at will, ad
nutum; but revocability may be provided for indirectly by means of a reserved power in
the donor to dispose of the properties conveyed; (3) That the transfer should be void if
the transferor should survive the transferee.

In the present case, the nature of the donations as mortis causa is confirmed by the fact
that the donations do not contain any clear provision that intends to pass proprietary
rights to petitioners prior to Cabatingan's death. The phrase "to become effective upon
the death of the DONOR" admits of no other interpretation but that Cabatingan did not
intend to transfer the ownership of the properties to petitioners during her lifetime.
Petitioners themselves expressly confirmed the donations as mortis causa in the
Acceptance and Attestation clauses of the Deed of Donation. That the donations were
made "in consideration of the love and affection of the donor" does not qualify the
donations as inter vivos because transfers mortis causa may also be made for the same
reason.

358
MANUEL G. ABELLO, ET. AL. VS. COMMISSIONER OF INTERNAL REVENUE

G.R. No. 120721


February 23, 2005

Facts:

During the 1987 national elections, petitioners contributed P882, 661.31 each to the
campaign funds to the then Senate candidate Edgardo Angara. The BIR assessed each
of the petitioners P263, 032.66 for their contributions. Petitioners questioned the
assessment through a letter to the BIR. They claimed that political or electoral
contributions are not considered gifts under the National Internal Revenue Code
(NIRC), and that, therefore, they are not liable for donor‘s tax. The claim for exemption
was denied by the Commissioner.

Petitioners filed a petition for review with the CTA, which ruled in favor of the petitioners
and ordered the Commissioner to desist from collecting donor‘s taxes from the
petitioners. On appeal, the Court of Appeals reversed and set aside the CTA decision
and ordered the petitioners to pay donor‘s tax. Petitioners motion for reconsideration,
which the Court of Appeals was denied. Hence, this petition.

Issue:

Whether or not the contributions to the campaign fund are gift and should be subject to
donor‘s tax.

Ruling:

The NIRC does not define transfer of property by gift. However, Article 18 of the Civil
Code defines donation as an act of liberality whereby a person disposes gratuitously of
a thing or right in favor of another, who accepts it. Donation has the following elements:
(a) the reduction of the patrimony of the donor; (b) the increase in the patrimony of the
donee; and, (c) the intent to do an act of liberality or animus donandi.

The present case falls squarely within the definition of a donation. Petitioners gave
P882,661.31 to the campaign funds without any material consideration. All three
elements of a donation are present. The patrimony of the four petitioners were reduced
by P882, 661.31 each. Senator Edgardo Angara‘s patrimony correspondingly increased
by P3,530,645.24. There was intent to do an act of liberality or animus donandi was
present since each of the petitioners gave their contributions without any consideration.
Taken together with the Civil Code definition of donation, Section 91 of the NIRC is
clear and unambiguous, thereby leaving no room for construction. Therefore, by virtue
of Section 91 of the NIRC, the contribution being gifts are subject to donor‘s tax

359
SPOUSES GESTOPA VS. COURT OF APPEALS

G.R. No. 111904


October 5, 2000

Facts:

Spouses Diego and Catalina Danlag were the owners of six parcels of unregistered
lands. They executed three deeds of donation mortis causa, in favor of private
respondent Mercedes Danlag-Pilapil, the ilegitimate daughter of Diego. All deeds
contained the reservation of the rights of the donors (1) to amend, cancel or revoke the
donation during their lifetime, and (2) to sell, mortgage, or encumber the properties
donated during the donors' lifetime, if deemed necessary. Diego, with the consent of his
wife, again executed a deed of donation inter vivos covering the aforementioned parcels
of land plus two other parcels, in favor of Mercedes. This contained two conditions, that
(1) the Danlag spouses shall continue to enjoy the fruits of the land during their lifetime,
and that (2) the donee can not sell or dispose of the land during the lifetime of the said
spouses, without their prior consent and approval. Mercedes caused the transfer of the
parcels' tax declaration to her name and paid the taxes on them.

Diego and Catalina Danlag then sold parcels 3 and 4 to herein petitioners, Mr. and Mrs.
Agripino Gestopa. The Danlags executed a deed of revocation recovering the six
parcels of land subject of the aforecited deed of donation inter vivos. Mercedes Pilapil
then filed with the RTC a petition against the Gestopas and the Danlags, for quieting of
title over the above parcels of land.

The trial court ruled in favor of the defendants ruling that the reservation clause in all the
deeds of donation indicated that Diego Danlag did not make any donation; that the
purchase by Mercedes of the two parcels of land covered by the Deed of Donation Inter
Vivos bolstered this conclusion; that Mercedes failed to rebut the allegations of
ingratitude she committed against Diego Danlag; and that Mercedes committed fraud
and machination in preparing all the deeds of donation without explaining to Diego
Danlag their contents.

Mercedes appealed to the Court of Appeals, which reversed the trial court‘s decision
ruling that the reservation by the donor of lifetime usufruct indicated that he transferred
to Mercedes the ownership over the donated properties; that the right to sell belonged
to the donee, and the donor's right referred to that of merely giving consent; that the
donor changed his intention by donating inter vivos properties already donated mortis
causa; that the transfer to Mercedes' name of the tax declarations pertaining to the
donated properties implied that the donation was inter vivos; and that Mercedes did not
purchase two of the six parcels of land donated to her. Hence, this instant petition.

Issue:

360
Whether or not the donation was inter vivos or mortis causa.
Ruling:

It was donation inter vivos. The spouses were aware of the difference between the two
donations, and that they needed to execute another deed of donation inter vivos, since
it has a different application to a donation mortis causa. Also, the court stated four
reasons to the matter: (1) that the spouses donated the parcels of land out of love and
affection, a clear indication of a donation inter vivos; (2) the reservation of a lifetime
usufruct; (3) reservation of sufficient properties for maintenance that shows the intention
to part with their six lot; and (4) respondent's acceptance, contained in the deed of
donation.

Further, the revocation was not valid because a valid donation, once accepted,
becomes irrevocable, except on account of officiousness, failure by the donee to comply
with the charges imposed in the donation, or ingratitude. The donor-spouses did not
invoke any of these reasons in the deed of revocation. The records do not show that the
donor-spouses instituted any action to revoke the donation in accordance with Article
769 of the Civil Code. Consequently, the supposed revocation on September 29, 1979,
had no legal effect.

361
REPUBLIC OF THE PHILIPPINES VS. DAVID REY GUZMAN

G.R. No. 132964


February 18, 2000

Facts:

David Rey Guzman, a natural-born American citizen, is the son of the spouses Simeon
Guzman (naturalized American) and Helen Meyers Guzman (American citizen). In
1968, Simeon died leaving to his heirs, Helen and David, an estate consisting of several
parcels of land in Bulacan. Helen and David executed a Deed of Extrajudicial
Settlement of the Estate, dividing and adjudicating to themselves all of the property, and
registered it to the RD a year after. In 1981, Helen executed a Deed of Quitclaim,
assigning, transferring and conveying her ½ share of the properties to David. But since
it was not registered, she executed another Deed of Quitclaim to confirm the first.
In 1994, Atty. Batongbacal wrote the OSG questioning David‘s ownership of ½ of the
estate as defective; arguing that the only instances when a foreigner may acquire
private property are by hereditary succession and if he was formerly a natural-born
citizen who lost his Filipino citizenship. Moreover, it contends that the Deeds of
Quitclaim executed by Helen were really donations inter vivos. Republic filed with RTC
a Petition for Escheat praying that ½ of David‘s interest be forfeited in its favor. RTC
dismissed the petition holding that the two (2) deeds of quitclaim executed by Helen
Meyers Guzman had no legal force and effect so that the ownership of the property
subject thereof remained with her, which the Court of Appeals affirmed. Hence, this
petition.

Issue:

Whether or not there was a donation inter vivos.

Ruling:

No. Not all the elements of a donation are present. The transfer of the properties by
virtue of a Deed of Quitclaim resulted in the (1) reduction of her patrimony as donor and
the (2) consequent increase in the patrimony of David as donee. However, Helen‘s (3)
intention to perform an act of liberality in favor of David was not sufficiently established.
The 2 Quitclaims reveal that Helen intended to convey to her son certain parcels of land
and to re-affirm it, she executed a waiver and renunciation of her rights over these
properties. It is clear that Helen merely contemplated a waiver of her rights, title, and
interest over the lands in favor of David, not a donation.

She was also aware that donation was not possible. Moreover, the essential element of
acceptance in the proper form and registration to make the donation valid is lacking.
The SPA executed by David in favor of Atty. Abela was not his acceptance, but an
acknowledgment that David owns the property referred to and that he authorizes Atty.
Abela to sell the same in his name. Further, there was nothing in the SPA to show that

362
he indeed accept the donation. However, the inexistence of a donation does not make
the repudiation of Helen in favor David valid. There is NO valid repudiation of
inheritance as Helen had already accepted her share of the inheritance when she,
together with David, executed a Deed of Extrajudicial Settlement of the Estate, dividing
and adjudicating between them all the properties. By virtue of that settlement, the
properties were registered in their names and for 11 years, they possessed the land in
the concept of owner. Thus, the 2 Quitclaims have no legal force and effect. Helen still
owns ½ of the property. Nevertheless, the nullity of the repudiation does not ipso facto
operate to convert the parcels of land into res nullius to be escheated in favor of the
Government. The repudiation being of no effect whatsoever the parcels of land should
revert to their private owner, Helen, who, although being an American citizen, is
qualified by hereditary succession to own the property subject of the litigation.

363

You might also like